You are on page 1of 367

Conceptual Trigonometry

Part I

A Companion
to
S. L. Loney’s
Plane Trigonometry Part I

By

Chandra Shekhar Kumar


Integrated M. Sc. in Physics, IIT Kanpur, India
CEO & Co-Founder, Ancient Kriya Yoga Mission
CTO & Co-Founder, Ancient Science Publishers

Ancient Science Publishers


test
TEX is a trademark of the American Mathematical Society.
METAFONT is a trademark of Addison-Wesley.

Care has been taken in the preparation of this book, but makes no
expressed or implied warranty of any kind and assumes no respon-
sibility for errors or omissions. No liability is assumed for incidental
or consequential damages in connection with or arising out of the
use of the information contained herein.

For comments, suggestions and or feedback, send mail to :


ancientsciencepublishers@gmail.com

Copyright ©2018 Chandra Shekhar Kumar

All rights reserved. This work is protected by copyright and per-


mission must be obtained prior to any prohibited reproduction, stor-
age in a retrieval system, or transmission in any form or by any
means, electronic, mechanical, photocopying, recording, or likewise
unless stated otherwise.

Ancient Science Publishers, # 404, Hari Kunj, Saraidhela, Dhanbad, Jharkhand 828127, India.
Highly Recommended Books for Self Study & Competitions
Author : Chandra Shekhar Kumar
Conceptual Kinematics
A Companion to I. E. Irodov’s Problems in General Physics
Conceptual Geometry of Straight Line
A Companion to S. L. Loney’s Co-ordinate Geometry
Conceptual Geometry
A Companion to S. L. Loney’s Co-ordinate Geometry
Conceptual Trigonometry Part II
A Companion to S. L. Loney’s Plane Trigonometry Part II
Conceptual Dynamics
A Companion to S. L. Loney’s Elements of Dynamics
Conceptual Statics
A Companion to S. L. Loney’s Elements of Statics
Conceptual Particle Dynamics
A Companion to S. L. Loney’s Dynamics of A Particle
Conceptual Rigid Body Dynamics
A Companion to S. L. Loney’s Dynamics of Rigid Bodies
Conceptual School Geometry
A Companion to Hall & Stevens’ School Geometry
Conceptual School Algebra
A Companion to Hall & Knight’s Elementary Algebra
Problems and Solutions in Plane Trigonometry
by
Isaac Todhunter & Neeru Singh
Solutions of the Examples in Higher Algebra
by
H. S. Hall, S. R. Knight, Neeru Singh
& C. S. Kumar
Questions and Problems in School Physics
A Companion to I. E. Irodov’s Problems in General Physics
by
Lev Tarasov, Aldina Tarasova
& Chandra Shekhar Kumar
Calculus
Basic Concepts for High Schools
by
Lev Tarasov & Chandra Shekhar Kumar

General Methods for Solving Physics Problems


A Companion to I. E. Irodov’s Problems in General Physics
by
B. S. Belikov & Chandra Shekhar Kumar
Preface

Back in 1990, solving the problems and exercises given in the


text-book of Plane Trigonometry Part I by S. L. Loney had a ter-
rorizing effect on me, irrespective of the outcome of the countless
hours, full of perspiration and inspiration, laced with joy and sur-
rendering to the sheer beauty and elegance of each problem, sub-
problem, ... woven with multi-concepts.. Whenever stuck, I used
to revise the concepts embedded in the text-book and related refer-
ences, monographs, take a break and start all over... an irresistible
journey... back n forth between the classics of Loney and others.
As time grew, I ended up stocking a huge pile of sheets comprising
of my notes as an endeavor to solve and devour the entire book and
beyond (needless to mention that I laid my hands on everything I
could in my pursuit).
Somewhere in 2005, I started collating and organizing my notes
to instill coherence and capture the elegance in the flow.
The present work is an outcome of this pursuit, which will serve as
a complete guide to private students reading the subject with few or
no opportunities of instruction. This will save the time and lighten
the work of Teachers as well. This book helps in acquiring a bet-
ter understanding of the basic principles of Plane Trigonometry and
in revising a large amount of the subject matter quickly. Care has
been taken, as in the forthcoming ones, to present the solutions with
multi-concepts and beyond in a simple natural manner, in order to
meet the difficulties which are most likely to arise, and to render the
work intelligible and instructive.
This work contains several variations of problems, solutions, meth-
ods, approaches to enrich, strengthen and enliven the inherent multi-
concepts.

Ancient Science Publishers Chandra Shekhar Kumar


July, 2018.
List of Chapters

Preface i

1 Measurement of Angles 1
1.1 Sexagesimal and Centesimal Measure . . . . . . . . . . 1
1.2 Circular Measure . . . . . . . . . . . . . . . . . . . . . . 7
1.3 The Radian . . . . . . . . . . . . . . . . . . . . . . . . . . 8
1.4 Measurement of Any Angle in Radians . . . . . . . . . 14

2 Trigonometrical Ratios for Angles Less Than A Right


Angle 21
2.1 Trigonometrical Ratios . . . . . . . . . . . . . . . . . . . 21
2.2 Relations and Trigonometrical Ratios . . . . . . . . . . 29
2.3 Values of Trigonometrical Ratios . . . . . . . . . . . . . 36

3 Simple Problems in Heights and Distances 41


3.1 Simple Problems . . . . . . . . . . . . . . . . . . . . . . 41

4 Applications of Algebraic Signs to Trigonometry 51


4.1 Tracing the changes in the ratios . . . . . . . . . . . . . 51

5 Trigonometrical Functions of Angles of Any Size and


Sign 55
5.1 Angles of Any Size and Sign . . . . . . . . . . . . . . . . 55

6 General Expressions for All Angles Having A Given Trigono-


metrical Ratio 65
6.1 Generic Values . . . . . . . . . . . . . . . . . . . . . . . 65
6.2 Trigonometrical Equations . . . . . . . . . . . . . . . . 73

7 Trigonometrical Ratios of The Sum and Difference of


Two Angles 85
7.1 Addition and Subtraction Theorems . . . . . . . . . . . 85
7.2 Product Formulae . . . . . . . . . . . . . . . . . . . . . . 88
7.3 Converse Formulae . . . . . . . . . . . . . . . . . . . . . 96
7.4 Tangent of The Sum of Two Angles . . . . . . . . . . . . 100

8 The Trigonometrical Ratios of Multiple and Submulti-


ple Angles 103
8.1 Multiple Angles . . . . . . . . . . . . . . . . . . . . . . . 103
8.2 Submultiple Angles . . . . . . . . . . . . . . . . . . . . . 114
LIST OF CHAPTERS

8.3 Angles of 9◦ , 18◦ , 36◦ , 81◦ . . . . . . . . . . . . . . . . . 128

9 Identities And Trigonometrical Equations 133


9.1 Tangent of The Sum of Angles . . . . . . . . . . . . . . 133
9.2 Identities . . . . . . . . . . . . . . . . . . . . . . . . . . . 133
9.3 Trigonometrical Equations . . . . . . . . . . . . . . . . 145

10 Logarithms 159
10.1 Characteristics, Mantissa and Properties . . . . . . . . 159

11 Tables of Logarithms And Trigonometrical Ratios, Prin-


ciple of Proportional Parts 165
11.1 Proportional Parts . . . . . . . . . . . . . . . . . . . . . 165
11.2 Logarithmic Sines, Tangents And Secants . . . . . . . 172

12 Relations Between The Sides and The Trigonometrical


Ratios of The Angles of Any Triangle 177
12.1 Basics . . . . . . . . . . . . . . . . . . . . . . . . . . . . . 177
12.2 Sides And Angles of A Triangle . . . . . . . . . . . . . . 180

13 Solution of Triangles 195


13.1 Solution of Right-Angled Triangles . . . . . . . . . . . . 195
13.2 Solution of Non-Right-Angled Triangles . . . . . . . . . 197
13.3 Two Sides and Included Angle . . . . . . . . . . . . . . 202
13.4 Ambiguous Case . . . . . . . . . . . . . . . . . . . . . . 213
13.5 One Side Two Angles vs. Three Angles . . . . . . . . . 220

14 Heights and Distances 225


14.1 Inaccessible Object and Distant Points . . . . . . . . . 225
14.2 Angle Subtended at Two Points . . . . . . . . . . . . . . 236

15 Properties of A Triangle 259


15.1 Area of a Given Triangle . . . . . . . . . . . . . . . . . . 259
15.2 The Circles Connected With A Triangle . . . . . . . . . 264
15.3 Orthocenter and Pedal Triangle . . . . . . . . . . . . . 272

16 On Quadrilaterals And Regular Polygons 291


16.1 Area of a Quadrilateral . . . . . . . . . . . . . . . . . . . 291
16.2 Regular Polygons . . . . . . . . . . . . . . . . . . . . . . 296

17 Trigonometrical Ratios of Small Angles, Area of A Cir-


cle, DIP of The Horizon. 307
17.1 Ratios of Small Angles . . . . . . . . . . . . . . . . . . . 307
17.2 Area of A Circle . . . . . . . . . . . . . . . . . . . . . . . 311
17.3 Dip of The Horizon . . . . . . . . . . . . . . . . . . . . . 314

18 Inverse Circular Functions 319


18.1 Identities and Equations . . . . . . . . . . . . . . . . . . 319

19 On Some Simple Trigonometrical Series 335


19.1 Simple Series . . . . . . . . . . . . . . . . . . . . . . . . 335

20 Elimination 347
LIST OF CHAPTERS

20.1 Elimination of Unknown Quantity . . . . . . . . . . . . 347


Chapter
1
Measurement of Angles

1.1 Sexagesimal and Centesimal Measure


Express in terms of a right angle the angles
§ Problem 1.1.1. 60◦ . ♢
60 2
§§ Solution. 60◦ = of a right angle = of a right angle. ■
90 3

§ Problem 1.1.2. 75◦ 15(


′.
) ♢
75 15 301
§§ Solution. 75◦ 15′ = + of a right angle = of a
90 90 × 60 360
right angle. ■

§ Problem 1.1.3. 63◦ 17′ 25′′ . ♢


§§ Solution. ( )
63 17 25
63◦ 17′ 25′′ = + + of a right angle
90 90 × 60 90 × 60 × 60
45569
= of a right angle. ■
64800
§ Problem 1.1.4. 130◦ 30(
′.
) ♢
130 30 9
§§ Solution. 130◦ 30′ = + of a right angle = 1 of
90 90 × 60 20
right angle. ■

§ Problem 1.1.5. 210◦ 30′ 30′′ . ♢


1.1. Sexagesimal and Centesimal Measure 2

§§ Solution. ( )
210 30 30
210◦ 30′ 30′′ = + + of a right angle
90 90 × 60 90 × 60 × 60
3661
=2 of a right angle. ■
10800
§ Problem 1.1.6. 370◦ 20′ 48′′ . ♢
§§ Solution. ( )
370 20 48
370◦ 20′ 48′′ = + + of a right angle
90 90 × 60 90 × 60 × 60
388
=4 of a right angle. ■
3376

Express in grades, minutes, and seconds the angles


§ Problem 1.1.7. 30(◦ . )g ♢
1 100g
§§ Solution. 30◦ = 30 + × 30 = = 33g 33′ 33.3̇′′ . ■
9 3

§ Problem 1.1.8. 81(◦ . )g ♢


1
§§ Solution. 81◦ = 81 + × 81 = 90g . ■
9

§ Problem 1.1.9. 138◦ 30′ . ♢


30◦
§§ Solution. 30′ = = .5◦ ;
60
138.5
∴ 138◦ 30′ = 138.5◦ = of a right angle
90
= 1.538̇ of a right angle = 153.8̇g = 153g 88′ 88.8̇′′ . ■

§ Problem 1.1.10. 35◦ 47′ 15′′ . ♢


15′
§§ Solution. 15′′ = = .25′ ;
60
47.25◦
∴ 47′ 15′′ = 47.25′ = = .7875◦ ;
60
35.7875
∴ 35◦ 47′ 15′′ = 35.7875◦ = of a right angle
90
= .397638̇ of a right angle = 39.7638̇g = 39g 76′ 38.8̇′′ . ■

§ Problem 1.1.11. 235◦ 12′ 36′′ . ♢


36′ 3′
§§ Solution. 36′′ = = = .6′ ;
60 5
12.6◦
∴ 12′ 36′′ = 12.6′ = = .21◦ ;
60
235.21
∴ 235◦ 12′ 36′′ = 235.21◦ = of a right angle
90
= 2.6134̇ of a right angle = 261.34̇g = 261g 34′ 44.4̇′′ . ■

§ Problem 1.1.12. 475◦ 13′ 48′′ . ♢


1.1. Sexagesimal and Centesimal Measure 3

48′ 4′
§§ Solution. 48′′ = = = .8′ ;
60 5
13.8◦
∴ 13′ 48′′ = 13.8′ = = .23◦ ;
60
475.23
∴ 475◦ 13′ 48′′ = 475.23◦ = of a right angle
90
= 5.2803̇ of a right angle = 528.03̇g = 528g 3′ 33.3̇′′ . ■

Express in terms of right angles, and also in degrees, minutes,


and seconds the angles
§ Problem 1.1.13. 120g . ♢
g 120 6
§§ Solution. 120 = of a right angle = of a right angle =
100 5
6
× 90◦ = 108◦ .
5 ( )◦
1
Otherwise thus : 120g = 120 − × 120 = 108◦ . ■
10

§ Problem 1.1.14. 45g 35′ 24′′ . ♢


§§ Solution. ( )
45 35 24
45g 35′ 24′′ = + + of a right angle
100 100 × 100 100 × 100 × 100
= .453524 of a right angle
= .453524 × 90◦ = 40.81716◦ = 40◦ + .81716◦
= 40◦ + .81716 × 60′ = 40◦ + 49.0296′ = 40◦ + 49′ + .0296′
= 40◦ + 49′ + .0296 × 60′′ = 40◦ + 49′ + 1.776′′
∴ 45g 35′ 24′′ = 40◦ 49′ 1.776′′ .

§ Problem 1.1.15. 39g 45′ 36′′ . ♢


§§ Solution.
39g 45′ 36′′ = .394536 of a right angle
= .394536 × 90◦ = 35.50824◦ = 35◦ + .50824◦
= 35◦ + .50824 × 60′ = 35◦ + 30.4944′ = 35◦ + 30′ + .4944′
= 35◦ + 30′ + .4944 × 60′′ = 35◦ + 30′ + 29.664′′
∴ 39g 45′ 36′′ = 35◦ 30′ 29.664′′ .

§ Problem 1.1.16. 255g 8′ 9′′ . ♢


§§ Solution.
255g 8′ 9′′ = 2.550809 of a right angle
= 2.550809 × 90◦ = 229.57281◦ = 229◦ + .57281◦
= 229◦ + .57281 × 60′ = 229◦ + 34.3686′ = 229◦ + 34′ + .3686′
= 229◦ + 34′ + .3686 × 60′′ = 229◦ + 34′ + 22.116′′
∴ 255g 8′ 9′′ = 229◦ 34′ 22.116′′ .

§ Problem 1.1.17. 759g 0′ 5′′ . ♢


1.1. Sexagesimal and Centesimal Measure 4

§§ Solution.
759g 0′ 5′′ = 7.590005 of a right angle
= 7.590005 × 90◦ = 683.10045◦ = 683◦ + .10045◦
= 683◦ + .10045 × 60′ = 683◦ + 6.0270′ = 683◦ + 6′ + .0270′
= 683◦ + 6′ + .0270 × 60′′ = 683◦ + 6′ + 1.620′′
∴ 759g 0′ 5′′ = 683◦ 6′ 1.62′′ .

Mark the position of the revolving line when it has traced out
the following angles

B
P2 P1

P
O
A′ A

P3 P4

B′

4
§ Problem 1.1.18. right angle. ♢
3
4 1
§§ Solution. Since right angle = 90◦ + × 90◦ = 90◦ + 30◦ , the
3 3
revolving line has turned through 30◦ more than a right angle and is
therefore in the second quadrant, i.e. is between OB and OA′ and
makes an angle of 30◦ with OB. ■

1
§ Problem 1.1.19. 3 right angles. ♢
2
1 1
§§ Solution. Since 3 right angles = 3×90◦ + ×90◦ = 3×90◦ +45◦ ,
2 2
the revolving line has turned through 45◦ more than three right an-
gles and is therefore in the fourth quadrant, i.e. is halfway between
OB ′ and OA. ■

1
§ Problem 1.1.20. 13 right angles. ♢
3
1 1
§§ Solution. Since 13 right angles = 13 × 90◦ + × 90◦ = 13 × 90◦ +
3 3
30◦ , the revolving line has turned through 30◦ more than thirteen
right angles and is therefore in the second quadrant, i.e. is between
OB and OA′ , and makes an angle of 30◦ with OB. ■

§ Problem 1.1.21. 120◦ . ♢


1.1. Sexagesimal and Centesimal Measure 5

§§ Solution. Since 120◦ = 90◦ + 30◦ , we have the same result as in §


Problem 1.1.18. ■

§ Problem 1.1.22. 315◦ . ♢


§§ Solution. Since 315◦ = 3 × 90◦ + 45◦ , we have the same result as
in § Problem 1.1.19. ■

§ Problem 1.1.23. 745◦ . ♢


§§ Solution. Since 745◦ = 8×90◦ +25◦ , the revolving line has turned
through 25◦ more than eight right angles and is therefore in the first
quadrant, i.e. is between OA and OB, and makes an angle of 25◦
with OA. ■

§ Problem 1.1.24. 1185◦ . ♢


§§ Solution. Since 1185◦ = 13 × 90◦ + 15◦ , the revolving line has
turned through 15◦ more than thirteen right angles and is therefore
in the second quadrant, i.e. is between OB and OA′ , and makes an
angle of 15◦ with OB. ■

§ Problem 1.1.25. 150g . ♢


§§ Solution. 150g = 100g +50g , the revolving line has turned through
50g more than a right angle, and is therefore in the second quadrant,
i.e. is halfway between OB and OA′ . ■

§ Problem 1.1.26. 420g . ♢


§§ Solution. 420g = 4 × 100g + 20g , the revolving line has turned
through 20g more than four right angles, and is therefore in the first
quadrant, i.e. is between OA and OB and makes an angle of 20g with
OA. ■

§ Problem 1.1.27. 875g . ♢


§§ Solution. 875g = 8 × 100g + 75g , the revolving line has turned
through 75g more than eight right angles, and is therefore in the
first quadrant, i.e. is between OA and OB and makes an angle of 75g
with OA. ■

§ Problem 1.1.28. How many degrees, minutes and seconds are re-
1
spectively passed over in 11 minutes by the hour and minute hands
9
of a watch ? ♢
360◦
§§ Solution. A minute-division on the face of a clock = = 6◦ .
60
Also, the minute-hand moves twelve times as fast as the hour-
hand.
1
Hence, in 11 minutes, the minute-hand passes over
9 ( )◦
1 2◦
11 × 6 = 66 = 66◦ 40′ .
9 3
and the hour-hand passes over
66◦ 40′
= 5◦ 33′ 20′′ . ■
12
1.1. Sexagesimal and Centesimal Measure 6

§ Problem 1.1.29. The number of degrees in one acute angle of a


right-angled triangle is equal to the number of grades in the other;
express both the angles in degrees. ♢
§§ Solution. If x be the number of degrees in one of the required
angles, then (90 − x) is the number of degrees in the other angle and
10
therefore (90 − x) is the number of grades in the other angle.
9
10 7
∴x= (90 − x), ∴ x = 47
9 ( 19 )
7 ◦ 7 ◦ 7 ◦
So that the required angles are 47 and 90 − 47 , i.e. 47
19 19 19
12 ◦
and 42 . ■
19

§ Problem 1.1.30. Prove that the number of Sexagesimal minutes


in any angle is to the number of Centesimal minutes in the same
angle as 27 : 50. ♢
§§ Solution. If S be the number of Sexagesimal minutes in any an-
gle and C be the number of Centesimal minutes in the same angle,
we have
S the given ∠ C
= = .
90 × 60 a right ∠ 100 × 100
S 9×6 27
∴ = =
C 10 × 10 50
∴ S : C = 27 : 50.
Otherwise thus :

1 10 1
1 Sexagesimal minute = minute = × grade
60 9 60
1 100
= grade = Centesimal minute
54 54
50
= Centesimal minute . ■
27
§ Problem 1.1.31. Divide 44◦ 8′ into two parts such that the number
of Sexagesimal seconds in one part may be equal to the number of
Centesimal seconds in the other part. ♢
( )
2
§§ Solution. If x be the number of degrees in one part, then 44 −x
( ) 15
10 2
is the number of degrees in the other part, i.e. 44 − x is the
9 15
number of grades in the other part.
In x◦ , there are (x × 60 × 60) Sexagesimal seconds and in
( )g
10 2
44 −x ,
9 15
there are [ ( ) ]
10 2
44 − x × 100 × 100
9 15
Centesimal seconds. ( )
10 2
∴ x × 60 × 60 = 44 − x × 100 × 100
9 15
100◦
∴x= = 33◦ 20′ ,
3
1.2. Circular Measure 7

◦ ′
( ◦ ′ parts◦ are
So that the required ) 33 20 and
44 8 − 33 20 , i.e. 33◦ 20′ and 10◦ 48◦ .
′ ■

1.2 Circular Measure


§ Problem 1.2.1. If the radius of the earth be 4000 miles, what is
the length of its circumference ? ♢
§§ Solution. The length of the circumference
= (2 × π × 4000) miles
≈ (3.14159265 × 8000) miles ≈ 25132.74 miles. ■

§ Problem 1.2.2. The wheel of a railway carriage is 3 feet in diam-


eter and makes 3 revolutions in a second; how fast is the train going
? ( ) ♢
3
§§ Solution. The circumference of the wheel = 2 × π × f eet =
2
3π f eet.
Hence the required rate ( )
60 × 60
= (3 × 3π) f eet per second = 9π × miles per hour
[ ) 1760 × 3
135 22
= taking π = ≈ 19.28 miles per hour. ■
7 7
§ Problem 1.2.3. A mill sail whose length is 18 feet makes 10 rev-
olutions per minute. What distance does its end travel in an hour
? ♢
§§ Solution. The circumference = (2 × π × 18) f eet = 36π f eet.
Hence the end travels (36π × 10) feet per minute, i.e.
( )
60
360π × miles per hour
[ ×3
1760 ]
90 22
i.e. taking π = ,
7 7
i.e. ≈ 12.85 miles per hour. ■

§ Problem 1.2.4. The diameter of a half-penny is an inch; what is


the length of a piece of string which would just surround its curved
edge ? ♢
§§ Solution. The required
( length
)
1
= 2×π× inches
2
= π inches = 3.14159 . . . inches. ■

§ Problem 1.2.5. Assuming that the earth describes in one year a


circle of 92500000 miles radius, whose center is the sun, how many
miles does the earth travel in a year ? ♢
§§ Solution. The required distance
= (2π × 92500000) miles
≈ (3.14159265 × 185000000) miles ≈ 581194640 miles. ■
1.3. The Radian 8

§ Problem 1.2.6. The radius of a carriage wheel is 1 f t. 9 ins. and in


1
th of a second it turns through 80◦ about its center, which is fixed;
9
how many miles does a point on the rim of the wheel travel in one
hour ? ♢
§§ Solution. The wheel
( makes) one complete revolution in
360 1 1
× sec., i.e. in sec.
80 9 2
Hence it makes (60 × 60 × 2) revolutions per hour.
The circumference of the wheel [ ]
22
= (2 × π × 21) inches ≈ (22 × 6) inches taking π = .
7
Hence the required number of miles
60 × 60 × 2 5
= (22 × 6) × = 22 × 6 × = 15.
1760 × 3 × 12 44
If we take π = 3.14159265, the required number ≈ 14.994. ■

1.3 The Radian


Express in degrees, minutes and seconds the angles :
πc
§ Problem 1.3.1. . ♢
3
π c 1
§§ Solution. = × 180◦ = 60◦ . ■
3 3

4π c
§ Problem 1.3.2. . ♢
3
4π c 4
§§ Solution. = × 180◦ = 240◦ . ■
3 3

§ Problem 1.3.3. 10π c . ♢


§§ Solution. 10π c = 10 × 180◦ = 1800◦ . ■

§ Problem 1.3.4. 1c . ♢
1 1
§§ Solution. 1c = × π c = × 180◦ = 57◦ 17′ 44.8′′ [see Art. 16]. ■
π π

§ Problem 1.3.5. 8c . ♢
8 8
§§ Solution. 8 = ×π = ×180 = 8×(57 17 44.8 ) = 458 21 58.4′′ .
c c ◦ ◦ ′ ′′ ◦ ′
π π

Express in grades, minutes and seconds the angles :


4π c
§ Problem 1.3.6. . ♢
5
4π c 4
§§ Solution. = × 200g = 160g . ■
5 5

7π c
§ Problem 1.3.7. . ♢
6
7π c 7 1g
§§ Solution. = × 200g = 233 = 233g 33′ 33.3̇′′ . ■
6 6 3
1.3. The Radian 9

§ Problem 1.3.8. 10π c . ♢


§§ Solution. 10π c = 10 × 200g = 2000g . ■

Express in radians the following angles :


§ Problem 1.3.9. 60◦ . ♢
πc π
§§ Solution. 60◦ = 60 × = radians. ■
180 3

§ Problem 1.3.10. 110◦ 30′ . ♢


1 πc 221
§§ Solution. 110◦ 30′ = 110 × = π radians. ■
2 180 360

§ Problem 1.3.11. 175◦ 45′ . ♢


3 πc 703
§§ Solution. 175◦ 45′ = 175 × = π radians. ■
4 180 720

§ Problem 1.3.12. 47◦ 25′ 36′′ . ♢


§§ Solution. Å ã◦
3′ 25 3
47◦ 25′ 36′′ = 47◦ 25 = 47 5
5 60
32 πc 3557
= 47 × = π radians. ■
75 180 13500
§ Problem 1.3.13. 395◦ . ♢
πc 79
§§ Solution. 395◦ = 395 × = π radians. ■
180 36

§ Problem 1.3.14. 60g . ♢


πc 3
§§ Solution. 60g = 60 × = π radians. ■
200 10

§ Problem 1.3.15. 110g 30′ . ♢


πc 1103
§§ Solution. 110g 30′ = 110.30g = 110.30 × = π radians. ■
200 2000

§ Problem 1.3.16. 345g 25′ 36′′ . ♢


§§ Solution.
345g 25′ 36′′ = 345.2536g
πc
= 345.2536 × = 1.726268π radians. ■
200
§ Problem 1.3.17. The difference between the two acute angles of
2
a right-angled triangle is π radians; express the angles in degrees.
5

2 2
§§ Solution. π radians = × 180◦ = 72◦ .
5 5
Hence, if x be the number of degrees in the larger of the required
angles, then (90 − x) is the number of degrees in the other angle.
Hence x−(90 − x) = 72, whence x = 81, so that the required angles
are 81◦ and 9◦ . ■
1.3. The Radian 10

2
§ Problem 1.3.18. One angle of a triangle is x grades and another
3
3 πx
is x degrees, whilst the third is radians; express them all in
2 75
degrees. ♢
§§ Solution. Since the sum of the three angles of a triangle is 180◦ ,
we have, by reducing the given angles to degrees,
9 2x 3x x
· + + × 180 = 180
10 3 2 75
x x 4x
∴ + + = 60, ∴ x = 40
5 2 5
so that the angles are 24◦ , 60◦ and 96◦ . ■

§ Problem 1.3.19. The circular measure of two angles of a triangle


1 1
are respectively and ; what is the number of degrees in the third
2 3
angle ? ♢
§§ Solution. The circular measure of the third angle
1 1 5
=π− − =π− .
2 3 6
Hence the required number of degrees
5 180◦ 5
= 180◦ − · = 180◦ − × 57◦ 17′ 44.8′′ [Art. 16] .
6 π 6
= 180◦ − 47◦ 44′ 47.3̇′′ = 132◦ 15′ 12.6̇′′ . ■

§ Problem 1.3.20. The angles of a triangle are in A. P. and the


number of degrees in the least is to the number of radians in the
greatest as 60 to π; find the angles in degrees. ♢
§§ Solution. Let the angles be (x − y)◦ , x◦ and (x + y)◦ .
Since the sum of the three angles of a triangle is 180◦ , we have
x − y + x + x + y = 180◦
∴ 3x = 180◦ , ∴ x = 60◦ .
The required angles are therefore
(60 − y)◦ , 60◦ and (60 + y)◦ .
π
Now (60 + y)◦ = (60 + y) radians.
180
π
∴ 60 − y : (60 + y) :: 60 : π.
180
180 60 − y 60
∴ · =
π 60 + y π
∴ 3 (60 − y) = 60 + y, ∴ y = 30.
The angles are therefore 30◦ , 60◦ and 90◦ . ■

§ Problem 1.3.21. The angles of a triangle are in A. P. and the


number of radians in the least angle is to the number of degrees in
the mean angle as 1 : 120. Find the angles in radians. ♢
§§ Solution. Let (x − y), x and (x + y) be the number of radians in
the three angles respectively.
∴ x − y + x + x + y = π.
π π
∴ 3x = π, ∴ x = . ∴ − y : 60 :: 1 : 120.
3 3
2π π 1
∴ − 2y = 1, ∴ y = − .
3 3 2
1.3. The Radian 11

1 π 2π 1
Thus the angles contain , and − radians. ■
2 3 3 2

§ Problem 1.3.22. Find the magnitude, in radians and degrees, of


the interior angle of

(1) a regular pentagon

(2) a regular heptagon

(3) a regular octagon

(4) a regular duodecagon, and

(5) a regular polygon of 17 sides. ♢


§§ Solution. Proceeding as in Ex. 2, Art. 20, we have
6
(1) 5x + 4 = 10, ∴ x = right angle.
5
Hence the required angle
6 π 3π 6
= × = radians = × 90◦ = 108◦ .
5 2 5 5
10
(2) 7x + 4 = 14, ∴ x = right angle.
7
Hence the required angle
10 π 5π 10 4◦
= × = radians = × 90◦ = 128 .
7 2 7 7 7
12 3
(3) 8x + 4 = 16, ∴ x = right angle = right angle.
8 2
Hence the required angle
3 π 3π 3
= × = radians = × 90◦ = 135◦ .
2 2 4 2
20 5
(4) 12x + 4 = 24, ∴ x = right angle = right angle.
12 3
Hence the required angle
5 π 5π 5
= × = radians = × 90◦ = 150◦ .
3 2 6 3
30
(5) 17x + 4 = 34, ∴ x = right angle.
17
Hence the required angle
30 π 15π 30 14 ◦
= × = radians = × 90◦ = 158 .
17 2 17 17 17 ■

§ Problem 1.3.23. The angle in one regular polygon is to that in


another as 3 : 2; also the number of sides in the first is twice that in
the second; how many sides have the polygons ? ♢
1.3. The Radian 12

§§ Solution. Let 2n and n be the number of sides in the two poly-


gons respectively. Since all the angles of the first polygon = (4n − 4)
(4n − 4)90
right angles, therefore each angle contains degrees.
2n
(2n − 4)90
Similarly, each angle of the second polygon contains
n
degrees. Hence we have
(4n − 4)90 (2n − 4)90
: =3:2
2n n
∴ 4n − 4 = 6n − 12; ∴ n = 4;
Hence the polygons have 8 sides and 4 sides respectively. ■

§ Problem 1.3.24. The number of sides in two regular polygons are


as 5 : 4 and the difference between their angles is 9◦ ; find the number
of sides in the polygons. ♢
§§ Solution. Let 5n and 4n be the number of sides in the two poly-
gons respectively.
Since all the angles of the first polygon = (10n − 4) right angles,
(10n − 4)90
therefore each angle contains degrees.
5n
(8n − 4)90
Similarly, each angle of the second polygon contains
4n
degrees. Hence we have
(10n − 4)90 (8n − 4)90
− = 9, ∴ n = 2,
5n 4n
so that the polygons have 10 sides and 8 sides respectively. ■

§ Problem 1.3.25. Find two regular polygons such that the number
of their sides may be as 3 to 4 and the number of degrees in an angle
of the first to the number of grades in an angle of the second as 4 to
5. ♢
§§ Solution. Let 3n and 4n be the number of sides in the two poly-
gons respectively.
Since all the angles of the first polygon = (6n − 4) right angles,
(6n − 4)90
therefore each angle contains degrees.
3n
(8n − 4)90
Similarly, each angle of the second polygon contains
4n
(8n − 4)100
degrees, i.e. grades.
4n
Hence we have
(6n − 4)90 (8n − 4)100
: = 4 : 5, ∴ n = 2,
3n 4n
so that the polygons have 6 sides and 8 sides respectively. ■

§ Problem 1.3.26. The angles of a quadrilateral are in A. P. and the


greatest is double the least; express the least angle in radians. ♢
§§ Solution. Let x − 3y, x − y, x + y and x + 3y be the number of
radians in the angles. Then
x − 3y + x − y + x + y + x + 3y = 2π;
π
∴ 4x = 2π, ∴ x = .
2
Also, we have x + 3y = 2(x − 3y).
1.3. The Radian 13

π π
∴ 9y = x = , ∴y= .
( 2 ) 18
π 3π π
Hence the least angle contains − radians, i.e. radians. ■
2 18 3

§ Problem 1.3.27. Find in radians, degrees and grades the angle


between the hour-hand and the minute-hand of a clock at

(1) half-past three

(2) twenty minutes to six

(3) a quarter past eleven. ♢


§§ Solution. Since the minute-hand moves twelve times as fast as
the hour-hand, it gains 11 minute-divisions in 12 minutes on the hour-
12
hand, i.e. 1 minute-division in minutes.
11
(1) At 3 o’clock there are 15 minute-divisions between the hands
11 × 30 55
and in 30 minutes the minute-hand gains , i.e. , minute-
12 2
divisions on the hour-hand.
( )
55 1
Hence at half-past 3, there are − 15 , i.e. 12 , minute-
2 2
divisions between the hands.
Now a minute-division on the face of a clock
πc 20g
= = 6◦ = [cf §P roblem 1.1.28] .
30 3
Hence the required answers are
1 πc 5π c 1 1 20g 1g
12 × , i.e. ; 12 × 6◦ , i.e. 75◦ ; and 12 × , i.e. 83 .
2 30 12 2 2 3 3
2
(2) Here there are 11 minute-divisions between the hands. Hence
3
the required answers are
2 πc 7π c 2 2 20g 7g
11 × , i.e. ; 11 × 6◦ , i.e. 70◦ ; and 11 × , i.e. 77 .
3 30 18 3 3 3 9
3
(3) Here there are 18 minute-divisions between the hands. Hence
4
the required answers are
3 πc 5π c 3 1◦
18 × , i.e. ; 18 × 6◦ , i.e. 112 ; and
4 30 8 4 2
3 20g
18 × , i.e. 125g .
4 3 ■

§ Problem 1.3.28. Find the times

(1) between four and five o’clock when the angle between the minute-
hand and the hour-hand is 78◦

(2) between seven and eight o’clock when this angle is 54◦ . ♢
1.4. Measurement of Any Angle in Radians 14

§§ Solution. (1) At 4 o’clock the minute-hand is 20 minute-divisions


behind the hour-hand.
360◦
Since a minute-division on the face of a clock = = 6◦ , the
60
78
hands are at an angle of 78◦ when they are separated by ,
6
i.e. 13, minute-divisions; and this will be the case when the
minute-hand has gained (20 − 13), i.e. 7, and also when it has
gained (20 + 13), i.e. 33 minute-divisions.
But the minute-hand gains
11 minute − divisions in 12 minutes
( )
12 7
∴ 7 minute − divisions in ×7 =7 minutes
( 11 ) 11
12
∴ 30 minute − divisions in × 33 = 36 minutes.
11
7
Hence the angle between the hands is 78◦ after intervals of 7
11
7
minutes and 36 minutes, i.e. at 7 and 36 minutes past 4.
11
(2) At 7 o’clock the minute-hand is 35 minute-divisions behind the
hour-hand.
360◦
Since a minute-division on the face of a clock = = 6◦ , the
60
54
hands are at angle of 54◦ when they are separated by =9
6
minute-divisions and this will be the case when the minute-
hand has gained (35 − 9) = 26, and also when it has gained
(35 + 9 = 44) minute-divisions. But the minute-hand gains
11 minute − divisions in 12 minutes
4
∴ 26 minute − divisions in 28 minutes
( 11)
12
∴ 44 minute − divisions in × 44 = 48 minutes.
11
Hence the angle between the hands is 54◦ after intervals of
4 4
28 minutes and 48 minutes, i.e. at 28 and 48 minutes past
11 11 ■
7.

1.4 Measurement of Any Angle in Radians


§ Problem 1.4.1. Find the number of degrees subtended at the cen-
ter of a circle by an arc whose length is .357 times the radius. ♢
.357r
§§ Solution. The number of radians in the angle = = .357.
r
Hence the angle
2
= .357 radian = .357 × right angle
π
.714
= × 90◦ = (.714 × .3183 × 90)◦ ≈ 20.454◦ . ■
π
§ Problem 1.4.2. Express in radians and degrees the angle sub-
tended at the center of a circle by an arc whose length is 15 feet, the
radius of the circle being 25 feet. ♢
1.4. Measurement of Any Angle in Radians 15

15 3
§§ Solution. The number of radians in the angle = = .
25 5
Hence the angle
3 3 2
= radian = × right angle
5 5 π
6 6
= × 90◦ = × 18◦ = 108◦ × .31831
5π π
= 34.37748◦ = 34◦ + .37748 × 60′ = 34◦ + 22.6488′
= 34◦ + 22′ + .6488 × 60′′ = 34◦ + 22′ + 38.928′′
≈ 34◦ 22′ 38.9′′ . ■

§ Problem 1.4.3. The value of the divisions on the outer rim of a


graduated circle is 5′ and the distance between successive gradua-
tions is .1 inch. Find the radius of the circle. ♢
§§ Solution. If r be the number of inches in the radius, we have
.1
= the number of radians in 5′
r
1 5 π
∴ = ×
10r 60 180
6 180
∴r= × = 216 × .31831 ≈ 68.75′′ . ■
5 π
§ Problem 1.4.4. The diameter of a graduated circle is 6 feet and
the graduations on its rim are 5′ apart; find the distance from one
graduation to another. ♢
§§ Solution. If x be the required distance in inches, we have
x 5 π
= the number of radians in 5′ = × .
36 60 180
π 3.14159
∴x= = ≈ .05236. ■
60 60
§ Problem 1.4.5. Find the radius of a globe which is such that the
distance between two places on the same meridian whose latitude
differs by 1◦ 10′ may be half-an-inch. ♢
§§ Solution. If r be the number of inches in the radius, we have
1
2 = the number of radians in 1◦ 10′ = 1 1 × π .
r 6 180
1 6 1 3
∴ r = × × 180 × = × 180 × .31831 ≈ 24.555′′ . ■
2 7 π 7
§ Problem 1.4.6. Taking the radius of the earth as 4000 miles, find
the difference in latitude of two places, one of which is 100 miles
north of the other. ♢
§§ Solution. The number of radians in the required difference =
100 1
= .
4000 40
Hence the difference
1 1 2
= radian = × right angle
40 (40 π )◦
1 9
= × 90◦ = × .31831
20π 2
= 1.43239◦ = 1◦ + .43239 × 60′ = 1◦ + 25.9434′
= 1◦ + 25′ + .9434 × 60′′ = 1◦ + 25′ + 56.604′′
≈ 1◦ 25′ 57′′ . ■
1.4. Measurement of Any Angle in Radians 16

§ Problem 1.4.7. Assuming the earth to be a sphere and the dis-


tance between two parallels of latitude, which subtends an angle
1
of 1◦ at the earth’s center, to be 69 miles, find the radius of the
9
earth. ♢
§§ Solution. If r be the number of miles in the radius, we have
1
69
9 = the number of radians in 1◦ = π .
r 180
622 180
∴r= × = 622 × 20 × .31831 ≈ 3959.8 miles. ■
9 π
§ Problem 1.4.8. The radius of a certain circle is 3 feet; find approx-
imately the length of an arc of this circle, if the length of the chord
of the arc be 3 feet also. ♢
§§ Solution. The arc subtends an angle of 60◦ at the center of the
circle, so that if x feet be the required length, we have
x π π
= the number of radians in 60◦ = 60 × = .
3 180 3
∴ x = π = 3.14159 . . . f eet. ■

§ Problem 1.4.9. What is the ratio of the radii of two circles at the
center of which two arcs of the same length subtend angles of 60◦
and 75◦ ? ♢
§§ Solution. If r and r′ be the radii respectively and x be the length
of an arc, we have
x π
= the number of radians in 60◦ = 60 ×
r 180
x π
and = the number of radians in 75◦ = 75 × .
r′ 180
r 75 5
Hence, by division, we have ′ = = , i.e. the required ratio is
r 60 4
5 : 4. ■

§ Problem 1.4.10. If an arc, of length 10 feet, on a circle of 8 feet


diameter subtend at the center an angle of 143◦ 14′ 22′′ ; find the value
of π to 4 places of decimals. ♢
§§ Solution. We have
10 [ ]
= the number of radians in 143◦ 14′ 22′′ i.e. in 515662′′
4
515662 π
= × .
60 × 60 180
10 60 × 60 × 180
∴π= × = 3.1416. ■
4 515662
§ Problem 1.4.11. If the circumference of a circle be divided into 5
parts which are in A. P. and if the greatest part be 6 times the least,
find in radians the magnitude of the angles that the parts subtend at
the center of the circle. ♢
§§ Solution. Let x − 2y, x − y, x, x + y and x + 2y be the parts into
which the circumference is divided.
2πr
Their sum = 5x = 2πr, ∴ x = .
5
Also, we have x + 2y = 6(x − 2y)
1.4. Measurement of Any Angle in Radians 17

5x 2πr πr
∴ 14y = 5x, ∴ y = = = .
14 14 7
Hence the arcs are
4πr 9πr 14πr 19πr 24πr
, , , and .
35 35 35 35 35
Thus the angles contain
4π 9π 14π 19π 24π
, , , and . ■
35 35 35 35 35
§ Problem 1.4.12. The perimeter of a certain sector of a circle is
equal to the length of the arc of a semi-circle having the same radius;
express the angle of the sector in degrees, minutes and seconds. ♢
§§ Solution. If r be the radius of the circle and θ be the number of
radians in the angle, then the perimeter of the sector = rθ + 2r.
∴ rθ + 2r = πr, ∴ θ = (π − 2) radians.
Hence the angle
2
= (π − 2) × right angle
π
2π − 4
= × 90◦ = 180◦ − 2 × 57◦ 17′ 44.8′′
π
= 180◦ − 114◦ 35′ 29.6′′ = 65◦ 24′ 30.4′′ . ■

§ Problem 1.4.13. At what distance does a man, whose height is 6


feet, subtend an angle of 10′ ? ♢
§§ Solution. If x be the required distance in feet, we have
6 10 π
= the number of radians in 10′ = × .
x 60 180
180
∴x=6×6× = 6 × 6 × 180 × .31831 ≈ 2062.65 f eet. ■
π
§ Problem 1.4.14. Find the length which at a distance of one mile
will subtend an angle of 1′ at the eye. ♢
§§ Solution. If x be the required length in feet, we have
x 1 π
= the number of radians in 1′ = × .
1760 × 3 60 180
1760 × 3 × 3.14159
∴x= ≈ 1.5359 f eet. ■
60 × 180
§ Problem 1.4.15. Find approximately the distance at which a globe,
1
5 inches in diameter, will subtend an angle of 6′ . ♢
2
§§ Solution. If x be the required distance in feet, we have
1
5
2 = the number of radians in 6′ = 6 × π .
12x 60 180
11
∴x= × 10 × 180 × .31831 ≈ 262.6 f eet. ■
24
§ Problem 1.4.16. Find approximately the distance of a tower whose
5 ′
height is 51 feet and which subtends at the eye an angle of 5 . ♢
11
§§ Solution. If x be the required distance in feet, we have
60
51 5 ′ π 1 π
= the number of radians in 5 = 11 × = × .
x 11 60 180 11 180
∴ x = 51 × 11 × 180 × .31831 ≈ 32142.9 f eet. ■
1.4. Measurement of Any Angle in Radians 18

§ Problem 1.4.17. A church spire, whose height is known to be


100 feet, subtends an angle of 9′ at the eye; find approximately its
distance. ♢
§§ Solution. If x be the required distance in feet, we have
100 9 π π
= the number of radians in 9′ = × = .
x 60 180 60 × 20
∴ x = 100 × 60 × 20 × .31831 = 3183.1 × 12 ≈ 38197.2 f eet. ■

§ Problem 1.4.18. Find approximately in minutes the inclination to


1
the horizon of an incline which rises 3 feet in 210 yards. ♢
2
§§ Solution. The number of radians in the angle
1
3 7 1
= 2 = = .
210 × 3 210 × 6 180
Hence the angle
1 1 2
= radian = × right angle
180 180 π
1
= × 90◦ = .31831◦ = (60 × .31831)′ = 19.099′ . ■
90π
§ Problem 1.4.19. The radius of the earth being taken to be 3960
miles and the distance of the moon from the earth being 60 times the
radius of the earth, find approximately the radius of the moon which
subtends at the earth an angle of 16′ . ♢
§§ Solution. If r be the required radius in miles, we have
r 16 π
= the number of radians in 16′ = × .
3960 × 60 60 180
3960 × 16 × π
∴r= = 22 × 16 × 3.14159 ≈ 1105.8 miles. ■
180
§ Problem 1.4.20. When the moon is setting at any given place,
the angle that is subtended at its center by the radius of the earth
passing through the given place is 57′ . If the earth’s radius be 3960
miles, find approximately the distance of the moon. ♢
§§ Solution. If x be the required distance in miles, we have
3960 57 π
= the number of radians in 57′ = × .
x 60 180
20
∴ x = 3960 × × 180 × .31831 = 238833 miles. ■
19
§ Problem 1.4.21. Prove that the distance of the sun is about 81 mil-
lion geographical miles, assuming that the angle which the earth’s
radius subtends at the distance of the sun is 8.76′′ and that a geo-
graphical mile subtends 1′ at the earth’s center. Find also the cir-
cumference and diameter of the earth in geographical miles. ♢
§§ Solution. If d be the distance of the sun and r be the radius of
the earth, we have
r 8.76 π
= the number of radians in 8.76′′ = × , and
d 60 × 60 180
1 1 π
= the number of radians in 1′ = × .
r 60 180
1.4. Measurement of Any Angle in Radians 19

Hence, by multiplication, we have


1 8.76 π2
= × .
(d )602 × 60 × 60 180 × ( 180)2
6×6×6 180 6×3 180
∴d= × × 105 = × × 106 ≈ 81 × 106 .
876 π 730 π
Again, if r be the number of geographical miles in the radius of the
earth, we have
1 1 π
= the number of radians in 1′ = ×
r 60 180
180
∴ r = 60 × = 60 × 57.2957795 ≈ 3437.75 miles.
π
∴ the diameter ≈ 6875.5 miles, and
the circumf erence = 2πr = (2 × 60 × 180) miles = 21600 miles. ■

§ Problem 1.4.22. The radius of the earth’s orbit, which is about


92700000 miles, subtends at the star Sirius an angle of about .4′′ ; find
roughly the distance of Sirius. ♢
§§ Solution. If d be the required distance in miles, we have
92700000 .4 π
= the number of radians in .4′′ = ×
d 60 × 60 180
60 × 600 180
∴ d = 92700000 × × = 92700000 × 9000 × 180 × .31831
4 π
≈ 478019 × 10 ≈ 478 × 10 miles.
8 11 ■
Chapter
2
Trigonometrical Ratios for Angles
Less Than A Right Angle

2.1 Trigonometrical Ratios


Prove the following statements.
§ Problem 2.1.1. cos4 A − sin4 A + 1 = 2 cos2 A. ♢
§§ Solution. ( )( )
cos4 A − sin4 A + 1 = cos2 A + sin2 A cos2 A − sin2 A + 1
= cos2 A − sin2 A + 1, ∵ cos2 A + sin2 A = 1
( )
= cos2 − 1 − cos2 A + 1 = 2 cos2 A. ■

§ Problem 2.1.2. (sin A + cos A) (1 − sin A cos A) = sin3 A + cos3 A. ♢


§§ Solution.
(sin A + cos A) (1 − sin A cos A)
( )
= (sin A + cos A) sin2 A + cos2 A − sin A cos A
= sin3 A + cos3 A. ■

sin A 1 + cos A
§ Problem 2.1.3. + = 2 cosec A. ♢
1 + cos A sin A
§§ Solution.
sin A 1 + cos A sin2 A + cos2 A + 2 cos A + 1
+ =
1 + cos A sin A (1 + cos A) sin A
2 (1 + cos A)
=
(1 + cos A) sin A
2 1
= =2× = 2 cosec A. ■
sin A sin A
2.1. Trigonometrical Ratios 22

§ Problem 2.1.4. cos6 A + sin6 A = 1 − 3 sin2 A cos2 A. ♢


§§ Solution. ( )( )
cos6 A + sin6 A = cos2 A + sin2 A cos4 A + sin4 A − sin2 A cos2 A
= cos4 A + sin4 A − sin2 A cos2 A
( )2
= cos2 A + sin2 A − 3 sin2 A cos2 A
= 1 − 3 sin2 A cos2 A. ■


1 − sin A
§ Problem 2.1.5. = sec A − tan A. ♢
1 + sin A
§§ Solution.
… …
1 − sin A (1 − sin A)2 1 − sin A
= = √
1 + sin A 1 − sin2 A 1 − sin2 A
1 − sin A 1 sin A
= = − = sec A − tan A. ■
cos A cos A cos A
cosec A cosec A
§ Problem 2.1.6. + = 2 sec2 A. ♢
cosec A − 1 cosec A + 1
§§ Solution.
1 1
cosec A cosec A sin A sin A
+ = +
cosec A − 1 cosec A + 1 1 1
−1 +1
sin A sin A
1 1 2
= + =
1 − sin A 1 + sin A 1 − sin2 A
2
= = 2 sec2 A. ■
cos2 A
cosec A
§ Problem 2.1.7. = cos A. ♢
cot A + tan A
§§ Solution.
1
cosec A sin A
=
cot A + tan A cos A sin A
+
sin A cos A
1
= sin A
cos2 A + sin2 A
sin A cos A
1 1
= ÷ = cos A. ■
sin A sin A cos A
§ Problem 2.1.8. (sec A + cos A) (sec A − cos A) = tan2 A + sin2 A. ♢
§§ Solution.
(sec A + cos A) (sec A − cos A) = sec2 A − cos2 A
( )
= 1 + tan2 A − 1 − sin2 A
= tan2 A + sin2 A. ■

1
§ Problem 2.1.9. = sin A cos A. ♢
cot A + tan A
2.1. Trigonometrical Ratios 23

§§ Solution.
1 1
=
cot A + tan A cos A sin A
+
sin A cos A
sin A cos A
= = sin A cos A. ■
cos2 A + sin2 A
1
§ Problem 2.1.10. = sec A + tan A. ♢
sec A − tan A
§§ Solution.
1 sec A + tan A
=
sec A − tan A sec2 A − tan2 A
= sec A + tan A, ∵ sec2 A − tan2 A = 1 [By Art. 27(3)] . ■
1 − tan A cot A − 1
§ Problem 2.1.11. = . ♢
1 + tan A cot A + 1
1
1 − tan A 1−
§§ Solution. = cot A = cot A − 1 . ■
1 + tan A 1 cot A + 1
1+
cot A

1 + tan2 A sin2 A
§ Problem 2.1.12. = . ♢
1 + cot2 A cos2 A
2
1 + tan A 2
sec A 1 1 sin2 A
§§ Solution. = = ÷ = . ■
1 + cot2 A cosec 2 A cos2 A sin2 A cos2 A

sec A − tan A
§ Problem 2.1.13. = 1 − 2 sec A tan A + 2 tan2 A. ♢
sec A + tan A
§§ Solution.
sec A − tan A (sec A − tan A)2
= = (sec A − tan A)2
sec A + tan A sec2 A − tan2 A
= sec2 A − 2 sec A tan A + tan2 A
= 1 + tan2 A − 2 sec A tan A + tan2 A
= 1 − 2 sec A tan A + 2 tan2 A. ■

tan A cot A
§ Problem 2.1.14. + = sec A cosec A + 1. ♢
1 − cot A 1 − tan A
§§ Solution.
sin A cos A
tan A cot A cos A + sin A
+ =
1 − cot A 1 − tan A cos A sin A
1− 1−
sin A cos A
sin2 A cos2 A
= −
cos A (sin A − cos A) sin A (sin A − cos A)
sin3 A − cos3 A
=
cos A sin A (sin A − cos A)
sin2 A + cos2 A + sin A cos A
=
cos A sin A
1 + sin A cos A
=
cos A sin A
1 1
= · + 1 = sec A cosec A + 1. ■
cos A sin A
2.1. Trigonometrical Ratios 24

cos A sin A
§ Problem 2.1.15. + = sin A + cos A. ♢
1 − tan A 1 − cot A
§§ Solution.
cos A sin A cos A sin A
+ = +
1 − tan A 1 − cot A sin A cos A
1− 1−
cos A sin A
cos2 A sin2 A
= +
cos A − sin A sin A − cos A
cos2 A − sin2 A
= = cos A + sin A. ■
cos A − sin A
§ Problem 2.1.16. (sin A + cos A) (cot A + tan A) = sec A+cosec A. ♢
§§ Solution. ( )
cos A sin A
(sin A + cos A) (cot A + tan A) = (sin A + cos A) +
Å sin 2A cos A ã
cos A + sin2 A
= (sin A + cos A)
sin A cos A
( )
1
= (sin A + cos A)
sin A cos A
sin A cos A
= +
sin A cos A sin A cos A
1 1
= + = sec A + cosec A. ■
cos A sin A
§ Problem 2.1.17. sec4 A − sec2 A = tan4 A + tan2 A. ♢
§§ Solution. ( )
sec4 A − sec2 A = sec2 A sec2 A − 1
( )
= 1 + tan2 A tan2 A = tan2 A + tan4 A. ■

§ Problem 2.1.18. cot4 A + cot2 A = cosec 4 A − cosec 2 A. ♢


§§ Solution. ( )
cot4 A + cot2 A = cot2 A cot2 A + 1
( )
= cosec 2 A − 1 cosec 2 A
= cosec 4 A − cosec 2 A. ■


§ Problem 2.1.19. cosec 2 A − 1 = cos A cosec A. ♢
§§ Solution.
√ √ cos A
cosec 2 A − 1 = cot2 A =
sin A
1
= cos A × = cos A cosec A. ■
sin A
§ Problem 2.1.20. sec2 A cosec 2 A = tan2 A + cot2 A + 2. ♢
§§ Solution. ( )( )
sec2 A cosec 2 A = 1 + tan2 A 1 + cot2 A
= 1 + tan2 A + cot2 A + tan2 A cot2 A
= 1 + tan2 A + cot2 A + 1 = tan2 A + cot2 A + 2. ■

§ Problem 2.1.21. tan2 A − sin2 A = sin4 A sec2 A. ♢


2.1. Trigonometrical Ratios 25

§§ Solution.
sin2 A
tan2 A − sin2 A = − sin2 A
cos2 A ( )
sin2 A 1 − cos2 A sin2 A sin2 A
= =
cos2 A cos2 A
1
= sin4 A × = sin 4
A sec 2
A. ■
cos2 A
§ Problem 2.1.22. (1 + cot A − cosec A) (1 + tan A + sec A) = 2. ♢
§§ Solution.
(1 + cot A − cosec A) (1 + tan A + sec A)
( )( )
cos A 1 sin A 1
= 1+ − 1+ +
( sin A sin A) ( cos A cos A)
sin A + cos A − 1 sin A + cos A + 1
=
sin A cos A
(sin A + cos A)2 − 1
=
sin A cos A
sin2 A + cos2 A + 2 sin A cos A − 1
=
sin A cos A
2 sin A cos A
= = 2. ■
sin A cos A
1 1 1 1
§ Problem 2.1.23. − = − .
cosec A − cot A sin A sin A cosec A + cot A

§§ Solution.
1 1 1 1
− = −
cosec A − cot A sin A 1 cos A sin A

sin A sin A
sin A 1 sin2 A − 1 + cos A
= − =
1 − cos A sin A sin A (1 − cos A)
− cos2 A + cos A cos A (1 − cos A)
= =
sin A (1 − cos A) sin A (1 − cos A)
cos A cos A (1 + cos A)
= =
sin A sin A (1 + cos A)
cos A + cos2 A cos A + 1 − sin2 A
= =
sin A (1 + cos A) sin A (1 + cos A)
1 sin A 1 1
= − = − .
sin A 1 + cos A sin A cosec A + cot A
Otherwise thus :
1 1
+
cosec A − cot A cosec A + cot A
cosec A + cot A + cosec A − cot A
=
cosec 2 A − cot2 A
2 cosec A [ ]
= = 2 cosec A ∵ cosec 2 A − cot2 A = 1
cosec 2 A − cot2 A
2 1 1
= = +
sin A sin A sin A
1 1 1 1
∴ − = − . ■
cosec A − cot A sin A sin A cosec A + cot A
2.1. Trigonometrical Ratios 26

cot A cos A cot A − cos A


§ Problem 2.1.24. = . ♢
cot A + cos A cot A cos A
§§ Solution.
cot A cos A cot A cos A (cot A − cos A)
= .
cot A + cos A cot2 A − cos2 A
cos A2
Now cot2 A − cos2 A = − cos2 A
sin2 A ( )
cos2 A 1 − sin2 A
=
sin2 A
cos2 A
= × cos2 A = cot2 A cos2 A
sin2 A
cot A cos A cot A cos A (cot A − cos A) cot A − cos A
∴ = = . ■
cot A + cos A cot2 A cos2 A cot A cos A
cot A + tan B
§ Problem 2.1.25. = cot A tan B. ♢
cot B + tan A
§§ Solution.
1
cot A + tan B + tan B
= tan A
cot B + tan A 1
+ tan A
[tan B ] [ ]
1 + tan A tan B 1 + tan A tan B
= ÷
tan A tan B
tan B
= = cot A tan B. ■
tan A

( 2.1.26.
§ Problem )
1 1
+ cos2 α sin2 α
sec2 α − cos2 α cosec 2 α − sin2 α
1 − cos2 α sin2 α
= . ♢
2 + cos2 α sin2 α
(
§§ Solution. )
1 1
+ cos2 α sin2 α
Ñ α − cos α cosec α − sin α é
sec 2 2 2 2

1 1
= + cos2 α sin2 α
1 1
− cos α
2 − sin α
2
Å cos22α 2
sin2ãα
cos α sin α
= + cos2 α sin2 α
1 − cos4 α 1 − sin4 α
cos4 α sin4 α
= 2
+ 2
1 + cos( α ) α 2 (
1 + sin )
cos2 α 1 − sin2 α sin α 1 − cos2 α
= +
1+ 2
( cos α 4 ) 1(+ sin2 α )
cos α 1 − sin α + sin2 α 1 − cos4 α
2
= ( )
(1 + cos2 α) 1 + sin2 α
( )( )
cos2 α + sin2 α 1 − cos2 α sin2 α
= ( )
1 + cos2 α + sin2 α + cos2 α sin2 α
1 − cos2 α sin2 α
= . ■
2 + cos2 α sin2 α
2.1. Trigonometrical Ratios 27

§ Problem 2.1.27. ( )
sin8 A − cos8 A = sin2 A − cos2 A
( )
× 1 − 2 sin2 A cos2 A . ♢
§§ Solution. ( )( )
sin8 A − cos8 A = sin4 A − cos4 A sin4 A + cos4 A
( )( )
= sin2 A + cos2 A sin2 A − cos2 A
î( )2 ó
× sin2 A + cos2 A − 2 sin2 A cos2 A
( )( )
= sin2 A − cos2 A 1 − 2 sin2 A cos2 A . ■

cos A cosec A − sin A sec A


§ Problem 2.1.28. = cosec A − sec A. ♢
cos A + sin A
§§ Solution.
cos A sin A
cos A cosec A − sin A sec A −
= sin A cos A
cos A + sin A cos A + sin A
cos2 A − sin2 A
=
sin A cos A (cos A + sin A)
cos A − sin A
=
sin A cos A
1 1
= − = cosec A − sec A. ■
sin A cos A
tan A + sec A − 1 1 + sin A
§ Problem 2.1.29. = . ♢
tan A − sec A + 1 cos A
§§ Solution.
tan A + sec A − 1 [tan A + (sec A − 1)]2
=
tan A − sec A + 1 tan2 A − (sec A − 1)2
tan2 A + sec2 A − 2 sec A + 1 + 2 tan A (sec A − 1)
=
tan2 A − sec2 A + 2 sec A − 1
2 sec2 A − 2 sec A + 2 tan A (sec A − 1)
=
2 sec A − 2
2 sec A (sec A − 1) + 2 tan A (sec A − 1)
=
2 (sec A − 1)
1 + sin A
= sec A + tan A = .
cos A
Otherwise thus :
( )
tan A + sec A − 1 tan A + sec A − sec2 A − tan2 A
=
tan A − sec A + 1 tan A − sec A + 1
tan A + sec A + tan2 A − sec2 A
=
tan A − sec A + 1
(tan A + sec A) (1 + tan A − sec A)
=
tan A − sec A + 1
1 + sin A
= tan A + sec A = . ■
cos A
§ Problem 2.1.30.
(tan α + cosec β)2 − (cot β − sec α)2
= 2 tan α cot β (cosec α + sec β) . ♢
2.1. Trigonometrical Ratios 28

§§ Solution.
(tan α + cosec β)2 − (cot β − sec α)2
= tan2 α + cosec 2 β + 2 tan α cosec β − cot2 β − sec2 α + 2 cot β sec α
= tan2 α − sec2 α + cosec 2 β − cot2 β + 2 (tan α cosec β + cot β sec α)
( )
sin α 1 cos β 1
= −1 + 1 + 2 · + ·
( cos)α sin β sin β ( cos α )
sin α + cos β 2 sin α cos β sin α + cos β
=2 =
cos α sin β cos α sin β sin α cos β
= 2 tan α cot β (sec β + cosec α) . ■

§ Problem 2.1.31.
2 sec2 α − sec4 α − 2 cosec 2 α + cosec 4 α
= cot4 α − tan4 α. ♢
§§ Solution.
2 sec2 α − sec4 α − 2 cosec 2 α + cosec 4 α
( )
= cosec 4 α − 2 cosec 2 α + 1 − sec4 α − 2 sec2 α + 1
( )2 ( )2
= cosec 2 α − 1 − sec2 α − 1 = cot4 α − tan4 α. ■

§ Problem 2.1.32.
(sin α + cosec α)2 + (cos α + sec α)2
= tan2 α + cot2 α + 7. ♢
§§ Solution.
(sin α + cosec α)2 + (cos α + sec α)2
= sin2 α + 2 sin α cosec α + cosec 2 α + cos2 α + 2 cos α sec α + sec2 α
= sin2 α + cos2 α + 2 + 2 + cosec 2 α + sec2 α
= 5 + 1 + cot2 α + 1 + tan2 α = tan2 α + cot2 α + 7. ■

§ Problem 2.1.33.
(cosec A + cot A) covers A − (sec A + tan A) vers A
= (cosec A − sec A) (2 − vers A covers A) .♢
§§ Solution.
(cosec A + cot A) covers A − (sec A + tan A) vers A
( ) ( )
1 cos A 1 sin A
= + (1 − sin A) − + (1 − cos A)
sin A sin A cos A cos A
(1 + cos A) (1 − sin A) (1 + sin A) (1 − cos A)
= −
sin A cos A
1 − sin A cos A + (cos A − sin A) 1 − sin A cos A − (cos A − sin A)
= −
sin(
A ) cos(A )
1 1 1 1
= (1 − sin A cos A) − + (cos A − sin A) +
sin A cos A sin A cos A
cos A − sin A
= (1 − sin A cos A + cos A + sin A)
(sin A cos A )
1 1
= − [2 − (1 − cos A) (1 − sin A)]
sin A cos A
= (cosec A − sec A) (2 − vers A covers A) . ■
2.2. Relations and Trigonometrical Ratios 29

§ Problem 2.1.34.
(1 + cot A + tan A) (sin A − cos A)
sec A cosec A
= − . ♢
cosec 2 A sec2 A
§§ Solution.
(1 + cot A + tan A) (sin A − cos A)
( )
cos A sin A
= 1+ + (sin A − cos A)
( 2 sin A cos A )
sin A + sin A cos A + cos2 A (sin A − cos A)
=
sin A cos A
sin3 A − cos3 A sin2 A cos2 A
= = −
sin A cos A cos A sin A
1 1 1 1
= · − ·
cos A cosec 2 A sin A sec2 A
sec A cosec A
= − . ■
cosec 2 A sec2 A
§ Problem 2.1.35. 2 versin A + cos2 A = 1 + versin2 A. ♢
§§ Solution.
2 versin A + cos2 A = 2 (1 − cos A) + cos2 A
= 1 + 1 − 2 cos A + cos2 A
= 1 + (1 − cos A)2 = 1 + versin2 A. ■

2.2 Relations and Trigonometrical Ratios


§ Problem 2.2.1. Express all the other trigonometrical ratios in
terms of the cosine. ♢
§§ Solution. Taking the figure in Art. 31, let the length OP be unity
and let the corresponding
√ length √ of OM be c.
Then M P = OP 2 − OM 2 = 1 − c2 .
OM c
∴ cos θ = = =c
OP 1

MP 1 − c2 √ √
sin θ = = = 1 − c2 = 1 − cos2 θ
OP √ 1 √
MP 1 − c2 1 − cos2 θ
tan θ = = =
OM c cos θ
OM c cos θ
cot θ = = √ = √
MP 1 − c2 1 − cos2 θ
OP 1 1
cosec θ = = √ = √
MP 1 − c2 1 − cos2 θ
OP 1 1
and sec θ = = = . ■
OM c cos θ
§ Problem 2.2.2. Express all the ratios in terms of the tangent. ♢
§§ Solution. Here let the length OM be unity and let the corre-
sponding length of M P be t.
√ √
Then OP = OM 2 + M P 2 = 1 + t2 .
MP t
∴ tan θ = = =t
OM 1
2.2. Relations and Trigonometrical Ratios 30

MP t tan θ
sin θ = = √ = √
OP 1 + t2 1 + tan2 θ
OM 1 1
cos θ = = √ = √
OP 1 + t2 1 + tan2 θ
OM 1 1
cot θ = = =
MP t tan θ √

OP 1 + t2 1 + tan2 θ
cosec θ = = =
MP √ t tan θ
OP 1 + t2 √
and sec θ = = = 1 + tan2 θ. ■
OM 1
§ Problem 2.2.3. Express all the ratios in terms of the cosecant. ♢
§§ Solution. Here let the length M P be unity and let the corre-
sponding length of OP be x.
√ √
Then OM = OP 2 − M P 2 = x2 − 1.
OP x
∴ cosec θ = = =x
MP 1
MP 1 1
sin θ = = =
OP x
√ cosec θ√
OM x2 − 1 cosec 2 θ − 1
cos θ = = =
OP x cosec θ
MP 1 1
tan θ = = √ = √
OM x 2−1 cosec 2θ−1
√ √
OM x2 − 1
cot θ = = = cosec 2 θ − 1
OP 1
OP x cosec θ
and sec θ = = √ = √ . ■
OM x −1
2 cosec 2 θ − 1
§ Problem 2.2.4. Express all the ratios in terms of the secant. ♢
§§ Solution. Here let the length OM be unity and let the corre-
sponding length of OP be x.
√ √
Then MP = OP 2 − OM 2 = x2 − 1.
OP x
∴ sec θ = = =x
OM 1
√ √
MP x2 − 1 sec2 θ − 1
sin θ = = =
OP x sec θ
OM 1 1
cos θ = = =
OP x
√ sec θ √
MP x2 − 1
tan θ = = = sec2 θ − 1
OM 1
OM 1 1
cot θ = = √ = √
MP x −1
2 sec θ − 1
2
OP x sec θ
and cosec θ = = √ = √ . ■
MP x2 − 1 sec2 θ − 1
1
§ Problem 2.2.5. The sine of a certain angle is ; find the numerical
4
values of the other trigonometrical ratios of this angle. ♢
2.2. Relations and Trigonometrical Ratios 31

§§ Solution. With the same figure, take P so that the length of OP


is 4.
MP 1
Then, since sin θ = = , we have M P = 1, and

OP 4
√ √
OM = OP 2 − M P 2 = 16 − 1 = 15.

OM 15 MP 1
∴ cos θ = = , tan θ = = √
OP 4 OM 15

OM 15 √ OP 4
cot θ = = = 15, sec θ = = √
MP 1 OM 15
OP 4
and cosec θ = = = 4.
MP 1
Otherwise thus :
1
Proceed as in Ex. 4, Art. 31, with sin θ = . ■
4

12
§ Problem 2.2.6. If sin θ = , find tan θ and versin θ. ♢
13
MP 12
§§ Solution. Here we have sin θ = = .
OP 13
∴ OP = 13, M P = 12;
√ √ √
∴ OM = OP 2 − M P 2 = 132 − 122 = 25 = 5.
MP 12
∴ tan θ = = .
OM 5
OM 5 8
∴ versin θ = 1 − cos θ = 1 − =1− = .
OP 13 13
Otherwise thus : … ( )
sin θ 12 12 2 12 12
tan θ = √ = ÷ 1− = √ = .
1 − sin θ
2 13 13 25 5

∴ versin θ = 1 − cos θ = 1 − 1 − sin2 θ
… ( )2
12 5 8
=1− 1− =1− = . ■
13 13 13
11
§ Problem 2.2.7. If sin A = , find tan A, cos A and sec A. ♢
61
§§ Solution. Here take OP = 61 and M P = 11 and let the angle θ
be denoted by A.
√ √
∴ OM = OP 2 − M P 2 = 612 − 112
√ √ √
= (61 + 11)(61 − 11) = 72 × 50 = 36 × 100 = 60.
MP 11
∴ tan A = =
OM 60
OM 60 OP 61
∴ cos A = = and sec A = = .
OP 61 OM 60
Otherwise thus :
… ( )2
sin A 11 11 11 60 11
tan A = √ = ÷ 1− = ÷ =
1− sin2 A 61 61 61 61 60
2.2. Relations and Trigonometrical Ratios 32

… ( )2
√ 11 60
cos A = 1− sin2 A= 1− =
61 61
1 1 61
and sec A = √ = … ( )2 = .
1 − sin2 A 11 60
1− ■
61
4
§ Problem 2.2.8. If cos θ =, find sin θ and cot θ. ♢
5
§§ Solution. Here take OM = 4 and proceed as in Ex. 3, Art. 31.
Otherwise thus :
… …
√ 16 9 3
sin θ = 1− cos2
θ= 1− = =
25 25 5
cos θ 4 3 4
and cot θ = √ = ÷ = . ■
1 − cos2 θ 5 5 3
9
§ Problem 2.2.9. If cos A = , find tan A and cosec A. ♢
41
§§ Solution.
Here we have OM = 9, OP = 41, and
√ √ √ √
MP = OP − OM =
2 2 412 − 92 = 50 × 32 = 100 × 16 = 40.
MP 40
∴ tan A = ==
OM 9
OP 41
and cosec A = = ,
MP 40
the angle ∠P OM being denoted by A. ■

3
§ Problem 2.2.10. If tan θ = , find the sine, cosine, versine and
4
cosecant of θ. ♢
§§ Solution. Here √ take OM = 4 and √
M P = 3. Then
OP = OM 2 + M P 2 = 16 + 9 = 5.
MP 3 OM 4
∴ sin θ = = , cos θ = =
OP 5 OP 5
4 1 OP 5
versin θ = 1 − cos θ = 1 − = and cosec θ = = .
5 5 MP 3
Otherwise thus :

tan θ 3 9 3 5 3
sin θ = √ = ÷ 1+ = ÷ =
1 + tan2 θ 4 16 4 4 5
1 5 4 1
cos θ = √ = 1 ÷ = , versin θ = 1 − cos θ =
1 + tan2 θ 4 5 5

1 + tan2 θ
5 3 5
and cosec θ = ÷ = .
= ■
tan θ 4 4 3
1 cosec 2 θ − sec2 θ
§ Problem 2.2.11. If tan θ = √ , find the value of .
7 cosec 2 θ + sec2 θ

2.2. Relations and Trigonometrical Ratios 33

§§ Solution.
1
We have cosec 2 θ = 1 + cot2 θ = 1 + =1+7=8
tan2 θ
2 2 1 8
and sec θ = 1 + tan θ = 1 + =
7 7
8
cosec θ − sec θ
2 2 8 −
∴ = 7 = 48 = 3 .
cosec 2 θ + sec2 θ 8 64 4
8+
7
Otherwise thus :
( )
cosec 2 θ − sec2 θ cosec 2 θ − sec2 θ sin2 θ
=
2
cosec θ + sec θ2 (cosec 2 θ + sec2 θ) sin2 θ
1
1 − tan2 θ 1−
= = 7 = 6 = 3.
1 + tan2 θ 1 8 4
1+ ■
7
15
§ Problem 2.2.12. If cot θ = , find cos θ and cosec θ. ♢
8
§§ Solution. Here √ take OM = 15 √ M P = 8. Then
and

OP = OM 2 + M P 2 = 152 + 82 = 289 = 17.
OM 15
∴ cos θ = =
OP 17
OP 17
and cosec θ = = .
MP 8
Otherwise thus :
… ( )2
cot θ 15 15 15 17 15
cos θ = √ = ÷ 1+ = ÷ =
1 + cot2 θ 8 8 8 8 17
… ( )2 …
√ 15 289 17
and cosec θ = 1 + cot2 θ = 1+ = = . ■
8 64 8
3
§ Problem 2.2.13. If sec A = , find tan A and cosec A. ♢
2
§§ Solution.
Here take OM = 2 and OP = 3 [cf. Ex. 3, Art. 31] .
√ √ √
Then MP = OP 2 − OM 2 = 32 − 22 = 5.

MP 5
∴ tan A = =
OM 2
OP 3 3√
and cosec A = = √ = 5,
MP 5 5
the angle ∠P OM being denoted by A.
Otherwise thus :
… … √
√ 9 5 5
tan A = A−1=
sec2 −1= =
4 4 2

sec A 3 5 3 3√
and cosec A = √ = ÷ = √ = 5. ■
sec2 A − 1 2 2 5 5
2.2. Relations and Trigonometrical Ratios 34

§ Problem 2.2.14. If 2 sin θ = 2 − cos θ, find sin θ. ♢


§§ Solution.
2 sin θ = 2 − cos θ
∴ cos θ = 2 (1 − sin θ) ; ∴ cos2 θ = 4 (1 − sin θ)2
∴ 1 − sin2 θ = 4 (1 − sin θ)2
∴ (1 − sin θ) (1 + sin θ) = 4 (1 − sin θ)2
∴ 1 − sin θ = 0 or 1 + sin θ = 4 (1 − sin θ) .
If 1 − sin θ = 0, then sin θ = 1.
If 1 + sin θ = 4 (1 − sin θ) , then 1 + sin θ = 4 − 4 sin θ
3
∴ 5 sin θ = 3, ∴ sin θ = . ■
5
§ Problem 2.2.15. If 8 sin θ = 4 + cos θ, find sin θ. ♢
§§ Solution.
8 sin θ = 4 + cos θ

∴ 4 (2 sin θ − 1) = cos θ = 1 − sin2 θ
( )
∴ 16 4 sin2 θ − 4 sin θ + 1 = 1 − sin θ 2

∴ 65 sin2 θ − 64 sin θ + 15 = 0
∴ (13 sin θ − 5) (5 sin θ − 3) = 0
∴ 13 sin θ − 5 = 0 or 5 sin θ − 3 = 0.
5
If 13 sin θ − 5 = 0 then sin θ = .
13
3
If 5 sin θ − 3 = 0 then sin θ = . ■
5
§ Problem 2.2.16. If tan θ + sec θ = 1.5, find sin θ. ♢
§§ Solution.
tan θ + sec θ = 1.5
sin θ 1 3 (1 + sin θ)2 9
∴ + = ; ∴ = ;
cos θ cos θ 2 cos2 θ 4
(1 + sin θ)2 9 1 + sin θ 9 9−4 5
∴ = ; ∴ = ; ∴ sin θ = = . ■
1 − sin2 θ 4 1 − sin θ 4 9+4 13
§ Problem 2.2.17. If cot θ + cosec θ = 5, find cos θ. ♢
§§ Solution.
cot θ + cosec θ = 5
cos θ 1 (1 + cos θ)2 (1 + cos θ)2
∴ + = 5; ∴ = 25; ∴ = 25
sin θ sin θ 2
sin θ 1 − cos2 θ
1 + cos θ 25 − 1 24 12
∴ = 25; ∴ cos θ = = = . ■
1 − cos θ 25 + 1 26 13
§ Problem 2.2.18. If 3 sec4 θ + 8 = 10 sec2 θ, find the values of tan θ.

§§ Solution.
3 sec4 θ + 8 = 10 sec2 θ
( )2 ( )
∴ 3 1 + tan2 θ + 8 = 10 1 + tan2 θ
∴ 3 tan4 θ − 4 tan2 θ + 1 = 0
( )( )
∴ 3 tan2 θ − 1 tan2 θ − 1 = 0
2.2. Relations and Trigonometrical Ratios 35

∴ 3 tan2 θ − 1 = 0 or tan2 θ − 1 = 0.
1 1
If 3 tan2 θ − 1 = 0, then tan2 θ = , ∴ tan θ = √ .
3 3
If tan2 θ − 1 = 0, then tan2 θ = 1, ∴ tan θ = 1.
Note : Negative values of the ratios have not yet been discussed;
hence, in this set of solutions, only positive ones are given.
Otherwise thus :

We have 3 sec4 θ − 10 sec2 θ + 8 = 0


( )( )
∴ 3 sec2 θ − 4 sec2 θ − 2 = 0
∴ 3 sec2 θ − 4 = 0 or sec2 θ − 2 = 0.
4
If 3 sec2 θ − 4 = 0, then sec2 θ = ,
…3
√ 4 1
and tan θ = sec2 θ − 1 = −1= √ .
3 3

If sec2 θ − 2 = 0, then sec2 θ = 2, and tan θ = 2 − 1 = 1. ■

§ Problem 2.2.19. If tan2 θ + sec θ = 5, find cos θ. ♢


§§ Solution.
tan2 θ + sec θ = 5
sin2 θ 1
∴ + =5
cos2 θ cos θ
∴ 1 − cos θ + cos θ = 5 cos2 θ
2

∴ 6 cos2 θ − cos θ − 1 = 0
∴ (2 cos θ − 1) (3 cos θ + 1) = 0
∴ 2 cos θ − 1 = 0 or 3 cos θ + 1 = 0.
1
If 2 cos θ − 1 = 0, then cos θ = .
2
1
If 3 cos θ + 1 = 0, then cos θ = − . [Refer note to the last solution] .
3
Otherwise thus :

sec2 θ − 1 + sec θ = 5
∴ sec θ + sec θ − 6 = 0; ∴ (sec θ − 2) (sec θ + 3) = 0
2

∴ sec θ − 2 = 0 or sec θ + 3 = 0.
1 1
If sec θ − 2 = 0, then sec θ = 2 and ∴ cos θ = = .
sec θ 2
1
If [sec θ + 3 = 0, then sec θ = −3 and ∴ cos θ = − .] ■
3
§ Problem 2.2.20. If tan θ + cot θ = 2, find sin θ. ♢
§§ Solution.
sin θ cos θ
tan θ + cot θ = 2; ∴ + =2
cos θ sin θ
∴ sin θ + cos θ = 2 sin θ cos θ
2 2

∴ 1 = 2 sin θ cos θ = 2 sin θ 1 − sin2 θ
∴ 1 = 4 sin θ − 4 sin θ; ∴ 4 sin θ − 4 sin2 θ + 1 = 0
2 4 4
2.3. Values of Trigonometrical Ratios 36

( )2 1
∴ 2 sin2 θ − 1 = 0; ∴ 2 sin2 θ − 1 = 0, ∴ sin θ = √ . ■
2
§ Problem 2.2.21. If sec2 θ = 2 + 2 tan θ, find tan θ. ♢
§§ Solution.
sec2 θ = 2 + 2 tan θ, ∴ 1 + tan2 θ = 2 + 2 tan θ
∴ tan2 θ − 2 tan θ + 1 = 2; ∴ (tan θ − 1)2 = 2
√ √
∴ tan θ − 1 = 2; ∴ tan θ = 1 + 2. ■

2x(x + 1)
§ Problem 2.2.22. If tan θ = , find sin θ and cos θ. ♢
2x + 1
§§ Solution. We have …
tan θ 2x(x + 1) 4x2 (x + 1)2
sin θ = √ = ÷ 1+
2
1 + tan θ 2x + 1 4x2 + 4x + 1

2x(x + 1) 4x4 + 8x3 + 8x2 + 4x + 1
= ÷
2x + 1 4x2 + 4x + 1
2x(x + 1) 2x2 + 2x + 1 2x(x + 1)
= ÷ =
2x + 1 2x + 1 2x2 + 2x + 1
2x + 1
and cos θ = sin θ ÷ tan θ = .
2x2 + 2x + 1
Otherwise thus :
With the figure of Art. 31, take OM = 2x + 1 and M P = 2x(x + 1).
Then √ √
OP = OM 2 + M P 2 = (2x + 1)2 + 4x2 (x + 1)2

= 4x4 + 8x3 + 8x2 + 4x + 1 = 2x2 + 2x + 1.
MP 2x(x + 1)
∴ sin θ = =
OP 2x2 + 2x + 1
OM 2x + 1
and cos θ = = . ■
OP 2x2 + 2x + 1

2.3 Values of Trigonometrical Ratios


§ Problem 2.3.1. If A = 30◦ , verify that

(1) cos 2A = cos2 A − sin2 A = 2 cos2 A − 1

(2) sin 2A = 2 sin A cos A

(3) cos 3A = 4 cos3 A − 3 cos A

(4) sin 3A = 3 sin A − 4 sin3 A, and


2 tan A
(5) tan 2A = .
1 − tan2 A ♢
1
§§ Solution. (1) cos 2A = cos 60◦ =
2 Å √ ã2 ( )2
3 1
cos2 A − sin2 A = cos2 30◦ − sin2 30◦ = −
2 2
3−1 1
= =
4 2
2.3. Values of Trigonometrical Ratios 37

Å √ ã2
◦ 3
and 2 cos A − 1 = 2 cos 30 − 1 = 2
2 2
−1
2
3 1
= −1= .
2 2

3
(2) sin 2A = sin 60◦ =
2 √ √
1 3 3
2 sin A cos A = 2 sin 30◦ cos 30◦ = 2 × × = .
2 2 2
(3) cos 3A = cos 90◦ = 0, and ( )
4 cos3 A − 3 cos A = cos A 4 cos2 A − 3
√ ñ Å √ ã2 ô

( ◦ 3) 3
= cos 30 4 cos 30 − 3 =
2
4 −3
2 2

3
= (3 − 3) = 0.
2
(4) sin 3A = sin 90◦ = 1, and ( )
3 sin A − 4 sin3 A = sin A 3 − 4 sin2 A
( )
= sin 30◦ 3 − 4 sin2 30◦
ï ( )2 ò
1 1 1
= 3−4 = (3 − 1) = 1.
2 2 2

(5) tan 2A = tan 60◦ = 3, and
2 tan A 2 tan 30◦
=
1 − tan2 A 1 − tan2 30◦
1
2× √ √
3 2 3 √
= ( )2 =
2
= 3.
1
1− √ ■
3
§ Problem 2.3.2. If A = 45◦ , verify that

(1) sin 2A = 2 sin A cos A

(2) cos 2A = 1 − 2 sin2 A, and


2 tan A
(3) tan 2A =
1 − tan2 A ♢
§§ Solution. (1) sin 2A = sin 90◦ = 1, and
1 1
2 sin A cos A = 2 sin 45◦ cos 45◦ = 2 × √ × √ = 1.
2 2
(2) cos 2A = cos 90◦ = 0, and
( )2
1
1 − 2 sin2 A = 1 − 2 sin2 45◦ = 1 − 2 × √ = 1 − 1 = 0.
2
(3) tan 2A = tan 90◦ = ∞, and
2 tan A 2 tan 45◦ 2×1 2
= = = = ∞. ■
1 − tan2 A 1 − tan2 45◦ 1−1 0

Verify that
2.3. Values of Trigonometrical Ratios 38

3
§ Problem 2.3.3. sin2 30◦ + sin2 45◦ + sin2 60◦ = . ♢
2
§§ Solution.
sin2 30◦ + sin2 45◦ + sin2 60◦
( )2 ( )2 Å √ ã2
1 1 3
= + √ +
2 2 2
1 1 3 3
= + + = . ■
4 2 4 2
1
§ Problem 2.3.4. tan2 30◦ + tan2 45◦ + tan2 60◦ = 4 . ♢
3
§§ Solution.
tan2 30◦ + tan2 45◦ + tan2 60◦
( )2 (√ )2 1
1 1
= √ +1+ 3 = +1+3=4 . ■
3 3 3
§ Problem 2.3.5. sin 30◦ cos 60◦ + cos 30◦ sin 60◦ = 1. ♢
§§ Solution.
sin 30◦ cos 60◦ + cos 30◦ sin 60◦
√ √
1 1 3 3 1 3 4
= × + × = + = = 1. ■
2 2 2 2 4 4 4

3 −1
§ Problem 2.3.6. cos 45◦ cos 60◦ − sin 45◦ sin 60◦ = − √ . ♢
2 2
§§ Solution.
cos 45◦ cos 60◦ − sin 45◦ sin 60◦
√ √
1 1 1 3 3−1
= √ × −√ × =− √ . ■
2 2 2 2 2 2
4 3
§ Problem 2.3.7. cot 30 + 3 sin 60 − 2 cosec 60◦ − tan2 30◦
2 ◦ 2 ◦ 2 =
3 4
1
3 . ♢
3
§§ Solution.
4 3
cot2 30◦ + 3 sin2 60◦ − 2 cosec 2 60◦ − tan2 30◦
3 Å √ ã2 4
( )2 ( )2
4 (√ )2 3 2 3 1
= 3 +3 −2 √ − √
3 2 3 4 3
9 8 1 2 1
=4+ − − =6−2 =3 . ■
4 3 4 3 3
1
§ Problem 2.3.8. cosec 2 45◦ · sec2 30◦ · sin3 90◦ · cos 60◦ = 1 . ♢
3
§§ Solution.
cosec 2 45◦ · sec2 30◦ · sin3 90◦ · cos 60◦
( √ )2 ( 2 ) 2 1
= 2 × √ × 13 ×
3 2
4 1 4 1
=2× ×1× = =1 . ■
3 2 3 3
1
§ Problem 2.3.9. 4 cot2 45◦ − sec2 60◦ + sin3 30◦ = . ♢
3
2.3. Values of Trigonometrical Ratios 39

§§ Solution.
( )3
1
4 cot2 45◦ − sec2 60◦ + sin3 30◦ = 4 × 12 − 22 +
2
1 1
=4−4+ = . ■
8 8
Chapter
3
Simple Problems in Heights and
Distances

3.1 Simple Problems


§ Problem 3.1.1. A person, standing on the bank of a river, observes
that the angle subtended by a tree on the opposite bank is 60◦ ; when
he retires 40 feet from the bank he finds the angle to be 30◦ ; find the
height of the tree and the breadth of the river. ♢
§§ Solution. Take the figure of Ex. 2, Art. 45. Let P M (x f eet, say)
be the tree, and B and A be the two positions of the observer respec-
tively, so that BM represents the breadth of the river. We are given
BA = 40 f eet, the ∠M BP = 60◦ and the ∠M AP = 30◦ . We then have
AM √ BM 1
= cot 30◦ = 3, and = cot 60◦ = √ .
x x 3
√ x
∴ AM = x 3, and BM = √ .
(√ ) (3 )
1 3−1 2x
∴ 40 = AM − BM = x 3− √ =x √ = √ .
√ 3 3 3
∴ x = 20 3 = 20 × 1.73205 = 34.64 f eet.

x 20 3
∴ BM = √ = √ = 20 f t. ■
3 3
§ Problem 3.1.2. At a certain point the angle of elevation of a tower
3
is found to be such that its cotangent is ; on walking 32 feet directly
5
toward the tower its angle of elevation is an angle whose cotangent
2
is . Find the height of the tower. ♢
5
3.1. Simple Problems 42

§§ Solution. Taking the same figure, let P M (x f eet, say) be the


tower and A and B be the two points at which the angles of elevation
are taken respectively.
3 2
We are given AB = 32 f eet, cot ∠M AP = and cot ∠M BP = .
5 5
AM 3 BM 2
We then have = , and = .
x 5 x 5
3 2
∴ AM = x and BM = x.
5 5
x
∴ 32 = AM − BM =
5
∴ x = 32 × 5 f eet = 160 f eet. ■

§ Problem 3.1.3. At a point A, the angle of elevation of a tower is


5
found to be such that its tangent is ; on walking 240 feet nearer
12
3
the tower the tangent of the angle of elevation is found to be ; what
4
is the height of the tower? ♢
§§ Solution. Taking the same figure, let P M (x f eet, say) be the
tower and A and B be the two points at which the angles of elevation
are taken respectively.
5 3
We are given AB = 240 f eet, tan ∠M AP = and tan ∠M BP = .
12 4
x 5 x 3
We then have = , and = .
AM 12 BM 4
12 4
∴ AM = x and BM = x.
5 ( )3
12 4 16
∴ 240 = AM − BM = − x= x.
5 3 15
∴ x = 15 × 15 f eet = 225 f eet. ■

§ Problem 3.1.4. Find the height of a chimney when it is found that,


on walking towards it 100 feet in a horizontal line through its base,
the angular elevation of its top changes from 30◦ to 45◦ . ♢
§§ Solution. Taking the same figure, with P M (x f eet, say) repre-
senting the chimney and A and B being the two points at which
the angles of elevation are taken respectively, we are given AB =
100 f eet, the ∠M AP = 30◦ , and the ∠M BP = 45◦ . We then have
AM √ BM
= cot 30◦ = 3, and = cot 45◦ = 1.
x √ x
∴ AM = x 3, and BM = x;
(√ )
∴ 100 = AM − BM = 3 − 1 x.
100 (√ )
∴x= √ = 50 3 + 1) = 50 × 2.73205 = 136.6 f eet. ■
3−1
§ Problem 3.1.5. An observer on the top of a cliff, 200 feet above the
sea-level, observes the angles of depression of two ships at anchor
to be 45◦ and 30◦ respectively ; find the distances between the ships
if the line joining them points to the base of the cliff. ♢
3.1. Simple Problems 43

§§ Solution. Take the same figure as before. Draw P N parallel to


M A, so that P N is the horizontal line passing through P . Let P be the
position of the observer on the cliff P M and B and A be the positions
of the ships respectively.
We are given P M = 200 f eet, the ∠N P B = 45◦ = ∠P BM (Euc. I. 29)
and the ∠N P A = 30◦ = ∠P AM .
We then have
AM √ BM
= cot 30◦ = 3, and = cot 45◦ = 1.
200 √ 200
∴ AM = 200 3, and BM = 200.
(√ )
∴ AB = AM − BM = 200 3 − 1 = 200 × .73205 = 146.4 f eet. ■

§ Problem 3.1.6. From the top of a cliff an observer finds that the
angles of depression of two buoys in the sea are 39◦ and 26◦ respec-
tively; the buoys are 300 yards apart and the line joining them points
straight at the foot of the cliff ; find the height of the cliff and the
distance of the nearest buoy from the foot of the cliff, given that
cot 26◦ = 2.0503, and cot 39◦ = 1.2349. ♢
§§ Solution. Construct a figure as in the last example, with B and
A as the positions of the two buoys.
We are given BA = 300 yards, the ∠N P B = 39◦ = ∠P BM , and the
∠N P A = 26◦ = ∠P AM .
Let x yards be the height of the cliff P M . We then have BM =
x cot 39◦ = x × 1.2349, and
x × 2.0503 = x cot 26◦ = AM = 300 + BM
= 300 + x × 1.2349.
∴ 300 = x(2.0503 − 1.2349) = .8154x.
300
∴x= = 367.9 yards.
.8154
Also, the required distance
= BM = x cot 39◦
= (367.9 × 1.2349) yards
= 454.3 yards. ■

§ Problem 3.1.7. The upper part of a tree broken over by the wind
makes an angle of 30◦ with the ground, and the distance from the
root to the point where the top of the tree touches the ground is
60 f eet ; what was the height of the tree? ♢
§§ Solution. Let y feet be the length of the broken part of the tree
and x feet be the height of the stump. We then have
50 100
x = 50 tan 30◦ = √ , and y = x cosec 30◦ = 2x = √ .
3 3
Hence the original height of the tree
=x+y
150 √
= √ = 50 3
3
= 50 × 1.73205 = 86.6 f eet. ■
3.1. Simple Problems 44

§ Problem 3.1.8. The horizontal distance between two towers is 60


feet and the angular depression of the top of the first as seen from
the top of the second, which is 150 feet high, is 30◦ ; find the height
of the first. ♢
§§ Solution. Take the figure of Ex. 3, Art. 45. Let AB be the second
tower, 150 feet high and CD be the first tower, x feet high.
We are given the ∠EAC = 30◦ , CE = 150 − x and DB = 60 f eet =
EA.
We then have
60 √
150 − x = 60 tan 30◦ = √ = 20 3 = 34.641
3
∴ x = 150 − 34.641 = 115.359 . . . f t. ■

§ Problem 3.1.9. The angle of elevation of the top of an unfinished


tower at a point distant 120 feet from its base is 45◦ ; how much
higher must the tower be raised so that its angle of elevation at the
same point may be 60◦ ? ♢
§§ Solution. Let h feet be the present height of the tower and x feet
be the required height. We then have
h h+x √
= tan 45◦ = 1, and = tan 60◦ = 3.
120 (√ ) 120
∴x= 3 − 1 120 = .73205 × 120 = 87.846 f eet. ■

§ Problem 3.1.10. Two pillars of equal height stand on either side of


a roadway which is 100 feet wide ; at a point in the roadway between
the pillars the elevations of the tops of the pillars are 60◦ and 30◦ ;
find their height and the position of the point. ♢
§§ Solution. Let h feet be the height of the pillars and x feet and y
feet be the distances of the point respectively. We then have

h h 3 √
x = h cot 60◦ = √ = , and y = h cot 30◦ = h 3
3 √ 3
h 3 √
∴ + h 3 = 100
3 √
∴ 4h 3 = 300

100 3 173.205
∴h= = = 43.3 f eet.
4 √ 4 √
100 3 3
Also, x = × = 25 f eet
4√ 3
100 3 √
and y = × 3 = 75 f eet. ■
4
§ Problem 3.1.11. The angle of elevation of the top of a tower is
observed to be 60◦ ; at a point 40 feet above the first point of obser-
vation the elevation is found to be 45◦ ; find the height of the tower
and its horizontal distance from the points of observation. ♢
§§ Solution. Let h feet be the height of the tower and x feet be the
required horizontal distance.
√ We then have
h = x tan 60◦ = x 3, and h − 40 = x tan 45◦ = x.
Hence, by subtraction, we have
(√ )
40 = x 3−1 .
3.1. Simple Problems 45

40 (√ )
∴x= √ = 20 3 + 1 = 20 × 2.73205 = 54.641 f eet.
3−1
∴ h = x + 40 = 94.641 f eet. ■

§ Problem 3.1.12. At the foot of a mountain the elevation of its sum-


mit is found to be 45◦ ; after ascending one mile towards the mountain
up a slope of 30◦ inclination the elevation is found to be 60◦ . Find
the height of the mountain . ♢
§§ Solution. Let A be the foot of the mountain and C be its summit,
AB be the slope and AD be the horizontal line through A and D being
vertically below C.
Join BC. We are given the ∠CAD = 45◦ , AB = 1 mile, ∠BAD =
30◦ and the ∠CBF = 60◦ , where BF is drawn parallel to AD, meeting
CD in F .
We then have
∠CAB = 45◦ − 30◦ = 15◦ , and ∠ACD = 45◦
∴ ∠ACB = 45◦ − ∠BCF = 45◦ − 30◦ = 15◦ = ∠CAB
∴ BC = AB = 1 mile.
Draw BE perpendicular to AD.

3
∴ CF = BC sin 60◦ = , and
2
◦ 1
F D = BE = AB sin 30 = .
√ 2
3+1 2.73205
∴ CD = CF + F D = = = 1.366 . . . mile. ■
2 2
§ Problem 3.1.13. What is the angle √ of elevation of the sun when
the length of the shadow of a pole is 3 times the height of the pole
? ♢
§§ Solution. In the figure, Art. 23, let P M represent the pole and
OM its shadow.
PM 1
∴ tan ∠P OM = = √
OM 3
∴ The angle of elevation = 30◦ . ■

§ Problem 3.1.14. The shadow of a tower standing on a level plane


is found to be 60 feet longer when the sun’s altitude is√30◦ than when
it is 45◦ . Prove that the height of the tower is 30(1 + 3) f eet . ♢
§§ Solution. Take the figure of Ex. 2, Art. 45. Let P M (x f eet, say)
be the tower.
We are given the ∠P BM = 45◦ , BA = 60 f eet and the ∠P AM =

30 .
We then have
BM AM √
= cot 45◦ = 1, and = cot 30◦ = 3.
x x √
∴ BM = x, and AM = AB + BM = 60 + x = x 3.
(√ )
∴ 3 − 1 x = 60
60 ( √ )
x= √ = 30 1 + 3 f eet. ■
3−1
3.1. Simple Problems 46

§ Problem 3.1.15. On a straight coast there are three objects A, B,


and C such that AB = BC = 2 miles. A vessel approaches B in a
line perpendicular to the coast and at a certain point AC is found to
subtend an angle of 60◦ ; after sailing in the same direction for ten
minutes AC is found to subtend 120◦ ; find the rate at which the ship
is going. ♢
§§ Solution. Let P and Q be the points at which AC subtends angles
of 60◦ and 120◦ respectively.
We are given the ∠AP C = 60◦ , ∠AQC = 120◦ and AB = BC =
2 miles.
We then have √
BP = AB cot 30◦ = 2 3 miles, and
2
BQ = AB cot 60◦ = √ miles.
3
Hence in 10 minutes the ship sails a distance
( √ )
2
= P Q = BP − BQ = 2 3 − √ miles
3
4
= √ miles.
(3 )
4
∴ the required rate = 6 × √ miles per hour
√ 3
= 8 3 = 8 × 1.73205
= 13.8564 miles per hour. ■

§ Problem 3.1.16. Two flagstaffs stand on a horizontal plane. A and


B are two points on the line joining the bases of the flagstaffs and
between them. The angles of elevation of the tops of the flagstaffs
as seen from A are 30◦ and 60◦ and, as seen from B, they are 60◦ and
45◦ . If the length AB be 30 feet, find the heights of the flagstaffs and
the distance between them. ♢
§§ Solution. Let EC(h f eet, say) be the flagstaff near A, C being its
base and F D(h′ f eet, say) be the flagstaff near B, D being its base.
Let the
∠F AD = 30◦ , ∠EAC = 60◦ , ∠F BD = 60◦ and ∠EBC = 45◦ .
We then have
h
CA = h cot 60◦ = √ , CB = h cot 45◦ = h,
3
√ h′
DA = h′ cot 30◦ = h′ 3, and DB = h′ cot 60◦ = √ .
3
h
∵ CB = CA + AB, ∴ h = √ + 30
3

30 3 √ (√ ) ( √ )
∴h= √ = 15 3 3 + 1 = 15 3 + 3
3−1
= 15 × 4.73205 = 70.98 . . . f eet.
√ h′
Also, since AD = AB + BD, we have h′ 3 = 30 + √
√ √ 3
∴ 2h′ = 30 3; ∴ h′ = 15 3 = 25.98 . . . f eet.
h h′
∴ CD = CA + AB + BD = √ + 30 + √
3 3
3.1. Simple Problems 47

(√ )
= 15 3 + 1 + 30 + 15 = 85.98 . . . f eet. ■

§ Problem 3.1.17. P is the top and Q the foot of a tower standing on


a horizontal plane. A and B are two points on this plane such that AB
2
is 32 feet and ∠QAB is a right angle. It is found that cot ∠P AQ =
5
and
3
cot ∠P BQ =
5
find the height of the tower. ♢
§§ Solution. If h feet be the height of the tower, we have
2
AQ = P Q cot ∠P AQ = h, and
5
3
BQ = P Q cot ∠P BQ = h.
5
Also, BQ2 = BA2 + AQ2
9 2 4 2
∴ h = (32)2 + h
25 25
∴ 5h = (32) × 25
2 2

∴ h = 32 5 = 32 × 2.236 . . . = 71.55 . . . f eet. ■

§ Problem 3.1.18. A square tower stands upon a horizontal plane.


From a point in this plane, from which three of its upper corners are
visible, their angular elevations are respectively 45◦ , 60◦ , and 45◦ .
Show ( that the)height of the tower is to the breadth of one of its sides
√ √
as 6 5 + 1 to 4. ♢
§§ Solution. Let P , Q and R be the three upper corners whose an-
gular elevations are 45◦ , 60◦ and 45◦ respectively at the point D.
Let C, B and A be the three lower corners vertically under P , Q
and R respectively.
Also, let h be the height of the tower, a be the breadth of one of
its sides and E be the middle point of its base. We then have
AD = h cot 45◦ = h = CD
h
BD = h cot 60◦ = √ , and
3
a
BE = EA = a cos 45◦ = √ .
2
Also, DA2 = DE 2 + EA2 = (DB + BE)2 + EA2
( )
h a 2 a2 h2 2ah a2 a2
∴ h2 = √ + √ + = + √ + +
3 2 2 3 6 2 2

∴ 2h2 − ah 6 − 3a2 = 0
√ √ √
a 6 ± 6a2 + 24a2 a 6( √ )
∴h= = 1+ 5
4 4
since the positive sign must obviously be taken.
√ (√ )
∴h:a= 6 5 + 1 : 4. ■

§ Problem 3.1.19. A lighthouse, facing north, sends out a fan-shaped


beam of light extending from north-east to north-west. An observer
on a steamer, sailing due west, first sees the light when he is 5 miles
3.1. Simple Problems 48


away from the lighthouse and continues to see it for 30 2 minutes.
What is the speed of the steamer ? ♢
§§ Solution. Let L be the lighthouse and S and H be respectively
the two positions of the steamer at which the light is visible. We then
have
LS = LH = 5 miles, and ∠SLH = 90◦ .

Hence in 30 2 minutes the steamer sails a distance

= SH = 5 sec 45◦ = 5 2 miles
Å √ ã
5 2
∴ The required speed = √ × 60 miles per hour
30 2
= 10 miles per hour. ■

§ Problem 3.1.20. A man stands at a point X on the bank XY of


a river with straight and parallel banks and observes that the line
joining X to a point Z on the opposite bank makes an angle of 30◦
with XY . He then goes along the bank a distance of 200 yards to Y
and finds that the ∠ZY X is 60◦ . Find the breadth of the river. ♢
§§ Solution. Let A be the point in XY opposite Z.
The ∠XZY = 90◦ and we have √
◦ 3 √
XZ = XY sin 60 = 200 × = 100 3 yards.
2
Hence the breadth of the river
= AZ = XZ sin 30◦
√ 1 √
= 100 3 × = 50 3 yards
2
= 50 × 1.73205 = 86.6 . . . yards.
Otherwise thus : We have
AY 1
= cot 60◦ = √
AZ 3
AX ◦

and = cot 30 = 3.
AZ
Hence, by addition, we have
XY 1 √ 4
= √ + 3= √
AZ 3 3
√ √
XY 3 200 3 √
∴ AZ = = = 50 3 = 86.6 . . . yards. ■
4 4
§ Problem 3.1.21. A man, walking due north, observes that the
elevation of a balloon, which is due east of him and is sailing toward
the north-west, is then 60◦ ; after he has walked 400 yards the balloon
is vertically over his head ; find its height supposing it to have always
remained the same. ♢
§§ Solution. Let M and M ′ be the two positions of the man, so that
M M ′ = 400 yards.
Let B and B ′ be two positions of the balloon and h yards be its
height above the ground, so that BA = h = B ′ M ′ , where A is the
point of the ground vertically below B.
3.1. Simple Problems 49

Then ∆BAM and ∆B ′ M ′ M are right angled triangles in two ver-


tical planes and ∆AM M ′ a right angled triangle in the horizontal
plane,
h
and we have AM = h cot 60◦ = √ .
3
Also, ∠AM M ′ = 90◦ , and ∠M AM ′ = 45◦
∴ AM = M M ′
h
∴ √ = 400
√ 3
∴ h = 400 3 yards = 4 × 173.205 . . . = 692.8 . . . yards. ■
Chapter
4
Applications of Algebraic Signs to
Trigonometry

4.1 Tracing the changes in the ratios


§ Problem 4.1.1. In a triangle one angle contains as many grades as
another contains degrees, and the third contains as many centesimal
seconds as there are sexagesimal seconds in the sum of the other
two; find the number of radians in each angle. ♢
§§ Solution. Let x, y and z be the number of radians in each angle
respectively.
We then have
x+y+z =π (4.1)
200 180
x× =y× (4.2)
π π
200 × 100 × 100 180 × 60 × 60
z× = (x + y) (4.3)
π π
From Eq. (4.2), we have 10x = 9y.
From Eq. (4.3), we have 250z = 81(x + y).
x y x+y 250z
∴ = = = .
9 10 19 81 × 19
x y z x+y+z π
∴ = = = =
2250 2500 1539 6289 6289
So that the angles contain ( )
2250 2500 1539 81
π, π, and π i.e. π radians. ■
6289 6289 6289 331
4.1. Tracing the changes in the ratios 52

§ Problem 4.1.2. Find the number of degrees, minutes, and sec-


onds in the angle at the center of a circle, whose radius is 5 feet,
which is subtended by an arc of length 6 feet. ♢
6
§§ Solution. The number of radians in the angle = .
5
Hence, in degrees, the angle
( )◦
6 180 6
= × = × 57◦ 17′ 44.8′′ [Art. 16] = 68◦ 45′ 17.8′′ . ■
5 π 5
§ Problem 4.1.3. To turn radians into seconds, prove that we must
multiply by 206265 nearly, and to turn seconds into radians the mul-
tiplier must be .0000048. ♢
§§ Solution. If θ be the number of radians in any angle and s be the
number of seconds in the same angle, we have
θ s
=
π 180 × 60 × 60
180
∴s=θ× × 60 × 60 ≈ 206265
π
π
and θ =s× ≈ .0000048. ■
180 × 60 × 60
x2 − y 2
§ Problem 4.1.4. If sin θ = , find the values of cos θ and cot θ.
x2 + y 2

§§ Solution. We have   Å ã2
√ x2 − y 2
cos θ = 1− sin2 θ= 1−
x2 + y 2

4x2 y 2 2xy
= =
(x2 + y 2 )2 x2 + y 2

1 − sin2 θ 2xy x2 − y 2 2xy
and cot θ = = 2 ÷ 2 = 2 .
sin θ x +y 2 x + y2 x − y2
Otherwise thus : Taking the figure of Art. 31, let the length OP be
x2 + y 2 and let the corresponding length of M P be x2 − y 2 . Then
√ √ √
OM = OP 2 − M P 2 = (x2 + y 2 )2 − (x2 − y 2 )2 = 4x2 y 2 = 2xy.
OM 2xy
∴ cos θ = = 2
OP x + y2
OM 2xy
and cot θ = = 2 . ■
MP x − y2
m2 + 2mn
§ Problem 4.1.5. If sin θ = ,
m2 + 2mn + 2n2
2
m + 2mn
prove that tan θ = .
2mn + 2n2 ♢
§§ Solution. With the same figure, let the length OP be m2 + 2mn +
2n2 and let the corresponding length of M P be m2 + 2mn. Then
√ √
OM = OP 2 − M P 2 = (m2 + 2mn + 2n2 )2 − (m2 + 2mn)2
√ [ ]
= (2m2 + 4mn + 2n2 ) 2n2 ∵ a2 − b2 = (a + b)(a − b)

= (m2 + 2mn + n2 ) 4n2 = (m + n)2n.
4.1. Tracing the changes in the ratios 53

MP m2 + 2mn
∴ tan θ == .
OM 2mn + 2n2
Otherwise thus : Substitute for sin θ in the formula
sin θ
tan θ = √ .

1 − sin2 θ
§ Problem 4.1.6. √
If cos θ − sin θ = 2 sin θ

prove that cos θ + sin θ = 2 cos θ.
(√ ) ♢

§§ Solution. If cos θ − sin θ = 2 sin θ, then cos θ = 2 + 1 sin θ.
(√ )
∴ multiplying both sides by 2 − 1 , we have
(√ ) (√ ) (√ )
2 − 1 cos θ = 2−1 2 + 1 sin θ = sin θ

∴ cos θ + sin θ = 2 cos θ. ■

§ Problem 4.1.7.
cosec 6 α − cot6 α = 3 cosec 2 α cot2 α + 1. ♢
§§ Solution.
cosec 6 α − cot6 α
( )( )
= cosec 2 α − cot2 α cosec 4 α + cosec 2 α cot2 α + cot4 α
( ) î( )2 ó
= cosec 2 α − cot2 α cosec 2 α − cot2 α + 3 cosec 2 α cot2 α
= 1 + 3 cosec 2 α cot2 α, ∵ cosec 2 α − cot2 α = 1. ■

§ Problem 4.1.8. Express


2 sec2 A − sec4 A − 2 cosec 2 A + cosec 4 A
in terms of tan A. ♢
§§ Solution.
2 sec2 A − sec4 A − 2 cosec 2 A + cosec 4 A
( ) ( )
= cosec 4 A − 2 cosec 2 A + 1 − sec4 A − 2 sec2 A + 1
( )2 ( )2
= cosec 2 A − 1 − sec2 A − 1
( )2 ( )2
= cot2 A) − tan2 A
= cot4 A − tan4 A
1
= − tan4 A. ■
tan4 A
§ Problem 4.1.9. Solve the equation 3 cosec 2 θ = 2 sec θ. ♢
§§ Solution.
3 cosec 2 θ = 2 sec θ
3 2
∴ =
sin2 θ ( cos θ )
∴ 3 cos θ = 2 1 − cos2 θ
∴ 2 cos2 θ + 3 cos θ − 2 = 0
∴ (2 cos θ − 1) (cos θ + 2) = 0
∴ 2 cos θ − 1 = 0, or cos θ + 2 = 0.
1
If 2 cos θ − 1 = 0 then cos θ = = cos 60◦ , i.e. θ = 60◦ .
2
If cos θ + 2 = 0 then cos θ = −2. This value is inadmissible, since the
cosine of an angle cannot be numerically greater than unity. ■
4.1. Tracing the changes in the ratios 54

§ Problem 4.1.10. A man on a cliff observes a boat at an angle of de-


pression of 30◦ , which is making for the shore immediately beneath
him. Three minutes later the angle of depression of the boat is 60◦ .
How soon will it reach the shore ? ♢
§§ Solution. Take the figure of Ex. 2, Art. 45. Draw P N parallel to
M A, so that P N is the horizontal line passing through P .
Let P be the position of the man and A and B be the positions of
the boat respectively.
We are given
∠N P A = 30◦ = ∠P AM
∠N P B = 60◦ = ∠P BM.
∵ ∠BAP = 30◦ = ∠AP B
∴ BP = BA.
1 1
∴ M B = BP cos 60◦ = BP = BA.
( 2 ) 2
1 1
∴ The required time = × 3 min. = 1 min. ■
2 2
1
§ Problem 4.1.11. Prove that the equation sin θ = x+ is impossible
x
if x be real. ♢
1
§§ Solution. By Algebra, (x−1)2 > 0, so that x2 +1 > 2x, ∴ x+ > 2.
x
1
∴ sin θ = x + is impossible if x be real, since the sine of an angle
x
cannot be numerically greater than unity. ■

4xy
§ Problem 4.1.12. Show that the equation sec2 θ = is only
(x + y)2
possible when x = y. ♢
§§ Solution. By Algebra, we know that if x and y be unequal, then
(x − y)2 > 0,
∴ x2 + y 2 > 2xy
∴ (x + y)2 > 4xy
4xy
∴ < 1.
(x + y)2
But sec θ cannot be < 1.
4xy
Hence the equation sec2 θ = is only possible when x = y,
(x + y)2
4
in which case sec2 θ = = 1. ■
4
Chapter
5
Trigonometrical Functions of
Angles of Any Size and Sign

5.1 Angles of Any Size and Sign


Prove that
§ Problem 5.1.1. sin 420◦ cos 390◦ + cos (−300◦ ) sin (−330◦ ) = 1. ♢
§§ Solution. √
3
sin 420◦ = sin (360◦ + 60◦ ) = sin 60◦ =
2

3
cos 390◦ = cos (360◦ + 30◦ ) = cos 30◦ =
2
cos (−300◦ ) = cos 300◦ = cos (180◦ + 120◦ ) = − cos 120◦
1
= − cos (90◦ + 30◦ ) = sin 30◦ =
2
sin (−330◦ ) = − sin 330◦ = − sin (180◦ + 150◦ ) = sin 150◦
1
= sin (90◦ + 60◦ ) = cos 60◦ = .
√ √ 2
3 3 1 1 3+1
Hence the expression = · + · = = 1. ■
2 2 2 2 4
§ Problem 5.1.2. cos 570◦ sin 510◦ − sin 330◦ cos 390◦ = 0. ♢
§§ Solution.
cos 570◦ = cos (360◦ + 210◦ ) = cos 210◦ = cos (180◦ + 30◦ )

3
= − cos 30◦ = −
2
1
sin 510◦ = sin (360◦ + 150◦ ) = sin 150◦ = sin (90◦ + 60◦ ) = cos 60◦ =
2
5.1. Angles of Any Size and Sign 56

sin 330◦ = sin (180◦ + 150◦ ) = − sin 150◦


1
= − sin (90◦ + 60◦ ) = − cos 60◦ = −
√ 2
3
cos 390◦ = cos (360◦ + 30◦ ) = cos 30◦ = .
√ Å 2 √ ã √ √
3 1 1 3 3 3
Hence the expression = − · − − · =− + = 0. ■
2 2 2 2 4 4
§ Problem 5.1.3. tan 225◦ cot 405◦ + tan 765◦ cot 675◦ = 0. ♢
§§ Solution.
tan 225◦ = tan (180◦ + 45◦ ) = tan 45◦ = 1
cot 405◦ = cot (360◦ + 45◦ ) = cot 45◦ = 1
tan 765◦ = tan (2 × 360◦ + 45◦ ) = tan 45◦ = 1
cot 675◦ = cot (360◦ + 315◦ ) = cot 315◦
= cot (180◦ + 135◦ ) = cot 135◦
= cot (90◦ + 45◦ ) = − tan 45◦ = −1.
Hence the expression = 1 − 1 = 0. ■

What are the values of cos A − sin A and tan A + cot A when A has
the values
π
§ Problem 5.1.4. ♢
3
§§ Solution. √
π π 1 3 .73205
cos − sin = − =− = −.366 . . .
3 3 2 2 2 √
π π √ 1 4 4 3
tan + cot = 3+ √ = √ =
3 3 3 3 3
4 × 1.73205 . . .
= = 2.3094 . . . . ■
3

§ Problem 5.1.5. ♢
3
§§ Solution. √
2π 2π 1 3
cos − sin =− −
3 3 2 √2
1+ 3 2.73205 . . .
=− =− = −1.366 . . .
2 √2
2π 2π √ 1 4 3
tan + cot =− 3− √ =− = −2.3094 . . . . ■
3 3 3 3

§ Problem 5.1.6. ♢
4
§§ Solution.
5π 5π Ä πä Ä πä
cos − sin = cos π + − sin π +
4 4 4 4
π π 1 1
= − cos + sin = − √ + √ = 0
4 4 2 2
5π 5π Ä πä Ä πä
tan + cot = tan π + + cot π +
4 4 4 4
π π
= tan + cot = 1 + 1 = 2. ■
4 4

§ Problem 5.1.7. ♢
4
5.1. Angles of Any Size and Sign 57

§§ Solution. ( ) ( )
7π 7π 3π 3π
cos − sin = cos π + − sin π +
4 4 4
3π 3π Ä4 π π ä Äπ πä
= − cos + sin = − cos + + sin +
4 4 2 4 2 4
π π 1 1 2 √
= sin + cos = √ + √ = √ = 2 = 1.414 . . .
4 4
( ) 2( 2 ) 2
7π 7π 3π 3π
tan + cot = tan π + + cot π +
4 4 4
3π 3π Äπ 4 πä Äπ πä
= tan + cot = tan + + cot +
4 4 2 4 2 4
π π
= − cot − tan = −1 − 1 = −2. ■
4 4
11π
§ Problem 5.1.8. ♢
3
§§ Solution. ( ) ( )
11π 11π 5π 5π
cos − sin = cos 2π + − sin 2π +
3 3 3 ( 3 ) ( )
5π 5π 2π 2π
= cos − sin = cos π + − sin π +
3 3 √3 3
2π 2π 1 3
= − cos + sin =+ +
3 3 2 2
2.73205
= = 1.366 . . .
2( ) ( )
11π 11π 5π 5π
tan + cot = tan 2π + + cot 2π +
3 3 3 ( 3 ) ( )
5π 5π 2π 2π
= tan + cot = tan π + + cot π +
3 3 3 3
2π 2π √ 1
= tan + cot =− 3− √
3 3 3

4 3
=− = −2.3094 . . . . ■
3
What values between 0◦ and 360◦ may A have when
1
§ Problem 5.1.9. sin A = √ ♢
2
§§ Solution. Since sin A is positive, A must be in the first or second
quadrant.
1
Now sin A = √ = sin 45◦ = sin (180◦ − 45◦ ).
2
Hence A = 45◦ , or 180◦ − 45◦ , i.e. 45◦ or 135◦ . ■

1
§ Problem 5.1.10. cos A = − ♢
2
§§ Solution. Since cos A is negative, A must be in the second or
third quadrant.
1
Now cos A = − = − cos 60◦ = cos (180◦ − 60◦ ) or cos (180◦ + 60◦ ).
2
Hence A = 120◦ or 240◦ . ■

§ Problem 5.1.11. tan A = −1 ♢


5.1. Angles of Any Size and Sign 58

§§ Solution. Since tan A is negative, A must be in the second or


fourth quadrant.
Now tan A = −1 = − tan 45◦ = tan (180◦ − 45◦ ) = tan 135◦
also, tan 135◦ = tan (180◦ + 135◦ ) = tan 315◦ .
∴ A = 135◦ or 315◦ . ■


§ Problem 5.1.12. cot A = − 3 ♢
§§ Solution. Since cot A is negative, A must be in the second or
fourth quadrant. √
Now cot A = − 3 = − cot 30◦ = cot (180◦ − 30◦ ) = cot 150◦
also, cot 150◦ = cot (180◦ + 150◦ ) = cot 330◦ .
∴ A = 150◦ or 330◦ . ■

2
§ Problem 5.1.13. sec A = − √ ♢
3
§§ Solution. Since sec A is negative, A must be in the second or
third quadrant.
Now
2
sec A = − √ = − sec 30◦ = sec (180◦ − 30◦ ) or sec (180◦ + 30◦ ) .
3
∴ A = 150◦ or 210◦ . ■

§ Problem 5.1.14. cosec A = −2 ♢


§§ Solution. Since cosec A is negative, A must be in the third or
fourth quadrant.
Now
cosec A = −2 = − cosec 30◦ or − cosec 150◦
= cosec (180◦ + 30◦ ) or cosec (180◦ + 150◦ ) .
∴ A = 210◦ or 330◦ . ■

Express in terms of the ratios of a positive angle, which is less


than 45◦ , the quantities
§ Problem 5.1.15. sin (−65◦ ) ♢
§§ Solution. sin (−65◦ ) = − sin 65◦ = − cos (90◦ − 65◦ ) = − cos 25◦ .

§ Problem 5.1.16. cos (−84◦ ) ♢


§§ Solution. cos (−84◦ ) = cos 84◦ = sin (90◦ − 84◦ ) = sin 6◦ . ■

§ Problem 5.1.17. tan 137◦ ♢


§§ Solution. tan 137◦ = − tan (180◦ − 137◦ ) = − tan 43◦ . ■

§ Problem 5.1.18. sin 168◦ ♢


§§ Solution. sin 168◦ = sin (180◦ − 168◦ ) = sin 12◦ . ■

§ Problem 5.1.19. cos 287◦ ♢


§§ Solution.
cos 287◦ = cos (180◦ + 107◦ ) = − cos 107◦
= − cos (90◦ + 17◦ ) = sin 17◦ . ■

§ Problem 5.1.20. tan (−246◦ ) ♢


5.1. Angles of Any Size and Sign 59

§§ Solution.
tan (−246◦ ) = − tan 246◦ = − tan (180◦ + 66◦ )
= − tan 66◦ = − cot (90◦ − 66◦ ) = − cot 24◦ . ■

§ Problem 5.1.21. sin 843◦ ♢


§§ Solution.
sin 843◦ = sin (2 × 360◦ + 123◦ ) = sin 123◦
= sin (180◦ − 123◦ ) = sin 57◦
= cos (90◦ − 57◦ ) = cos 33◦ . ■

§ Problem 5.1.22. cos (−928◦ ) ♢


§§ Solution.
cos (−928◦ ) = cos 928◦ = cos (2 × 360◦ + 208◦ )
= cos 208◦ = cos (180◦ + 28◦ ) = − cos 28◦ . ■

§ Problem 5.1.23. tan 1145◦ ♢


§§ Solution.
tan 1145◦ = tan (3 × 360◦ + 65◦ ) = tan 65◦
= cot (90◦ − 65◦ ) = cot 25◦ . ■

§ Problem 5.1.24. cos 1410◦ ♢


§§ Solution.
cos 1410◦ = cos (3 × 360◦ + 330◦ ) = cos 330◦
= cos (180◦ + 150◦ ) = − cos 150◦
= − cos (180◦ − 30◦ ) = cos 30◦ . ■

§ Problem 5.1.25. cot (−1054◦ ) ♢


§§ Solution.
cot (−1054◦ ) = − cot 1054◦ = − cot (2 × 360◦ + 334◦ ) = − cot 334◦
= − cot (180◦ + 154◦ ) = − cot 154◦
= − cot (180◦ − 26◦ ) = cot 26◦ . ■

§ Problem 5.1.26. sec 1327◦ ♢


§§ Solution.
sec 1327◦ = sec (3 × 360◦ + 247◦ ) = sec 247◦ = sec (180◦ + 67◦ )
= − sec 67◦ = − cosec (90◦ − 67◦ ) = − cosec 23◦ . ■

§ Problem 5.1.27. cosec (−756◦ ) ♢


§§ Solution.
cosec (−756◦ ) = − cosec 756◦
= − cosec (2 × 360◦ + 36◦ ) = − cosec 36◦ . ■

What sign has sin A + cos A for the following values of A ?


§ Problem 5.1.28. 140◦ ♢
§§ Solution.
sin 140◦ = sin (180◦ − 140◦ ) = sin 40◦
cos 140◦ = − cos (180◦ − 140◦ ) = − cos 40◦
∴ cos A is numerically greater than sin A and is negative
∴ sin A + cos A is negative. ■
5.1. Angles of Any Size and Sign 60

§ Problem 5.1.29. 278◦ ♢


§§ Solution.
sin 278◦ = − sin 98◦ = − sin (180◦ − 98◦ ) = − sin 82◦
cos 278◦ = cos (180◦ + 98◦ ) = − cos 98◦ = cos (180◦ − 98◦ ) = cos 82◦ .
∴ sin A is numerically greater than cos A and is negative
∴ sin A + cos A is negative. ■

§ Problem 5.1.30. −356◦ ♢


§§ Solution.
sin (−356◦ ) = − sin 356◦ = − sin (180◦ + 176◦ )
= sin 176◦ = sin (180◦ − 176◦ ) = sin 4◦ .
cos (−356◦ ) = cos 356◦ = cos (180◦ + 176◦ )
= − cos 176◦ = cos (180◦ − 176◦ ) = cos 4◦ .
∴ sin A and cos A are both positive
∴ sin A + cos A is positive. ■

§ Problem 5.1.31. −1125◦ ♢


§§ Solution.
−1125◦ = − (3 × 360 + 45◦ )
1 1
∴ sin (−1125◦ ) = − √ , and cos (−1125◦ ) = √ .
2 2

∴ sin A + cos A = 0.
What sign has sin A − cos A for the following values of A ?
§ Problem 5.1.32. 215◦ ♢
§§ Solution.
sin 215◦ = sin (180◦ + 35◦ ) = − sin 35◦
cos 215◦ = cos (180◦ + 35◦ ) = − cos 35◦ .
∴ sin A and cos A are both negative and cos A is numerically greater
than sin A ;
∴ sin A − cos A is positive. ■

§ Problem 5.1.33. 825◦ ♢


§§ Solution.
825◦ = (2 × 360◦ + 105◦ )
sin 825◦ = sin 105◦ = sin (180◦ − 105◦ ) = sin 75◦
cos 825◦ = cos 105◦ = − cos (180◦ − 105◦ ) = − cos 75◦ .
∴ sin A is positive and cos A is negative ;
∴ sin A − cos A is positive. ■

§ Problem 5.1.34. −634◦ ♢


§§ Solution.
−634◦ = −720◦ + 86◦
∴ sin A and cos A are both positive and sin A is numerically greater
than cos A ;
∴ sin A − cos A is positive. ■
5.1. Angles of Any Size and Sign 61

§ Problem 5.1.35. −457◦ ♢


§§ Solution.
−457◦ = − (360◦ + 97◦ )
∴ sin (−457◦ ) = sin (−97◦ ) = − sin 97◦ = − sin (180◦ − 97◦ ) = − sin 83◦
cos (−457◦ ) = cos 97◦ = − cos (180◦ − 97◦ ) = − cos 83◦
∴ sin A and cos A are both negative and sin A is numerically greater
than cos A ;
∴ sin A − cos A is negative. ■

§ Problem 5.1.36. Find the sines and cosines of all angles in the
first four quadrants whose tangents are equal to cos 135◦ . ♢
◦ 1 1
§§ Solution. If tan θ = cos 135 = − √ , then sin θ = ± √ and cos θ =
√ 2 3
2
± .
3
Also, since tan θ is negative, θ is in the second or fourth quadrant.
In the second quadrant,

1 2
sin θ = √ , and cos θ = − .
3 3
In the fourth quadrant,

1 2
sin θ = − √ , and cos θ = . ■
3 3
Prove that
§ Problem 5.1.37.
sin (270◦ + A) = − cos A and
tan (270◦ + A) = − cot A. ♢
§§ Solution.
sin (270◦ + A) = sin [180◦ + (90◦ + A)]
= − sin (90◦ + A) , [Art. 73] = − cos A, [Art. 70].
tan (270◦ + A) = tan [180 + (90◦ + A)]
= tan (90◦ + A) = − cot A.
Otherwise thus: Cf. the figure of Art. 50. Let the revolving line,
starting from OA, trace out any angle A in the first quadrant and let
OP1 be the position of the revolving line then, so that the ∠AOP1 is
A.
Let the revolving line then turn through three right angles (270◦ )
in the positive direction to the position OP4 , so that the ∠AOP4 is
(270◦ + A).
Draw P1 M1 and P4 M4 perpendicular to OA.
By Euc. I. 26, the triangles P1 OM1 and OP4 M4 are geometrically
equal and we have
OP1 = OP4 in magnitude and sign,
P1 M1 = OM4 in magnitude and sign,
and P4 M4 = OM1 in magnitude, but of opposite sign.
P4 M4 OM1
∴ sin (270◦ + A) = =− = − cos A, and
OP4 OP1
P4 M4 OM1
tan (270◦ + A) = =− = − cot A. ■
OM4 P1 M1
5.1. Angles of Any Size and Sign 62

§ Problem 5.1.38. cos (270◦ − A) = − sin A and cot (270◦ − A) = tan A.



§§ Solution.
cos (270◦ − A) = cos [180◦ + (90◦ − A)]
= − cos (90◦ − A) = − sin A
cot (270◦ − A) = cot [180 + (90◦ − A)]
= cot (90◦ − A) = tan A.
Otherwise thus: Cf. the figure of Art. 50. Let the revolving line,
starting from OA, trace out any angle A in the first quadrant and let
OP1 be the position of the revolving line then, so that the ∠AOP1 is
A.
To obtain the angle (270◦ − A), let the revolving line start from OA
and after turning through three right angles, i.e. into the position
OB ′ , then turn back through an angle A into the position OP3 , so
that the ∠AOP3 is (270◦ − A).
Draw P1 M1 and P3 M3 perpendicular to AOA′ .
By Euc. I. 26, the triangles P1 OM1 and OP3 M3 are geometrically
equal and we have
OP1 = OP3 in magnitude and sign,
OM3 = P1 M1 in magnitude, but of opposite sign
and P3 M3 = OM1 .
OM3 P1 M1
∴ cos (270◦ − A) = =− = − sin A, and
OP3 OP1
OM3 −P1 M1
cot (270◦ − A) = = = tan A. ■
P3 M3 −OM1
§ Problem 5.1.39.
cos A + sin (270◦ + A) − sin (270◦ − A)
+ cos (180◦ + A) = 0. ♢
§§ Solution.
sin (270◦ + A) = − cos A by §P roblem 5.1.37
sin (270◦ − A) = sin [180◦ + (90◦ − A)]
= − sin (90◦ − A) = − cos A,
and cos (180◦ + A) = − cos A, by Art. 73.

Hence the given expression


= cos A − cos A + cos A − cos A = 0. ■

§ Problem 5.1.40.
sec (270◦ − A) sec (90◦ − A)
− tan (270◦ − A) tan (90◦ + A) + 1 = 0. ♢
§§ Solution.
sec (270◦ − A) = sec [180◦ + (90◦ − A)]
= − sec (90◦ − A) = − cosec A
sec (90◦ − A) = cosec A
tan (270◦ − A) = tan [180◦ + (90◦ − A)]
= tan (90◦ − A) = cot A
tan (90◦ + A) = − cot A.
Hence the given expression
( )
= − cosec 2 A − − cot2 A + 1 = − cosec 2 A + cosec 2 A = 0. ■
5.1. Angles of Any Size and Sign 63

§ Problem 5.1.41.
cot A + tan (180◦ + A) + tan (90◦ + A)
+ tan (360◦ − A) = 0. ♢
§§ Solution.
The given expression
= cot A + tan A + (− cot A) + (− tan A) = 0. ■
Chapter
6
General Expressions for All
Angles Having A Given
Trigonometrical Ratio

6.1 Generic Values


What are the most general values of θ which satisfy the equa-
tions
1
§ Problem 6.1.1. sin θ = . ♢
2
§§ Solution.
1 π
sin θ = = sin
2 6
π
∴ θ = nπ + (−1)n . ■
6

3
§ Problem 6.1.2. sin θ = − . ♢
2
§§ Solution. √
3 π Ä πä
sin θ = − = − sin = sin −
2 Ä π ä3 3
π
∴ θ = nπ + (−1)n − = nπ − (−1)n . ■
3 3
1
§ Problem 6.1.3. sin θ = √ . ♢
2
1 π
§§ Solution. sin θ = √ = sin .
2 4 Äπä
∴ θ = nπ + (−1)n . ■
4
6.1. Generic Values 66

1
§ Problem 6.1.4. cos θ = − . ♢
2
§§ Solution.
1 2π
cos θ = − = cos
2 3

∴ θ = 2nπ ± . ■
3

3
§ Problem 6.1.5. cos θ = . ♢
2
§§ Solution. √
3 π
cos θ = = cos
2 6
π
∴ θ = 2nπ ± . ■
6
1
§ Problem 6.1.6. cos θ = − √ . ♢
2
§§ Solution.
1 3π
cos θ = − √ = cos
2 4

∴ θ = 2nπ ± . ■
4

§ Problem 6.1.7. tan θ = 3. ♢
§§ Solution.
√ π
tan θ = 3 = tan
3
π
∴ θ = nπ + . ■
3
§ Problem 6.1.8. tan θ = −1. ♢
§§ Solution.

tan θ = −1 = tan
4

∴ θ = nπ + . ■
4
§ Problem 6.1.9. cot θ = 1. ♢
§§ Solution.
π
cot θ = 1 = cot
4
π
∴ θ = nπ + . ■
4
§ Problem 6.1.10. sec θ = 2. ♢
§§ Solution.
π
sec θ = 2 = sec
3
π
∴ θ = 2nπ ± . ■
3
2
§ Problem 6.1.11. cosec θ = √ . ♢
3
§§ Solution.
2 π
cosec θ = √ = cosec
3 3
π
∴ θ = nπ + (−1)n . ■
3
6.1. Generic Values 67

§ Problem 6.1.12. sin2 θ = 1. ♢


§§ Solution.
π
sin2 θ = 1 = sin2
2
π
∴ θ = nπ ± .
2
Notes : θ = nπ ± α is the general solution of any one of the following
equations:
sin2 θ = sin2 α
cos2 θ = cos2 α
tan2 θ = tan2 α
cosec 2 θ = cosec 2 α
sec2 θ = sec2 α
and
cot2 θ = cot2 α. ■

1
§ Problem 6.1.13. cos2 θ = . ♢
4
§§ Solution.
1 π
cos2 θ = = cos2
4 3
π
∴ θ = nπ ± . ■
3
1
2
§ Problem 6.1.14. tan θ = . ♢
3
§§ Solution.
1 π
tan2 θ = = tan2
3 6
π
∴ θ = nπ ± . ■
6
§ Problem 6.1.15. 4 sin2 θ = 3. ♢
§§ Solution.
3 π
sin2 θ = = sin2
4 3
π
∴ θ = nπ ± . ■
3
§ Problem 6.1.16. 2 cot2 θ = cosec 2 θ. ♢
§§ Solution.
2 cot2 θ = cosec 2 θ = 1 + cot2 θ
π
∴ cot2 θ = 1 = cot2
4
π
∴ θ = nπ ± . ■
4
4
2
§ Problem 6.1.17. sec θ = . ♢
3
§§ Solution.
4 π
sec2 θ = = sec2
3 6
π
∴ θ = nπ ± . ■
6
6.1. Generic Values 68

§ Problem 6.1.18. What is the most general value of θ that satisfies


both of the equations
1
cos θ = − √ , and
2
tan θ = 1. ♢
§§ Solution. The only value of θ between 0◦ and 360◦ satisfying both

conditions is 225◦ , i.e. .
4

Hence the general value of θ is 2nπ + . ■
4

§ Problem 6.1.19. What is the most general value of θ that satisfies


both of the equations √
cot θ = − 3, and
cosec θ = −2. ♢
§§ Solution. θ must be either in the 4th quadrant or in the 1st neg-
π
ative quadrant and its value is − .
6
π
Hence the general value of θ is 2nπ − . ■
6

1 1
§ Problem 6.1.20. If cos (A − B) = , and sin (A + B) = , find the
2 2
smallest positive values of A and B and also their most general val-
ues. ♢
1 π 1 π
§§ Solution. cos(A − B) = = cos and sin(A + B) = = sin .
2 3 2 6
π

Hence A − B = 
3
5π  give the smallest positive values of A and B.
and A + B =
î π
6 ó
N ot A + B = , ∵ A + B > A − B.
6
7π 7π
Hence, by addition, we have 2A = , so that A = = 105◦
6 12
π π
and, by subtraction, we have 2B = , so that B = = 45◦ .
2 4
The general values are given by
π
A − B = 2nπ ± , and
3
π
A + B = mπ ± (−1)m ,
6
where m and n are anyÄintegers.ä
m π π
Hence we have A= n+ π ± + (−1)m
Äm 2 ä 6
π
12
π
and B= − n π ∓ + (−1)m . ■
2 6 12
2
§ Problem 6.1.21. If tan (A − B) = 1, and sec (A + B) = √ , find
3
the smallest positive values of A and B and also their most general
values. ♢
6.1. Generic Values 69

π 2 π
§§ Solution. tan (A − B) = 1 = tan and sec (A + B) = √ = sec .
4 3 6
π

Hence A − B = 
4
11π  give the smallest positive values of A and B.
and A + B =
î π
6 ó
N ot A + B = , ∵ A + B > A − B.
6
25π 25π 1◦
Hence, by addition, we have 2A = , so that A = = 187
12 24 2 ◦
19π 19π 1
and, by subtraction, we have 2B = , so that B = = 142 .
12 24 2
The general values are given by
π
A − B = mπ + , and
4
π
A + B = 2nπ ± ,
6
where m and n are any integers.
Ä ä
m π π
Hence we have A= n+ π+ ±
Ä mä
2 8
π
12
π
and B = n− π− ± . ■
2 8 12
§ Problem 6.1.22. Find the angles between 0◦ and 360◦ which have
respectively

3
(1) their sines equal to ,
2
1
(2) their cosines equal to − , and
2
1
(3) their tangents equal to √ .
3 ♢

3
§§ Solution. (1) If sin θ = , then θ is in the first or second quad-
2
rant and is equal to 60◦ or 120◦ .
1
(2) If cos θ = − , then θ is in second or third quadrant and is equal
2
to 120◦ or 240◦ .
1
(3) If tan θ = √ , then θ is in the first or third quadrant and is
3
equal to 30◦ or 210◦ . ■

§ Problem 6.1.23. Taking into consideration only angles between


0◦ and 180◦ how many values of x are there if
5
(1) sin x = ,
7
1
(2) cos x = ,
5
4
(3) cos x = − ,
5
6.1. Generic Values 70

2
(4) tan x = , and
3
(5) cot x = −7 ? ♢
§§ Solution. (1) Two values, supplementary.
(2) One value, between 0◦ and 90◦ .
(3) One value, between 90◦ and 180◦ .
(4) One value, acute.

(5) One value, obtuse.

§ Problem 6.1.24. Given the angle x construct the angle y if


(1) sin y = 2 sin x,
(2) tan y = 3 tan x,
1
(3) cos y = cos x,
2
(4) sec y = cosec x. ♢
§§ Solution. (1) Describe a semicircle AP B on AB as diameter.
Make an angle ∠P AB equal to x. Join BP . On the semicircle,
take a point Q such that BQ is equal to 2 · BP . Join AQ. We
then have
BQ 2 · BP
sin ∠BAQ = = = 2 sin x.
AB AB
∴ ∠BAQ = y.
1
Notes : This construction is impossible if sin x > .
2
(2) Draw a straight line AB the length of which is unity and erect
a perpendicular BP equal in length to x. Join AP . Produce BP
to Q so that BQ = 3 · BP . Join AQ. We then have
BQ 3 · BP
tan ∠BAQ = = = 3 tan x.
AB AB
∴ ∠BAQ = y.
Notes : This construction is always possible.
(3) Describe a semicircle AP B on AB as diameter. Make an angle
∠P AB equal to x.
1
On the semicircle, take a point Q such that AQ is equal to AP .
2
Join BP and BQ. We then have
AQ 1 AP 1
cos ∠BAQ = = · = cos x.
AB 2 AB 2
∴ ∠BAQ = y.
Notes : This construction is always possible.
(4) In the right-angled triangle ∆P OM (figure of Art. 23), let the
angle ∠P OM = x. We then have
OP
sec ∠OP M = = cosec ∠P OM = cosec x.
PM
∴ ∠OP M = y.
Notes : y is obviously complement of x. ■
6.1. Generic Values 71

§ Problem 6.1.25. Show that the same angles are indicated by the
two following formulae :
π π
(2n − 1) + (−1)n , and (6.1)
2 3
π
2nπ ± (6.2)
6
n being any integer. ♢
§§ Solution.
π π
If θ = (2n − 1) + (−1)n
2 3
π π
then θ + = nπ + (−1)n
2 3
this is the solution of the equation
Ä π ä π
sin θ + = sin
2√ 3
3 π
∴ cos θ = = cos
2 6
π
the solution of which is θ = 2nπ ± .
6
Otherwise thus : Putting n = 0, 1, 2, 3, . . . in Eq. (6.1), we have
the series of angles
π π 11π 13π
− , , , ,...
6 6 6 6
Putting n = 0, 1, 2, 3, . . . in Eq. (6.2), we have the series of angles
π π 11π 13π
− , , , ,...
6 6 6 6
each value of n giving two angles. ■

§ Problem 6.1.26. Prove ( that the


) two formulae
1
2n + π ± α, and (6.3)
2 Äπ ä
nπ + (−1)n −α (6.4)
2
denote the same angles, n being any integer.
Illustrate by a figure. ♢
( )
1
§§ Solution. From the formula 2n + π ± α, we have, by putting
2
n = 0, 1, 2, . . . the series of angles
π 5π 9π
± α, ± α, ± α, . . . . (6.5)
2 2 2
From the second formula, Ä π weähave the series of angles
Äπ ä
π π
− α, π − − α , 2π + − α, 3π − −α
2 2 2 2
π π 5π 5π
i.e. − α, + α, − α, + α, . . . .
2 2 2 2
These two series are clearly the same.
As in the figure of Art. 50, let OA be the initial line and OA′ in the
direction opposite to the initial line and OB the line bounding the
first quadrant.
Let OP and OQ be lines, respectively between OB and OA and
between OB and OA′ , such that ∠P OB = ∠BOQ = α.
Then, when the angle considered is one of those of Eq. (6.3), the
bounding line is either in the position OP or in the position OQ.
6.1. Generic Values 72

For Eq. (6.4), firstly, let n be even and equal to 2m ; then the for-
π
mula = 2mπ + −α and the corresponding bounding line is therefore
2
in the position OP .
Secondly, Älet n be
ä odd andπ equal to 2m + 1 ; then the formula
π
= (2m+1)π− − α = 2mπ+ +α and the corresponding bounding
2 2
line is in the position OQ. Hence the proposition. ■

§ Problem 6.1.27. If θ − α = nπ + (−1)n β, prove that


θ = 2mπ + α + β
or else that
θ = (2m + 1)π + α − β
where m and n are any integers. ♢
§§ Solution. If n be even (= 2m, say) we have
θ − α = 2mπ + β
∴ θ = 2mπ + α + β
If n be odd (= 2m + 1, say) we have
θ − α = (2m + 1)π − β
∴ θ = (2m + 1)π + α − β. ■

§ Problem 6.1.28. If cos pθ + cos qθ = 0, prove that the different


values of θ form two arithmetical progressions in which the common
2π 2π
differences are and respectively. ♢
p+q p∼q
§§ Solution.
cos pθ + cos qθ = 0
∴ cos pθ = − cos qθ = cos (π − qθ)
∴ pθ = 2nπ ± (π − qθ) .
Talking the upper sign, we have
(p + q)θ = (2n + 1)π
(2n + 1)
∴θ= π.
p+q
By putting n = 0, 1, 2, . . . in succession, we have the series of angles
π 3π 5π
, , , ...
p+q p+q p+q

which are in A. P., with common difference .
p+q
Taking the lower sign, we have
(p ∼ q)θ = (2n − 1)π
(2n − 1)
∴θ= π,
p∼q
and we have the series of angles
π π 3π
− , , , ...
p∼q p∼q p+q

which are in A. P., with common difference . ■
p∼q

3
§ Problem 6.1.29. Construct the angle whose sine is √ . ♢
2+ 5
6.2. Trigonometrical Equations 73

§§ Solution. Take a straight line AB of length 2a and draw AC per-


pendicular to it of length a. √
Join BC, then BC is equal to a 5.
Produce AB to O, making BO equal to BC.
On OA, describe a semicircle and in it draw AP , equal to 3a and
meeting the semicircle in P .
Join OP . The angle ∠AP O, being an angle in a semicircle, is right
angle. We then have
AP AP 3a 3
sin ∠AOP = = = √ = √ .
OA AB + BO 2a + a 5 2+ 5
Hence ∠AOP is the required angle. ■

6.2 Trigonometrical Equations


Solve the equations
1
§ Problem 6.2.1. cos2 θ − sin θ − = 0. ♢
4
§§ Solution.
1
cos2 θ − sin θ − =0
4
1
∴ 1 − sin θ − sin θ − = 0
2
4
∴ 4 sin2 θ + 4 sin θ − 3 = 0
∴ (2 sin θ − 1) (2 sin θ + 3) = 0
∴ 2 sin θ − 1 = 0 or 2 sin θ + 3 = 0.
If 2 sin θ − 1 = 0, then
1 π
sin θ = = sin
2 6
π
∴ θ = nπ + (−1)n .
6
If 2 sin θ + 3 = 0, then
3
sin θ = − ;
2
this value is inadmissible, since the sine of an angle cannot be nu-
merically greater than unity. ■

§ Problem 6.2.2. 2 sin2 θ + 3 cos θ = 0. ♢


§§ Solution.
2 sin2 θ + 3 cos θ = 0
( )
∴ 2 1 − cos2 θ + 3 cos θ = 0
∴ 2 cos2 θ − 3 cos θ − 2 = 0
∴ (2 cos θ + 1) (cos θ − 2) = 0
∴ 2 cos θ + 1 = 0, or cos θ − 2 = 0.
If 2 cos θ + 1 = 0, then
1 2π
cos θ = − = cos
2 3

∴ θ = 2nπ ± .
3
If cos θ − 2 = 0, then
cos θ = 2;
6.2. Trigonometrical Equations 74

this value is inadmissible, since the cosine of an angle cannot be


numerically greater than unity. ■


§ Problem 6.2.3. 2 3 cos2 θ = sin θ. ♢
§§ Solution. √
2 3 cos2 θ = sin θ
√ ( )
2 3 1 − sin2 θ = sin θ
√ √
∴ 2 3 sin2 θ + sin θ − 2 3 = 0
( √ ) (√ )
∴ 2 sin θ − 3 3 sin θ + 2 = 0
√ √
∴ 2 sin θ − 3 = 0, or 3 sin θ + 2 = 0.

If 2 sin θ − 3 = 0, then

3 π
sin θ = = sin
2 3
π
∴ θ = nπ + (−1)n .
√ 3
If 3 sin θ + 2 = 0, then
2
sin θ = − √ ;
3
this value is inadmissible. ■

§ Problem 6.2.4. cos θ + cos2 θ = 1. ♢


§§ Solution.
cos θ + cos2 θ = 1
∴ cos2 θ + cos θ − 1 = 0
√ √
−1 ± 1 + 4 −1 ± 5
∴ cos θ = = .
√ 2 2
− 5−1
The value is inadmissible.
2 √
5−1
∴ cos θ = . ■
2
§ Problem 6.2.5. 4 cos θ − 3 sec θ = 2 tan θ. ♢
§§ Solution.
4 cos θ − 3 sec θ = 2 tan θ
3 sin θ
∴ 4 cos θ − =2
cos θ cos θ
∴ 4 cos2 θ − 3 = 2 sin θ (6.6)
( )
4 1 − sin2 θ − 3 = 2 sin θ
∴ 4 sin2 θ + 2 sin θ − 1 = 0
√ √ √
−2 ± 4 + 16 −2 ± 2 5 ± 5−1
∴ sin θ = = = .
8 8 4
Taking the upper sign, we have

5−1 π
sin θ = = sin
4 10
π
∴ θ = nπ + (−1)n .
10
6.2. Trigonometrical Equations 75

Taking the lower sign, we have


√ √ ( )
− 5−1 5+1 3π
sin θ = =− = sin −
4 4 10

∴ θ = nπ − (−1)n .
10
Otherwise thus : From Eq. (6.6), we have
4 cos3 θ − 3 cos θ = 2 sin θ cos θ
∴ cos 3θ = sin 2θ.
For the solution of this equation, see § Problem 6.2.20.
π
The result θ = 2nπ − (giving cos θ = 0) in that problem is not a
2

solution here, but is due to the factor cos θ introduced.
1
§ Problem 6.2.6. sin2 θ − 2 cos θ + = 0. ♢
4
§§ Solution.
1
sin2 θ − 2 cos θ + =0
4
1
∴ 1 − cos θ − 2 cos θ + = 0
2
4
∴ 4 cos2 θ + 8 cos θ − 5 = 0
∴ (2 cos θ − 1) (2 cos θ + 5) = 0
∴ 2 cos θ − 1 = 0, or 2 cos θ + 5 = 0.
If 2 cos θ − 1 = 0, then
1 π
cos θ = = cos
2 3
π
∴ θ = 2nπ ± .
3
If 2 cos θ + 5 = 0, then
5
cos θ = − ;
2

this value is inadmissible.
( √ ) √
§ Problem 6.2.7. tan2 θ − 1 + 3 tan θ + 3 = 0. ♢
§§ Solution. ( √ ) √
tan2 θ − 1 + 3 tan θ + 3 = 0
( √ )
∴ (tan θ − 1) tan θ − 3 = 0
π √ π
∴ tan θ = 1 = tan or tan θ = 3 = tan
4 3
π π
∴ θ = nπ + or nπ + . ■

(√ 4 ) 3
1
§ Problem 6.2.8. cot2 θ + 3+ √ cot θ + 1 = 0. ♢
3
§§ Solution. (√ )
1
cot2 θ + 3+ √ cot θ + 1 = 0
( ) (3 √ )
1
∴ cot θ + √ cot θ + 3 = 0
3
1 2π √ 5π
∴ cot θ = − √ = cot or cot θ = − 3 = cot
3 3 6
2π 5π
∴ θ = nπ + or nπ + . ■
3 6
6.2. Trigonometrical Equations 76

§ Problem 6.2.9. cot θ − ab tan θ = a − b. ♢


§§ Solution.
cot θ − ab tan θ = a − b
1
∴ − ab tan θ = a − b
tan θ
∴ ab tan θ + (a − b) tan θ − 1 = 0
2

∴ (a tan θ − 1) (b tan θ + 1) = 0
1 1
∴ tan θ = or − . ■
a b
§ Problem 6.2.10. tan2 θ + cot2 θ = 2. ♢
§§ Solution.
tan2 θ + cot2 θ = 2
1
∴ tan2 θ + =2
tan2 θ
∴ tan4 θ − 2 tan2 θ + 1 = 0
( )2
∴ tan2 θ − 1 =0
∴ tan θ = ±1
π
∴ θ = nπ ± . ■
4
(√ )
§ Problem 6.2.11. sec θ − 1 = 2 − 1 tan θ. ♢
§§ Solution. (√ )
sec θ − 1 = 2 − 1 tan θ
(√ )√
∴ sec θ − 1 = 2−1 sec2 θ − 1

∴ (i) sec θ − 1 = 0, i.e. sec θ = 1, i.e. θ = 2nπ
and √ (√ )√
(ii) sec θ − 1 =2−1 sec θ + 1
( √ )
∴ sec θ − 1 = 3 − 2 2 (sec θ + 1)
(√ ) √ (√ )
∴ 2 − 1 sec θ = 2 2−1
√ π
sec θ = 2, i.e. θ = 2nπ + .
4
π
The value θ = 2nπ − is inadmissible ; it is introduced by the squar-
4
ing and is the solution of the equation
(√ )
sec θ − 1 = − 2 − 1 tan θ. ■

( )
§ Problem 6.2.12. 3 sec2 θ + tan2 θ = 5. ♢
§§ Solution. ( )
3 sec2 θ + tan2 θ = 5
( )
∴ 3 1 + tan2 θ + tan2 θ = 5
∴ 6 tan2 θ = 2
1
∴ tan2 θ =
3
1
∴ tan θ = ± √
3
π
∴ θ = nπ ± . ■
6
6.2. Trigonometrical Equations 77

§ Problem 6.2.13. cot θ + tan θ = 2 cosec θ. ♢


§§ Solution.
cot θ + tan θ = 2 cosec θ
1 + tan2 θ 2
∴ = (6.7)
tan θ sin θ
∴ sec θ = 2 sec θ
2
(6.8)
sec θ = 2
π
∴ θ = 2nπ ± .
3
The value sec θ = 0 from Eq. (6.8) is inadmissible ; the original equa-
tion is also satisfied by the solution sin θ = 0, i.e. θ = nπ. ■

√ (√ )
§ Problem 6.2.14. 4 cos2 θ + 3=2 3 + 1 cos θ. ♢
§§ Solution. √ (√ )
4 cos2 θ + 3=2 3 + 1 cos θ
Å√ ã √
3+1 3
∴ cos2 θ − cos θ + =0
2 4
( )Å √ ã
1 3
∴ cos θ − cos θ − =0
2 2

1 π 3 π
∴ cos θ = = cos or cos θ = = cos
2 3 2 6
π π
∴ θ = 2nπ ± or 2nπ ± . ■
3 6
§ Problem 6.2.15. 3 sin2 θ − 2 sin θ = 1. ♢
§§ Solution.
3 sin2 θ − 2 sin θ = 1
∴ (3 sin θ + 1) (sin θ − 1) = 0
1
∴ sin θ = − or sin θ = 1. ■
3
1
§ Problem 6.2.16. sin 5θ = √ . ♢
2
§§ Solution.
1 π
sin 5θ = √ = sin
2 4
π
∴ 5θ = nπ + (−1)n
4
nπ π
∴θ= + (−1)n . ■
5 20
§ Problem 6.2.17. sin 9θ = sin θ. ♢
§§ Solution.
sin 9θ = sin θ
∴ 9θ = mπ + (−1)m θ.
When m is even, (= 2n, say),
9θ = 2nπ + θ
∴ 8θ = 2nπ

∴θ= .
4
6.2. Trigonometrical Equations 78

When m is odd, (= 2n + 1, say),


9θ = (2n + 1)π − θ
∴ 10θ = (2n + 1)π
(2n + 1)π
∴θ= . ■
10
§ Problem 6.2.18. sin 3θ = sin 2θ. ♢
§§ Solution.
sin 3θ = sin 2θ
∴ 3θ = mπ + (−1)m 2θ.
When m is even, (= 2n, say),
3θ = 2nπ + 2θ
∴ θ = 2nπ.
When m is odd, (= 2n + 1, say),
3θ = (2n + 1)π − 2θ
∴ 5θ = (2n + 1)π
(2n + 1)π
∴θ= . ■
5
§ Problem 6.2.19. cos mθ = cos nθ. ♢
§§ Solution.
cos mθ = cos nθ
∴ mθ = 2rθ ± nθ, where r is any integer.
Taking the upper sign, we have
(m − n)θ = 2rπ
2rπ
∴θ= .
m−n
Taking the lower sign, we have
(m + n)θ = 2rπ
2rπ
∴θ= . ■
m+n
§ Problem 6.2.20. sin 2θ = cos 3θ. ♢
§§ Solution.
sin 2θ = cos 3θ
Äπ ä
∴ cos 3θ = cos − 2θ
Ä π2 ä
∴ 3θ = 2nπ ± − 2θ .
2
Taking the upper sign, we have
π
5θ = 2nπ +
( )2
1 π
∴ θ = 2n + .
2 5
Taking the lower sign, we have
π
θ = 2nπ − . ■
2
§ Problem 6.2.21. cos 5θ = cos 4θ. ♢
6.2. Trigonometrical Equations 79

§§ Solution.
cos 5θ = cos 4θ
∴ 5θ = 2nπ ± 4θ.
Taking the upper sign, we have
θ = 2nπ.
Taking the lower sign, we have
9θ = 2nπ
2nπ
∴θ= . ■
9
§ Problem 6.2.22. cos mθ = sin nθ. ♢
§§ Solution. Äπ ä
cos mθ = sin nθ = cos − nθ
Äπ ä 2
∴ mθ = 2rπ ± − nθ , where r is any integer.
2
Taking the upper sign, we have
( )
1
(m + n)θ = 2r + π
( ) 2
1 π
∴θ= 2r + .
2 m+n
Taking the lower sign, we have
( )
1
(m − n)θ = 2r − π
( ) 2
1 π
∴θ= 2r − . ■
2 m−n
§ Problem 6.2.23. cot θ = tan 8θ. ♢
§§ Solution.
cot θ = tan 8θ
Äπ ä
∴ tan 8θ = tan −θ
Ä π2 ä
∴ 8θ = nπ + −θ
( 2)
1
∴ 9θ = n + π
( 2)
1 π
∴θ = n+ . ■
2 9
§ Problem 6.2.24. cot θ = tan nθ. ♢
§§ Solution.
cot θ = tan nθ
Äπ ä
∴ tan nθ = tan −θ
Äπ ä 2
∴ nθ = mπ + − θ , where m is any integer
2 ( )
1
∴ (n + 1)θ = m + π
( ) 2
1 π
∴θ = m+ . ■
2 n+1
2
§ Problem 6.2.25. tan 2θ = tan . ♢
θ
6.2. Trigonometrical Equations 80

§§ Solution.
2
tan 2θ = tan
θ
2
∴ 2θ = nπ +
θ
∴ 2θ2 − nπθ − 2 = 0

nπ ± n2 π 2 + 16
∴θ= . ■
4
§ Problem 6.2.26. tan 2θ tan θ = 1. ♢
§§ Solution.
tan 2θ tan θ = 1
1 Äπ ä
∴ tan 2θ = = cot θ = tan −θ
tan θ Äπ ä 2
∴ 2θ = nπ + −θ
( 2)
1
∴ 3θ = n + π
( 2)
1 π
∴θ = n+ . ■
2 3
§ Problem 6.2.27. tan2 3θ = cot2 α. ♢
§§ Solution.
tan2 3θ = cot2 α
Äπ ä
∴ tan2 3θ = tan2 −α
Äπ 2 ä
∴ 3θ = nπ ± −α
(
2 )
π α 1 π α
∴ θ = (2n + 1) ± = n+ ± . ■
6 3 2 3 3
§ Problem 6.2.28. tan 3θ = cot θ. ♢
§§ Solution.
tan 3θ = cot θ
Äπ ä
∴ tan 3θ = tan −θ
Ä π2 ä
∴ 3θ = nπ + −θ
( 2)
1
∴ 4θ = n + π
( 2)
1 π
∴θ = n+ . ■
2 4
§ Problem 6.2.29. tan2 3θ = tan2 α. ♢
§§ Solution.
tan2 3θ = tan2 α
∴ 3θ = nπ ± α
nπ α
∴θ= ± . ■
3 3
§ Problem 6.2.30. 3 tan2 θ = 1. ♢
6.2. Trigonometrical Equations 81

§§ Solution.
3 tan2 θ = 1
1
∴ tan θ = ± √
3
π
∴ θ = nπ ± . ■
6
§ Problem 6.2.31. tan mx + cot nx = 0. ♢
§§ Solution.
tan mx + cot nx = 0
Äπ ä
∴ tan mx = − cot nx = tan + nx
Äπ ä 2
∴ mx = rπ + + nx , where r is any integer
2 ( )
1
∴ (m − n)x = r+ π
( ) 2
1 π
∴x= r+ . ■
2 m−n
§ Problem 6.2.32. tan (π cot θ) = cot (π tan θ) . ♢
§§ Solution.
tan (π cot θ) = cot (π tan θ)
Äπ ä
∴ tan (π cot θ) = tan − π tan θ
2
π
∴ π cot θ = nπ + − π tan θ
2
1
∴ cot θ + tan θ = n +
2
1 1
∴ + tan θ = n +
tan θ 2
∴ 2 tan2 θ − (2n + 1) tan θ + 2 = 0

(2n + 1) ± (2n + 1)2 − 16
∴ tan θ =
1î √ ó
4
= 2n + 1 ± 4n2 + 4n − 15 .
4
3
Since 4n2 + 4n − 15 = (2n − 3)(2n + 5), we see that unless n > or
2
5
< − , 4n2 + 4n − 15 will be negative and therefore tan θ imaginary.
2
Hence in the above value, n is any integer except 1, 0, −1, −2. ■

1 1
§ Problem 6.2.33. sin(θ − ϕ) = and cos(θ + ϕ) = . ♢
2 2
§§ Solution.
1 π 1 π
sin(θ − ϕ) = = sin and cos(θ + ϕ) = = cos
2 6 2 3
π π
∴ θ − ϕ = nπ + (−1)n , and θ + ϕ = 2mπ ± ,
6 3
where n and m are any integers.
Ä nä π π
∴θ = m+ π ± + (−1)n , and
Ä 2n ä 6π 12
π
ϕ= m− π ± − (−1)n . ■
2 6 12
6.2. Trigonometrical Equations 82


1 3
§ Problem 6.2.34. cos(2x + 3y) = and cos(3x + 2y) = . ♢
2 2
§§ Solution. √
1 π 3 π
cos(2x + 3y) = = cos and cos(3x + 2y) = = cos
2 3 2 6
π π
∴ 2x + 3y = 2nπ ± and 3x + 2y = 2mπ ± ,
3 6
where n and m are any integers
2π π
∴ 4x + 6y = 4nπ ± and 9x + 6y = 6mπ ±
3 2
hence, by subtraction,
π 2π
5x = (6m − 4n)π ± ∓
[ 2 3 ]
1 π 2π
∴x= (6m − 4n)π ± ∓ .
5 2 3
1î πó
Similarly, y= (6n − 4m)π ± π ∓ . ■
5 3
§ Problem 6.2.35. Find all the angles between 0◦ and 90◦ which
satisfy the equation
sec2 θ cosec 2 θ + 2 cosec 2 θ = 8. ♢
§§ Solution.
sec2 θ cosec 2 θ + 2 cosec 2 θ = 8
( )( ) ( )
∴ 1 + tan2 θ 1 + cot2 θ + 2 1 + cot2 θ = 8
1 + tan2 θ + cot2 θ + 1 + 2 + 2 cot2 θ = 8
∴ tan2 θ + 3 cot2 θ − 4 = 0
3
∴ tan2 θ + −4=0
tan2 θ
∴ tan4 θ − 4 tan2 θ + 3 = 0
( )( )
∴ tan2 θ − 1 tan2 θ − 3 = 0
∴ tan2 θ − 1 = 0 or tan2 θ − 3 = 0

∴ tan θ = ±1 or ± 3.
The required angles are therefore 45◦ and 60◦ .
Otherwise thus :
sec2 θ cosec 2 θ + 2 cosec 2 θ = 8
1 2
∴ + =8
sin2 θ cos2( θ sin2 θ)
∴ 1 + 2 cos θ = 8 sin θ cos θ = 8 1 − cos θ cos2 θ = 8 cos2 θ − 8 cos4 θ
2 2 2 2

8 cos4 θ − 6 cos2 θ + 1 = 0
( )( )
∴ 4 cos2 θ − 1 2 cos2 θ − 1 = 0
∴ 4 cos2 θ − 1 = 0 or 2 cos2 θ − 1 = 0
1 1
∴ cos θ = ± or ± √ .
2 2
The required angles are therefore 45◦ and 60◦ . ■

5
§ Problem 6.2.36. If tan2 θ = , find versinθ and explain the double
4
result. ♢
6.2. Trigonometrical Equations 83

§§ Solution.
5
If tan2 θ =
4
5 9
then sec2 θ = 1 + =
4 4
2
∴ cos θ = ± , and
3
2 1 5
versinθ = 1 − cos θ = 1 ∓ = or .
√ 3 3 3
5
Since tan θ = ± , there are four angles, one in each quadrant ;
2
1
those in the first and fourth quadrants give versinθ = , and those
3
5
in the second and third quadrants give versinθ = . ■
3

1
§ Problem 6.2.37. If the coversin of an angle be , find its cosine
3
and cotangent. ♢
1 2
§§ Solution. If θ be the angle, we have 1−sin θ = , so that sin θ = .
… … 3 3
√ 4 5 1√
∴ cos θ = 1 − sin θ =
2 1− = =± 5, and
9 9 3
cos θ 1√
cot θ = =± 5. ■
sin θ 2
Chapter
7
Trigonometrical Ratios of The
Sum and Difference of Two Angles

7.1 Addition and Subtraction Theorems


3 9
§ Problem 7.1.1. If sin α = and cos β = , find the values of
5 41
sin (α − β) and cos (α + β). Verify by a graph and accurate measure-
ment. ♢
§§ Solution. … ( )2
√ 3 4
cos α = 1 − sin2 α = 1− = .
5 5
… ( )2 …
√ 9 1600 40
sin β = 1− cos2 β= 1− = = .
41 1681 41
∴ sin(α − β) = sin α cos β − cos α sin β
3 9 4 40 27 − 160 133
= × − × = =− .
5 41 5 41 205 205
Also, cos(α + β) = cos α cos β − sin α sin β
4 9 3 40 36 − 120 84
= × − × = =− . ■
5 41 5 41 205 205
45 33
§ Problem 7.1.2. If sin α = and sin β = , find the values of
53 65
sin (α − β) and sin (α + β). ♢
§§ Solution. … ( )2
√ 45
cos α = 1 − sin2 α = 1−
53
7.1. Addition and Subtraction Theorems 86

… …
(53 + 45)(53 − 45) 784 28
= = =
(53)2 (53)2 53
… ( )2
√ 33
and cos β = 1 − sin2 β = 1−
65
… …
(65 + 33)(65 − 33) 3136 56
= = =
(65)2 (65)2 65
45 56 28 33
∴ sin(α − β) = sin α cos β − cos α sin β = × − ×
53 65 53 65
28(90 − 33) 28 × 3(30 − 11) 84 × 19 1596
= = = = .
53 × 65 53 × 65 53 × 65 3445
Also, sin(α + β) = sin α cos β + cos α sin β
28(90 + 33) 84 × 41 3444
= = = . ■
53 × 65 53 × 65 3445
15 12
§ Problem 7.1.3. If sin α = and cos β = , find the values of
17 13
sin(α + β), cos(α − β) and tan(α + β). Verify by a graph and accurate
measurement. ♢
§§ Solution. … ( )2 …
√ 15 64 8
cos α = 1 − sin2 α = 1− = =
17 (17)2 17
… ( )2 …
√ 12 25 5
and sin β = 1− cos2 β= 1− ==
13 (13)2 13
sin α 15 8 15
∴ tan α = = ÷ =
cos α 17 17 8
sin β 5 12 5
and tan β = = ÷ = .
cos β 13 13 12
∴ sin(α + β) = sin α cos β + cos α sin β
15 12 8 5 20(9 + 2) 220
= × + × = = .
17 13 17 13 17 × 13 221
cos(α − β) = cos α cos β + sin α sin β
8 12 15 5 3(32 + 25) 171
= × + × = = .
17 13 17 13 17 × 13 221
15 5
tan α + tan β + 220
tan(α + β) = = 8 12 = .
1 − tan α tan β 15 5 21
1− × ■
8 12
Prove that
§ Problem 7.1.4.
cos (45◦ − A) cos (45◦ − B) − sin (45◦ − A) sin (45◦ − B)
= sin(A + B). ♢
§§ Solution.
cos (45◦ − A) cos (45◦ − B) − sin (45◦ − A) sin (45◦ − B)
= cos [(45◦ − A) + (45◦ − B)]
= cos [90◦ − (A + B)]
= sin(A + B), by Art. 69. ■
7.1. Addition and Subtraction Theorems 87

§ Problem 7.1.5.
sin (45◦ + A) cos (45◦ − B) + cos (45◦ + A) sin (45◦ − B)
= cos (A − B) . ♢
§§ Solution.
sin (45◦ + A) cos (45◦ − B) + cos (45◦ + A) sin (45◦ − B)
= sin [(45◦ + A) + (45◦ − B)]
= sin [90◦ + (A − B)]
= cos(A − B), by Art. 70. ■

sin(A − B) sin(B − C) sin(C − A)


§ Problem 7.1.6. + + = 0. ♢
cos A cos B cos B cos C cos C cos A
§§ Solution.
sin(A − B) sin A cos B − cos A sin B
= = tan A − tan B
cos A cos B cos A cos B
sin(B − C) sin B cos C − cos B sin C
= = tan B − tan C
cos B cos C cos B cos C
sin(C − A) sin C cos A − cos C sin A
= = tan C − tan A
cos C cos A cos C cos A
sin(A − B) sin(B − C) sin(C − A)
∴ + + = 0. ■
cos A cos B cos B cos C cos C cos A
§ Problem 7.1.7. sin 105◦ + cos 105◦ = cos 45◦ . ♢
§§ Solution.
sin 105◦ + cos 105◦ = sin (180◦ − 105◦ ) − cos (180◦ − 105◦ )
= sin 75◦ − cos 75◦ = sin 75◦ − sin (90◦ − 75◦ )
= sin 75◦ − sin 15◦ = 2 cos 45◦ sin 30◦ [Art. 94.]
◦ 1 ◦
= 2 cos 45 × = cos 45 .
2
Otherwise thus :
sin 105◦ + cos 105◦ = sin 75◦ − cos 75◦
√ √
3+1 3−1
= √ − √
2 2 2 2
2 1
= √ = √ = cos 45◦ . ■
2 2 2
§ Problem 7.1.8. sin 75◦ − sin 15◦ = cos 105◦ + cos 15◦ . ♢
§§ Solution.
sin 75◦ − sin 15◦ = cos (90◦ − 75◦ ) + cos (90◦ + 15◦ ) , by Arts. 69 and 70
= cos 15◦ + cos 105◦ . ■

§ Problem 7.1.9. cos α cos(γ − α) − sin α sin(γ − α) = cos γ. ♢


§§ Solution.
cos α cos(γ − α) − sin α sin(γ − α)
= cos [α + (γ − α)] = cos γ. ■

§ Problem 7.1.10. cos(α + β) cos γ −cos(β + γ) cos α = sin β sin(γ − α).



§§ Solution.
cos(α + β) cos γ − cos(β + γ) cos α
= (cos α cos β − sin α sin β) cos γ
− (cos β cos γ − sin β sin γ) cos α
= sin β (sin γ cos α − cos γ sin α) = sin β sin(γ − α). ■
7.2. Product Formulae 88

§ Problem 7.1.11. sin(n + 1)A sin(n − 1)A+cos(n + 1)A cos(n − 1)A =


cos 2A. ♢
§§ Solution.
sin(n + 1)A sin(n − 1)A + cos(n + 1)A cos(n − 1)A
= cos [(n + 1)A − (n − 1)A] = cos 2A. ■

§ Problem 7.1.12. sin(n + 1)A sin(n + 2)A+cos(n + 1)A cos(n + 2)A =


cos A. ♢
§§ Solution.
sin(n + 1)A sin(n + 2)A + cos(n + 1)A cos(n + 2)A
= cos [(n + 1)A − (n + 2)A]
= cos (−A) = cos A, by Art. 68. ■

7.2 Product Formulae


Prove that
sin 7θ − sin 5θ
§ Problem 7.2.1. = tan θ. ♢
cos 7θ + cos 5θ
§§ Solution.
1 1
sin 7θ − sin 5θ 2 cos (7θ + 5θ) sin (7θ − 5θ)
= 2 2
cos 7θ + cos 5θ 1 1
2 cos (7θ + 5θ) cos (7θ − 5θ)
2 2
sin θ
= = tan θ. ■
cos θ
cos 6θ − cos 4θ
§ Problem 7.2.2. = − tan θ. ♢
sin 6θ + sin 4θ
§§ Solution.
1 1
cos 6θ − cos 4θ −2 sin (6θ + 4θ) sin (6θ − 4θ)
= 2 2
sin 6θ + sin 4θ 1 1
2 sin (6θ + 4θ) cos (6θ − 4θ)
2 2
sin θ
=− = − tan θ. ■
cos θ
sin A + sin 3A
§ Problem 7.2.3. = tan 2A. ♢
cos A + cos 3A
§§ Solution.
1 1
sin A + sin 3A 2 sin (A + 3A) cos (A − 3A)
= 2 2
cos A + cos 3A 1 1
2 cos (A + 3A) cos (A − 3A)
2 2
sin 2A
= = tan 2A. ■
cos 2A
sin 7A − sin A
§ Problem 7.2.4. = cos 4A sec 5A. ♢
sin 8A − sin 2A
7.2. Product Formulae 89

§§ Solution.
1 1
sin 7A − sin A 2 cos (7A + A) sin (7A − A)
= 2 2
sin 8A − sin 2A 1 1
2 cos (8A + 2A) sin (8A − 2A)
2 2
cos 4A sin 3A 1
= = cos 4A ·
cos 5A sin 3A cos 5A
= cos 4A sec 5A. ■

cos 2B + cos 2A
§ Problem 7.2.5. = cot(A + B) cot(A − B). ♢
cos 2B − cos 2A
§§ Solution.
cos 2B + cos 2A cos 2A + cos 2B
=
cos 2B − cos 2A cos 2B − cos 2A
1 1
2 cos (2A + 2B) cos (2A − 2B)
= 2 2
1 1
2 sin (2A + 2B) sin (2A − 2B)
2 2
cos(A + B) cos(A − B)
=
sin(A + B) sin(A − B)
= cot(A + B) cot(A − B). ■

sin 2A + sin 2B tan(A + B)


§ Problem 7.2.6. = . ♢
sin 2A − sin 2B tan(A − B)
§§ Solution.
1 1
sin 2A + sin 2B 2 sin (2A + 2B) cos (2A − 2B)
= 2 2
sin 2A − sin 2B 1 1
2 cos (2A + 2B) sin (2A − 2B)
2 2
sin(A + B) cos(A − B)
=
cos(A + B) sin(A − B)
tan(A + B)
= . ■
tan(A − B)
sin A + sin 2A A
§ Problem 7.2.7. = cot . ♢
cos A − cos 2A 2
§§ Solution.
1 1
sin A + sin 2A 2 sin (A + 2A) cos (A − 2A)
= 2 2
cos A − cos 2A 1 1
2 sin (2A + A) sin (2A − A)
(2 ) 2
A
cos −
2
=
A
sin
2
A
cos
= 2 , by Art. 68
A
sin
2
A
= cot . ■
2
sin 5A − sin 3A
§ Problem 7.2.8. = tan A. ♢
cos 3A + cos 5A
7.2. Product Formulae 90

§§ Solution.
1 1
sin 5A − sin 3A 2 cos (5A + 3A) sin (5A − 3A)
= 2 2
cos 3A + cos 5A 1 1
2 cos (3A + 5A) cos (3A − 5A)
2 2
sin A sin A
= = = tan A. ■
cos(−A) cos A
cos 2B − cos 2A
§ Problem 7.2.9. = tan(A − B). ♢
sin 2B + sin 2A
§§ Solution.
1 1
cos 2B − cos 2A 2 sin (2A + 2B) sin (2A − 2B)
= 2 2
sin 2B + sin 2A 1 1
2 sin (2A + 2B) cos (2A − 2B)
2 2
sin(A − B)
= = tan(A − B). ■
cos(A − B)
§ Problem 7.2.10.
cos(A + B) + sin(A − B) = 2 sin (45◦ + A)
cos (45◦ + B) . ♢
§§ Solution.
cos(A + B) + sin(A − B) = sin [90◦ + (A + B)] + sin(A − B), by Art. 70
1
= 2 sin (90◦ + A + B + A − B)
2
1
cos (90◦ + A + B − A + B)
2
= 2 sin (45◦ + A) cos (45◦ + B) . ■

§ Problem 7.2.11.
cos 3A − cos A cos 2A − cos 4A
+
sin 3A − sin A sin 4A − sin 2A
sin A
= . ♢
cos 2A cos 3A
§§ Solution.
cos 3A − cos A cos 2A − cos 4A
+
sin 3A − sin A sin 4A − sin 2A
1 1 1 1
−2 sin (3A + A) sin (3A − A) 2 sin (4A + 2A) sin (4A − 2A)
= 2 2 + 2 2
1 1 1 1
2 cos (3A + A) sin (3A − A) 2 cos (4A + 2A) sin (4A − 2A)
2 2 2 2
sin 2A sin 3A sin 3A cos 2A − cos 3A sin 2A
=− + =
cos 2A cos 3A cos 2A cos 3A
sin(3A − 2A) sin A
= = . ■
cos 2A cos 3A cos 2A cos 3A
sin(4A − 2B) + sin(4B − 2A)
§ Problem 7.2.12. = tan(A + B). ♢
cos(4A − 2B) + cos(4B − 2A)
§§ Solution.
sin(4A − 2B) + sin(4B − 2A)
cos(4A − 2B) + cos(4B − 2A)
1 1
2 sin (4A − 2B + 4B − 2A) cos (4A − 2B − 4B + 2A)
= 2 2
1 1
2 cos (4A − 2B + 4B − 2A) cos (4A − 2B − 4B + 2A)
2 2
7.2. Product Formulae 91

sin(A + B)
= = tan(A + B). ■
cos(A + B)
tan 5θ + tan 3θ
§ Problem 7.2.13. = 4 cos 2θ cos 4θ. ♢
tan 5θ − tan 3θ
§§ Solution.
sin 5θ sin 3θ
tan 5θ + tan 3θ +
= cos 5θ cos 3θ
tan 5θ − tan 3θ sin 5θ sin 3θ

cos 5θ cos 3θ
sin 5θ cos 3θ + cos 5θ sin 3θ
=
sin 5θ cos 3θ − cos 5θ sin 3θ
sin(5θ + 3θ) sin 8θ
= =
sin(5θ − 3θ) sin 2θ
2 sin 4θ cos 4θ 4 sin 2θ cos 2θ cos 4θ
= =
sin 2θ sin 2θ
= 4 cos 2θ cos 4θ. ■

cos 3θ + 2 cos 5θ + cos 7θ


§ Problem 7.2.14. = cos 2θ − sin 2θ tan 3θ. ♢
cos θ + 2 cos 3θ + cos 5θ
§§ Solution.
cos 3θ + 2 cos 5θ + cos 7θ (cos 7θ + cos 3θ) + 2 cos 5θ
=
cos θ + 2 cos 3θ + cos 5θ (cos 5θ + cos θ) + 2 cos 3θ
1 1
2 cos (7θ + 3θ) cos (7θ − 3θ) + 2 cos 5θ
= 2 2
1 1
2 cos (5θ + θ) cos (5θ − θ) + 2 cos 3θ
2 2
2 cos 5θ cos 2θ + 2 cos 5θ
=
2 cos 3θ cos 2θ + 2 cos 3θ
2 cos 5θ (cos 2θ + 1) cos 5θ cos(3θ + 2θ)
= = =
2 cos 3θ (cos 2θ + 1) cos 3θ cos 3θ
cos 3θ cos 2θ − sin 3θ sin 2θ
=
cos 3θ
= cos 2θ − sin 2θ tan 3θ. ■

sin A + sin 3A + sin 5A + sin 7A


§ Problem 7.2.15. = tan 4A. ♢
cos A + cos 3A + cos 5A + cos 7A
§§ Solution.
sin A + sin 3A + sin 5A + sin 7A
cos A + cos 3A + cos 5A + cos 7A
1 1 1 1
2 sin (A + 3A) cos (A − 3A) + 2 sin (5A + 7A) cos (5A − 7A)
= 2 2 2 2
1 1 1 1
2 cos (A + 3A) cos (A − 3A) + 2 cos (5A + 7A) cos (5A − 7A)
2 2 2 2
2 sin 2A cos(−A) + 2 sin 6A cos(−A)
=
2 cos 2A cos(−A) + 2 cos 6A cos(−A)
sin 2A + sin 6A
=
cos 2A + cos 6A
1 1
2 sin (2A + 6A) cos (2A − 6A) sin 4A
= 2 2 = = tan 4A.
1 1 cos 4A
2 cos (2A + 6A) cos (2A − 6A) ■
2 2
7.2. Product Formulae 92

sin(θ + ϕ) − 2 sin θ + sin(θ − ϕ)


§ Problem 7.2.16. = tan θ. ♢
cos(θ + ϕ) − 2 cos θ + cos(θ − ϕ)
§§ Solution.
sin(θ + ϕ) − 2 sin θ + sin(θ − ϕ)
cos(θ + ϕ) − 2 cos θ + cos(θ − ϕ)
sin(θ + ϕ) + sin(θ − ϕ) − 2 sin θ
=
cos(θ + ϕ) + cos(θ − ϕ) − 2 cos θ
1 1
2 sin [(θ + ϕ) + (θ − ϕ)] cos [(θ + ϕ) − (θ − ϕ)] − 2 sin θ
= 2 2
1 1
2 cos [(θ + ϕ) + (θ − ϕ)] cos [(θ + ϕ) − (θ − ϕ)] − 2 cos θ
2 2
2 sin θ cos ϕ − 2 sin θ
=
2 cos θ cos ϕ − 2 cos θ
2 sin θ (cos ϕ − 1) sin θ
= = = tan θ. ■
2 cos θ (cos ϕ − 1) cos θ
sin A + 2 sin 3A + sin 5A sin 3A
§ Problem 7.2.17. = . ♢
sin 3A + 2 sin 5A + sin 7A sin 5A
§§ Solution.
sin A + 2 sin 3A + sin 5A sin A + sin 5A + 2 sin 3A
=
sin 3A + 2 sin 5A + sin 7A sin 3A + sin 7A + 2 sin 5A
1 1
2 sin (A + 5A) cos (A − 5A) + 2 sin 3A
= 2 2
1 1
2 sin (3A + 7A) cos (3A − 7A) + 2 sin 5A
2 2
2 sin 3A cos(−2A) + 2 sin 3A
=
2 sin 5A cos(−2A) + 2 sin 5A
2 sin 3A (cos 2A + 1) sin 3A
= = . ■
2 sin 5A (cos 2A + 1) sin 5A
sin(A − C) + 2 sin A + sin(A + C) sin A
§ Problem 7.2.18. = . ♢
sin(B − C) + 2 sin B + sin(B + C) sin B
§§ Solution.
sin(A − C) + 2 sin A + sin(A + C)
sin(B − C) + 2 sin B + sin(B + C)
sin(A − C) + sin(A + C) + 2 sin A
=
sin(B − C) + sin(B + C) + 2 sin B
1 1
2 sin [(A − C) + (A + C)] cos [(A − C) − (A + C)] + 2 sin A
= 2 2
1 1
2 sin [(B − C) + (B + C)] cos [(B − C) − (B + C)] + 2 sin B
2 2
2 sin A cos(−C) + 2 sin A 2 sin A (cos C + 1) sin A
= = = . ■
2 sin B cos(−C) + 2 sin B 2 sin B (cos C + 1) sin B
sin A − sin 5A + sin 9A − sin 13A
§ Problem 7.2.19. = cot 4A. ♢
cos A − cos 5A − cos 9A + cos 13A
§§ Solution.
sin A − sin 5A + sin 9A − sin 13A
cos A − cos 5A − cos 9A + cos 13A
(sin A − sin 5A) + (sin 9A − sin 13A)
=
(cos A − cos 5A) − (cos 9A − cos 13A)
7.2. Product Formulae 93

1 1 1 1
2 cos (A + 5A) sin (A − 5A) + 2 cos (9A + 13A) sin (9A − 13A)
= 2 2 2 2
1 1 1 1
2 sin (A + 5A) sin (5A − A) − 2 sin (9A + 13A) sin (13A − 9A)
2 2 2 2
2 cos 3A sin(−2A) + 2 cos 11A sin(−2A)
=
2 sin 3A sin 2A − 2 sin 11A sin 2A
−2 cos 3A sin 2A − 2 cos 11A sin 2A cos 11A + cos 3A
= =
2 sin 3A sin 2A − 2 sin 11A sin 2A sin 11A − sin 3A
1 1
2 cos (11A + 3A) cos (11A − 3A)
= 2 2
1 1
2 cos (11A + 3A) sin (11A − 3A)
2 2
cos 4A
= = cot 4A. ■
sin 4A
sin A + sin B A+B A−B
§ Problem 7.2.20. = tan cot . ♢
sin A − sin B 2 2
§§ Solution.
A+B A−B
sin A + sin B 2 sin cos
= 2 2
sin A − sin B A+B A−B
2 cos sin
2 2
A+B A−B
= tan cot . ■
2 2
cos A + cos B A+B A−B
§ Problem 7.2.21. = cot cot . ♢
cos B − cos A 2 2
§§ Solution.
A+B A−B
cos A + cos B 2 cos cos
= 2 2
cos B − cos A A+B A−B
2 sin sin
2 2
A+B A−B
= cot cot . ■
2 2
sin A + sin B A+B
§ Problem 7.2.22. = tan . ♢
cos A + cos B 2
§§ Solution.
A+B A−B
sin A + sin B 2 sin cos
= 2 2
cos A + cos B A+B A−B
2 cos cos
2 2
A+B
sin A+B
= 2 = tan .
A+B 2
cos ■
2
sin A − sin B A+B
§ Problem 7.2.23. = cot . ♢
cos B − cos A 2
§§ Solution.
A+B A−B A+B
sin A − sin B 2 cos sin cos A+B
= 2 2 = 2 = cot .
cos B − cos A A+B A−B A+B 2
2 sin sin sin ■
2 2 2
7.2. Product Formulae 94

§ Problem 7.2.24.
cos(A + B + C) + cos(−A + B + C) + cos(A − B + C) + cos(A + B − C)
sin(A + B + C) + sin(−A + B + C) − sin(A − B + C) + sin(A + B − C)
= cot B. ♢
§§ Solution.
The numerator of the expression
1
= 2 cos [(A + B + C) + (−A + B + C)]
2
1
× cos [(A + B + C) − (−A + B + C)]
2
1
+ 2 cos [(A − B + C) + (A + B − C)]
2
1
× cos [(A − B + C) − (A + B − C)]
2
= 2 cos(B + C) cos A + 2 cos A cos(C − B)
= 2 cos A [cos(B + C) + cos(C − B)]
= 2 cos A · 2 cos C cos B.
The denominator
1
= 2 sin [(A + B + C) + (−A + B + C)]
2
1
× cos [(A + B + C) − (−A + B + C)]
2
− {sin(A − B + C) − sin(A + B − C)}
1
= 2 sin [(A + B + C) + (−A + B + C)]
2
1
× cos [(A + B + C) − (−A + B + C)]
{ 2
1
− 2 cos [(A − B + C) + (A + B − C)]
2 }
1
× sin [(A − B + C) − (A + B − C)]
2
= 2 sin(B + C) cos A − 2 cos A sin(C − B)
= 2 cos A [sin(B + C) + sin(B − C)]
= 2 cos A · sin B cos C.
4 cos A cos B cos C
Hence the expression = = cot B. ■
4 cos A sin B cos C
§ Problem 7.2.25.
cos 3A + cos 5A + cos 7A + cos 15A
= 4 cos 4A cos 5A cos 6A. ♢
§§ Solution.
cos 3A + cos 5A + cos 7A + cos 15A
1 1 1 1
= 2 cos (3A + 5A) cos (3A − 5A) + 2 cos (7A + 15A) cos (7A − 15A)
2 2 2 2
= 2 cos 4A cos(−A) + 2 cos 11A cos(−4A)
= 2 cos 4A (cos A + cos 11A)
1 1
= 2 cos 4A · 2 cos (A + 11A) cos (A − 11A)
2 2
= 2 cos 4A · 2 cos 6A cos(−5A)
= 4 cos 4A cos 5A cos 6A. ■
7.2. Product Formulae 95

§ Problem 7.2.26.
cos(−A + B + C) + cos(A − B + C) + cos(A + B − C) + cos(A + B + C)
= 4 cos A cos B cos C. ♢
§§ Solution.
The expression
1
= 2 cos [(−A + B + C) + (A − B + C)]
2
1
× cos [(−A + B + C) − (A − B + C)]
2
1
+ 2 cos [(A + B − C) + (A + B + C)]
2
1
× cos [(A + B − C) − (A + B + C)]
2
= 2 cos C cos(B − A) + 2 cos(A + B) cos(−C)
= 2 cos C [cos(A − B) + cos(A + B)]
= 2 cos C · 2 cos A · cos(−B)
= 4 cos A cos B cos C. ■

§ Problem 7.2.27.
sin 50◦ − sin 70◦ + sin 10◦ = 0. ♢
§§ Solution.
1 1
sin 50◦ − sin 70◦ + sin 10◦ = 2 cos (50◦ + 70◦ ) sin (50◦ − 70◦ ) + sin 10◦
2 2
= −2 cos 60◦ sin 10◦ + sin 10◦
1
= − sin 10◦ + sin 10◦ = 0. ∵ cos 60◦ = .
2
Otherwise thus :
1 1
sin 50◦ + sin 10◦ − sin 70◦ = 2 sin (50◦ + 10◦ ) cos (50◦ − 10◦ ) − sin 70◦
2 2
= 2 sin 30◦ cos 20◦ − sin 70◦
1
= cos 20◦ − cos (90◦ − 70◦ ) , ∵ sin 30◦ =
2
= cos 20◦ − cos 20◦ = 0.
Otherwise thus :
1 1
sin 10◦ − sin 70◦ + sin 50◦ = 2 cos (10◦ + 70◦ ) sin (10◦ − 70◦ ) + sin 50◦
2 2
= −2 cos 40◦ sin 30◦ + sin 50◦
= − cos 40◦ + cos (90◦ − 50◦ )
= − cos 40◦ + cos 40◦ = 0. ■

§ Problem 7.2.28. sin 10◦ +sin 20◦ +sin 40◦ +sin 50◦ = sin 70◦ +sin 80◦ .

§§ Solution.
1 1
sin 10◦ + sin 50◦ = 2 sin (10◦ + 50◦ ) cos (10◦ − 50◦ )
2 2
= 2 sin 30◦ cos(−20◦ ) = cos 20◦
1 1
sin 20◦ + sin 40◦ = 2 sin (20◦ + 40◦ ) cos (20◦ − 40◦ )
2 2
= 2 sin 30◦ cos(−10◦ ) = cos 10◦
and cos 20◦ + cos 10◦ = sin (90◦ − 20◦ ) + cos (90◦ − 10◦ )
= sin 70◦ + sin 80◦ . ■
7.3. Converse Formulae 96

§ Problem 7.2.29.
sin α + sin 2α + sin 4α + sin 5α
α 3α
= 4 cos cos sin 3α. ♢
2 2
§§ Solution.
sin α + sin 2α + sin 4α + sin 5α
3α α 9α α
= 2 sin cos + 2 sin cos
2( 2 2 ) 2
α 3α 9α
= 2 cos sin + sin
2 2 2
α 3α
= 2 cos · 2 sin 3α cos
2 2
α 3α
= 4 cos cos sin 3α. ■
2 2
Simplify
{ ( ) } { ( ) }
3 3
§ Problem 7.2.30. cos θ+ n− ϕ − cos θ+ n+ ϕ . ♢
2 2
§§ Solution. The expression
[ ( ) { ( ) }]
1 3 3
= 2 sin θ+ n+ ϕ+ θ+ n− ϕ
2[ ( )
2 { ( )
2 }]
1 3 3
× sin θ+ n+ ϕ− θ+ n− ϕ
2 2 2
3
= 2 sin (θ + nϕ) sin ϕ. ■

{ (2 ) } { ( ) }
1 1
§ Problem 7.2.31. sin θ + n − ϕ + sin θ + n + ϕ . ♢
2 2
§§ Solution. The expression
[ ( ) { ( ) }]
1 1 1
= 2 sin θ+ n− ϕ+ θ+ n+ ϕ
2[ ( 2) { ( 2) }]
1 1 1
× cos θ+ n− ϕ− θ+ n+ ϕ
2 2( ) 2
ϕ
= 2 sin (θ + nϕ) cos −
2
ϕ
= 2 sin (θ + nϕ) cos . ■
2

7.3 Converse Formulae


Express as a sum or difference the following :
§ Problem 7.3.1. 2 sin 5θ sin 7θ. ♢
§§ Solution.
2 sin 5θ sin 7θ = cos(5θ − 7θ) − cos(5θ + 7θ)
= cos(−2θ) − cos 12θ
= cos 2θ − cos 12θ, by Art. 68. ■

§ Problem 7.3.2. 2 cos 7θ sin 5θ. ♢


§§ Solution.
2 cos 7θ sin 5θ = sin(7θ + 5θ) − sin(7θ − 5θ)
= sin 12θ − sin 2θ. ■
7.3. Converse Formulae 97

§ Problem 7.3.3. 2 cos 11θ cos 3θ. ♢


§§ Solution.
2 cos 11θ cos 3θ = cos(11θ + 3θ) + cos(11θ − 3θ)
= cos 14θ + cos 8θ. ■

§ Problem 7.3.4. 2 sin 54◦ sin 66◦ . ♢


§§ Solution.
2 sin 54◦ sin 66◦ = cos (54◦ − 66◦ ) − cos (54◦ + 66◦ )
= cos (−12◦ ) − cos 120◦ = cos 12◦ − cos 120◦ . ■

Prove that
θ 7θ 3θ 11θ
§ Problem 7.3.5. sin sin + sin sin = sin 2θ sin 5θ. ♢
2 2 2 2
§§ Solution.
θ 7θ 3θ 11θ
sin sin + sin sin
2 2 [ ( 2 )
2 ( )]
1 θ 7θ θ 7θ
= cos − − cos +
2[ (2 2 ) 2( 2 )]
1 3θ 11θ 3θ 11θ
+ cos − − cos +
2 2 2 2 2
1
= [cos(−3θ) − cos 4θ + cos(−4θ) − cos 7θ]
2
1
= (cos 3θ − cos 4θ + cos 4θ − cos 7θ)
2
1
= (cos 3θ − cos 7θ)
2
1 7θ + 3θ 7θ − 3θ
= × 2 sin sin
2 2 2
= sin 5θ sin 2θ. ■

θ 9θ 5θ
§ Problem 7.3.6. cos 2θ cos − cos 3θ cos = sin 5θ sin . ♢
2 2 2
§§ Solution.
θ 9θ
cos 2θ cos − cos 3θ cos
2 [ ( 2 ) ( )]
1 θ θ
= cos 2θ + + cos 2θ −
[2 ( 2) ( 2 )]
9θ 9θ
− cos 3θ + + cos 3θ −
[ 2 2 ( )]
1 5θ 3θ 15θ 3θ
= cos + cos − cos − cos −
2( 2 2 2 )2
1 5θ 3θ 15θ 3θ
= cos + cos − cos − cos
2( 2 2 ) 2 2
1 5θ 15θ
= cos − cos
2 2 2
1 15θ + 5θ 15θ − 5θ
= × 2 sin sin
2 4 4

= sin 5θ sin . ■
2
§ Problem 7.3.7.
sin A sin(A + 2B) − sin B sin(B + 2A)
= sin(A − B) sin(A + B). ♢
7.3. Converse Formulae 98

§§ Solution.
sin A sin(A + 2B) − sin B sin(B + 2A)
1
= [cos(A − A − 2B) − cos(A + A + 2B)]
2
1
− [cos(B − B − 2A) − cos(B + B + 2A)]
2
1
= [cos(−2B) − cos 2(A + B) − cos(−2A) − cos 2(B + A)]
2
1 1 2A + 2B 2A − 2B
= (cos 2B − cos 2A) = × 2 sin sin
2 2 2 2
= sin(A + B) sin(A − B).
Otherwise thus :

sin A sin(A + 2B) = sin A sin [(A + B) + B]


= sin A [sin(A + B) cos B + cos(A + B) sin B]
sin B sin(B + 2A) = sin B sin [(B + A) + A]
= sin B [sin(B + A) cos A + cos(B + A) sin A]
∴ sin A sin(A + 2B) − sin B sin(B + 2A)
= sin(A + B) (sin A cos B − cos A sin B)
= sin(A + B) sin(A − B). ■

§ Problem 7.3.8. (sin 3A + sin A) sin A + (cos 3A − cos A) cos A = 0. ♢


§§ Solution.
(sin 3A + sin A) sin A + (cos 3A − cos A) cos A
( )
3A + A 3A − A
= 2 sin cos sin A
( 2 2 )
A + 3A A − 3A
+ 2 sin sin cos A
2 2
= 2 sin 2A cos A sin A − 2 sin 2A sin A cos A = 0. ■

2 sin(A − C) cos C − sin(A − 2C) sin A


§ Problem 7.3.9. = . ♢
2 sin(B − C) cos C − sin(B − 2C) sin B
§§ Solution.
2 sin(A − C) cos C − sin(A − 2C)
2 sin(B − C) cos C − sin(B − 2C)
sin(A − C + C) + sin(A − C − C) − sin(A − 2C)
=
sin(B − C + C) + sin(B − C − C) − sin(B − 2C)
sin A + sin(A − 2C) − sin(A − 2C) sin A
= = . ■
sin B + sin(B − 2C) − sin(B − 2C) sin B
§ Problem 7.3.10.
sin A sin 2A + sin 3A sin 6A + sin 4A sin 13A
sin A cos 2A + sin 3A cos 6A + sin 4A cos 13A
= tan 9A. ♢
§§ Solution. The expression
1 1 1
(cos A − cos 3A) + (cos 3A − cos 9A) + (cos 9A − cos 17A)
= 2 2 2
1 1 1
(sin 3A − sin A) + (sin 9A − sin 3A) + (sin 17A − sin 9A)
2 2 2
cos A − cos 17A 2 sin 9A sin 8A
= = = tan 9A. ■
sin 17A − sin A 2 cos 9A sin 8A
7.3. Converse Formulae 99

§ Problem 7.3.11.
cos 2A cos 3A − cos 2A cos 7A + cos A cos 10A
sin 4A sin 3A − sin 2A sin 5A + sin 4A sin 7A
= cot 6A cot 5A. ♢
§§ Solution. The expression
1 1 1
(cos A + cos 5A) − (cos 5A + cos 9A) + (cos 9A + cos 11A)
= 2 2 2
1 1 1
(cos A − cos 7A) − (cos 3A − cos 7A) + (cos 3A − cos 11A)
2 2 2
cos A + cos 11A 2 cos 6A cos 5A
= =
cos A − cos 11A 2 sin 6A sin 5A
= cot 6A cot 5A. ■

§ Problem 7.3.12.
cos (36◦ − A) cos (36◦ + A) + cos (54◦ + A) cos (54◦ − A)
= cos 2A. ♢
§§ Solution. We have
cos (36◦ − A) = sin [90◦ − (36◦ − A)] = sin (54◦ + A)
and cos (36◦ + A) = sin [90◦ − (36◦ + A)] = sin (54◦ − A) .
Hence the expression
= sin (54◦ + A) sin (54◦ − A) + cos (54◦ + A) cos (54◦ − A)
= cos [(54◦ + A) − (54◦ − A)] = cos 2A. ■

§ Problem 7.3.13.
cos A sin(B − C) + cos B sin(C − A)
+ cos C sin(A − B) = 0. ♢
§§ Solution. The expression
1
= [sin(A + B − C) − sin(A − B + C)]
2
1
+ [sin(B + C − A) − sin(B − C + A)]
2
1
+ [sin(C + A − B) − sin(C − A + B)] = 0. ■
2
1
§ Problem 7.3.14. sin (45◦ + A) sin (45◦ − A) = cos 2A. ♢
2
§§ Solution. sin (45◦ + A) sin (45◦ − A)
1
= [cos (45◦ + A − 45◦ + A) − cos (45◦ + A + 45◦ − A)]
2
1 1
= (cos 2A − cos 90◦ ) = cos 2A ∵ cos 90◦ = 0. ■
2 2
§ Problem 7.3.15. versin(A + B)versin(A − B) = (cos A − cos B)2 . ♢
§§ Solution.
versin(A + B)versin(A − B)
= [1 − cos(A + B)] [1 − cos(A − B)]
= 1 − [cos(A + B) + cos(A − B)] + cos(A + B) cos(A − B)
= 1 − 2 cos A cos B + cos2 A − sin2 B [Ex. 2, Art. 93]
( )
= 1 − 2 cos A cos B + cos2 A − 1 − cos2 B
= cos2 A − 2 cos A cos B + cos2 B = (cos A − cos B)2 . ■
7.4. Tangent of The Sum of Two Angles 100

§ Problem 7.3.16.
sin (β − γ) cos (α − δ) + sin (γ − α) cos (β − δ)
+ sin (α − β) cos (γ − δ) = 0. ♢
§§ Solution. The expression
1
= [sin (β − γ + α − δ) + sin (β − γ − α + δ)]
2
1
+ [sin (γ − α + β − δ) + sin (γ − α − β + δ)]
2
1
+ [sin (α − β + γ − δ) + sin (α − β − γ + δ)]
2
1
= [sin (β − γ + α − δ) + sin (β − γ − α + δ)
2
+ sin (γ − α + β − δ) − sin (−γ + α + β − δ)
− sin (−α + β − γ + δ) − sin (−α + β + γ − δ)] by Art. 68
= 0. ■

π 9π 3π 5π
§ Problem 7.3.17. 2 cos cos + cos + cos = 0. ♢
13 13 13 13
§§ Solution.
π 9π 3π 5π
2 cos cos + cos + cos
13 13 13 13
π 9π π 4π
= 2 cos cos + 2 cos cos
13 ( 13 13 ) 13
π 9π 4π
= 2 cos cos + cos = 0.
(
13 )
13 13
9π 9π 4π
∵ cos = − cos π − = − cos .
13 13 13
Otherwise thus :
π 9π 3π 5π
2 cos cos + cos + cos
13 13 13 13
8π 10π 3π 5π
= cos + cos + cos + cos = 0.
13( )
13 13 13
8π 8π 5π
∵ cos = − cos π − = − cos
13 ( 13 ) 13
10π 10π 3π
and cos = − cos π − = − cos . ■
13 13 13

7.4 Tangent of The Sum of Two Angles


1 1
§ Problem 7.4.1. It tan A = and tan B = , find the values of
2 3
tan(2A + B)) and tan(2A − B). Verify by a graph and accurate mea-
surement. ♢
§§ Solution.
1
2 tan A 2× 4
tan 2A =
1 − tan2 A
= ( 2)2 = 3 .
1
1−
2
4 1
tan 2A + tan B + 5 5
∴ tan(2A + B) = = 3 3 = ÷ = 3.
1 − tan 2A tan B 4 1 3 9
1− ×
3 3
7.4. Tangent of The Sum of Two Angles 101

4 1
tan 2A − tan B − 13 9
Also tan(2A − B) = = 3 3 =1÷ = .
1 + tan 2A tan B 4 1 9 13 ■
1+ ×
√ 3 3 √
3 3
§ Problem 7.4.2. If tan A = √ and tan B = √ , prove that
4− 3 4+ 3
tan(A − B) = ·375. ♢
§§ Solution.
tan A − tan B
tan(A − B) =
1 + tan
√ A tan B√
3 3
√ − √
4− 3 4+ 3
= √ √
3 3
1+ √ × √
√4 − 3 4
√ + 3
4 3+3−4 3+3 6 6 3
= ( √ )( √ ) = = = = ·375. ■
4− 3 4+ 3 +3 13 + 3 16 8
n 1
§ Problem 7.4.3. If tan A = and tan B = , find tan(A + B).
n+1 2n + 1

§§ Solution.
tan A + tan B
tan(A + B) =
1 − tan A tan B
n 1
+
n+1 2n + 1
=
n 1
1− ×
n+1 2n + 1
2n2 + n + n + 1 2n2 + 2n + 1
= = = 1. ■
(n + 1)(2n + 1) − n 2n2 + 2n + 1
5 1 π
§ Problem 7.4.4. If tan α = and tan β = , prove that α + β = .
6 11 4
Verify by a graph and accurate measurement. ♢
§§ Solution.
tan α + tan β
tan(α + β) =
1 − tan α tan β
5 1
+ 55 + 6 61 π
= 6 11 = = = 1 = tan .
5 1 66 − 5 61 4
1− ×
6 11
π
Hence one of the values of α + β is . ■
4

Prove that ( )
Äπ ä 3π
§ Problem 7.4.5. tan + θ × tan +θ = −1. ♢
4 4
§§ Solution.
π
Äπ ä tan
+ tan θ 1 + tan θ
tan +θ = 4 =
4 π 1 − tan θ
1 − tan tan θ
4
7.4. Tangent of The Sum of Two Angles 102


( ) tan + tan θ
3π 4 −1 + tan θ
and tan +θ = =
4 3π 1 + tan θ
1 − tan tan θ
Äπ ä ( ) 4
3π 1 + tan θ −(1 − tan θ)
∴ tan + θ × tan +θ = × = −1. ■
4 4 1 − tan θ 1 + tan θ
Äπ ä Äπ ä
§ Problem 7.4.6. cot + θ cot − θ = 1. ♢
4 4
§§ Solution.
π
Äπ ä cot cot θ − 1 cot θ − 1
cot +θ = 4 =
4 π 1 + cot θ
cot + cot θ
4
π
Äπ ä cot cot θ + 1 cot θ + 1
and cot −θ = 4
4 π = cot θ − 1
cot θ − cot
Äπ ä Ä π 4ä
∴ cot + θ cot − θ = 1. ■
4 4
A A
§ Problem 7.4.7. 1 + tan A tan = tan A cot − 1 = sec A. ♢
2 2
§§ Solution.
A
A sin A sin 2
1 + tan A tan =1+ ·
2 cos A A
cos
2
A A
cos A cos + sin A sin
= 2 2
A
cos A cos
( ) 2
A A
cos A − cos
2 2
= =
A A
cos A cos cos A cos
2 2
1
= = sec A.
cos A
Again,
A
A sin A cos 2
tan A cot −1= · −1
2 cos A A
sin
2
A A
sin A cos − cos A sin
= 2 2
A
cos A sin
( ) 2
A
sin A −
2
=
A
cos A sin
2
A
sin 1
= 2 = = sec A.
A cos A ■
cos A sin
2
Chapter
8
The Trigonometrical Ratios of
Multiple and Submultiple Angles

8.1 Multiple Angles


§ Problem 8.1.1. Find the value of sin 2α when
3
(1) cos α =
5
12
(2) sin α = , and
13
16
(3) tan α = .
63 ♢
§§ Solution. (1) … …
√ 9 16 4
sin α = 1 − cos2 α = 1−= =±
25 25 5
4 3 24
∴ sin 2α = 2 sin α cos α = ±2 × × = ± .
5 5 25
(2) … …
√ 144 25 5
cos α = 1− α= 1−
sin2 = =±
169 169 13
12 5 120
∴ sin 2α = ±2 × × =± .
13 13 169
8.1. Multiple Angles 104

(3)
16
2 tan α 2×
sin 2α =
1 + tan2 α
[Art. 109] = ( 63)2
16
1+
[ ] 63
32 256 32 3969 2016
= ÷ 1+ = × = . ■
63 3969 63 4225 4225
§ Problem 8.1.2. Find the value of cos 2α when
15
(1) cos α =
17
4
(2) sin α = , and
5
5
(3) tan α = .
12
Verify by a graph and accurate measurement. ♢
§§ Solution. (1)
225 450 − 289 161
cos 2α = 2 cos2 α − 1 = 2 × −1= =
289 289 289
(2)
16 25 − 32 7
cos 2α = 1 − 2 sin2 α = 1 − 2 × = =−
25 25 25
(3)
1 − tan2 α
cos 2α = [Art. 109]
1 + tan2 α
25
1−
= 144 = 144 − 25 = 119 .
25 144 + 25 169
1+ ■
144
b
§ Problem 8.1.3. If tan θ = , find the value of a cos 2θ + b sin 2θ. ♢
a
§§ Solution.
1 − tan2 θ 2 tan θ
a cos 2θ + b sin 2θ = a × +b×
1 + tan2 θ 1 + tan2 θ
a2 − b2 2ab2
=a× 2 2
+b× 2 2
Å a2 + b2 ãa +Åb 2 2 ã
a − b + 2b 2 a +b
=a =a = a.
a2 + b2 a2 + b2
Otherwise thus : ( )
b
a cos 2θ + b sin 2θ = a cos 2θ + sin 2θ
a
= a (cos 2θ + tan θ sin 2θ)
a
= (cos 2θ cos θ + sin θ sin 2θ)
cos θ
a a
= cos(2θ − θ) = × cos θ = a. ■
cos θ cos θ
Prove that
sin 2A
§ Problem 8.1.4. = tan A. ♢
1 + cos 2A
8.1. Multiple Angles 105

sin 2A 2 sin A cos A sin A


§§ Solution. = = = tan A. ■
1 + cos 2A 2 cos2 A cos A

sin 2A
§ Problem 8.1.5. = cot A. ♢
1 − cos 2A
sin 2A 2 sin A cos A cos A
§§ Solution. = = = cot A. ■
1 − cos 2A 2 sin2 A sin A

1 − cos 2A
§ Problem 8.1.6. = tan2 A. ♢
1 + cos 2A
1 − cos 2A 2 sin2 A
§§ Solution. = = tan2 A. ■
1 + cos 2A 2 cos2 A

§ Problem 8.1.7. tan A + cot A = 2 cosec 2A. ♢


§§ Solution.
sin A cos A sin2 A + cos2 A
tan A + cot A = + =
cos A sin A sin A cos A
2 2
= = = 2 cosec 2A. ■
2 sin A cos A sin 2A
§ Problem 8.1.8. tan A − cot A = −2 cot 2A. ♢
§§ Solution.
sin A cos A sin2 A − cos2 A
tan A − cot A = − =
cos (
A sin A )sin A cos A
−2 cos2 A − sin2 A −2 cos 2A
= = = −2 cot 2A. ■
2 sin A cos A sin 2A
§ Problem 8.1.9. cosec 2A + cot 2A = cot A. ♢
1 + cos 2A 2 cos2 A
§§ Solution. cosec 2A + cot 2A = = = cot A.
sin 2A 2 sin A cos A

1 − cos A + cos B − cos(A + B) A B


§ Problem 8.1.10. = tan cot . ♢
1 + cos A − cos B − cos(A + B) 2 2
§§ Solution.
1 − cos A + cos B − cos(A + B)
1 + cos A − cos B − cos(A + B)
1 − cos(A + B) − (cos A − cos B)
=
1 − cos(A + B) + (cos A − cos B)
A+B A+B A−B
2 sin2 + 2 sin sin
= 2 2 2
A+B A+B A−B
2 sin 2 − 2 sin sin
2 [ 2 2 ]
A+B A+B A−B
2 sin sin + sin
2 2 2
= [ ]
A+B A+B A−B
2 sin sin − sin
2 2 2
A+B A−B
sin + sin
= 2 2
A+B A−B
sin − sin
2 2
8.1. Multiple Angles 106

A B
2 sin cos
= 2 2 = tan A cot B .
A B 2 2
2 cos sin ■
2 2
( )
cos A A
§ Problem 8.1.11. = tan 45◦ ± . ♢
1 ∓ sin A 2
§§ Solution.
A A
cos A cos2 − sin2
= 2 2
1 ∓ sin A A A A A
cos2 + sin2 ∓ 2 sin cos
( 2 2) ( 2 2 )
A A A A
cos − sin cos + sin
2 2 2 2
= ( )
A A 2
cos ∓ sin
2 2
A A
cos ± sin
= 2 2 ,
A A
cos ∓ sin
2 2
the upper or lower signs to be taken together,
A
1 ± tan ( )
= 2 = tan 45◦ ± A .
A 2
1 ∓ tan ■
2
sec 8A − 1 tan 8A
§ Problem 8.1.12. = . ♢
sec 4A − 1 tan 2A
§§ Solution.
1
sec 8A − 1 −1 cos 4A (1 − cos 8A)
= cos 8A =
sec 4A − 1 1 cos 8A (1 − cos 4A)
−1
cos 4A
cos 4A 2 sin2 4A 2 sin 4A cos 4A sin 4A
= · = ·
cos 8A 2 sin2 2A cos 8A 2 sin2 2A
sin 8A 2 sin 2A cos 2A sin 8A cos 2A
= · = ·
cos 8A 2 sin2 2A cos 8A sin 2A
tan 8A ■
= tan 8A cot 2A = .
tan 2A

1 + tan (45 − A)
2
§ Problem 8.1.13. = cosec 2A. ♢
1 − tan2 (45◦ − A)
§§ Solution.
sin2 (45◦ − A)
◦ 1+
1 + tan (45 − A)
2 cos2 (45◦ − A)
=

1 − tan (45 − A)
2 sin2 (45◦ − A)
1−
cos2 (45◦ − A)
cos2 (45◦ − A) + sin2 (45◦ − A)
=
cos2 (45◦ − A) − sin2 (45◦ − A)
1
=
cos 2 (45◦ − A)
1 1
= = = cosec 2A. ■
cos (90◦ − 2A) sin 2A
8.1. Multiple Angles 107

α+β
sin α + sin β tan
§ Problem 8.1.14. = 2 . ♢
sin α − sin β α−β
tan
2
§§ Solution.
α+β α−β
sin α + sin β 2 sin cos
= 2 2
sin α − sin β α+β α−β
2 cos sin
2 2
α+β
α+β α−β tan
= tan cot = 2 .
2 2 α−β ■
tan
2
sin A − sin B
2 2
§ Problem 8.1.15. = tan(A + B). ♢
sin A cos A − sin B cos B
§§ Solution.
sin2 A − sin2 B 2 sin2 A − 2 sin2 B
=
sin A cos A − sin B cos B 2 sin A cos A − 2 sin B cos B
1 − cos 2A − (1 − cos 2B) cos 2B − cos 2A
= =
sin 2A − sin 2B sin 2A − sin 2B
1 1
2 sin (2A + 2B) sin (2A − 2B)
= 2 2
1 1
2 cos (2A + 2B) sin (2A − 2B)
2 2
sin(A + B)
= = tan(A + B). ■
cos(A + B)
Äπ ä Äπ ä
§ Problem 8.1.16. tan + θ − tan − θ = 2 tan 2θ. ♢
4 4
§§ Solution. Ä ä Äπ ä
π
tan + θ − tan −θ
4 4
1 + tan θ 1 − tan θ
= − , by Art. 100,
1 − tan θ 1 + tan θ
(1 + tan θ) − (1 − tan θ)2
2
=
1 − tan2 θ
2 × 2 tan θ
= = 2 tan 2θ, by Art. 105. ■
1 − tan2 θ
cos A + sin A cos A − sin A
§ Problem 8.1.17. − = 2 tan 2A. ♢
cos A − sin A cos A + sin A
§§ Solution.
cos A + sin A cos A − sin A

cos A − sin A cos A + sin A
(cos A + sin A)2 − (cos A − sin A)2
=
cos2 A − sin2 A
4 cos A sin A 2 sin 2A
= = = 2 tan 2A. ■
cos 2A cos 2A
4 cos 2A
§ Problem 8.1.18. cot (A + 15◦ ) − tan (A − 15◦ ) = . ♢
1 + 2 sin 2A
8.1. Multiple Angles 108

§§ Solution.
cot (A + 15◦ ) − tan (A − 15◦ )
cos (A + 15◦ ) sin (A − 15◦ )
= −
sin (A + 15◦ ) cos (A − 15◦ )
cos (A + 15◦ ) cos (A − 15◦ ) − sin (A + 15◦ ) sin (A − 15◦ )
=
sin (A + 15◦ ) cos (A − 15◦ )
cos [(A + 15◦ ) + (A − 15◦ )] 2 cos 2A
= =
1 sin 2A + sin 30◦
(sin 2A + sin 30◦ )
2
2 cos 2A 4 cos 2A
= = .
1 1 + 2 sin 2A ■
sin 2A +
2
sin θ + sin 2θ
§ Problem 8.1.19. = tan θ. ♢
1 + cos θ + cos 2θ
§§ Solution.
sin θ + sin 2θ sin θ + 2 sin θ cos θ
=
1 + cos θ + cos 2θ 1 + cos θ + 2 cos2 θ − 1
sin θ (1 + 2 cos θ) sin θ
= = = tan θ. ■
cos θ (1 + 2 cos θ) cos θ
1 + sin θ − cos θ θ
§ Problem 8.1.20. = tan . ♢
1 + sin θ + cos θ 2
§§ Solution.
1 + sin θ − cos θ (1 − cos θ) + sin θ
=
1 + sin θ + cos θ (1 + cos θ) + sin θ
θ θ θ
2 sin2 + 2 sin cos
= 2 2 2
θ θ θ
2 cos2 + 2 sin cos
(2 2 )2
θ θ θ θ
sin sin + cos sin
2 2 2 θ
= ( )= 2 = tan .
θ θ θ θ 2
cos cos + sin cos ■
2 2 2 2
§ Problem 8.1.21.
sin(n + 1)A − sin(n − 1)A
cos(n + 1)A + 2 cos nA + cos(n − 1)A
A
= tan . ♢
§§ Solution. 2
sin(n + 1)A − sin(n − 1)A
cos(n + 1)A + 2 cos nA + cos(n − 1)A
1 1
2 cos (nA + A + nA − A) sin (nA + A − nA + A)
= 2 2
1 1
2 cos (nA + A + nA − A) cos (nA + A − nA + A) + 2 cos nA
2 2
2 cos nA · sin A
=
2 cos nA cos A + 2 cos nA
sin A A ■
= = tan [by XV II. 4].
cos A + 1 2
8.1. Multiple Angles 109

§ Problem 8.1.22.
sin(n + 1)A + 2 sin nA + sin(n − 1)A
cos(n − 1)A − cos(n + 1)A
A
= cot . ♢
§§ Solution. 2
sin(n + 1)A + 2 sin nA + sin(n − 1)A
cos(n − 1)A − cos(n + 1)A
1 1
2 sin (nA + A + nA − A) cos (nA + A − nA + A) + 2 sin nA
= 2 2
1 1
2 sin (nA + A + nA − A) sin (nA + A − nA + A)
2 2
2 sin nA cos A + 2 sin nA
=
2 sin nA sin A
cos A + 1 A ■
= = cot [by XV II. 4].
sin A 2
§ Problem 8.1.23. sin(2n + 1)A sin A = sin2 (n + 1)A − sin2 nA. ♢
§§ Solution.
1
sin(2n + 1)A sin A = [cos(2nA + A − A) − cos(2nA + A + A)]
2
1
= [cos 2nA − cos 2(n + 1)A]
2
1[ { }]
= 1 − 2 sin2 nA − 1 − 2 sin2 (n + 1)A
2
1[ ]
= 2 sin2 (n + 1)A − 2 sin2 nA
2
= sin2 (n + 1)A − sin2 nA. ■

sin(A + 3B) + sin(3A + B)


§ Problem 8.1.24. = 2 cos(A + B). ♢
sin 2A + sin 2B
§§ Solution.
1 1
sin(A + 3B) + sin(3A + B) 2 sin (4A + 4B) cos (2A − 2B)
= 2 2
sin 2A + sin 2B 1 1
2 sin (2A + 2B) cos (2A − 2B)
2 2
2 sin 2(A + B) cos(A − B)
=
2 sin(A + B) cos(A − B)
sin 2(A + B)
=
sin(A + B)
2 sin(A + B) cos(A + B)
=
sin(A + B)
= 2 cos(A + B). ■

A 3A
§ Problem 8.1.25. sin 3A + sin 2A − sin A = 4 sin A cos cos . ♢
2 2
§§ Solution.
sin 3A + sin 2A − sin A = (sin 3A − sin A) + sin 2A
1 1
= 2 cos (3A + A) sin (3A − A) + sin 2A
2 2
= 2 cos 2A sin A + 2 sin A cos A
= 2 sin A (cos 2A + cos A)
3A A
= 2 sin A · 2 cos cos
2 2
8.1. Multiple Angles 110

A 3A
= 4 sin A coscos . ■
2 2

§ Problem 8.1.26. tan 2A = (sec 2A + 1) sec2 A − 1. ♢
§§ Solution.
sin 2A 2 sin A cos A
tan 2A = =
cos 2A cos 2A ( )
2 cos2 A sin A 2 1 − sin2 A
= · = · tan A
cos 2A cos A cos 2A
1 − 2 sin A + 1
2 cos 2A + 1
= · tan A = · tan A
cos 2A √ cos 2A
= (sec 2A + 1) sec2 A − 1. ■

( )
§ Problem 8.1.27. cos3 2θ + 3 cos 2θ = 4 cos6 θ − sin6 θ . ♢
§§ Solution. ( )
cos3 2θ + 3 cos 2θ = cos 2θ cos2 2θ + 3
( )
= cos 2θ 4 − sin2 2θ
( )
= 4 cos 2θ 1 − sin2 θ cos2 θ
î( )2 ó
= 4 cos 2θ cos2 θ + sin2 θ − sin2 θ cos2 θ
( )( 4 )
= 4 cos2 θ − sin2 θ cos θ + cos2 θ sin2 θ + sin4 θ
( )
= 4 cos6 θ − sin6 θ . ■

( )
§ Problem 8.1.28. 1 + cos2 2θ = 2 cos4 θ + sin4 θ . ♢
§§ Solution.
( )2 ( )2
1 + cos2 2θ = cos2 θ + sin2 θ + cos2 θ − sin2 θ
= cos4 θ + 2 cos2 θ sin2 θ + sin4 θ
+ cos4 θ − 2 cos2 θ sin2 θ + sin4 θ
( )
= 2 cos4 θ + sin4 θ . ■

§ Problem 8.1.29. sec2 A (1 + sec 2A) = 2 sec 2A. ♢


§§ Solution. ( )
1 1
sec2 A (1 + sec 2A) = 2
1+
cos A ( cos 2A )
1 cos 2A + 1
=
cos2 A cos 2A
1 2 cos2 A
= ·
cos2 A cos 2A
2
= = 2 sec 2A. ■
cos 2A
§ Problem 8.1.30. cosec A − 2 cot 2A cos A = 2 sin A. ♢
§§ Solution.
1 2 cos 2A cos A
cosec A − 2 cot 2A cos A = −
sin A sin 2A
1 2 cos 2A cos A 1 cos 2A
= − = −
sin A 2 sin A cos A sin A sin A
1 − cos 2A 2 sin2 A
= = = 2 sin A. ■
sin A sin A
8.1. Multiple Angles 111

( )
1 A A
§ Problem 8.1.31. cot A = cot − tan . ♢
2 2 2
§§ Solution.
cos A cos A
cot A = =
sin A A A
2 sin cos
Ñ A
2
A

1 cos2 − sin2
= 2 2
2 A A
sin cos
Ñ 2
A
2
A
é
1 cos2 sin2
= 2 − 2
2 A A A A
sin cos sin cos
( 2 2 ) 2 2
1 A A
= cot − tan . ■
2 2 2
1
§ Problem 8.1.32. sin α sin (60◦ − α) sin (60◦ + α) = sin 3α. ♢
4
§§ Solution.
sin α sin (60◦ − α) sin (60◦ + α)
( )
= sin α sin2 60◦ − sin2 α , by Ex. 2, Art. 93
( )
3 1( ) 1
= sin α − sin2 α = 3 sin α − 4 sin3 α = sin 3α.
4 4 4
Otherwise thus :
sin α sin (60◦ − α) sin (60◦ + α)
1
= sin α × (cos 2α − cos 120◦ )
(2 )
1 1
= sin α cos 2α +
2 2
1
= (2 sin α cos 2α + sin α)
4
1 1
= (sin 3α − sin α + sin α) = sin 3α. ■
4 4
◦ ◦ 1
§ Problem 8.1.33. cos α cos (60 − α) cos (60 + α) = cos 3α. ♢
4
§§ Solution.
cos α cos (60◦ − α) cos (60◦ + α)
( )
= cos α cos2 60◦ − sin2 α , by Ex. 2, Art. 93
( ) ( )
3
= cos α cos2 α − sin2 60◦ = cos α cos2 α −
4
1( ) 1
= 4 cos3 α − 3 cos α = cos 3α.
4 4
Otherwise thus :
◦ ◦
cos α cos (60 − α) cos (60 + α)
( )
1 1 1
= cos α × (cos 120◦ + cos 2α) = cos α cos 2α −
2 2 2
1
= (2 cos α cos 2α − cos α)
4
1 1
= (cos 3α + cos α − cos α) = cos 3α. ■
4 4
8.1. Multiple Angles 112

§ Problem 8.1.34. cot α + cot (60◦ + α) − cot (60◦ − α) = 3 cot 3α. ♢


§§ Solution.
cot α + cot (60◦ + α) − cot (60◦ − α)
cos α cos (60◦ + α) cos (60◦ − α)
= + −
sin α sin (60◦ + α) sin (60◦ − α)
cos α sin (60◦ − α) cos (60◦ + α) − cos (60◦ − α) sin (60◦ + α)
= +
sin α sin (60◦ + α) sin (60◦ − α)
cos α sin [(60◦ − α) − (60◦ + α)]
= +
sin α sin (60◦ + α) sin (60◦ − α)
cos α 2 sin 2α
= −
sin α 2 sin (60◦ + α) sin (60◦ − α)
cos α 2 sin 2α
= −
sin α cos 2α − cos 120◦
cos α 2 × 4 sin α cos α
= − ( )
sin α 1
2 1 − 2 sin2 α +
2
cos α 8 sin2 α cos α
= − ( )
sin α sin α 3 − 4 sin2 α
3 cos α − 4 sin2 α cos α − 8 sin2 α cos α
=
( 3 sin α − 4 sin)3 α
3 cos α − 4 sin2 α cos α
=
[ sin(3α ) ]
3 cos α − 4 1 − cos2 α cos α
=
( sin 3α )
3 4 cos3 α − 3 cos α
=
sin 3α
3 cos 3α
= = 3 cot 3α.
sin 3α
Otherwise thus :
cot α + cot (60◦ + α) − cot (60◦ − α)
1 1 1
= + −
tan α tan (60◦ + α) tan (60◦ − α)
1 1 − tan 60◦ tan α 1 + tan 60◦ tan α
= + ◦
− ◦
tan α tan√60 + tan α √tan 60 − tan α
1 1 − 3 tan α 1 + 3 tan α
= + √ − √
tan α 3 + tan α 3 − tan α
1 8 tan α 3 − tan2 α − 8 tan2 α
= − =
tan α 3 − tan2 α tan α (3 − tan2 α)
( )
3 1 − 3 tan α2
=
3 tan α − tan3 α
1
=3× = 3 cot 3α. ■
tan 3α
1
§ Problem 8.1.35. cos 20◦ cos 40◦ cos 60◦ cos 80◦ = . ♢
16
§§ Solution.
1
cos 20◦ cos 40◦ cos 80◦ = cos 20◦ (cos 120◦ + cos 40◦ )
2
8.1. Multiple Angles 113

[ ]
1 1 ( )
= cos 20◦ − + 2 cos2 20◦ − 1
2 ( 2 )
1 3
= cos 20◦ 2 cos2 20◦ −
2 2
1( ) 1
= 4 cos 20 − 3 cos 20 = cos 60◦ .
3 ◦ ◦
4 4
1 1 1 1
∴ cos 20◦ cos 40◦ cos 60◦ cos 80◦ = cos2 60◦ = × = . ■
4 4 4 16
3
§ Problem 8.1.36. sin 20◦ sin 40◦ sin 60◦ sin 80◦ = . ♢
16
§§ Solution.
sin 20◦ sin 40◦ sin 60◦ sin 80◦
1
= sin 20◦ (cos 40◦ − cos 120◦ )
2 [ ( )]
1 1
= sin 20◦ 1 − 2 sin2 20◦ − −
2 ( ) 2
1 ◦ 3 ◦
= sin 20 − 2 sin 20
2
2 2
1( ) 1
= 3 sin 20◦ − 4 sin3 20◦ = sin 60◦ .
4 4
1 1 3 3
∴ sin 20◦ sin 40◦ sin 60◦ sin 80◦ = sin2 60◦ = × = . ■
4 4 4 16
§ Problem 8.1.37. cos 4α = 1 − 8 cos2 α + 8 cos4 α. ♢
§§ Solution.
( )2
cos 4α = 2 cos2 2α − 1 = 2 2 cos2 α − 1 − 1
( )
= 2 4 cos4 α − 4 cos2 α + 1 − 1
= 1 − 8 cos2 α + 8 cos4 α. ■

§ Problem 8.1.38. sin 4A = 4 sin A cos3 A − 4 cos A sin3 A. ♢


§§ Solution. ( )
sin 4A = 2 sin 2A cos 2A = 4 sin A cos A cos2 A − sin2 A
= 4 sin A cos3 A − 4 cos A sin3 A. ■

§ Problem 8.1.39. cos 6α = 32 cos6 α − 48 cos4 α + 18 cos2 α − 1. ♢


§§ Solution.
( )2
cos 6α = 2 cos2 3α − 1 = 2 4 cos3 α − 3 cos α − 1
( )
= 2 16 cos6 α − 24 cos4 α + 9 cos2 α − 1
= 32 cos6 α − 48 cos4 α + 18 cos2 α − 1. ■

§ Problem 8.1.40. tan 3A tan 2A tan A = tan 3A − tan 2A − tan A. ♢


§§ Solution. By Art. 107, we have
tan A + tan 2A
tan 3A = tan(A + 2A) = .
1 − tan A tan 2A
∴ tan 3A (1 − tan A tan 2A) = tan A + tan 2A
∴ tan 3A tan 2A tan A = tan 3A − tan 2A − tan A. ■
8.2. Submultiple Angles 114

§ Problem 8.1.41.
2 cos 2n θ + 1
= (2 cos θ − 1) (2 cos 2θ − 1)
2 cos θ + 1 ( ) ( )
2 cos 22 θ − 1 . . . 2 cos 2n−1 θ − 1 .

§§ Solution.
We have
(2 cos θ + 1) (2 cos θ − 1) = 4 cos2 θ − 1
= 2 (1 + cos 2θ) − 1
= 2 cos 2θ + 1.
Similarly,
(2 cos 2θ + 1) (2 cos 2θ − 1) = 2 cos 22 θ + 1.
( )( )
2 cos 22 θ + 1 2 cos 22 θ − 1 = 2 cos 23 θ + 1.
... = ...
( )( )
2 cos 2n−1 θ + 1 2 cos 2n−1 θ − 1 = 2 cos 2n θ + 1.

Hence,by multiplication and canceling, we have


( )
(2 cos θ + 1) (2 cos θ − 1) (2 cos 2θ − 1) 2 cos 22 θ − 1
( )
. . . 2 cos 2n−1 θ − 1 = 2 cos 2n θ + 1.
2 cos 2n θ + 1
∴ = (2 cos θ − 1) (2 cos 2θ − 1)
2 cos θ + 1 ( ) ( )
2 cos 22 θ − 1 . . . 2 cos 2n−1 θ − 1 . ■

8.2 Submultiple Angles


1 1
§ Problem 8.2.1. If sin θ = and sin ϕ = , find the values of
2 3
sin(θ + ϕ) and sin(2θ + 2ϕ). ♢
§§ Solution. … ( )2 √
1 1
3
sin θ = ; ∴ cos θ = 1− =±
2 22
… ( ) √
1 1 2 2 2
sin ϕ = ; ∴ cos ϕ = 1− =±
3 3 3
∴ sin(θ + ϕ) = sin θ cos ϕ + cos θ sin ϕ
Å √ ã Å √ ã
1 2 2 3 1
= ± + ±
2 3 2 3
√ √
±2 2 ± 3
= .
6
Again,
( )2
1 1 1
cos 2θ = 1 − 2 sin2 θ = 1 − 2 = =1−
… ( ) 2

2 2
√ 1 2 3
∴ sin 2θ = 1 − cos2 2θ = 1− =±
2 2
( )2
1 2 7
also, cos 2ϕ = 1 − 2 sin2 ϕ = 1 − 2 =1− =
3 9 9
8.2. Submultiple Angles 115

… ( )2 √
7 4 2
∴ sin 2ϕ = 1− =±
9 9
∴ sin (2θ + 2ϕ) = sin 2θ cos 2ϕ + cos 2θ sin 2ϕ
√ Å √ ã √ √
3 7 1 4 2 ±7 3 ± 4 2
=± · + ± = . ■
2 9 2 9 18

§ Problem 8.2.2. The tangent of an angle is 2 · 4. Find its cosecant,


the cosecant of half the angle, and the cosecant of the supplement
of double the angle. ♢
§§ Solution. Cf. the figures of Arts. 31 and 50. Let θ be the angle
and let the length OM be unity and let the corresponding length of
M P be 2.4. Then √ √ √
OP = OM 2 + M P 2 = 1 + (2.4)2 = 6.76 = 2.6.
OP 2.6 13
∴ cosec θ = =± =±
MP 2.4 12
OM 1 5
and cos θ = =± =± .
MP 2.6 13
Otherwise thus :

Substitute in the formulae
1 + tan2 θ 1
cosec θ = and cos θ = √ .
tan θ 1 + tan2 θ
… … ( )
θ 1 1 5
∴ sin = ± (1 − cos θ) = ± 1∓
2 2 2 13
2 3
= ±√ or ± √
√ 13 √13
θ 13 13
∴ cosec = ± or ± .
2 2 3
Again,
1 1
cosec (180◦ − 2θ) = =
sin (180◦ − 2θ) sin 2θ
1 1 169
= = ( )( )= .
2 sin θ cos θ 12 5 120
2 ± ± ■
13 13
11 4
§ Problem 8.2.3. If cos α = and sin β = , find the values of
61 5
α−β α+β
sin2 and cos2 , the angles α and β being positive acute
2 2
angles. ♢
11
§§ Solution. If cos α = , then
61 … ( )
√ 11 2
sin α = 1 − cos α =
2 1−
61
… …
(61)2 − (11)2 (61 + 11)(61 − 11)
= =
(61)2 (61)2

3600 60
= = .
(61)2 61
8.2. Submultiple Angles 116

4
If sin β = , then
5 …
√ 16 3
cos β = 1 − sin2 β = 1− =
25 5
∴ cos(α − β) = cos α cos β + sin α sin β
11 3 60 4
= · + ·
61 5 61 5
33 + 240 273
= =
305 305
α−β 1 1 32 16
∴ sin2 = [1 − cos(α − β)] = × = .
2 2 2 305 305
Also,
[ ]
α+β 1 1 33 − 240
cos2 = [1 + cos(α + β)] = 1+
2 2[ ] 2 305
1 207 1 98 49
= 1− = × = . ■
2 305 2 305 305
3 4
§ Problem 8.2.4. If cos α = and cos β = , find the value of
5 5
α−β
cos , the angles α and β being positive acute angles. ♢
2
3 4
§§ Solution. If cos α = and cos β = , then
5 √ 5
… 1+
3 …
α 1 + cos α 5 8 2
cos = = = = √
2 2 2 10 5

… 4 …
β 1 + cos β 1+
9 3
cos = = 5 = = √
2 2 2 10 10

… 1−
3 …
α 1 − cos α 5 = 2 1
sin = = = √
2 2 2 10 5

… 4 …
β 1 − cos β 1− 1 1
sin
= = 5 = = √ .
2 2 2 10 10
α−β α β α β
∴ cos = cos cos + sin sin
2 2 2 2 2
2 3 1 1 7 7
= √ ·√ +√ ·√ = √ = √ . ■
5 10 5 10 50 5 2
1 θ
§ Problem 8.2.5. Given sec θ = 1 , find tan and tan θ. Verify by a
4 2
graph. ♢
1 5 4
§§ Solution. If sec θ = 1 = , then cos θ = .
4 4 Õ 5
… 1−
4
θ 1 − cos θ 5 = ±1.
∴ tan = ± =±
2 1 + cos θ 4 3
1+
5
8.2. Submultiple Angles 117

Also,

√ 25 3
tan θ = ± sec2 θ−1=±
−1=± . ■
16 4
A
§ Problem 8.2.6. If cos A = ·28, find the value of tan and explain
2
the resulting ambiguity. ♢
§§ Solution. … …
A 1 − cos A 1 − .28
tan =± =±
2 1 + cos A 1 + .28
… … …
.72 9 72 3
=± =± =± .=±
1.28 16 128 4
A
Since cos A = cos (2nπ ± A), any equation giving tan in terms of
( ) ( ) 2
A A A
cos A will give also tan nπ ± , i.e. tan ± , i.e. ± tan . ■
2 2 2
§ Problem 8.2.7. Find the values of
1◦
(1) sin 7 ,
2
1◦
(2) cos 7 ,
2
1◦
(3) tan 22 , and
2
1◦
(4) tan 11 .
4

§§ Solution. (1)   Å
… √ ã
1◦
1 1 3+1
sin 7 (1 − cos 15◦ ) =
= 1− √
2
2 2 2 2
… √ √ … √ √
2 2− 3−1 4− 6− 2
= √ =
8
√ √4 2 √
4− 6− 2
= √
2 2
√ √ √ √
1◦ 1 ◦
4+ 6+ 2
(2) cos 7 = (1 + cos 15 ) = √ ,
2 2 2 2
√ √
1◦ 1 + tan2 45◦ − 1 1+1−1 √
(3) tan 22 = = = 2 − 1.
2 tan 45◦ 1
(4) √
1◦ √ √
1◦ 1 + tan2 22 −1 4−2 2 1
tan 11 = 2 = √ −√

4
tan 22
1 2−1 2−1
»( √ ) (√
2
)2 (√ )
= 4−2 2 2+1 − 2+1
8.2. Submultiple Angles 118

»( √ )( √ ) (√ )
= 4−2 2 3+2 2 − 2+1
√ √ (√ )
= 4+2 2− 2+1 . ■

§ Problem 8.2.8. If sin θ + sin ϕ = a and cos θ + cos ϕ = b, find the


θ−ϕ
value of tan . ♢
2
§§ Solution.
( 2 Squaring
) ( and adding the
) given equations, we have
sin θ + cos2 θ + sin2 ϕ + cos2 ϕ + 2 (sin θ sin ϕ + cos θ cos ϕ)
= a2 + b2
∴ 1 + 1 + 2 cos(θ − ϕ) = a2 + b2
∴ 2 [1 + cos(θ − ϕ)] = a2 + b2
a2 + b2 − 2
∴ cos(θ − ϕ) = .
… 2
θ−ϕ 1 − cos(θ − ϕ)
∴ tan =± , [by Art. 110, (3)] ,
2 1 + cos(θ − ϕ)
  ( ) …
2 − a2 + b2 − 2 4 − a2 − b2
=± =± . ■
2 + (a2 + b2 − 2) a2 + b2
Prove that
α+β
§ Problem 8.2.9. (cos α + cos β)2 + (sin α − sin β)2 = 4 cos2 . ♢
2
§§ Solution.
(cos α + cos β)2 + (sin α − sin β)2
= cos2 α + cos2 β + 2 cos α cos β + sin2 α + sin2 β − 2 sin α sin β
( ) ( )
= cos2 α + sin2 α + cos2 β + sin2 β + 2 (cos α cos β − sin α sin β)
α+β
= 1 + 1 + 2 cos(α + β) = 2 [1 + cos(α + β)] = 4 cos2 . ■
2
α−β
§ Problem 8.2.10. (cos α + cos β)2 +(sin α + sin β)2 = 4 cos2 . ♢
2
§§ Solution.
(cos α + cos β)2 + (sin α + sin β)2
= cos2 α + cos2 β + 2 cos α cos β + sin2 α + sin2 β + 2 sin α sin β
α−β
= 2 [1 + cos(α − β)] = 4 cos2 . ■
2
α−β
§ Problem 8.2.11. (cos α − cos β)2 + (sin α − sin β)2 = 4 sin2 . ♢
2
§§ Solution.
(cos α − cos β)2 + (sin α − sin β)2
= cos2 α + cos2 β − 2 cos α cos β + sin2 α + sin2 β − 2 sin α sin β
= 2 − 2 (cos α cos β + sin α sin β) = 2 − 2 cos(α − β)
α−β
= 2 [1 − cos(α − β)] = 4 sin2 . ■
2
A
2 tan
§ Problem 8.2.12. sin A = 2 . ♢
A
1 + tan2
2
§§ Solution. See Art. 109. ■
8.2. Submultiple Angles 119

A
1 − tan2
§ Problem 8.2.13. cos A = 2 . ♢
2
A
1 + tan
2
§§ Solution. See Art. 109. ■

Äπ ä Äπ ä
§ Problem 8.2.14. sec + θ sec − θ = 2 sec 2θ. ♢
4 4
§§ Solution.
Äπ ä Äπ ä 1 1
sec + θ sec −θ = π · π
4 4 cos + θ cos − θ
2 î ó
4 4
1 π
= Ä ä= ∵ cos = 0
1 π cos 2θ 2
cos + cos 2θ
2 2
= 2 sec 2θ.
Otherwise thus Äπ: ä îπ Äπ äó Äπ ä
∵ cos + θ = sin − +θ = sin −θ
4 2 4 4
We have Ä ä Äπ ä
π 2
sec
4
+ θ sec
4
−θ = Äπ ä (θ )
2 sin − θ cos −θ
4 4
2 2
= Äπ ä= = 2 sec 2θ.
sin − 2θ cos 2θ
2
Otherwise thus :
By Ex. 2, ä we
Ä πArt. 93, Ä πhave ä π
cos + θ cos − θ = cos2 − sin2 θ
Äπ4 ä Äπ 4 ä 4
1 1
∴ sec + θ sec −θ = π =
4 4 cos2 − sin2 θ 1
− sin2 θ
4 2
2 2
= = = 2 sec 2θ. ■
1 − 2 sin2 θ cos 2θ
( ) …
A 1 + sin A
§ Problem 8.2.15. tan 45◦ + = = sec A + tan A. ♢
2 1 − sin A
§§ Solution.
A
( ) 1 + tan
A
◦ 2 [By Art. 100]
tan 45 + =
2 A
1 − tan
2
A
sin Œ
1+ 2 ( )
A A A 2
cos cos + sin
2 = 2 2
= ( )
A A A 2
sin cos − sin
1− 2 2 2
A
cos
2
8.2. Submultiple Angles 120

Õ
A A A A
cos2 + sin2 + 2 cos sin
= 2 2 2 2
A A A A
cos2 + sin2 − 2 cos sin
2 2 2 2
… …
1 + sin A (1 + sin A)2 1 + sin A
= = =
1 − sin A 1 − sin2 A cos A
1 sin A
= + = sec A + tan A.
cos A cos A
Otherwise thus : ( )
A
( ) sin 45◦ +
A 2
tan 45◦ + = ( )
2 A
cos 45◦ +
Õ 2
( )
2 sin2 45◦ +
A …
2 1 − cos (90◦ + A)
= ( )=
A 1 + cos (90◦ + A)
2 cos2 45◦ +
2
… …
1 + sin A (1 + sin A)2 1 + sin A
= = =
1 − sin A 1 − sin2 A cos A
1 sin A
= + = sec A + tan A. ■
cos A cos A
( ) ( )
π A π A 1
§ Problem 8.2.16. sin2 + − sin2 − = √ sin A. ♢
8 2 8 2 2
(
§§ Solution. ) ( )
π A π A
sin2 + − sin2 −
8 2 8
1î Ä2 π äó 1 î Äπ äó
= 1 − cos +A − 1 − cos −A
1î Äπ 4 ä Ä π2 äó 4
2
= cos − A − cos +A
2î 4 ó 4
1 π π 1
= 2 sin sin A = sin sin A = √ sin A. ■
2 4 4 2
3
§ Problem 8.2.17. cos α + cos (α + 120 ) + cos (α − 120◦ ) = .
2 2 ◦ 2 ♢
2
§§ Solution.
cos2 α + cos2 (α + 120◦ ) + cos2 (α − 120◦ )
1
= [1 + cos 2α + 1 + cos (2α + 240◦ ) + 1 + cos (2α − 240◦ )]
2
1
= (3 + cos 2α + 2 cos 2α cos 240◦ )
2 [ ]
1 1
= (3 + cos 2α − cos 2α) ∵ cos 240◦ = −
2 2
3
= . ■
2
π 3π 5π 7π 3
§ Problem 8.2.18. cos4 + cos4 + cos4 + cos4 = . ♢
8 8 8 8 2
8.2. Submultiple Angles 121

§§ Solution.
π 3π 5π 7π
cos4 + cos4 + cos4 + cos4
8 8 8 Ñ 8 é2 Ñ é2
( π )2 3π 5π
1 + cos 1 + cos 1 + cos
= 4 + 4 + 4
2 2 2
Ñ 7π
é2
1 + cos
+ 4
2
Å√ ã2 Å √ ã2 Å √ ã2 Å √ ã2
2+1 2−1 2−1 2+1
= √ + √ + √ + √
2 2 2 2 2 2 2 2
ï √ √ ò
3+2 2 3−2 2 3 3
=2 + =2× = . ■
8 8 4 2
π 3π 5π 7π 3
§ Problem 8.2.19. sin4 + sin4 + sin4 + sin4 = . ♢
8 8 8 8 2
§§ Solution.
π 3π 5π 7π
sin4 + sin4 + sin4 + sin4
8 8 8 Ñ 8 é2 Ñ é2
( π )2 3π 5π
1 − cos 1 − cos 1 − cos
= 4 + 4 + 4
2 2 2
Ñ 7π
é2
1 − cos
+ 4
2
Å√ ã2 Å √ ã2 Å √ ã2 Å √ ã2
2−1 2+1 2+1 2−1
= √ + √ + √ + √
2 2 2 2 2 2 2 2
ï √ √ ò
3−2 2 3+2 2 3 3
=2 + =2× = . ■
8 8 4 2

§ Problem 8.2.20.
cos 2θ cos 2ϕ + sin2 (θ − ϕ) − sin2 (θ + ϕ)
= cos (2θ + 2ϕ) . ♢
§§ Solution.
cos 2θ cos 2ϕ + sin2 (θ − ϕ) − sin2 (θ + ϕ)
1
= [cos (2θ + 2ϕ) + cos (2θ − 2ϕ) + 1 − cos 2 (θ − ϕ) − 1 + cos 2 (θ + ϕ)]
2
1
= × 2 cos (2θ + 2ϕ) = cos 2 (θ + ϕ) . ■
2
§ Problem 8.2.21. ( )
(tan 4A + tan 2A) 1 − tan2 3A tan2 A = 2 tan 3A sec2 A. ♢
§§ Solution. ( )
(tan 4A + tan 2A) 1 − tan2 3A tan2 A
( )Å ã
sin 4A sin 2A sin2 3A sin2 A
= + 1−
cos 4A cos 2A cos2 3A cos2 A
8.2. Submultiple Angles 122

( )Å ã
sin 4A cos 2A + cos 4A sin 2A cos2 3A cos2 A − sin2 3A sin2 A
=
cos 4A cos 2A cos2 3A cos2 A
[ ]
sin(4A + 2A)
=
[ cos 4A cos 2A ]
(cos 3A cos A − sin 3A sin A) (cos 3A cos A + sin 3A sin A)
2 2
[ ] [ cos 3A cos A ]
sin 6A cos(3A + A) cos(3A − A)
= 2 2
cos 4A cos 2A cos 3A cos A
sin 6A cos 4A cos 2A
= ·
cos 4A cos 2A cos2 3A cos2 A
sin 6A 2 sin 3A cos 3A
= =
cos2 3A cos2 A cos2 3A cos2 A
2 sin 3A 1
= · = 2 tan 3A sec2 A. ■
cos 3A cos2 A
Ä
§ Problem 8.2.22.
α αäÄ α αä
1 + tan − sec 1 + tan + sec
2 2 2 2
α
= sin α sec2 . ♢
2
Ä
§§ Solution.
α αäÄ α αä
1 + tan − sec 1 + tan + sec
2 2 Ä 2
α ä2
2
α
= 1 + tan − sec2
α Ä αä
2 2
α α
= 1 + tan2 + 2 tan − 1 + tan2 = 2 tan
2 2 2 2
α α
2 sin cos sin α
2 2
= α · α = α
cos cos cos2
2 2 2
1 2 α
= sin α · α = sin α sec .
cos2 2 ■
2
Find the proper signs to be applied to the radicals in the three
following formulae.
A √ √ A
§ Problem 8.2.23. 2 cos = ± 1 − sin A ± 1 + sin A, when =
2 2
278◦ . ♢
A A
§§ Solution. If = 278◦ , then sin is negative and numerically
2 2
A
> cos .
2
A A √
∴ sin + cos = − 1 + sin A, and
2 2
A A √
sin − cos = − 1 − sin A.
2 2
Hence, by subtraction, we have
A √ √
2 cos = + 1 − sin A − 1 + sin A. ■
2
A √ √ A
§ Problem 8.2.24. 2 sin = ± 1 − sin A ± 1 + sin A, when =
2 2
19π
. ♢
11
8.2. Submultiple Angles 123

A 19π 19π 7π
§§ Solution. If = , then, since is slightly less than ,
2 11 11 4
A A
sin is negative and numerically > cos . Hence
2 2
A A √
sin + cos = − 1 + sin A, and
2 2
A A √
sin − cos = − 1 − sin A.
2 2
Hence, by addition, we have
A √ √
2 sin = − 1 − sin A − 1 + sin A. ■
2
A √ √ A
§ Problem 8.2.25. 2 cos = ± 1 − sin A ± 1 + sin A, when =
2 2
−140◦ . ♢
A ◦ A A
§§ Solution. If = −140 , then sin and cos are both negative,
2 2 2
A
but cos is numerically the greater. Hence
2
A A √
sin + cos = − 1 + sin A, and
2 2
A A √
sin − cos = + 1 − sin A.
2 2
Hence, by subtraction, we have
A √ √
2 cos = − 1 − sin A − 1 + sin A. ■
2
§ Problem 8.2.26. If A = 340◦ , prove that
A √ √
2 sin = − 1 + sin A + 1 − sin A, and
2
A √ √
2 cos = − 1 + sin A − 1 − sin A. ♢
2 A A

§§ Solution. If A = 340 , then ◦
= 170 and cos is negative and
2 2
A
numerically > sin . Hence
2
A A √
sin + cos = − 1 + sin A, and
2 2
A A √
sin − cos = + 1 − sin A.
2 2
Hence, by addition, we have
A √ √
2 sin = − 1 + sin A + 1 − sin A
2
and, by subtraction, we have
A √ √
2 cos = − 1 + sin A − 1 − sin A. ■
2
§ Problem 8.2.27. If A = 460◦ , prove that
A √ √
2 cos = − 1 + sin A + 1 − sin A. ♢
2 A A

§§ Solution. If A = 460 , then ◦
= 230 and sin is negative and
2 2
A
numerically > cos . Hence
2
A A √
sin + cos = − 1 + sin A, and
2 2
8.2. Submultiple Angles 124

A A √
sin − cos = − 1 − sin A,
2 2
Hence, by subtraction, we have
A √ √
2 cos = − 1 + sin A + 1 − sin A. ■
2
§ Problem 8.2.28. If A = 580◦ , prove that
A √ √
2 sin = − 1 + sin A − 1 − sin A. ♢
2 A A

§§ Solution. If A = 580 then ◦
= 290 and sin is negative and
2 2
A
numerically > cos . Hence
2
A A √
sin + cos = − 1 + sin A, and
2 2
A A √
sin − cos = − 1 − sin A.
2 2
Hence, by addition, we have
A √ √
2 sin = − 1 + sin A − 1 − sin A. ■
2
A
§ Problem 8.2.29. Within what respective limits must lie when
2
A √ √
(1) 2 sin = 1 + sin A + 1 − sin A
2
A √ √
(2) 2 sin = − 1 + sin A + 1 − sin A
2
A √ √
(3) 2 sin = + 1 + sin A − 1 − sin A
2
A √ √
(4) 2 cos = 1 + sin A − 1 − sin A.
2 ♢
A √ √
§§ Solution. (1) 2 sin = 1 + sin A + 1 − sin A,
2
A A √
when sin + cos = + 1 + sin A
2 2
A A √
and sin − cos = + 1 − sin A
2 2
A A
i.e. when sin is positive and numerically > cos
2 2
A π 3π
i.e. when lies between 2nπ + and 2nπ + .
2 4 4
A √ √
(2) 2 sin = − 1 + sin A + 1 − sin A
2
A A √
when sin + cos = − 1 + sin A
2 2
A A √
and sin − cos = + 1 − sin A
2 2
A A
i.e. when cos is positive and numerically > sin
2 2
A 3π 5π
i.e. when lies between 2nπ + and 2nπ + .
2 4 4
8.2. Submultiple Angles 125

A √ √
(3) 2 sin = + 1 + sin A − 1 − sin A
2
A A √
when sin + cos = + 1 + sin A
2 2
A A √
and sin − cos = − 1 − sin A
2 2
A A
i.e. when cos is positive and numerically > sin
2 2
A π π
i.e. when lies between 2nπ − and 2nπ + .
2 4 4
A √ √
(4) 2 cos = 1 + sin A − 1 − sin A.
2
A A √
when sin + cos = + 1 + sin A
2 2
A A √
and sin − cos = + 1 − sin A
2 2
A A
i.e. when sin is positive and numerically > cos
2 2
A π 3π
i.e. when lies between 2nπ + and 2nπ + . ■
2 4 4
§ Problem 8.2.30. In the formula
A √ √
2 cos = ± 1 + sin A ± 1 − sin A
2
A
find within what limits must lie when
2
(1) the two positive signs are taken

(2) the two negative signs are taken, and

(3) the first sign is negative and the second positive. ♢


A √ √
§§ Solution. (1) 2 cos = + 1 + sin A + 1 − sin A,
2
A A √
when sin + cos = + 1 + sin A
2 2
A A √
and sin − cos = − 1 − sin A
2 2
A A
i.e. when cos is positive and numerically > sin
2 2
A π π
i.e. when lies between 2nπ − and 2nπ + .
2 4 4
A √ √
(2) 2 cos = − 1 + sin A − 1 − sin A
2
A A √
when sin + cos = − 1 + sin A
2 2
A A √
and sin − cos = + 1 − sin A
2 2
A A
i.e. when cos is negative and numerically > sin
2 2
A 3π 5π
i.e. when lies between 2nπ + and 2nπ + .
2 4 4
8.2. Submultiple Angles 126

A √ √
(3) 2 cos = − 1 + sin A + 1 − sin A
2
A A √
when sin + cos = − 1 + sin A
2 2
A A √
and sin − cos = − 1 − sin A
2 2
A A
i.e. when sin is positive and numerically > cos
2 2
A 5π 7π
i.e. when lies between 2nπ + and 2nπ + . ■
2 4 4
§ Problem 8.2.31. Prove that the sine is algebraically less than the
3π π
cosine for any angle between 2nπ − and 2nπ + where n is any
4 4
integer. ♢

√ ( )
§§ Solution. We have
1 1
sin A − cos A = 2 sin A √ − cos A √
√ Ä πä √ Ä πä
2 2
π
= 2 sin A cos − cos A sin = 2 sin A − .
4 4 Ä4 π ä
Hence sin A − cos A is negative, i.e. cos A > sin A when sin A −
Ä πä
4
is negative, i.e. when A − lies between 2nπ − π and 2nπ, in the
4

third and fourth quadrants, i.e. when A lies between 2nπ − and
4
π
2nπ + .
4
Otherwise thus :
Since 2nπ is equivalent to n complete revolutions of the revolving

line, the angle 2nπ − corresponds to the position of the revolving
4
line at OR, [cf. figure of Art. 116], which bisects the third quadrant
π
and the angle 2nπ + to the position of the revolving line at OP ,
4
which bisects the first quadrant.
By Arts. 53, 54 and 55, we see that from R to B ′ the sine and
the cosine are both negative and the sine numerically > the cosine;
hence the sine is algebraically < the cosine.
From B ′ to A the sine is negative and the cosine is positive; hence
the sine is algebraically < the cosine.
From A to P the sine and the cosine are both positive and the
sine numerically < the cosine; hence the sine is algebraically < the
cosine. ■

A
§ Problem 8.2.32. If sin be determined from the equation
3
A A
sin A = 3 sin − 4 sin3
3 3
prove that we should expect to obtain also the values of
π−A π+A
sin and − sin .
3 3
Give also a geometrical illustration. ♢
8.2. Submultiple Angles 127

A
§§ Solution. ∵ sin A = sin [nπ + (−1)n A], any equation giving sin
3
1
in terms of sin A should give also sin [nπ + (−1)n A].
3
Now n is of the form 3m, or 3m ± 1.
If n = 3m, then [ ]
1 A
sin [nπ + (−1)n A] = sin mπ + (−1)3m
3 3
A A
= sin mπ cos(−1)3m + cos mπ sin(−1)3m
3 3
A A
= cos mπ sin(−1)3m = sin .
3 3
whether m be even or odd, for sin mπ = 0 and cos mπ = +1 or −1
according as m is even or odd.
If n = 3m + 1, then ï ò
1 π + (−1)3m+1 A
sin [nπ + (−1)n A] = sin mπ +
3 3
π + (−1)3m+1 A
= cos mπ sin
3
π−A π+A
= sin or − sin ,
3 3
according as m is even or odd.
If n = 3m − 1, then ï ò
1 π − (−1)3m−1 A
sin [nπ + (−1)n A] = sin mπ −
3 3
π − (−1)3m−1 A
= − cos mπ sin
3
π−A π+A
= sin or − sin ,
3 3
according as m is odd or even.
A
Hence we have two values in addition to sin ,
3
π−A π+A
viz. sin and − sin . ■
3 3
A
§ Problem 8.2.33. If cos be found from the equation
3
A A
cos A = 4 cos3 − 3 cos
3 3
prove that we should expect to obtain also the values of
2π − A 2π + A
cos and cos .
3 3
Give also a geometrical illustration. ♢
A
§§ Solution. ∵ cos A = cos (2nπ ± A), any equation giving cos in
3
1
terms of cos A should give also cos (2nπ ± A).
3
Now n is of the form 3m, or 3m ± 1.
If n = 3m, then ( )
1 A A
cos (2nπ ± A) = cos 2mπ ± = cos .
3 3 3
8.3. Angles of 9◦ , 18◦ , 36◦ , 81◦ 128

If n = 3m + 1, then ( )
1 2π A 2π ± A
cos (2nπ ± A) = cos 2mπ + ± = cos .
3 3 3 3
If n = 3m − 1, then ( )
1 2π A
cos (2nπ ± A) = cos 2mπ − ±
3 ( 3 3)
2π ∓ A 2π ∓ A
= cos 2mπ − = cos .
3 3
A
Hence we have two values in addition to cos ,
3
2π − A 2π + A
viz. cos and − cos . ■
3 3

8.3 Angles of 9◦ , 18◦ , 36◦ , 81◦


Prove that √
5−1
§ Problem 8.3.1. sin2 72◦ − sin2 60◦ = . ♢
8
§§ Solution.
ñ√ √ ô2 Å √ ã2
10 + 2 5 3
sin2 72◦ − sin2 60◦ = −
4 2
√ √ √
10 + 2 5 3 5+ 5−6 5−1
= − = = . ■
16 4 8 8

5+1
§ Problem 8.3.2. cos2 48◦ − sin2 12◦ = . ♢
8
§§ Solution.
cos2 48◦ − sin2 12◦ = cos (48◦ + 12◦ ) cos (48◦ − 12◦ ) [By Ex. 2, Art. 93]
√ √
1 5+1 5+1
= cos 60◦ cos 36◦ = · = . ■
2 4 8
§ Problem 8.3.3. cos 12◦ + cos 60◦ + cos 84◦ = cos 24◦ + cos 48◦ . Verify
by a graph. ♢
§§ Solution.
cos 12◦ + cos 60◦ + cos 84◦ = cos 12◦ + 2 cos 72◦ cos 12◦
= cos 12◦ (1 + 2 cos 72◦ )
Å √ ã
5−1
= cos 12◦ 1 +
2

5+1
= · cos 12◦ = 2 cos 36◦ cos 12◦
2
= cos 24◦ + cos 48◦ . ■

π 2π 3π 4π 5
§ Problem 8.3.4. sin sin sin sin = . ♢
5 5 5 5 16
§§ Solution.
π 2π 3π 4π π 2π 2π π
sin sin sin sin = sin sin sin sin
5 5 5 5 5 5 5 5
π 2π
= sin2 sin2 = sin2 36◦ sin2 72◦
5√ 5 √
10 − 2 5 10 + 2 5 5
= · = . ■
16 16 16
8.3. Angles of 9◦ , 18◦ , 36◦ , 81◦ 129

π 13π 1
§ Problem 8.3.5. sin + sin =− . ♢
10 10 2
§§ Solution.
π 13π 7π 6π
sin + sin = 2 sin cos
10 10 10 ( 10 )
3π 4π
= 2 sin − cos
10 10 √ √
5+1 5−1 2 1
= −2 sin 54◦ cos 72◦ = −2 · · =− =− .
4 4 4 2
Otherwise thus :

π 13π π 3π
sin + sin = sin − sin , by Art. 73,
10 10 10 10 √ √
5−1 5+1
= sin 18◦ − sin 54◦ = −
4 4
2 1
=− =− . ■
4 2
π 13π 1
§ Problem 8.3.6. sin sin =− . ♢
10 10 4
§§ Solution. ( )
π 13π π 3π
sin sin = sin − sin
10 10 √10 √ 10
5−1 5+1 1
=− · =− . ■
4 4 4
§ Problem 8.3.7. tan 6◦ tan 42◦ tan 66◦ tan 78◦ = 1. ♢
§§ Solution.
sin 6◦ sin 42◦ sin 66◦ sin 78◦
tan 6◦ tan 42◦ tan 66◦ tan 78◦ =
cos 6◦ cos 42◦ cos 66◦ cos 78◦
2 sin 6◦ sin 66◦ × 2 sin 42◦ sin 78◦
=
2 cos 6◦ cos 66◦ × 2 cos 42◦ cos 78◦
(cos 60◦ − cos 72◦ ) (cos 36◦ − cos 120◦ )
=
(cos 60◦ + cos 72◦ ) (cos 36◦ + cos 120◦ )
Å √ ãÅ√ ã
1 5−1 5+1 1
− +
2 4 4 2
= Å √ ãÅ√ ã
1 5−1 5+1 1
+ −
2 4 4 2
( √ )( √ )
3− 5 3+ 5 4
= (√ ) (√ ) = = 1.
5+1 5−1 4 ■

π 2π 3π 4π 5π 6π 7π 1
§ Problem 8.3.8. cos cos cos cos cos cos cos = 7.
15 15 15 15 15 15 15 2

§§ Solution.
π 4π 1Ä π πä
cos cos = cos + cos
15 15 2Å 3√ 5ã √
1 1 5+1 3+ 5
= + =
2 2 4 8
( )
2π 7π 1 π 3π
cos cos = cos + cos
15 15 2 3 5
8.3. Angles of 9◦ , 18◦ , 36◦ , 81◦ 130

Å √ ã √
1 1 5−1 3− 5
= − =
2 2 4 8
( )
3π 6π 1 π 3π
cos cos = cos + cos
15 15 2Å 5 5 ã
√ √
1 5+1 5−1 1
= − = , and
2 4 4 4
5π π 1
cos = cos = .
15 3 2
π 2π 3π 4π 5π 6π 7π
∴ cos cos cos cos cos cos cos
15 15 √ 15 √15 15 15 15
3+ 5 3− 5 1 1
= × × ×
8 8 4 2
1 1 1 1 1
= × × × = 7.
2 8 4 2 2
Otherwise thus :
π 2π 3π 4π 5π 6π 7π
cos cos cos cos cos cos cos
15 15 15 15 15 15 15
π π 2π 4π 7π
2 sin cos cos cos cos
= 15 15 15 15 15
π
2 sin
15
3π 3π 6π
2 sin cos cos
× 15 15 15 × cos 5π
3π 15
2 sin
15
2π 2π 4π 7π 2π 2π
sin cos cos cos sin cos
= 15 15 15 15 × 5 5 ×1
π π 2
2 sin 2 sin
15 5
4π 4π 7π 4π
sin cos cos sin 1
15 15 15 × 5
= π π ×2
2 × 2 sin 2 × 2 sin
15 5
8π 7π π
sin cos sin 1
15 15 × 5
= π π ×2
2 × 2 × 2 sin 2 × 2 sin
15 5
7π 7π 14π
sin cos sin
= 15 15 × 1 = 15 = 1 .
π 2 3 π 27
23 sin 27 sin ■
15 15
2π 4π 8π 14π
§ Problem 8.3.9. 16 cos cos cos cos = 1. ♢
15 15 15 15
§§ Solution.
2π 4π 8π 14π
16 cos cos cos cos
15 15 15 15
2π 2π 4π 8π 14π
16 sin cos cos cos cos
= 15 15 15 15 15

sin
15
8.3. Angles of 9◦ , 18◦ , 36◦ , 81◦ 131

4π 4π 8π 14π
8 sin cos cos cos
= 15 15 15 15

sin
15
8π 8π 14π 16π 14π
4 sin cos cos 2 sin cos
= 15 15 15 = 15 15
2π 2π
sin sin
Ä πä Ä
15
πä
15
2 × − sin × − cos
= 15 15

sin
15
π π 2π
2 sin cos sin
= 15 15 = 15 = 1.
2π 2π
sin sin ■
15 15
§ Problem 8.3.10. Two parallel chords of a circle, which are on the
same side of the center, subtend angles of 72◦ and 144◦ respectively
at the center. Prove that the perpendicular distance between the
chords is half the radius of the circle. ♢
§§ Solution. See figure of Art. 130. Let O be the center and r be
the radius of the circle and let P P ′ and QQ′ be the two chords, P P ′
being the one nearer to O.
Join OQ, OQ′ , OP and OP ′ . Draw OM N perpendicular to P P ′ and
QQ′ , bisecting them in M and N respectively.
We have the ∠QOQ′ = 72◦ , and the ∠P OP ′ = 144◦ . Hence the
required distance
= M N = ON − OM = r cos ∠QON − r cos ∠P OM
= r cos 36◦ − r cos 72◦ = r (cos 36◦ − cos 72◦ )
Å√ √ ã
5+1 5−1 2 r
=r − =r× = . ■
4 4 4 2

§ Problem 8.3.11. In any circle prove that the chord which sub-
tends 108◦ at the center is equal to the sum of the two chords which
subtend angles of 36◦ and 60◦ . ♢
§§ Solution. If r be the radius of the circle, the lengths of the chords
are 2r sin 54◦ , 2r sin 18◦ and 2r sin 30◦ respectively.
√ √
5+1 5−1 1
Now sin 54◦ = = + = sin 18◦ + sin 30◦ .
4 4 2
Hence we have the required result. ■

§ Problem 8.3.12. Construct the angle whose cosine is equal to its


tangent. ♢
§§ Solution. If θ be the angle, we have
cos θ = tan θ; ∴ cos2 θ = sin θ; ∴ 1 − sin2 θ = sin θ
∴ sin2 θ + sin θ − 1 = 0;
√ √
−1 + 1 + 4 5−1
∴ sin θ = = = 2 sin 18◦ .
2 2
Describe a semicircle AP B on AB as diameter. Make an angle ∠P AB
equal to 18◦ . Join BP .
8.3. Angles of 9◦ , 18◦ , 36◦ , 81◦ 132

On this semicircle take a point Q, such that BQ equals 2BP . Join


AQ. We then have
BQ 2BP
sin ∠BAQ = = = 2 sin 18◦ .
AB AB
Hence ∠BAQ is the required angle. ■

§ Problem 8.3.13. Solve the equation


sin 5θ cos 3θ = sin 9θ cos 7θ. ♢
§§ Solution.
sin 5θ cos 3θ = sin 9θ cos 7θ
∴ 2 sin 5θ cos 3θ = 2 sin 9θ cos 7θ
∴ sin 8θ + sin 2θ = sin 16θ + sin 2θ
∴ sin 8θ = sin 16θ = 2 sin 8θ cos 8θ
∴ sin 8θ = 0 or 2 cos 8θ = 1.
If sin 8θ = 0, then 8θ = nπ.
1 π
If 2 cos 8θ = 1, then cos 8θ = = cos
2 3
π
∴ 8θ = 2nπ ± .
( 3 )
nπ 1 π
∴θ= or 2n ± . ■
8 3 8
Chapter
9
Identities And Trigonometrical
Equations

9.1 Tangent of The Sum of Angles


§ Problem 9.1.1. Prove that
5 tan θ − 10 tan3 θ + tan5 θ
tan 5θ = . ♢
1 − 10 tan2 θ + 5 tan4 θ
§§ Solution.
s1 − s3 + s5
tan 5θ =
1 − s2 + s4
5C tan θ − 5C tan3 θ + 5C tan5 θ
1 3 5
=
1 − 5C2 tan2 θ + 5C4 tan4 θ
5 tan θ − 10 tan3 θ + tan5 θ
= . ■
1 − 10 tan2 θ + 5 tan4 θ

9.2 Identities
If A + B + C = 180◦ , prove that
§ Problem 9.2.1. sin 2A + sin 2B − sin 2C = 4 cos A cos B sin C. ♢
§§ Solution.
sin 2A + sin 2B − sin 2C = 2 sin(A + B) cos(A − B) − 2 sin C cos C
= 2 sin C cos(A − B) − 2 sin C cos C
= 2 sin C [cos(A − B) − cos C]
= 2 sin C [cos(A − B) + cos(A + B)]
= 2 sin C · 2 cos A cos B
= 4 cos A cos B sin C. ■
9.2. Identities 134

§ Problem 9.2.2. cos 2A+cos 2B +cos 2C = −1−4 cos A cos B cos C. ♢


§§ Solution.
cos 2A + cos 2B + cos 2C = 2 cos(A + B) cos(A − B) + 2 cos2 C − 1
= 2 cos(A + B) cos(A − B) + 2 cos C · cos C − 1
= −2 cos C cos(A − B) + 2 cos C [− cos(A + B)] − 1
= −2 cos C [cos(A − B) + cos(A + B)] − 1
= −2 cos C · 2 cos A cos B − 1
= −1 − 4 cos A cos B cos C. ■

§ Problem 9.2.3. cos 2A + cos 2B − cos 2C = 1 − 4 sin A sin B cos C. ♢


§§ Solution. ( )
cos 2A + cos 2B − cos 2C = 2 cos(A + B) cos(A − B) − 2 cos2 C − 1
= 2 cos(A + B) cos(A − B) − 2 cos C · cos C + 1
= −2 cos C cos(A − B) − 2 cos C [− cos(A + B)] + 1
= −2 cos C [cos(A − B) − cos(A + B)] + 1
= −2 cos C · 2 sin A sin B + 1
= 1 − 4 sin A sin B cos C. ■

A B C
§ Problem 9.2.4. sin A + sin B + sin C = 4 cos cos cos . ♢
2 2 2
§§ Solution.
A+B A−B C C
sin A + sin B + sin C = 2 sin cos + 2 sin cos
2 2 2 2
C A−B A+B C
= 2 cos cos + 2 cos cos
2 [ 2 2 ] 2
C A−B A+B
= 2 cos cos + cos
2 2 2
C A B
= 2 cos · 2 cos cos
2 2 2
A B C
= 4 cos cos cos . ■
2 2 2
A B C
§ Problem 9.2.5. sin A + sin B − sin C = 4 sin sin cos . ♢
2 2 2
§§ Solution.
A+B A−B C C
sin A + sin B − sin C = 2 sin cos − 2 sin cos
2 2 2 2
C A−B A+B C
= 2 cos cos − 2 cos cos
2 [ 2 2 ] 2
C A−B A+B
= 2 cos cos − cos
2 2 2
C A B
= 2 cos · 2 sin sin
2 2 2
A B C
= 4 sin sin cos . ■
2 2 2
A B C
§ Problem 9.2.6. cos A + cos B + cos C = 1 + 4 sin sin sin . ♢
2 2 2
9.2. Identities 135

§§ Solution.
A+B A−B C
cos A + cos B + cos C = 2 cos cos + 1 − 2 sin2
2 2 2
A+B A−B C C
= 2 cos cos + 1 − 2 sin sin
2 2 2 2
C A−B A+B C
= 2 sin cos + 1 − 2 cos sin
2 [ 2 ]2 2
C A−B A+B
= 2 sin cos − cos +1
2 2 2
C A B
= 2 sin · 2 sin sin +1
2 2 2
A B C
= 1 + 4 sin sin sin . ■
2 2 2
§ Problem 9.2.7. sin2 A + sin2 B − sin2 C = 2 sin A sin B cos C. ♢
§§ Solution.
sin2 A + sin2 B − sin2 C
1 1 1
= (1 − cos 2A) + (1 − cos 2B) − (1 − cos 2C)
2 2 2
1
= [1 − (cos 2A + cos 2B − cos 2C)]
2
1
= [1 − (1 − 4 sin A sin B cos C)] {By §P roblem 9.2.3}
2
= 2 sin A sin B cos C. ■

§ Problem 9.2.8. cos2 A + cos2 B + cos2 C = 1 − 2 cos A cos B cos C. ♢


§§ Solution.
1
cos2 A + cos2 B + cos2 C = (1 + cos 2A + 1 + cos 2B + 1 + cos 2C)
2
1
= [3 + (cos 2A + cos 2B + cos 2C)]
2
1
= [3 − 1 − 4 cos A cos B cos C] {By §P roblem 9.2.2}
2
= 1 − 2 cos A cos B cos C. ■

§ Problem 9.2.9. cos2 A + cos2 B − cos2 C = 1 − 2 sin A sin B cos C. ♢


§§ Solution.
1 1 1
cos2 A + cos2 B − cos2 C = (1 + cos 2A) + (1 + cos 2B) − (1 + cos 2C)
2 2 2
1
= [1 + (cos 2A + cos 2B − cos 2C)]
2
1
= [1 + 1 − 4 sin A sin B cos C] {By §P roblem 9.2.3}
2
= 1 − 2 sin A sin B cos C. ■

A B C A B C
§ Problem 9.2.10. sin2 + sin2 + sin2 = 1 − 2 sin sin sin .
2 2 2 2 2 2

§§ Solution.
A B C
sin2 + sin2 + sin2
2 2 2
1
= (1 − cos A + 1 − cos B + 1 − cos C)
2
9.2. Identities 136

1
= [3 − (cos A + cos B + cos C)]
2[ ]
1 A B C
= 3 − 1 − 4 sin sin sin {By §P roblem 9.2.6}
2 2 2 2
A B C
= 1 − 2 sin sin sin . ■
2 2 2
A B C A B C
§ Problem 9.2.11. sin2 + sin2 − sin2 = 1 − 2 cos cos sin .
2 2 2 2 2 2

§§ Solution.
A B C
sin2 + sin2 − sin2
2 2 2
1 1 1
= (1 − cos A) + (1 − cos B) − (1 − cos C)
2 2 2
1
= [1 − (cos A + cos B − cos C)]
2[ ]
1 A B C
= 1 + 1 − 4 cos cos sin {By Ex. 2, Art. 127}
2 2 2 2
A B C
= 1 − 2 cos cos sin . ■
2 2 2
A B B C C A
§ Problem 9.2.12. tan tan + tan tan + tan tan = 1. ♢
2 2 2 2 2 2
A+B C
§§ Solution. ∵ = 90◦ −
2 ( 2 )
A+B C C 1
∴ tan = tan 90◦ − = cot =
2 2 2 C
tan
2
A B
tan + tan 1
∴ 2 2 = , by Art. 98
A B C
1 − tan tan tan
2 2 2
A C B C A B
∴ tan tan + tan tan = 1 − tan tan
2 2 2 2 2 2
A B B C C A
∴ tan tan + tan tan + tan tan = 1. ■
2 2 2 2 2 2
A B C A B C
§ Problem 9.2.13. cot + cot + cot = cot cot cot . ♢
2 2 2 2 2 2
A+B C
§§ Solution. ∵ = 90◦ −
2 ( 2 )
A+B C C 1
∴ cot = cot 90◦ − = tan =
2 2 2 C
cot
2
A B
cot cot −1 1
∴ 2 2 = , by Art. 100
A B C
cot + cot cot
2 2 2
A B C C A B
∴ cot cot cot − cot = cot + cot
2 2 2 2 2 2
A B C A B C
∴ cot + cot + cot = cot cot cot . ■
2 2 2 2 2 2
§ Problem 9.2.14. cot B cot C + cot C cot A + cot A cot B = 1. ♢
9.2. Identities 137

§§ Solution. ∵ A + B + C = 180◦ , ∴ A + B = 180◦ − C


∴ cot(A + B) = cot(180◦ − C) = − cot C
cot A cot B − 1
∴ = − cot C
cot A + cot B
∴ cot A cot B − 1 = − cot C (cot A + cot B)
∴ cot B cot C + cot C cot A + cot A cot B = 1. ■

§ Problem 9.2.15.
sin(B + 2C) + sin(C + 2A) + sin(A + 2B)
B−C C−A A−B
= 4 sin sin sin . ♢
2 2 2
§§ Solution.
∵ B + 2C = 180◦ + C − A = 180◦ − (A − C)
∴ sin(B + 2C) = sin(A − C)
∴ sin(B + 2C) + sin(C + 2A) + sin(A + 2B)
= sin(A − C) + sin(B − A) + sin(C − B)
B−C 2A − B − C B−C B−C
= 2 sin cos − 2 sin cos
2 ( 2 2 ) 2
B−C 2A − B − C B−C
= 2 sin cos − cos
2 2 2
B−C A−C B−A
= 2 sin × 2 sin sin
2 2 2
B−C C−A A−B
= 4 sin sin sin . ■
2 2 2
§ Problem 9.2.16.
A B C
sin + sin + sin −1
2 2 2
π−A π−B π−C
= 4 sin sin sin . ♢
4 4 4
§§ Solution.
A B C
sin + sin + sin −1
2 (
2 2) ( ) ( )
π A π B π C
= cos − + cos − + cos − −1
(2 2 ) 2 2 2 2
π A+B A−B π−C
= 2 cos − cos + 1 − 2 sin2 −1
2 [ 4 4 ( )] 4
π−C A−B π A+B
= 2 sin cos − cos −
{ ( 4 )4 2 4 }
π A+B A+B π−C
∵ cos − = sin = sin
2 4 ( )4 ( )
4
π−C π B π A
= 2 sin × 2 sin − sin −
4 4 4 4 4
π−A π−B π−C
= 4 sin sin sin . ■
4 4 4
§ Problem 9.2.17.
A B C
cos + cos − cos
2 2 2
π+A π+B π−C
= 4 cos cos cos . ♢
4 4 4
9.2. Identities 138

§§ Solution.
A B A+B A−B
cos + cos = 2 cos cos
2 2 4 4
π−C A−B
= 2 cos cos
4 4
C A+B A+B A+B
cos = sin = 2 sin cos
2 2 4 4 ( )
π−C A+B π−C π A+B
= 2 cos sin = 2 cos cos −
4 4 4 2 4
A B C
∴ cos + cos − cos
2 2 2 [ ( )]
π−C A−B π A+B
= 2 cos cos − cos −
4 4 2 4
π−C π−A π−B
= 2 cos × 2 sin sin
4 ( 4 ) 4( )
π−C π π−A π π−B
= 4 cos cos − cos −
4 2 4 2 4
π−C π+A π+B
= 4 cos cos cos . ■
4 4 4
sin 2A + sin 2B + sin 2C A B C
§ Problem 9.2.18. = 8 sin sin sin . ♢
sin A + sin B + sin C 2 2 2
§§ Solution.
sin 2A + sin 2B + sin 2C
sin A + sin B + sin C
4 sin A sin B sin C
= [by Ex. 1, Art. 127 and §P roblem 9.2.4]
A B C
4 cos cos cos
2 2 2
A A B B C C
2 sin cos × 2 sin cos × 2 sin cos
= 2 2 2 2 2 2
A B C
cos cos cos
2 2 2
A B C
= 8 sin sin sin . ■
2 2 2
§ Problem 9.2.19.
sin(B + C − A) + sin(C + A − B) + sin(A + B − C)
= 4 sin A sin B sin C. ♢
§§ Solution.
sin(B + C − A) + sin(C + A − B) + sin(A + B − C)
= sin (180◦ − 2A) + sin (180◦ − 2B) + sin (180◦ − 2C)
= sin 2A + sin 2B + sin 2C

= 4 sin A sin B sin C [by Ex. 1, Art. 127] .

If A + B + C = 2S, prove that


§ Problem 9.2.20. sin(S − A) sin(S − B)+sin S sin(S − C) = sin A sin B.

§§ Solution.
sin(S − A) sin(S − B) + sin S sin(S − C)
1
= [cos(B − A) − cos(2S − A − B) + cos C − cos(2S − C)]
2
1
= [cos(A − B) − cos C + cos C − cos(A + B)]
2
9.2. Identities 139

1
= [cos A cos B + sin A sin B − (cos A cos B − sin A sin B)]
2
= sin A sin B. ■

§ Problem 9.2.21.
4 sin S sin(S − A) sin(S − B) sin(S − B) sin(S − C)
= 1 − cos2 A − cos2 B − cos2 C + 2 cos A cos B cos C. ♢
§§ Solution.
4 sin S sin(S − A) sin(S − B) sin(S − B) sin(S − C)
= [cos A − cos(2S − A)] [cos(B − C) − cos(2S − B − C)]
= [cos A − cos(B + C)] [cos(B − C) − cos A]
= cos A [cos(B − C) + cos(B + C)]
− cos2 A − cos(B + C) cos(B − C)
1
= cos A · 2 cos B cos C − cos2 A − (cos 2B + cos 2C)
2
1( )
= 2 cos A cos B cos C − cos2 A − 2 cos2 B − 1 + 2 cos2 C − 1
2
= 1 − cos2 A − cos2 B − cos2 C + 2 cos A cos B cos C. ■

§ Problem 9.2.22.
sin(S − A) + sin(S − B) + sin(S − C) − sin S
A B C
= 4 sin sin sin . ♢
2 2 2
§§ Solution.
sin(S − A) + sin(S − B) + sin(S − C) − sin S
2S − A − B A−B C 2S − C
= 2 sin cos − 2 sin cos
2 2 2 2
C A−B C A+B
= 2 sin cos − 2 sin cos
2 [ 2 2 ]2
C A−B A+B
= 2 sin cos − cos
2 2 2
C A B
= 2 sin · 2 sin sin
2 2 2
A B C
= 4 sin sin sin . ■
2 2 2
§ Problem 9.2.23.
cos2 S + cos2 (S − A) + cos2 (S − B) + cos2 (S − C)
= 2 + 2 cos A cos B cos C. ♢
§§ Solution.
cos2 S + cos2 (S − A) + cos2 (S − B) + cos2 (S − C)
1
= [1 + cos 2S + 1 + cos(2S − 2A) + 1 + cos(2S − 2B)+
2
1 + cos(2S − 2C)]
1
= 2 + [cos(A + B + C) + cos(B + C − A)
2
+ cos(A + C − B) + cos(B + A − C)]
1
= 2 + [2 cos(B + C) cos A + 2 cos A cos(B − C)]
2
= 2 + cos A [cos(B + C) + cos(B − C)]
= 2 + cos A · 2 cos B cos C
= 2 + 2 cos A cos B cos C. ■
9.2. Identities 140

§ Problem 9.2.24.
cos2 A + cos2 B + cos2 C + 2 cos A cos B cos C
= 1 + 4 cos S cos(S − A) cos(S − B) cos(S − C). ♢
§§ Solution.
cos2 A + 2 cos A cos B cos C + cos2 B + cos2 C − 1
1
= cos2 A + 2 cos A cos B cos C + (cos 2B + cos 2C)
2
= cos2 A + cos A [cos(B + C) + cos(B − C)] + cos(B + C) cos(B − C)
= [cos A + cos(B + C)] [cos A + cos(B − C)]
= [cos A + cos(2S − A)] [cos(2S − B − C) + cos(B − C)]
= 4 cos S cos(S − A) cos(S − B) cos(S − C). ■

§ Problem 9.2.25. If α + β + γ + δ = 2π, prove that

(1)
cos α + cos β + cos γ + cos δ
α+β α+γ α+δ
+ 4 cos cos cos =0
2 2 2
(2)
sin α − sin β + sin γ − sin δ
α+β α+γ α+δ
+ 4 cos sin cos =0
2 2 2
(3)
tan α + tan β + tan γ + tan δ
= tan α tan β tan γ tan δ (cot α + cot β + cot γ + cot δ) . ♢
α+β γ+δ
§§ Solution. If α + β + γ + δ = 2π, then =π− .
2 2
(1)
cos α + cos β + cos γ + cos δ
α+β α−β γ+δ γ−δ
= 2 cos cos + 2 cos cos
2 ( 2 2 ) 2
α+β α−β γ−δ
= 2 cos cos − cos
2 [ 2 2 ]
α+β α − (2π − α − γ − δ) γ−δ
= 2 cos cos − cos
2 2 2
α+β α+γ−π π−α−δ
= 2 cos × 2 sin sin
2 [ Ä ( 2 )]
πä
2
α+β α+γ π α+δ
= 4 cos sin − sin −
2 ( 2 2 )2 2
α+β α+γ α+δ
= 4 cos − cos cos .
2 2 2
∴ cos α + cos β + cos γ + cos δ
α+β α+γ α+δ
+ 4 cos cos cos = 0.
2 2 2
(2)
sin α − sin β + sin γ − sin δ
α+β α+γ α+δ
+ 4 cos sin cos
2 2 2
9.2. Identities 141

α+β α−β γ+δ γ−δ


= 2 cos sin + 2 cos sin
2 2 2 2
α+β α+γ α+δ
+ 4 cos sin cos
2 ( 2 2 )
α+β α−β γ−δ α+γ α+δ
= 2 cos sin − sin + 2 sin cos
2 [ 2 2 ( 2 )2
α+β α−β γ−δ γ+δ
= 2 cos sin − sin + sin α +
2 2 ] 2 2
γ−δ
+ sin
[ 2 ( )]
α+β α−β γ+δ
= 2 cos sin + sin α +
2 [ 2 ( 2 )]
α+β α−β α+β
= 2 cos sin + sin α + π −
2 [ 2 ( )]
2
α+β α−β α−β
= 2 cos sin + sin π +
2 [ 2 ( )]
2
α+β α−β α−β
= 2 cos sin − sin = 0.
2 2 2
(3)
tan(α + β) = tan [2π − (γ + δ)]
tan α + tan β tan γ + tan δ
∴ =−
1 − tan α tan β 1 − tan γ tan δ
∴ tan α + tan β + tan γ + tan δ
= tan α tan γ tan δ + tan β tan γ tan δ
+ tan α tan β tan γ + tan α tan β tan δ
= tan α tan β tan γ tan δ (cot α + cot β + cot γ + cot δ) . ■

§ Problem 9.2.26. If the sum of four angles be 180◦ , prove that the
sum of the products of their cosines taken two and two together is
equal to the sum of the products of their sines taken similarly. ♢
§§ Solution. Let the four angles be A, B, C and D.
∵ A + B + C + D = 180◦ ∴ A + B = 180◦ − (C + D)
∴ cos(A + B) = − cos(C + D)
∴ cos A cos B − sin A sin B = − cos C cos D + sin C sin D
∴ cos A cos B + cos C cos D = sin A sin B + sin C sin D.
Similarly, cos A cos C + cos B cos D = sin A sin C + sin B sin D
and cos A cos D + cos B cos C = sin A sin D + sin B sin C.
Hence, by addition, we obtain the required result. ■

§ Problem 9.2.27. Prove that


sin 2α + sin 2β + sin 2γ
= 2 (sin α + sin β + sin γ) (1 + cos α + cos β + cos γ)
if α + β + γ = 0. ♢
§§ Solution. Since α + β = −γ, we have
sin(α + β) = − sin γ, and
cos(α + β) = cos γ.
sin 2α + sin 2β + sin 2γ
= 2 sin(α + β) cos(α − β) + 2 sin γ cos γ
= −2 sin γ cos(α − β) + 2 sin γ cos(α + β)
9.2. Identities 142

= −2 sin γ [cos(α − β) − cos(α + β)]


= −2 sin γ · 2 sin α sin β = −4 sin α sin β sin β
α α β β γ γ
= −32 sin cos sin cos sin cos
2 2 2 2 2 2
α β γ α β γ
= −32 sin sin sin · cos cos cos
[( 2 2 2 2 ) 2 ] 2
α−β α+β γ
= −8 cos − cos sin
[( 2 2 ) 2 ]
α+β α−β γ
cos + cos cos
[ 2 2 ]2
α−β α+β γ γ
= 8 cos sin + sin cos
[ 2 2 2 2]
γ α+β α−β
cos2 + cos cos
2 2 2
= 2 (sin α + sin β + sin γ) (1 + cos γ + cos α + cos β) . ■

§ Problem 9.2.28. Verify that


sin3 a sin(b − c) + sin3 b sin(c − a) + sin3 c sin(a − b)
+ sin(a + b + c) sin(b − c) sin(c − a) sin(a − b) = 0. ♢
§§ Solution. Since sin 3A = 3 sin A − 4 sin3 A, we have
sin3 a sin(b − c) + sin3 b sin(c − a) + sin3 c sin(a − b)
( ) ( )
3 1 3 1
= sin a − sin 3a sin(b − c) + sin b − sin 3b sin(c − a)
4 4 ( 4 )
4
3 1
+ sin c − sin 3c sin(a − b).
4 4
Now
sin a sin(b − c) + sin b sin(c − a) + sin c sin(a − b)
= sin a sin b cos c − sin a cos b sin c + sin b sin c cos a
− sin b cos c sin a + sin c sin a cos b − sin c cos a sin b = 0
∴ sin3 a sin(b − c) + sin3 b sin(c − a) + sin3 c sin(a − b)
1
= − [sin 3a sin(b − c) + sin 3b sin(c − a) + sin 3c sin(a − b)]
4
1
= [cos(3a + b − c) − cos(3a − b + c) + cos(3b + c − a)
8
− cos(3b − c + a) + cos(3c + a − b) − cos(3c − a + b)] .
Hence taking the first and last terms together, the second and third
terms together and the fourth and fifth terms together, within the
square brackets, we have
sin3 a sin(b − c) + sin3 b sin(c − a) + sin3 c sin(a − b)
1
= sin(a + b + c) [sin 2(c − a) + sin 2(a − b) + sin 2(b − c)] .
4
Now
sin 2(c − a) + sin 2(a − b) + sin 2(b − c)
= 2 sin(c − b) cos(c − 2a + b) + 2 sin(b − c) cos(b − c)
= 2 sin(b − c) [cos(b − c) − cos(c − 2a + b)]
= 2 sin(b − c) · 2 sin(b − a) sin(c − a)
= −4 sin(b − c) sin(c − a) sin(a − b).
Whence the required result follows. ■
9.2. Identities 143

If A, B, C and D be any angles, prove that


§ Problem 9.2.29.
sin A sin B sin(A − B) + sin B sin C sin(B − C)
+ sin C sin A sin(C − A)
+ sin(A − B) sin(B − C) sin(C − A) = 0. ♢
§§ Solution.
sin A sin B sin(A − B) + sin B sin C sin(B − C) + sin C sin A sin(C − A)
1
= sin B [cos B − cos(2A − B) + cos(2C − B) − cos B]
2
+ sin C sin A sin(C − A)
= − sin B sin(A − B + C) sin(C − A) + sin C sin A sin(C − A)
1
= − sin(C − A) [cos(A − 2B + C) − cos(A + C) − cos(C − A)
2
+ cos(A + C)]
= − sin(C − A) sin(C − B) sin(B − A)
= − sin(A − B) sin(B − C) sin(C − A)
∴ sin A sin B sin(A − B) + sin B sin C sin(B − C)
+ sin C sin A sin(C − A)
+ sin(A − B) sin(B − C) sin(C − A) = 0. ■

§ Problem 9.2.30.
sin(A − B) cos(A + B) + sin(B − C) cos(B + C)
+ sin(C − D) cos(C + D) + sin(D − A) cos(D + A) = 0. ♢
§§ Solution. The expression
1
= (sin 2A − sin 2B + sin 2B − sin 2C + sin 2C
2
− sin 2D + sin 2D − sin 2A) = 0. ■

§ Problem 9.2.31.
sin(A + B − 2C) cos B − sin(A + C − 2B) cos C
= sin(B − C) {cos(B + C − A) + cos(C + A − B) + cos(A + B − C)} . ♢
§§ Solution.
sin(A + B − 2C) cos B − sin(A + C − 2B) cos C
1
= [sin(A + 2B − 2C) + sin(A − 2C) − sin(A + 2C − 2B) − sin(A − 2B)]
2
by taking the first and third terms together and the second and
fourth terms together,
= cos A sin(2B − 2C) + cos(A − B − C) sin(B − C)
= sin(B − C) {2 cos A cos(B − C) + cos(A − B − C)}
= sin(B − C) {cos(A + B − C) + cos(A − B + C) + cos(B + C − A)} . ■

§ Problem 9.2.32.
sin(A + B + C + D) + sin(A + B − C − D) + sin(A + B − C + D)
+ sin(A + B + C − D) = 4 sin(A + B) cos C cos D.♢
§§ Solution. The expression
= 2 sin(A + B) cos(C + D) + 2 sin(A + B) cos(C − D)
= 2 sin(A + B) [cos(C + D) + cos(C − D)]
= 2 sin(A + B) · 2 cos C cos D
= 4 sin(A + B) cos C cos D. ■
9.2. Identities 144

§ Problem 9.2.33. If any theorem be true for values of A, B and C


such that
A + B + C = 180◦
prove that the theorem is still true if we substitute for A, B and C
respectively the quantities
A B C
(1) 90◦ − , 90◦ − and 90◦ − , or
2 2 2
(2) 180◦ − 2A, 180◦ − 2B and 180◦ − 2C.
Hence deduce § Problem 9.2.16 and § Problem 9.2.17. ♢
◦ A ◦ B ◦ C
§§ Solution. (1) If A1 = 90 − , B1 = 90 − and C1 = 90 − ,
2 2 2
then
A+B+C
A1 + B1 + C1 = 270◦ − = 270◦ − 90◦ = 180◦ .
2
(2) If A2 = 180◦ − 2A, B2 = 180◦ − 2B and C2 = 180◦ − 2C, then
A2 + B2 + C2 = 540◦ − 2(A + B + C) = 540◦ − 360◦ = 180◦ .
In § Problem 9.2.16, let
π α π β π γ
A= − , B= − , C= − .
2 2 2 2 2 2
α A π−α
Then cos A = sin and sin = sin .
2 2 4
Also, α + β + γ = 3π − 2(A + B + C) = 3π − 2π = π.
α β γ π−α π−β π−γ
∴ sin + sin + sin = 1 + 4 sin sin sin .
2 2 2 4 4 4
In § Problem 9.2.17, let
π α π β π γ
A= − , B= − , C= − .
2 2 2 2 2 2
α A π−α C π−γ
Then sin A = cos , sin = sin , cos = cos .
2 2 4 2 4

α β γ π−α π−β π−γ


∴ cos + cos − cos = 4 sin sin cos
2 2 2 4 4 4
π+α π+β π−γ
= 4 cos cos cos . ■
4 4 4
If x + y + z = xyz, prove that
§ Problem 9.2.34.
3x − x3 3y − y 3 3z − z 3 3x − x3 3y − y 3 3z − z 3
+ + = · · .
1 − 3x 2 1 − 3y 2 1 − 3z 2 1 − 3x2 1 − 3y 2 1 − 3z 2 ♢
§§ Solution. As in Ex. 5, Art. 127, if
x = tan A, y = tan B, and z = tan C,
we have A + B + C = nπ + π
3x − x3 3y − y 3 3z − z 3
∴ + +
1 − 3x2 1 − 3y 2 1 − 3z 2
3 tan A − tan3 A 3 tan B − tan3 B 3 tan C − tan3 C
= + +
1 − 3 tan A
2 1 − 3 tan B
2 1 − 3 tan2 C
= tan 3A + tan 3B + tan 3C
By a proof similar to that of Ex. 5, Art. 127
= tan 3A tan 3B tan 3C
9.3. Trigonometrical Equations 145

3x − x3 3y − y 3 3z − z 3
= · · .
1 − 3x2 1 − 3y 2 1 − 3z 2
Proof :
∵ A + B + C = (n + 1)π = mπ, say, where m is any integer,
∴ 3A + 3B + 3C = 3mπ, ∴ 3A + 3B = 3mπ − 3C
tan 3A + tan 3B
∴ tan(3A + 3B) = − tan 3C, ∴ = − tan 3C.
1 − tan 3A tan 3B
∴ tan 3A + tan 3B + tan 3C = tan 3A tan 3B tan 3C. ■

(
§ Problem 9.2.35. )( ) ( )( )
x 1 − y 2 1 − z 2 + y 1 − z 2 1 − x2
( )( )
+ z 1 − x2 1 − y 2 = 4xyz. ♢
§§ Solution. As in Ex. 5, Art. 127, we have
2x 2y 2z 2x 2y 2z
+ + = · ·
1 − x2 1 − y2 1 − z2 1 − x2 1 − y 2 1 − z 2
x y z 4xyz
+ + = .
1 − x2 1 − y2 1 − z2 (1 − x2 ) (1 − y 2 ) (1 − z 2 )
( )( ) ( )( )
x 1 − y2 1 − z2 + y 1 − z2 1 − x2
( ) ( )
+z 1−x 2
1−y2
= 4xyz. ■

9.3 Trigonometrical Equations


Solve the equations
§ Problem 9.3.1. sin θ + sin 7θ = sin 4θ. ♢
§§ Solution.
sin θ + sin 7θ = sin 4θ; ∴ 2 sin 4θ cos 3θ = sin 4θ
∴ sin 4θ = 0 or 2 cos 3θ = 1.
If sin 4θ = 0, then 4θ = nπ.
1 π
If 2 cos 3θ = 1, then cos 3θ = = cos
( 2 ) 3
π 1
∴ 3θ = 2nπ ± = 2n ± π.
3( )3
nπ 1 π
∴θ= or 2n ± . ■
4 3 3

§ Problem 9.3.2. cos θ + cos 7θ = cos 4θ. ♢


§§ Solution.
cos θ + cos 7θ = cos 4θ; ∴ 2 cos 4θ cos 3θ = cos 4θ
∴ cos 4θ = 0 or 2 cos 3θ = 1.
π π π π
If cos 4θ = 0, then 4θ = 2pπ ± = 4p × ± = (4p ± 1) .
2 2 2 ( )2
π π 1
= an odd multiple of = (2n + 1) = n + π.
( 2 ) 2 2
1
If 2 cos 3θ = 1, then 3θ = 2n ± π[ see last example].
( ) 3( )
1 π 1 π
∴θ = n+ or 2n ± . ■
2 4 3 3

§ Problem 9.3.3. cos θ + cos 3θ = 2 cos 2θ. ♢


9.3. Trigonometrical Equations 146

§§ Solution.
cos θ + cos 3θ = 2 cos 2θ; ∴ 2 cos 2θ cos θ = 2 cos 2θ
∴ cos 2θ = 0 or cos θ = 1.
( )
1
If cos 2θ = 0, then 2θ = n + π.[ see last example].
2
If cos θ = 1, then θ = 2nπ.
( )
1 π
∴θ = n+ or 2nπ. ■
2 2

§ Problem 9.3.4. sin 4θ − sin 2θ = cos 3θ. ♢


§§ Solution.
sin 4θ − sin 2θ = cos 3θ; ∴ 2 cos 3θ sin θ = cos 3θ
∴ cos 3θ = 0 or 2 sin θ = 1.
( )
1
If cos 3θ = 0 then 3θ = n + π.
2
1 π
If 2 sin θ = 1 then sin θ = = sin
2 6
π
∴ θ = nπ + (−1)n .
( ) 6
1 π π
∴θ = n+ or nπ + (−1)n . ■
2 3 6

§ Problem 9.3.5. cos θ − sin 3θ = cos 2θ. ♢


§§ Solution.
cos θ − sin 3θ = cos 2θ; ∴ cos θ − cos 2θ = sin 3θ
3θ θ 3θ 3θ
∴ 2 sin sin = 2 sin cos
2 2 2 2
3θ θ 3θ
∴ sin = 0 or sin = cos .
2 2 2
3θ 3θ
If sin = 0 then = nπ.
2 2( )
3θ θ π θ
If cos = sin = cos − ,
2 2 ( 2 ) 2
3θ π θ
we have = 2nπ ± − ;
2 2 2
Taking the upper sign,
π
2θ = 2nπ +
2
Taking the lower sign,
π
θ = 2nπ − .
( ) 2 ( )
2nπ 1 1
∴θ= or n + π or 2n − π. ■
3 4 2

§ Problem 9.3.6. sin 7θ = sin θ + sin 3θ. ♢


§§ Solution.
sin 7θ = sin θ + sin 3θ
∴ sin 7θ − sin θ = sin 3θ
∴ 2 cos 4θ sin 3θ = sin 3θ
∴ sin 3θ = 0 or 2 cos 4θ = 1.
If sin 3θ = 0 then 3θ = nπ.
9.3. Trigonometrical Equations 147

1 π
If 2 cos 4θ = 1, then cos 4θ = = cos
2 3
π
∴ 4θ = 2nπ ±
( 3 )
nπ 1 π
∴θ= or 2n ± . ■
3 3 4

§ Problem 9.3.7. cos θ + cos 2θ + cos 3θ = 0. ♢


§§ Solution.
cos θ + cos 2θ + cos 3θ = 0
∴ cos θ + cos 3θ + cos 2θ = 0
∴ 2 cos 2θ cos θ + cos 2θ = 0
∴ cos 2θ (2 cos θ + 1) = 0
∴ cos 2θ = 0 or 2 cos θ + 1 = 0.
( )
1
If cos 2θ = 0 then 2θ = n + π.
2
1 2π
If 2 cos θ + 1 = 0, then cos θ = − = cos
2 3

∴ θ = 2nπ ± .
( ) 3
1 π 2π
∴θ = n+ or 2nπ ± . ■
2 2 3

§ Problem 9.3.8. sin θ + sin 3θ + sin 5θ = 0. ♢


§§ Solution.
sin θ + sin 3θ + sin 5θ = 0
∴ sin θ + sin 5θ + sin 3θ = 0
∴ 2 sin 3θ cos 2θ + sin 3θ = 0
∴ sin 3θ (2 cos 2θ + 1) = 0
∴ sin 3θ = 0 or 2 cos 2θ + 1 = 0.
If sin 3θ = 0 then 3θ = nπ.
1 2π
If 2 cos 2θ + 1 = 0 then cos 2θ = − = cos
2 3

∴ 2θ = 2nπ ± .
( 3 )
nπ 1
∴θ= or n ± π. ■
3 3
§ Problem 9.3.9. sin 2θ − cos 2θ − sin θ + cos θ = 0. ♢
§§ Solution.
sin 2θ − cos 2θ − sin θ + cos θ = 0
∴ sin 2θ − sin θ = cos 2θ − cos θ
3θ θ 3θ θ
∴ 2 cos sin = −2 sin sin
2 2 2 2
θ 3θ 3θ
∴ sin = 0 or cos = − sin .
2 2 2
θ θ
If sin = 0 then = nπ.
2 2
3θ 3θ 3θ 3π
If cos = − sin then = nπ + .
2 2 ( 2 ) 4
2n 1
∴ θ = 2nπ or + π. ■
3 2
9.3. Trigonometrical Equations 148

§ Problem 9.3.10. sin(3θ + α) + sin(3θ − α) + sin(α − θ) − sin(α + θ) =


cos α. ♢
§§ Solution.
sin(3θ + α) + sin(3θ − α) + sin(α − θ) − sin(α + θ) = cos α
∴ 2 sin 3θ cos α − 2 cos α sin θ = cos α
1 1
∴ sin 3θ − sin θ = , ∴ 2 cos 2θ sin θ =
2 2
( ) 1
2 1 − 2 sin2 θ sin θ =
2
∴ 8 sin3 θ − 4 sin θ + 1 = 0
( )
∴ (2 sin θ − 1) 4 sin2 θ + 2 sin θ − 1 = 0
∴ 2 sin θ − 1 = 0 or 4 sin2 θ + 2 sin θ − 1 = 0.
1 π
If 2 sin θ − 1 = 0 then sin θ = = sin .
2 6
If 4 sin2 θ + 2 sin θ − 1 = 0
√ ( )
± 5−1 π 3π
then sin θ = = sin or sin −
4 10 10
π
∴ θ = nπ + (−1)n
6
π
or nπ + (−1)n
10

or nπ − (−1)n . ■
10
§ Problem 9.3.11. cos(3θ + α) cos(3θ − α) + cos(5θ + α) cos(5θ − α) =
cos 2α. ♢
§§ Solution.
cos(3θ + α) cos(3θ − α) + cos(5θ + α) cos(5θ − α) = cos 2α
∴ cos 6θ + cos 2α + cos 10θ + cos 2α = 2 cos 2α
∴ cos 6θ + cos 10θ = 0; ∴ 2 cos 8θ cos 2θ = 0
∴ cos 8θ = 0 or cos 2θ = 0.
π
If cos 8θ = 0, then 8θ = nπ + .
2
π
If cos 2θ = 0, then 2θ = nπ + .
( ) ( )2
1 π 1 π
∴θ = n+ or n + . ■
2 8 2 2
§ Problem 9.3.12. cos nθ = cos(n − 2)θ + sin θ. ♢
§§ Solution.
cos nθ = cos(n − 2)θ + sin θ
∴ cos nθ − cos(n − 2)θ = sin θ
∴ −2 sin(n − 1)θ sin θ = sin θ
∴ sin θ = 0 or 2 sin(n − 1)θ = −1.
If sin θ = 0 then θ = mπ, where m is any integer.
1 Ä πä
If 2 sin(n − 1)θ = −1 then sin(n − 1)θ = − = sin −
2 6
π
∴ (n − 1)θ = mπ − (−1)m .
1 î πó
6
∴ θ = mπ or mπ − (−1)m . ■
n−1 6
9.3. Trigonometrical Equations 149

n+1 n−1
§ Problem 9.3.13. sin θ = sin θ + sin θ. ♢
2 2
§§ Solution.
n+1 n−1
sin θ = sin θ + sin θ
2 2
n+1 n−1
∴ sin θ − sin θ = sin θ
2 2
nθ θ θ θ
∴ 2 cos sin = 2 sin cos .
2 2 2 2
θ nθ θ
∴ sin = 0 or cos = cos .
2 2 2
θ θ
If sin = 0 then = mπ.
2 2
nθ θ nθ θ
If cos = cos , then = 2mπ ±
2 2 2 2
taking the upper sign,
(n − 1)θ = 4mπ
taking the lower sign,
(n + 1)θ = 4mπ.
4mπ
∴ θ = 2mπ or . ■
n∓1
§ Problem 9.3.14. sin mθ + sin nθ = 0. ♢
§§ Solution.
sin mθ + sin nθ = 0
m+n m−n
∴ 2 sin θ · cos θ=0
2 2
m+n m−n
∴ sin θ = 0 or cos θ = 0.
2 2
m+n m+n
If sin θ = 0 then θ = rπ,
2 2
where r is any integer.
m−n m−n π
If cos θ = 0, then θ = rπ + .
2 2 2
2rπ π
∴θ= or (2r + 1) . ■
m+n m−n
§ Problem 9.3.15. cos mθ + cos nθ = 0. ♢
§§ Solution.
cos mθ + cos nθ = 0 [Cf. §P roblem 6.1.28.]
m+n m−n
∴ 2 cos θ · cos θ=0
2 2
m+n m−n
∴ cos θ = 0 or cos θ = 0.
2 2
m+n m+n π
If cos θ = 0 then θ = rπ + .
2 2 2
m−n m−n π
If cos θ = 0, then θ = rπ + .
2 2 2
π
∴ θ = (2r + 1) . ■
m±n
§ Problem 9.3.16. sin2 nθ − sin2 (n − 1)θ = sin2 θ. ♢
9.3. Trigonometrical Equations 150

§§ Solution.
sin2 nθ − sin2 (n − 1)θ = sin2 θ
∴ sin [nθ + (n − 1)θ] sin [nθ − (n − 1)θ] = sin2 θ, by Ex. 2, Art. 93
∴ sin(2n − 1)θ sin θ = sin2 θ
∴ sin θ = 0 or sin(2n − 1)θ = sin θ.
If sin θ = 0 then θ = mπ.
If sin(2n − 1)θ = sin θ,
then sin(2n − 1)θ − sin θ = 0
∴ 2 cos nθ sin(n − 1)θ = 0
∴ cos nθ = 0 or sin(n − 1)θ = 0
π
If cos nθ = 0 then nθ = mπ +
2
If sin(n − 1)θ = 0, then (n − 1)θ = mπ.
( )
1 π mπ
∴ θ = mπ or m + or . ■
2 n n−1
§ Problem 9.3.17. sin 3θ + cos 2θ = 0. ♢
§§ Solution.
sin 3θ + cos 2θ = 0
Äπ ä
∴ cos 2θ − cos + 3θ = 0
( ) 2( )
π 5θ π θ
∴ 2 sin + sin + =0
( 4 ) 2 4( 2 )
π 5θ π θ
∴ sin + = 0 or sin + = 0.
( 4 2 ) 4 2
π 5θ π 5θ
If sin + = 0 then + = nπ.
(4 2 ) 4 2
π θ π θ
If sin + = 0 then + = nπ.
4 2 4 2
1Ä πä π
∴θ= 2nπ − or 2nπ − . ■
5 2 2
√ √
§ Problem 9.3.18. 3 cos θ + sin θ = 2. ♢
√ √
§§ Solution. 3 cos θ + sin θ = 2.

Dividing both sides of the equation by 3 + 1, i.e. 2, we have

3 1 1
cos θ + sin θ = √ (9.1)
2 2 2
Äπ ä π
∴ sin + θ = sin
Äπ ä 3 4
π
∴ + θ = nπ + (−1)n
3 4
nπ π
∴ θ = nπ + (−1) − (9.2)
4 3
Otherwise thus :
From Eq. (9.1), we have
Ä πä π π π
cos θ − = cos ; ∴ θ − = 2nπ ±
6 4 6 4
5π π
∴ θ = 2nπ + or 2nπ − (9.3)
12 12
9.3. Trigonometrical Equations 151

To show that Eq. (9.2) and Eq. (9.3) give the same values for θ :
in Eq. (9.2) let n be even (= 2m, say) and we have
π π π
θ = 2mπ + − = 2mπ −
4 3 12
let n be odd (= 2m + 1, say) and we have
( )
π π 7π 5π
θ = (2m + 1)π − − = 2mπ + π − = 2mπ + . ■
4 3 12 12

§ Problem 9.3.19. sin θ + cos θ = 2. ♢

§§ Solution. sin θ + cos θ = 2.
√ √
Dividing both sides of the equation by 1 + 1, i.e. 2, we have
1 1
√ sin θ + √ cos θ = 1.
Ä 2 2 ( )
πä π 1
∴ cos θ − = 1; θ − = 2nπ i.e. θ = 2n + π.
4 4 4
Otherwise thus : Ä πä
we have sin θ + = 1.
4
π π
∴ θ + = 2nπ +
4( ) 2
1
∴ θ = 2n + π. ■
4
√ √
§ Problem 9.3.20. 3 sin θ − cos θ = 2. ♢
√ √
§§ Solution. 3 sin θ − cos θ = 2.

Dividing both sides of the equation by 3 + 1, i.e. 2, we have

3 1 1
sin θ − cos θ = √ .
2 2
Ä πä π π
2
π
∴ sin θ − = sin ; θ − = nπ + (−1)n
6 4 6 4
π π
∴ θ = nπ + (−1)n + . ■
4 6

§ Problem 9.3.21. sin x + cos x = 2 cos A. ♢

§§ Solution. sin x + cos x = 2 cos A.
√ √
Dividing both sides of the equation by 1 + 1, i.e. 2, we have
1 1
√ sin x + √ cos x = cos A.
2 Ä 2π ä
∴ cos x − = cos A
4
π
∴ x − = 2nπ ± A
4
π
∴ x = 2nπ + ± A. ■
4
§ Problem 9.3.22. 5 sin θ + 2 cos θ = 5 (given tan 21◦ 48′ = ·4). ♢
§§ Solution. 5 sin θ + 2 cos θ = 5.
√ √
Dividing both sides of the equation by 52 + 22 , i.e. 29, we have
5 2 5
√ sin θ + √ cos θ = √ .
29 29 29
∴ cos 21◦ 48′ sin θ + sin 21◦ 48′ cos θ = cos 21◦ 48′
( ) ( )
∴ sin θ + 21◦ 48′ = cos 21◦ 48′ = sin 90◦ − 21◦ 48′ = sin 68◦ 12′
( )
∴ θ + 21◦ 48′ = n × 180◦ + (−1)n 68◦ 12′
9.3. Trigonometrical Equations 152

( )
∴ θ = −21◦ 48′ + n × 180◦ + (−1)n 68◦ 12′ . ■

§ Problem 9.3.23. 6 cos x + 8 sin x = 9 (given tan 53◦ 8′ = 1 13 and


cos 25◦ 50′ = ·9). ♢
§§ Solution. 6 cos x + 8 sin x = 9.

Dividing both sides of the equation by 62 + 82 , i.e. 10, we have
3 4 9
cos x + sin x = .
5 5 10
◦ ′ ◦ ′
∴ cos 53 8 cos x + sin 53 8 sin x = ·9
( )
∴ cos x − 53◦ 8′ = cos 25◦ 50′
∴ x − 53◦ 8′ = 2n × 180◦ ± 25◦ 50′
∴ x = 2n × 180◦ + 78◦ 58′ or 2n × 180◦ + 27◦ 18′ . ■

§ Problem 9.3.24. 1 + sin2 θ = 3 sin θ cos θ (given tan 71◦ 34′ = 3). ♢
§§ Solution. 1 + sin2 θ = 3 sin θ cos θ.
∴ 2 + 2 sin2 θ = 6 sin θ cos θ
∴ 2 + 1 − cos 2θ = 3 sin 2θ
∴ cos 2θ + 3 sin 2θ = 3.
√ √
Dividing both sides of this last equation by 1 + 32 , i.e. 10, we have
1 3 3
√ cos 2θ + √ sin 2θ = √ .
10 10 10
∴ cos 71◦ 34′ cos 2θ + sin 71◦ 34′ sin 2θ = sin 71◦ 34′
( ) ( )
∴ cos 2θ − 71◦ 34′ = cos 90◦ − 71◦ 34′ = cos 18◦ 26′
∴ 2θ − 71◦ 34′ = 2n × 180◦ ± 18◦ 26′

∴ 2θ = 2n × 180◦ + 90◦ or 2n × 180◦ + 52◦ 8′


∴ θ = n × 180◦ + 45◦ or n × 180◦ + 26◦ 34′ . ■


§ Problem 9.3.25. cosec θ = cot θ + 3. ♢

§§ Solution. cosec θ = cot θ + 3.
1 cos θ √
∴ = + 3
sin θ √sin θ
∴ cos θ + 3 sin θ = 1.

Dividing both sides of this last equation by 1 + 3, i.e. 2, we have

1 3 1
cos θ + sin θ = .
2 Ä 2π ä 2
π
∴ cos θ − = cos
3 3
π π
∴ θ − = 2nπ ±
3 3

∴ θ = 2nπ or 2nπ + . ■
3
§ Problem 9.3.26. cosec x = 1 + cot x. ♢
§§ Solution. cosec x = 1 + cot x.
1 cos x
∴ =1+ ; ∴ cos x + sin x = 1.
sin x sin x √ √
Dividing both sides of this last equation by 1 + 1, i.e. 2, we have
1 1 1
√ cos x + √ sin x = √ .
2 2 2
9.3. Trigonometrical Equations 153

Ä πä π
∴ cos x − = cos
4 4
π π
∴ x − = 2nπ ±
4 4
π
∴ x = 2nπ or 2nπ + . ■
2
( √ )
§ Problem 9.3.27. 2 + 3 cos θ = 1 − sin θ. ♢
( √ )
§§ Solution. 2 + 3 cos θ = 1 − sin θ.
Äπ ä
√ 1 + cos +θ ( )
1 − sin θ π θ
∴2+ 3= = Äπ 2 ä = cot + [§P roblem 8.1.4]
cos θ sin +θ 4 2
( 2 )
π π θ π θ π
∴ cot = cot + ; ∴ + = nπ +
12 4 2 4 2 12
θ π π
∴ = nπ − ; ∴ θ = 2nπ − .
2 6 3
Also, both sides of the original equation become zero if
π
cos θ = 0 and 1 − sin θ = 0, i.e. if θ = 2nπ + . ■
2

§ Problem 9.3.28. tan θ + sec θ = 3. ♢

§§ Solution. tan θ + sec θ = 3.
sin θ 1 √
∴ + = 3
√cos θ cos θ
∴ 3 cos θ − sin θ = 1.

Dividing both sides of this last equation by 3 + 1, i.e. 2, we have

3 1 1
cos θ − sin θ =
2 Ä 2ä 2
π π
∴ cos θ + = cos
6 3
π π
∴ θ + = 2nπ ±
6 3
π π
∴ θ = 2nπ + or 2nπ − . ■
6 2
§ Problem 9.3.29. cos 2θ = cos2 θ. ♢
§§ Solution. cos 2θ = cos2 θ.
∴ cos2 θ − sin2 θ = cos2 θ; ∴ sin2 θ = 0; ∴ θ = nπ. ■

§ Problem 9.3.30. 4 cos θ − 3 sec θ = tan θ. ♢


§§ Solution. 4 cos θ − 3 sec θ = tan θ.
3 sin θ
∴ 4 cos θ − =
cos(θ cos θ )
∴ 4 cos2 θ − 3 = sin θ; ∴ 4 1 − sin2 θ − 3 = sin θ
∴ 4 sin2 θ + sin θ − 1 = 0
√ √
−1 ± 1 + 16 ± 17 − 1
∴ sin θ = = . ■
8 8
§ Problem 9.3.31. cos 2θ + 3 cos θ = 0. ♢
9.3. Trigonometrical Equations 154

§§ Solution. cos 2θ + 3 cos θ = 0.


∴ 2 cos2 θ − 1 + 3 cos θ = 0
√ √
−3 ± 9 + 8 ± 17 − 3
∴ cos θ = = .
√ 4 4√
− 17 − 3 17 − 3
The value is inadmissible, ∴ cos θ = . ■
4 4

§ Problem 9.3.32. cos 3θ + 2 cos θ = 0. ♢


§§ Solution. cos 3θ + 2 cos θ = 0.
∴ 4 cos3 θ − 3 cos θ + 2 cos θ = 0 [Art. 107]
( )
∴ 4 cos3 θ − cos θ = 0; ∴ cos θ 4 cos2 θ − 1 = 0
∴ cos θ = 0 or 4 cos2 θ − 1 = 0.
π
If cos θ = 0 then θ = nπ + .
2
1 π
If 4 cos2 θ − 1 = 0 then cos θ = ± = ± cos .
( ) 2 3
1 π
∴θ = n+ π or nπ ± . ■
2 3
(√ )( 1
)
§ Problem 9.3.33. cos 2θ = 2 + 1 cos θ − √ . ♢
(√ )( 1
) 2
§§ Solution. cos 2θ = 2 + 1 cos θ − √ .
2
(√ ) 1
∴ 2 cos θ − 1 =
2
2 + 1 cos θ − 1 − √
2
(√ ) 1
∴ 2 cos θ −
2
2 + 1 cos θ + √ = 0
( )2
1
∴ (2 cos θ − 1) cos θ − √ = 0.
2
1
∴ 2 cos θ − 1 = 0 or cos θ − √ = 0.
2
1 π
If 2 cos θ − 1 = 0, then cos θ = = cos .
2 3
1 1 π
If cos θ − √ = 0, then cos θ = √ = cos .
2 2 4
π π
∴ θ = 2nπ ± or 2nπ ± . ■
3 4
§ Problem 9.3.34. cot θ − tan θ = 2. ♢
§§ Solution. cot θ − tan θ = 2.
cos θ sin θ
− = 2; ∴ cos2 θ − sin2 θ = 2 sin θ cos θ
sin θ cos θ Äπ ä Äπ ä
∴ cos 2θ = sin 2θ = cos − 2θ ; ∴ 2θ = 2nπ ± − 2θ
2 ( ) 2
π 1 π
∴ 4θ = 2nπ + ; ∴ θ = n + .
2 4 2
Otherwise thus :
By § Problem 8.1.5:
cot θ − tan θ = 2 cot 2θ
π
∴ 2 cot 2θ = 2; ∴ cot 2θ = 1 = cot
4
9.3. Trigonometrical Equations 155

( )
π 1 π
∴ 2θ = nπ + ; ∴θ= n+ . ■
4 4 2
§ Problem 9.3.35. 4 cot 2θ = cot2 θ − tan2 θ. ♢
§§ Solution. 4 cot 2θ = cot2 θ − tan2 θ.
∴ 2 (cot θ − tan θ) = cot2 θ − tan2 θ [see last example]
∴ cot θ − tan θ = 0 or 2 = cot θ + tan θ.
1
If cot θ − tan θ = 0, then − tan θ = 0
tan θ
π
∴ 1 − tan θ = 0; ∴ tan θ = ±1, i.e. θ = nπ ± .
2
4
1
If cot θ + tan θ = 2, then + tan θ = 2
tan θ
∴ tan2 θ − 2 tan θ + 1 = 0, i.e. (tan θ − 1)2 = 0
∴ tan θ = 1, which is included in the former solution.
Otherwise thus :
The given equation may be written
4 1
= − tan2 θ
(
tan 2θ )
tan2θ

4 1 − tan2 θ 1 − tan4 θ
∴ =
2 tan θ tan2 θ
1 + tan2 θ
∴ 1 − tan θ = 0 or 2 =
2
,
tan θ

and the solution follows as before.

§ Problem 9.3.36. 3 tan (θ − 15◦ ) = tan (θ + 15◦ ). ♢


§§ Solution. 3 tan (θ − 15◦ ) = tan (θ + 15◦ ).
tan (θ − 15◦ ) 1
∴ = .
tan (θ + 15◦ ) 3
tan (θ − 15◦ ) sin (θ − 15◦ ) sin (θ + 15◦ )
Now ◦
= ◦
÷
tan (θ + 15 ) cos (θ − 15 ) cos (θ + 15◦ )
2 cos (θ + 15◦ ) sin (θ − 15◦ )
=
2 sin (θ + 15◦ ) cos (θ − 15◦ )
sin 2θ − sin 30◦
=
sin 2θ + sin 30◦
sin 2θ − sin 30◦ 1 2 sin 2θ 4
∴ = ; ∴ =
sin 2θ + sin 30◦ 3 2 sin 30◦ 2
1
∴ sin 2θ = 2 × sin 30◦ = 2 × = 1
2
π π
∴ 2θ = 2nπ + ; ∴ θ = nπ + . ■
2 4
§ Problem 9.3.37. tan θ + tan 2θ + tan 3θ = 0. ♢
§§ Solution. tan θ + tan 2θ + tan 3θ = 0.
sin θ sin 2θ sin 3θ
∴ + + =0
cos θ cos 2θ cos 3θ
sin θ cos 3θ + sin 3θ cos θ sin 2θ
∴ + =0
cos θ cos 3θ cos 2θ
sin(θ + 3θ) sin 2θ
∴ + =0
cos θ cos 3θ cos 2θ
sin 4θ sin 2θ
∴ + =0
cos θ cos 3θ cos 2θ
9.3. Trigonometrical Equations 156

2 sin 2θ cos 2θ sin 2θ


∴ + =0
cos θ cos 3θ cos 2θ

∴ either sin 2θ = 0, ∴ 2θ = nπ, i.e. θ = ; or
2
2
2 cos 2θ + cos θ cos 3θ = 0.
∴ 2 (1 + cos 4θ) + cos 4θ + cos 2θ = 0
∴ 3 cos 4θ + cos 2θ + 2 = 0
( )
∴ 3 2 cos2 2θ − 1 + cos 2θ + 2 = 0
∴ 6 cos2 2θ + cos 2θ − 1 = 0
∴ (2 cos 2θ + 1) (3 cos 2θ − 1) = 0
∴ 2 cos 2θ + 1 = 0 or 3 cos 2θ − 1 = 0.
1 2π
If 2 cos 2θ + 1 = 0, then cos 2θ = − = cos
2 3
2π π
∴ 2θ = 2nπ ± , i.e. θ = nπ ±
3 3
1
If 3 cos 2θ − 1 = 0, then cos 2θ =
3
1 α
∴ 2θ = 2nπ ± α, where cos α = , i.e. θ = nπ ± . ■
3 2
√ √
§ Problem 9.3.38. tan θ + tan 2θ + 3 tan θ tan 2θ = 3. ♢
√ √
§§ Solution. tan θ + tan 2θ + 3 tan √ θ tan 2θ = 3.
∴ tan θ + tan 2θ = 3 (1 − tan θ tan 2θ)
tan θ + tan 2θ √ π
∴ = 3 = tan
1 − tan θ tan 2θ 3
π
∴ tan 3θ = tan [Art. 107]
3 ( )
π 1 π
3θ = nπ + ; ∴ θ = n + . ■
3 3 3
§ Problem 9.3.39. sin 3α = 4 sin α sin(x + α) sin(x − α). ♢
§§ Solution. sin 3α = 4 sin α sin(x + α) sin(x − α).
( )
∴ 3 sin α − 4 sin3 α = 4 sin α sin2 x − sin2 α [By Arts. 107 and 93]
3
∴ 3 sin α = 4 sin α sin2 x; ∴ sin2 x =
√ 4
3 Ä πä π
∴ sin x = ± = sin ± ; ∴ x = nπ ± . ■
2 3 3
§ Problem 9.3.40. Prove that the equation x3 −2x+1 = 0 is satisfied
by putting for x either
√ of the values
2 sin 45◦ , 2 sin 18◦ and 2 sin 234◦ . ♢
§§ Solution. x3 − 2x + 1 = 0. ( )
∴ (x − 1) x2 + x − 1 = 0
∴ x − 1 = 0 or x2 + x − 1 = 0.
If x − 1 = 0 then x = 1.

± 5−1
If x2 + x − 1 = 0 then x = .
2
√ √ 1
Now 2 sin 45◦ = 2 × √ = 1
√ √2
◦ 5−1 5−1
2 sin 18 = 2 × = and
4 2
9.3. Trigonometrical Equations 157

√ √
5+1 − 5−1
2 sin 234◦ = −2 sin 54◦ = −2 × = . ■
4 2
§ Problem 9.3.41. If sin (π cos θ) = cos (π sin θ), prove that
Ä πä 1
cos θ ± = √ . ♢
4 2 2
§§ Solution. sin (π cos θ) = cos (π sin θ).
Äπ ä
∴ cos − π cos θ = cos (π sin θ)
2
π
∴ − π cos θ = 2nπ ± π sin θ, where n is any integer.
2
1
∴ cos θ ± sin θ = − 2n
2
1 1 1 − 4n
∴ cos θ · √ ± sin θ · √ = √
Ä2 π ä 2
1 − 4n
2 2
∴ cos θ ∓ = √ .
4 2 2
Now n must be zero; for otherwise, since it is an integer, the right-
hand member would be numerically greater than unity
Ä πä 1
cos θ ± = √ . ■
4 2 2
§ Problem 9.3.42. If sin (π cot θ) = cos (π tan θ), prove that either
1
cosec 2θ or cot 2θ is equal to n + where n is a positive or negative
4
integer. ♢
§§ Solution. sin (π cot θ) = cos (π tan θ).
Äπ ä
∴ cos (π tan θ) = cos − π cot θ .
Ä π2 ä
∴ π tan θ = 2nπ ± − π cot θ ,
2
where n is zero or some positive or negative integer.
Taking the upper sign,
1 1 2
2n + = tan θ + cot θ = =
2 sin θ cos θ sin 2θ
1 1
∴ = cosec 2θ = n + .
sin 2θ 4
Taking the lower sign,
1 2 cos 2θ 1
2n − = tan θ − cot θ = − ; ∴ cot 2θ = − n.
2 sin 2θ 4
1
Thus either cosec 2θ or cot 2θ is equal n + . ■
4
Chapter 10
Logarithms

10.1 Characteristics, Mantissa and Proper-


ties
§ Problem 10.1.1. Given log 4 = .60206 and log 3 = .4771213, find the
logarithms of
1
.8, .003, .0108 and (.00018) 7 . ♢
8 23
§§ Solution. .8 = =
10 10 (√ )
∴ log(.8) = 3 log 2 − log 10 = 3 log 4 − log 10
3
= log 4 − log 10 = .90309 − 1 = 1̄.90309
2
3 3
.003 = =
1000 103
∴ log(.003) = log 3 − 3 log 10 = .4771213 − 3 = 3̄.4771213
108 4 × 27 4 × 33
.0108 = = =
104 104 104
∴ log(.0108) = log 4 + 3 log 3 − 4 log 10
= .60206 + 1.4313639 − 4 = 2̄.0334239.

18 2 × 32 4 × 32
.00018 = = =
( )
10 5 10 5 105
1 1 1
∴ log(.00018) 7 = log 4 + 2 log 3 − 5 log 10
7 2
1
= (.30103 + .9542426 − 5) = 1̄.4650389. ■
7
§ Problem 10.1.2. Given log 11 = 1.0413927 and log 13 = 1.1139434,
find the values of
10.1. Characteristics, Mantissa and Properties 160

(1) log 1.43

(2) log 133.1



(3) log 4 143, and

(4) log 3 .00169.

143 11 × 13
§§ Solution. (1) 1.43 = =
100 102
∴ log 1.43 = log 11 + log 13 − 2 log 10
= 1.0413927 + 1.1139434 − 2 = .1553361.

1331 113
(2) 133.1 = =
10 10
∴ log 133.1 = 3 log 11 − log 10 = 3.1241781 − 1 = 2.1241781.

(3)

4 √ 1
log 143 = log 4
11 × 13 = (log 11 + log 13)
4
1
= (2.1553361) = .5288340.
4
169 132
(4) .00169 = 5
=
10 105
√3 1 1
∴ log .00169 = (2 log 13 − 5 log 10) = (2.2278868 − 5)
3 3
1( )
= 3̄.2278868 = 1̄.0759623. ■
3
§ Problem 10.1.3. What are the characteristics of the logarithms
√ 3
of 243.7, .0153, 2.8713, .00057, .023, 5 24615 and (24589) 4 ? ♢
§§ Solution.
log 243.7 = 2. . . . [Art. 142, (i)] .
log(.0153) = 2̄. . . . [Art. 142, (ii)] .
log 2.8713 = 0. . . . [Art. 142, (i)] .
log(.00057) = 4̄. . . . [Art. 142, (ii)] .
log(.023) = 2̄. . . . [Art. 142, (ii)] .
log 24615 = 4. . . . [Art. 142, (i)] .
√5 1
∴ log 24615 = log 24615 = 0. . . .
5
log 24589 = 4. . . . [Art. 142, (i)] .
3 3
∴ log (24589) 4 = log 24589 = 3. . . . ■
4
§ Problem 10.1.4. Find the 5th root of .003, having given log 3 =
.4771213 and
log 312936 = 5.4954243. ♢
§§ Solution.
1 1
log(.003) 5 = (3.4771213) , by §P roblem 10.1.1
5
1( )
= 5̄ + 2.4771213 = 1̄.4954243 = log(.312936)
5
1
∴ (.003) 5 = .312936. ■
10.1. Characteristics, Mantissa and Properties 161

§ Problem 10.1.5. Find the value of


1
(1) 7 7
2
(2) (84) 5 , and
1
(3) (.021) 5

having given
log 2 = .30103, log 3 = .4771213
log 7 = .8450980, log 132057 = 5.1207283
log 588453 = 5.7697117 and log 461791 = 5.6644438. ♢
§§ Solution. (1)
1 1 1
log 7 7 = log 7 = (.8450980)
7 7
= .1207283 = log 1.32057
1
∴ 7 7 = 1.32057.

(2)
84 = 12 × 7 = 22 × 3 × 7
2 2
∴ log(84) 5 = (2 log 2 + log 3 + log 7)
5
2
= (.60206 + .4771213 + .8450980)
5
2
= (1.9242793) = .7697117 = log 5.88453
5
2
∴ (84) 5 = 5.88453.
1 21 3×7
(3) (.021) 5 = =
103 103
1 1 1( )
∴ log(.021) 5 = (log 3 + log 7 − 3 log 10) = 2̄.3222193
5 5
1( )
= 5̄ + 3.3222193 = 1̄.6644439 = log(.461791)
5
1
∴ (.021) 5 = .461791. ■

§ Problem 10.1.6. Having given log 3 = .4771213, find the number


of digits in

(1) 343

(2) 327 and

(3) 362

and the position of the first significant figure in

(1) 3−13

(2) 3−43 and

(3) 3−65 . ♢
§§ Solution. (1) log 343 = 43 log 3 = 43 × .4771213 = 20.5162159.
Hence there are 21 digits in 343 .
10.1. Characteristics, Mantissa and Properties 162

(2) log 327 = 27 log 3 = 27 × .4771213 = 12.8822751.


Hence there are 13 digits in 327 .
(3) log 362 = 62 log 3 = 62 × .4771213 = 29.5815206.
Hence there are 30 digits in 362 .
(4) log 3−13 = −13 log 3 = −13 × .4771213 = −6.2025769 = 7̄.7974231.
Hence the first significant figure is in the seventh place of dec-
imals.
(5) log 3−43 = −43 log 3 = −43×.4771213 = −20.5162159 = 21.4837841.
¯
Hence the first significant figure is in the twenty-first place of
decimals.
(6) log 3−65 = −65 log 3 = −65×.4771213 = −31.0128845 = 32.9871155.
¯
Hence the first significant figure is in the thirty-second place

of decimals.
§ Problem 10.1.7. Given log 2 = .30103, log 3 = .4771213 and log 7 =
.8450980, solve the equations
(1) 2x · 3x+4 = 7x
(2) 22x+1 · 33x+2 = 74x
(3) 72x ÷ 2x−4 = 33x−7 , and
(4) ™
7x+y × 32x+y = 9
3x−y ÷ 2x−2y = 3x ♢
§§ Solution. (1) 2x · 3x+4 = 7x .
Taking logarithms of both sides, we have
x log 2 + (x + 4) log 3 = x log 7
∴ x (log 2 + log 3 − log 7) = −4 log 3
4 log 3
∴x=
log 7 − (log 2 + log 3)
1.9084852 1.9084852
= = = 28.5 . . .
.8450980 − .7781513 .0669467
(2) 22x+1 · 33x+2 = 74x
Taking logarithms of both sides, we have
(2x + 1) log 2 + (3x + 2) log 3 = 4x log 7
log 2 + 2 log 3
∴x=
4 log 7 − (2 log 2 + 3 log 3)
1.2552726 1.2552726
= = = .93 . . .
3.3803920 − 2.0334239 1.3469681
(3) 72x ÷ 2x−4 = 33x−7
Taking logarithms of both sides, we have
2x log 7 − (x − 4) log 2 = (3x − 7) log 3
7 log 3 + 4 log 2
∴x=
3 log 3 + log 2 − 2 log 7
3.3398491 + 1.20412 4.5439691
= = = 107.68 . . .
1.7323939 − 1.6901960 .0421979
10.1. Characteristics, Mantissa and Properties 163

(4) We have
}
(x + y) log 7 + (2x + y) log 3 = 2 log 3
(x − y) log 3 + (2y − x) log 2 = x log 3
or }
(x + y)c + (2x + y)b = 2b
(x − y)b + (2y − x)a = xb
where a = log 2, b = log 3 and c = log 7
}
∴ (c + 2b)x + (c + b)y = 2b
ax + (b − 2a)y = 0
Solving these equations, we have
2b(2a − b) 2ab
x= and y = . ■
5ab + 3ac − 2b2 − bc 5ab + 3ac − 2b2 − bc
§ Problem 10.1.8. From the tables, find the seventh root of .000026751.

§§ Solution.
log(.000026751) = 5̄.4273400
1 1( )
∴ log(.000026751) 7 = 5̄.4273400
7
1( )
= 7̄ + 2.4273400
7
= 1̄.3467629 ≈ log(.22221)
1
∴ (000026751) 7 ≈ .22221. ■

Making use of the tables, find the approximate values of



§ Problem 10.1.9. 3 645.3. ♢
§§ Solution.
log 645.3 = 2.8097617

3 1
∴ log 645.3 = (2.8097617)
3
= .9365872 ≈ log 8.6415
√3
∴ 645.3 ≈ 8.6415. ■


§ Problem 10.1.10. 5 82357. ♢
§§ Solution.
log 82357 = 4.9157005
√5 1
∴ log 82357 = (4.9157005)
5
= .9831401 ≈ log 9.6192

5
∴ 82357 ≈ 9.6192. ■

√ √
5× 37
§ Problem 10.1.11. √ √ . ♢
4
8× 59
√ √
5× 37
§§ Solution. Let x = √ √ .
4
8× 59 ( )
1 1 1 1
∴ log x = log 5 + log 7 − log 8 + log 9
2 3 4 5
= .6311843 − .4166210 = .2145633 ≈ log 1.6389
∴ x ≈ 1.6389. ■
10.1. Characteristics, Mantissa and Properties 164


7.2 × 8.3
§ Problem 10.1.12. 3
. ♢
9.4 ÷ 16.5

3 7.2 × 8.3
§§ Solution. Let x = .
9.4 ÷ 16.5
1
∴ log x = [log 7.2 + log 8.3 − (log 9.4 − log 16.5)]
3
1
= [(log 7.2 + log 8.3 + log 16.5) − log 9.4]
3
1
= [2.9938945 − .9731279]
3
1
= (2.0207666) = .6735889 ≈ log 4.7162
3
∴ x ≈ 4.7162. ■

 
1 1
8 5 × 11 3
§ Problem 10.1.13. √ √ . ♢
74 × 5 62
 
1 1
8 5 × 11 3
§§ Solution. Let x = √ √ .
74 × 5 62
[ ( )]
1 1 1 1 1
∴ log x = log 8 + log 11 − log 74 + log 62
2 5 3 2 5
1 1( )
= [.5277489 − 1.2930942] = 1̄.2346547
2 2
1( )
= 2̄ + 1.2346547 = 1̄.6173274 ≈ log(.41431)
2
∴ x ≈ .41431. ■
Chapter 11
Tables of Logarithms And
Trigonometrical Ratios, Principle
of Proportional Parts

11.1 Proportional Parts


§ Problem 11.1.1.
Given log 35705 = 4.5527290
and log 35706 = 4.5527412
find the values of log 35705.7 and log 35.70585. ♢
§§ Solution.
The difference for 1 = 4.5527412 − 4.5527290 = .0000122.
∴ the difference for .7 = .0000085
∴ log 35705.7 = 4.5527290 + .0000085 = 4.5527375.
Again, we have log 35.705 = 1.5527290
and log 35.706 = 1.5527412.
Hence the difference for .001 = .0000122
∴ the difference for .00085 = .0000104
∴ log 35.70585 = 1.5527290 + .0000104 = 1.5527394. ■

§ Problem 11.1.2.
Given log 5.8743 = .7689487
and log 587.44 = 2.7689561
find the values of log 58743.57 and log .00587432. ♢
§§ Solution.
Given log 58743 = 4.7689487
and log 58744 = 4.7689561.
11.1. Proportional Parts 166

The difference for 1 = .0000074.


∴ the difference for .57 = .0000042
∴ log 58743.57 = 4.7689487 + .0000042 = 4.7689529.
Again the difference for .2 = .0000015
∴ log 58743.2 = 4.7689487 + .0000015 = 4.7689502
∴ log .00587432 = 3̄.7689502. ■

§ Problem 11.1.3.
Given log 47847 = 4.6798547
and log 47848 = 4.6798638
find the numbers whose logarithms are respectively
2.6798593 and 3̄.6798617. ♢
§§ Solution. We have
log 478.47 = 2.6798547 (11.1)
and log 478.48 = 2.6798638 (11.2)
Let log (478.47 + x) = 2.6798593 (11.3)
From Eq. (11.1) and Eq. (11.2), we have the difference for
.01 = .0000091.
From Eq. (11.1) and Eq. (11.3), we have the difference for
x = .0000046.
x .0000046
∴ = .
.01 .0000091
46 .46
∴x= × .01 = ≈ .005.
91 91
∴ the required number = 478.47 + .005 = 478.475.
Again, let log (478.47 + y) = 2.6798617 (11.4)
From Eq. (11.1) and Eq. (11.4), we have the difference for y = .0000070.
70 .7
∴y= × .01 = ≈ .0077.
91 91
∴ log(478.4777) = 2.6798617; ∴ log .004784777 = 3̄.6798617.
∴ the required number = .004784777. ■

§ Problem 11.1.4.
Given log 258.36 = 2.4122253
and log 2.5837 = .4122421,
find the numbers whose logarithms are
.4122378 and 2̄.4122287. ♢
§§ Solution. We have
log 2.5836 = .4122253 (11.5)
and log 2.5837 = .4122421 (11.6)
Let log (2.5836 + x) = .4122378 (11.7)
From Eq. (11.5) and Eq. (11.6), we have the difference for
.0001 = .0000168.
From Eq. (11.5) and Eq. (11.7), we have the difference for
x = .0000125.
125 .0125
Hence we have x = × .0001 = ≈ .000074.
168 168
11.1. Proportional Parts 167

Hence the required number


= 2.5836 + .000074 = 2.583674.
Again, we have log .025836 = 2̄.4122253 (11.8)
and log .025837 = 2̄.4122421 (11.9)
Let log (.025836 + y) = 2̄.4122287 (11.10)
From Eq. (11.8) and Eq. (11.9), we have the difference for
.000001 = .0000168.
From Eq. (11.8) and Eq. (11.10), we have the difference for
y = .000034.
Hence we have
34 .000034
y= × .000001 = = .0000002.
168 168
Hence the required number
= .025836 + .0000002 = .0258362. ■

§ Problem 11.1.5. From the table in Art. 144, find the logarithms
of
(1) 52538.97
(2) 527.286
(3) .000529673,
and the numbers whose logarithms are
(4) 3.7221098
(5) 2̄.7210075, and
(6) .7210386.

§§ Solution. (1)
log 52538 = 4.7204735
diff. for .9 = .0000074
diff. for .07 = .0000006
∴ log 52538.97 = 4.7204815
(2)
log 52728 = 4.7220413
diff. for .6 = .0000049
∴ log 52728.6 = 4.7220462
∴ log 527.286 = 2.7220462
(3)
log 52967 = 4.7240054
diff. for .3 = .0000025
∴ log 52967.3 = 4.7240079
∴ log .000529673 = 4̄.7240079
(4)
Let log x = 4.7221098
We have log 52736 = 4.7221072
diff. = .0000026
11.1. Proportional Parts 168

diff. for .3 = .0000025


∴ log 52736.3 = log x
∴ x = 52736.3.
Hence the required number is 5273.63.

(5)
Let log x = 4.7240075
We have log 52967 = 4.7240054
diff. = .0000021
diff. for .2 = .0000016
diff. for .06 = .00000049
∴ log 52967.26 = log x
∴ x = 52967.26.
Hence the required number is .05296726.

(6)
Let log x = 4.7210386
We have log 52606 = 4.7210353
diff. = .0000033
diff. for .4 = .0000033
∴ log 52606.4 = log x
∴ x = 52606.4.
Hence the required number is 5.26064. ■

§ Problem 11.1.6.
Given sin 43◦ 23′ = .6868761
and sin 43◦ 24′ = .6870875,
find the value of sin 43◦ 23′ 47′′ . ♢
§§ Solution. For an increase of 60′′ in the angle, there is an increase
of
.6870875 − .6868761, i.e. .0002114,
in the logarithm.
Hence for an increase of 47′′ in the angle, the corresponding in-
crease in the logarithm
47
= × .0002114 = .0001656.
60
◦ ′ ′′
∴ sin 43 23 47 = .6868761 + .0001656 = .6870417. ■

§ Problem 11.1.7. Find also the angle whose sine is .6870349. ♢


§§ Solution. Let the required angle ◦ ′ ′′
( ) be 43 23 + x , so that
sin 43◦ 23′ + x′′ = .6870349.
x′′ .6870349 − .6868761 .0001588
∴ = =
60′′ .6870875 − .6868761 .0002114
′′ 1588 ′′
∴ x = 60 × ≈ 45 .
2114
Hence the required angle is 43◦ 23′ 45′′ . ■

§ Problem 11.1.8.
Given cos 32◦ 16′ = .8455726
and cos 32◦ 17′ = .8454172
find the values of cos 32 16′ 24′′ and of cos 32◦ 16′ 47′′ .
◦ ♢
11.1. Proportional Parts 169

§§ Solution. For an increase of 60′′ in the angle, there is a decrease


of
.8455726 − .8454172 = .0001554 in the logarithm.
Hence, for an increase of 24′′ in the angle, the corresponding de-
crease in the logarithm
24
= × .0001554 = .0000622.
60
And, for an increase of 47′′ in the angle, the corresponding decrease
in the logarithm
47
= × .0001554 = .0001217.
60
◦ ′ ′′
∴ cos 32 16 24 = .8455726 − .0000622 = .8455104
and cos 32◦ 16′ 47′′ = .8455726 − .0001217 = .8454509. ■

§ Problem 11.1.9. Find also the angles whose cosines are


.8454832 and .8455176. ♢
§§ Solution. (1) Let the(required angle ◦ ′ ′′
) be 32 16 + x .
∴ cos 32◦ 16′ + x′′ = .8454832.
x′′ .8455726 − .8454832 .0000894
∴ = =
60′′ .8455726 − .8454172 .0001554
894
∴ x = 60′′ × ≈ 35′′ .
1554
Hence the required angle is 32◦ 16′ 35′′ .

(2) Let the required angle ◦ ′ ′′


( be◦ 32′ 16 ′′+) y .
∴ cos 32 16 + y = .8455176.
y ′′ .8455726 − .8455176 .0000550
∴ = =
60′′ .8455726 − .8454172 .0001554
550
∴ x = 60′′ × ≈ 21′′ .
1554
Hence the required angle is 32◦ 16′ 21′′ . ■

§ Problem 11.1.10.
Given tan 76◦ 21′ = 4.1177784
and tan 76◦ 22′ = 4.1230079,
find the values of tan 76◦ 21′ 29′′ and tan 76◦ 21′ 47′′ . ♢
§§ Solution.
tan 76◦ 22′ = 4.1230079
tan 76◦ 21′ = 4.1177784
∴ diff. for 60′′ = .0052295.
29
∴ diff. for 29′′ = × .0052295 = .0025276
60
47
∴ diff. for 47′′ = × .0052295 = .0040964.
60
◦ ′ ′′
∴ tan 76 21 29 = 4.1177784 + .0025276 = 4.1203060
∴ tan 76◦ 21′ 47′′ = 4.1177784 + .0040964 = 4.1218748. ■

§ Problem 11.1.11.
Given cosec 13◦ 8′ = 4.4010616
and cosec 13◦ 9′ = 4.3955817,
find the values of cosec 13◦ 8′ 19′′ and cosec 13◦ 8′ 37′′ . ♢
11.1. Proportional Parts 170

§§ Solution.
cosec 13◦ 9′ = 4.3955817
cosec 13◦ 8′ = 4.4010616
∴ diff. for 60′′ = −.0054799.
19
∴ diff. for 19′′ = × (−.0054799) = −.0017353
60
37
∴ diff. for 37′′ = × (−.0054799) = −.0033793.
60
cosec 13◦ 8′ 19′′ = 4.4010616 − .0017353 = 4.3993263
cosec 13◦ 8′ 37′′ = 4.4010616 − .0033793 = 4.3976823. ■

§ Problem 11.1.12. Find also the angle whose cosecant is 4.396789.



§§ Solution.
Let cosec 13◦ 8′ x′′ = 4.3967890
cosec 13◦ 8′ = 4.4010616
∴ diff. for x′′ = −.0042726.
x′′ .0042726
∴ = , ∴ x ≈ 47′′
60′′ .0054799
∴ required angle = 13◦ 8′ 47′′ . ■

§ Problem 11.1.13.
Given L cos 34◦ 44′′ = 9.9147729
and L cos 34◦ 45′′ = 9.9146852,
find the value of L cos 34◦ 44◦ 27′′ . ♢
§§ Solution.
L cos 34◦ 45′′ = 9.9146852
L cos 34◦ 44′′ = 9.9147729
∴ diff. for 60′′ = −.0000877.
27
∴ diff. for 27′′ = × (−.0000877) = −.0000395
60
◦ ◦ ′′
∴ L cos 34 44 27 = 9.9147729 − .0000395 = 9.9147334. ■

§ Problem 11.1.14. Find also the angle θ, where


L cos θ = 9.9147328. ♢
§§ Solution.
Let L cos 34◦ 44′ x′′ = 9.9147328
L cos 34◦ 44′ = 9.9147729
∴ diff. for x′′ = −.0000401.
But diff. for 60′′ = −.0000877
401
∴x= × 60 ≈ 27
877
∴ θ = 34 44 27′′ .
◦ ′ ■

§ Problem 11.1.15.
Given L cot 71◦ 27′ = 9.5257779
and L cot 71◦ 28′ = 9.5253589,
find the value of L cot 71◦ 27′ 47′′ ,
and solve the equation L cot θ = 9.5254782. ♢
11.1. Proportional Parts 171

§§ Solution.
L cot 71◦ 28′ = 9.5253589
L cot 71◦ 27′ = 9.5257779
∴ diff.for 60′′ = −.0004190.
47
∴ diff. for 47′′ = × (−.0004190) = −.0003282.
60
◦ ′ ′′
∴ L cot 71 27 47 = 9.5257779 − .0003282 = 9.5254497.
Again, let θ = 71◦ 27′ x′′ . Then
L cot 71◦ 27′ x′′ = 9.5254782
L cot 71◦ 27′ = 9.5257779
∴ diff. for x′′ = −.0002997.
But diff. for 60′′ = −.0004190
2997
∴x= × 60 = 43; ∴ θ = 71◦ 27′ 43′′ . ■
4190
§ Problem 11.1.16.
Given L sec 18◦ 27′ = 10.0229168
and L sec 18◦ 28′ = 10.0229590,
find the value of L sec 18◦ 27′ 35′′ . ♢
§§ Solution.
L sec 18◦ 28′ = 10.0229590
L sec 18◦ 27′ = 10.0229168
∴ diff. for 60′′ = .0000422.
35
∴ diff. for 35′′ = × (.0000422) = .0000246
60
∴ L sec 18◦ 27′ 35′′ = 10.0229168 + .0000246 = 10.0229414. ■

§ Problem 11.1.17. Find also the angle whose L sec is 10.0229285. ♢


§§ Solution.
Let L sec 18◦ 27′ x′′ = 10.0229285
L sec 18◦ 27′ = 10.0229168
∴ diff. for x′′ = .0000117.
But diff. for 60′′ = .0000422
117
∴x= × 60 = 17
422
∴ required angle = 18◦ 27′ 17′′ . ■

§ Problem 11.1.18. Find in degrees, minutes and seconds the angle


whose sine is .6, given that
log 6 = 7781513, L sin 36◦ 52′ = 9.7781186
and L sin 36◦ 53′ = 9.7782870. ♢
§§ Solution.
6
Let sin θ = .6 =
10
∴ L sin θ = 10 + log 6 − log 10 = 9.7781513.
Let θ = 36◦ 52′ x′′ .
Then L sin 36◦ 52′ x′′ = 9.7781513
L sin 36◦ 52′ = 9.7781186
∴ diff. for x′′ = .0000327
L sin 36◦ 53′ = 9.7782870
11.2. Logarithmic Sines, Tangents And Secants 172

L sin 36◦ 52′ = 9.7781186


∴ diff. for 60′′ = .0001684
327
∴x= × 60 ≈ 12.
1684
∴ the angle = 36◦ 52′ 12′′ . ■

11.2 Logarithmic Sines, Tangents And Secants


§ Problem 11.2.1. Find θ, given that cos θ = .9725382,
cos 13◦ 27′ = .9725733, diff. for 1′ = 677. ♢
§§ Solution.
Since cos θ < cos 13◦ 27′ , ∴ θ > 13◦ 27′ .
( )
Let then cos 13◦ 27′ x′′ = .9725382.
Since .9725733 − .9725382 = .0000351
351
we have x = 60′′ × ≈ 31′′ .
677
Hence θ = 13 27 31′′ .
◦ ′ ■

3
§ Problem 11.2.2. Find the angle whose sine is , given
8
◦ ′ ′ ♢
sin 22 1 = .3748763, diff. for 1 = 2696.
3
§§ Solution. Since = .375, let the required angle be 22◦ 1′ + x′′ , so
8
that ( )
sin 22◦ 1′ + x′′ = .375.
∵ .375 − .3748763 = .0001237,
1237
we have x = 60′′ × ≈ 28′′ .
2696
Hence the required angle is 22◦ 1′ 28′′ . ■

§ Problem 11.2.3.
Given cosec 65◦ 24′ = 1.0998243
diff. for 1′ = 1464,
find the value of cosec 65◦ 24′ 37′′ .
and the angle whose cosec is 1.0997938. ♢
37
§§ Solution. Since × 1464 ≈ 903, we have
60
◦ ′ ′′
cosec 65 24 37 = 1.0998243 − .0000903 = 1.0997340.
Again, let θ be the required angle.
∵ cosec θ < cosec 65◦ 24′ , ∴ θ > 65◦ 24′ .
Let then θ = 65◦ 24′ + x′′
( )
∴ cosec 65◦ 24′ + x′′ = 1.0997938.
∵ 1.0998243 − 1.0997938 = .0000305,
305
∴ x = 60′′ × = 12.5′′ .
1464
∴ θ = 65 24 12.5′′ .
◦ ′ ■
11.2. Logarithmic Sines, Tangents And Secants 173

§ Problem 11.2.4.
Given L tan 22◦ 37′ = 9.6197205
diff. for 1′′ = 3557,
find the value of L tan 22◦ 37′ 22′′ ,
and the angle whose L tan is 9.6195283. ♢
22
§§ Solution. Since × 3557 ≈ 1304, we have
60
◦ ′ ′′
L tan 22 37 22 = 9.6197205 + .0001304 = 9.6198509.
Again, let θ be the required angle.
∵ L tan θ < L tan 22◦ 37′ , ∴ θ < 22◦ 37′ .
( )
Let then θ = 22◦ 37′ − x′′ , ∴ L tan 22◦ 37′ − x′′ = 9.6195283.
∵ 9.6197205 − 9.6195283 = .0001922
1922
∴ x = 60′′ × ≈ 32′′ .
3557
∴ θ = 22◦ 37′ − 32′′ = 22◦ 36′ 28′′ . ■

§ Problem 11.2.5. Find the angle whose L cos is 9.993, given


L cos 10◦ 15′ = 9.9930131, diff. for 1′ = 229. ♢
§§ Solution. Let θ be the required angle.
∵ L cos θ < L cos 10◦ 15′′ , ∴ θ > 10◦ 15′ .
Let then θ = 10◦ 15′ + x′′ , ∴ L cos θ = 9.993.
∵ 9.9930131 − 9.993 = .0000131
131
∴ x = 60′′ × ≈ 34′′ .
229
∴ θ = 10◦ 15′ 34′′ . ■

§ Problem 11.2.6. Find the angle whose L sec is 10.15, given


L sec 44◦ 55′ = 10.1498843, diff. for 1′ = 1260. ♢
§§ Solution. Let θ be the required angle.
∵ L sec θ > L sec 44◦ 55′ , ∴ θ > 44◦ 55′ .
( )
Let then θ = 44◦ 55′ + x′′ , ∴ L sec 44◦ 55′ + x′′ = 10.15.
∵ 10.15 − 10.1498843 = .0001157,
1157
∴ x = 60 × ≈ 55.
1260
∴ θ = 44 55 55′′ .
◦ ′ ■

§ Problem 11.2.7. From the table in Art. 159, find the values of

(1) L sin 32◦ 18′ 23′′

(2) L cos 32◦ 16′ 49′′

(3) L cot 32◦ 29′ 43′′

(4) L sec 32◦ 52′ 27′′

(5) L tan 57◦ 45′ 28′′

(6) L cosec 57◦ 48′ 21′′ , and

(7) L cos 57◦ 58′ 29′′ .



11.2. Logarithmic Sines, Tangents And Secants 174

23
§§ Solution. (1) Since × 1998 ≈ 766, we have
60
◦ ′ ′′
L sin 32 18 23 = 9.7278277 + .0000766 = 9.7279043.
49
(2) Since × 798 ≈ 652, we have
60
L cos 32◦ 16′ 49′′ = 9.9271509 − .0000652 = 9.9270857.
43
(3) Since × 2788 ≈ 1998, we have
60
L cot 32◦ 29′ 43′′ = 10.1960915 − .0001998 = 10.1958917.
27
(4) Since × 817 ≈ 368, we have
60
L sec 32 52′ 27′′ = 10.0757539 + .0000368 = 10.0757907.

28
(5) Since × 2800 ≈ 1307, we have
60
L tan 57 45′ 28′′ = 10.2000030 + .0001307 = 10.2001337.

21
(6) Since × 795 ≈ 278, we have
60
L cosec 57 48′ 21′′ = 10.0725305 − .0000278 = 10.0725027.

29
(7) Since × 2020 ≈ 976, we have
60
L cos 57◦ 58′ 29′′ = 9.7246138 − .0000976 = 9.7245162. ■

§ Problem 11.2.8. With the help of the table in Art. 159, solve the
equations

(1) L tan θ = 10.1959261

(2) L cosec θ = 10.0738125

(3) L cos θ = 9.9259283, and

(4) L sin θ = 9.9241352.



§§ Solution. (1)
∵ L tan 57◦ 30′ = 10.1958127
diff. for 1′ = 2788
Let θ = 57◦ 30′ + x′′
( )
∴ L tan 57◦ 30′ + x′′ = 10.1959261.
Since 10.1959261 − 10.1958127 = .0001134
1134
∴ x = 60′′ × ≈ 24′′ .
2788
∴ θ = 57◦ 30′ 24′′ .

(2)
∵ L cosec 57◦ 32′ = 10.0738099
diff. for 1′ = 805
Let θ = 57◦ 32′ − x′′
( )
∴ L cosec 57◦ 32′ − x′′ = 10.0738125.
Since 10.0738125 − 10.0738099 = .0000026
11.2. Logarithmic Sines, Tangents And Secants 175

26
∴ x = 60′′ × ≈ 2′′ .
805
∴ θ = 57 32 − x = 57◦ 31′ 58′′ .
◦ ′ ′′

(3)
∵ L cos 32◦ 32′ = 9.9258681
diff. for 1′ = 806
Let θ = 32◦ 32′ − x′′
( )
∴ L cos 32◦ 32′ − x′′ = 9.9259283.
Since 9.9259283 − 9.9258681 = .0000602
602
∴ x = 60′′ × ≈ 45′′ .
805
∴ θ = 32 32 − 45 = 32◦ 31′ 15′′ .
◦ ′ ′′

(4)
∵ L sin 57◦ 6′ = 9.9240827
diff. for 1′ = 817
Let θ = 57◦ 6′ + x′′
( )
∴ L sin 57◦ 6′ + x′′ = 9.9241352.
Since 9.9241352 − 9.9240827 = .0000525
525
∴ x = 60′′ × ≈ 39′′ .
817
∴ θ = 57 6 39′′ .
◦ ′ ■

§ Problem 11.2.9. Take out of the tables L tan 16◦ 6′ 23′′ and calcu-
late the value of the square root of the tangent. ♢
§§ Solution.
L tan 16◦ 6′ = 9.4603492
23
diff. for 23′′ = × 4740 = 1817
60
◦ ′ ′′
∴ L tan 16 6 23 = 9.4603492 + .0001817 = 9.4605309
∴ log tan 16◦ 6′ 23′′ = L tan 16◦ 6′ 23′′ − 10 = 1̄.4605309.

Let x = tan 16◦ 6′ 23′′ .
Taking logarithms of both sides, we have
1( ) 1( )
log x = log tan 16◦ 6′ 23′′ = 1̄.4605309
2 2
1( )
= 2̄ + 1.4605309 = 1̄.7302655.
2
Now log 53736 = 4.7302653, diff. for 1 = 81.
Let log (53736 + y) = 4.7302655.
2
We then have y= = .02
81
∴ log 53736.02 = 4.7302655.
∴ x = .5373602. ■

§ Problem 11.2.10. Change into a form more convenient for loga-


rithmic computation (i.e. express in the form of products of quanti-
ties) the quantities

(1) 1 + tan x tan y


11.2. Logarithmic Sines, Tangents And Secants 176

(2) 1 − tan x tan y

(3) cot x + tan y

(4) cot x − tan y

1 − cos 2x
(5) , and
1 + cos 2x
tan x + tan y
(6) .
cot x + cot y ♢
§§ Solution. (1)
sin x sin y
1 + tan x tan y = 1 +
cos x cos y
cos x cos y + sin x sin y
=
cos x cos y
= cos(x − y) sec x sec y.

(2)
sin x sin y
1 − tan x tan y = 1 −
cos x cos y
cos x cos y − sin x sin y
=
cos x cos y
= cos(x + y) sec x sec y.

(3)
cos x sin y
cot x + tan y = +
sin x cos y
cos x cos y + sin x sin y
=
sin x cos y
= cos(x − y) cosec x sec y.

(4)
cos x sin y
cot x − tan y = −
sin x cos y
cos x cos y − sin x sin y
=
sin x cos y
= cos(x + y) cosec x sec y.

1 − cos 2x 2 sin2 x
(5) = = tan2 x.
1 + cos 2x 2 cos2 x
(6) [ ] [ ]
tan x + tan y sin x sin y cos x cos y
= + ÷ +
cot x + cot y [ cos x cos y sin x
] [sin y ]
sin x cos y + cos x sin y cos x sin y + sin x cos y
= ÷
cos x cos y sin x sin y
sin x sin y
= = tan x tan y. ■
cos x cos y
Chapter 12
Relations Between The Sides and
The Trigonometrical Ratios of
The Angles of Any Triangle

12.1 Basics
In a triangle
§ Problem 12.1.1. Given
a = 25, b = 52 and c = 63;
A B C
find tan , tan and tan . ♢
2 2 2
§§ Solution. We have
1
s = (25 + 52 + 63) = 70, s − a = 45, s − b = 18 and s − c = 7.
2 … …
A 18 × 7 1 B 7 × 45 1
∴ tan = = , tan = = ,
2 70 × 45 5 2 70 × 18 2

C 45 × 18 9
and tan = = . ■
2 70 × 7 7
§ Problem 12.1.2. Given
a = 125, b = 123 and c = 62;
find the sines of half the angles and the sines of the angles. ♢
§§ Solution.
1
s = (125 + 123 + 62) = 155, s − a = 30, s − b = 32 and s − c = 93.
2√

∴ s(s − a)(s − b)(s − c) = 155 × 30 × 32 × 93

= 31 × 5 × 5 × 3 × 64 × 31 × 3
12.1. Basics 178

= 31 × 5 × 3 × 8.
… …
A 32 × 93 4 B 93 × 30 3
∴ sin = = √ , sin = =
2 123 × 62 41 2 62 × 125 5

C 30 × 32 8
sin = = √ .
2 125 × 123 5 41
2 40
sin A = × 31 × 5 × 3 × 8 =
123 × 62 41
2 24
sin B = × 31 × 5 × 3 × 8 =
125 × 62 25
2 496
and sin C = × 31 × 5 × 3 × 8 = . ■
125 × 123 1025
§ Problem 12.1.3. Given
a = 18, b = 24 and c = 30,
find sin A, sin B and sin C.
Verify by a graph. ♢
§§ Solution.
1
s = (18 + 24 + 30) = 36, s − a = 18, s − b = 12 and s − c = 6.
2√

∴ s(s − a)(s − b)(s − c) = 36 × 18 × 12 × 6

= 36 × 36 × 6 × 6 = 36 × 6.
2 3
∴ sin A = × 36 × 6 =
24 × 30 5
2 4
sin B = × 36 × 6 =
18 × 30 5
2
and sin C = × 36 × 6 = 1. ■
18 × 24
§ Problem 12.1.4. Given
a = 35, b = 84 and c = 91,
find tan A, tan B and tan C. ♢
§§ Solution.
1
s = (35 + 84 + 91) = 105, s − a = 70, s − b = 21 and s − c = 14.
2 …
A 21 × 14 1
∴ tan = = ,
2 105 × 70 5
A
2 tan ( )
2 2 1 2 25 5
and tan A = = ÷ 1− = × =
A 5 25 5 24 12
1 − tan 2
2 …
B 14 × 70 2
tan = =
2 105 × 21 3
B
2 tan ( )
2 4 4 4 9 12
and tan B = = ÷ 1− = × =
B 3 9 3 5 5
1 − tan 2
2 …
C 70 × 21
tan = =1
2 105 × 14
12.1. Basics 179

C
2 tan 2 2
and tan C = 2 = = = ∞.
C 1−1 0
1− tan2 ■
2
§ Problem 12.1.5. Given
a = 13, b = 14 and c = 15,
find the sines of the angles. Verify by a graph. ♢
§§ Solution.
1
s = (13 + 14 + 15) = 21, s − a = 8, s − b = 7 and s − c = 6
√ 2
√ √
∴ s(s − a)(s − b)(s − c) = 21 × 8 × 7 × 6 = 42 × 4 × 42 = 42 × 2.
2 4 2 56
∴ sin A = × 42 × 2 = , sin B = × 42 × 2 =
14 × 15 5 13 × 15 65
2 12
and sin C = × 42 × 2 = . ■
13 × 14 13
§ Problem 12.1.6. Given
a = 287, b = 816 and c = 865,
A
find the values of tan and tan A. ♢
2
§§ Solution.
1
s = (287 + 816 + 865) = 984, s − a = 697, s − b = 168, s − c = 119.
2 … …
A 168 × 119 7 × 24 × 17 × 7 7
∴ tan = = =
2 984 × 697 41 × 24 × 17 × 41 41
A
2 tan ( )
2 14 49 14 41 × 41 287
∴ tan A = = ÷ 1− = × = .
A 41 41 × 41 41 1632 816 ■
1 − tan2
2
§ Problem 12.1.7. Given √ √
√ √ 6+ 2
a = 3, b = 2 and c = ,
2
find the angles. ♢
§§ Solution. We have
Å √ √ ã2
(√ ) 2 6+ 2 (√ )2
2 + − 3
b +c −a
2 2 2 2
cos A = = Å√ √ ã
2bc √ 6+ 2
2 2
2
√ √
2+2+ 3−3 3+1 1
= √ = (√ ) = = cos 60◦
2 3+2 2 3+1 2
∴ A = 60◦ .

+ a2 − b2
c2 2+ 3+3−2
cos B = = √ √
2ca 3 2+ 6

3+ 3 1
= √ ( √ ) = √ = cos 45◦
2 3+ 3 2
∴ B = 45◦
∴ C = 180◦ − (A + B) = 180◦ − 105◦ = 75◦ . ■
12.2. Sides And Angles of A Triangle 180

12.2 Sides And Angles of A Triangle


In any triangle ABC, prove that
B−C b−c A
§ Problem 12.2.1. sin = cos . ♢
2 a 2
a b c
§§ Solution. ∵ = = = k (say), we have
sin A sin B sin C
b−c k sin B − k sin C sin B − sin C
= =
a k sin A sin A
B+C B−C
2 cos sin
= 2 2
A A
2 sin cos
2 2
A B−C B−C
sin sin sin
= 2 2 = 2 .
A A A
sin cos cos
2 2 2
B−C b−c A
∴ sin = cos .
2 a 2
Otherwise thus :
B−C B C B C
sin = sin cos − cos sin
2 … 2 2 …2 2 … …
(s − c)(s − a) s(s − c) s(s − b) (s − a)(s − b)
= −
ca ab ca ab
… …
s−c s(s − a) s−b s(s − a)
= −
a bc a bc
( )…
s−c s−b s(s − a)
= −
a a bc

b−c s(s − a) b−c A
= = cos . ■
a bc a 2
§ Problem 12.2.2. b2 sin 2C + c2 sin 2B = 2bc sin A. ♢
§§ Solution.
b2 sin 2C + c2 sin 2B = 2b2 sin C cos C + 2c2 sin B cos B
= 2b sin C (b cos C + c cos B)
= 2ab sin C = 2bc sin A. ■

§ Problem 12.2.3. a (b cos C − c cos B) = b2 − c2 . ♢


§§ Solution.
a2 + b2 − c2 a2 + c2 − b2
a (b cos C − c cos B) = −
2 2
= b 2 − c2 . ■

§ Problem 12.2.4. (b+c) cos A+(c+a) cos B+(a+b) cos C = a+b+c. ♢


§§ Solution.
(b + c) cos A + (c + a) cos B + (a + b) cos C
= (b cos A + a cos B) + (c cos B + b cos C) + (c cos A + a cos C)
= c + a + b, by Art. 170. ■

A
§ Problem 12.2.5. a (cos B + cos C) = 2(b + c) sin2 . ♢
2
12.2. Sides And Angles of A Triangle 181

§§ Solution. From the equations


b = c cos A + a cos C, and
c = a cos B + b cos A
we have, by addition,
c cos A + a cos C + a cos B + b cos A = b + c
A
∴ a (cos B + cos C) = (b + c) (1 − cos A) = 2(b + c) sin2 .
2
Otherwise thus :

B+C B−C
cos B + cos C = 2 cos cos
2 2
A B−C
= 2 sin cos
2 2
A B−C
2 sin cos sin A
= 2 2
sin A
A A B−C
2 sin 2 × 2 cos cos
= 2 2 2
sin A
A B+C B−C
2 sin2 × 2 sin cos
= 2 2 2
sin A
A
2 sin2 (sin B + sin C)
= 2
sin A
A
2 sin2 (b + c)
= 2
a
A
∴ a (cos B + cos C) = 2(b + c) sin2 . ■
2
A
§ Problem 12.2.6. a (cos C − cos B) = 2(b − c) cos2 . ♢
2
§§ Solution. From the equations
b = c cos A + a cos C, and
c = a cos B + b cos A
we have, by subtraction,
a (cos C − cos B) + (c − b) cos A = b − c
A
∴ a (cos C − cos B) = (b − c) (1 + cos A) = 2(b − c) cos2 .
2
Otherwise thus :

B+C B−C
cos C − cos B = 2 sin sin
2 2
A B−C
= 2 cos sin
2 2
A B−C
2 cos sin sin A
= 2 2
sin A
A A B−C
2 cos2 × 2 sin sin
= 2 2 2
sin A
12.2. Sides And Angles of A Triangle 182

A B+C B−C
2 cos2 × 2 cos sin
= 2 2 2
sin A
A
2 cos2 (sin B − sin C)
= 2
sin A
A
2 cos2 (b − c)
= 2
a
A
∴ a (cos C − cos B) = 2(b − c) cos2 . ■
2
sin(B − C) b 2 − c2
§ Problem 12.2.7. = . ♢
sin(B + C) a2
§§ Solution.
sin(B − C) sin(B − C) sin(B + C)
=
sin(B + C) sin2 (B + C)
( ) ( )
sin B − sin2 C
2 sin B 2 sin C 2
= 2A
= −
( )sin sin A sin A
b 2 Ä c ä2 b2 − c2
= − = . ■
a a a2
a+b A+B A−B
§ Problem 12.2.8. = tan cot . ♢
a−b 2 2
a sin A
§§ Solution. We have = .
b sin B
a+b sin A + sin B
∴ =
a−b sin A − sin B
A+B A−B
2 sin cos
= 2 2
A+B A−B
2 cos sin
2 2
A+B A−B
= tan cot . ■

( )2 2
A A
§ Problem 12.2.9. a sin + B = (b + c) sin . ♢
2 2
§§ Solution.
b+c k sin B + k sin C sin B + sin C
= =
a k sin A sin A( )
A A
2 sin B + cos
sin B + sin(A + B) 2 2
= =
sin A A A
2 sin cos
( ) 2 2
A A
∴ a sin + B = (b + c) sin . ■
2 2
a2 sin(B − C) b2 sin(C − A) c2 sin(A − B)
§ Problem 12.2.10. + + =
sin B + sin C sin C + sin A sin A + sin B
0. ♢
12.2. Sides And Angles of A Triangle 183

a b c
§§ Solution. Since = = = k (say), the given ex-
sin A sin B sin C
pression
[
sin A sin(B + C) sin(B − C) sin B sin(C + A) sin(C − A)
= k2 +
sin B + sin C sin C + sin A ]
sin C sin(A + B) sin(A − B)
+
ñ ( 2 ) ( 2 sin A +2sin)B
sin A sin B − sin 2C sin B sin C − sin A
= k2 +
sin B + sin C sin C + sin A
( )ô
sin C sin2 A − sin2 B
+
sin A + sin B
= k2 [sin A (sin B − sin C) + sin B (sin C − sin A) + sin C (sin A − sin B)]
= 0. ■

( )
B C A
§ Problem 12.2.11. (b + c − a) cot + cot = 2a cot . ♢
2 2 2
§§ Solution. ( )
B C
(b + c − a) cot + cot
2 …2ï … ò
s(s − b) s(s − c)
= 2(s − a) +
(s − c)(s − a) (s − a)(s − b)
ñ ô
√ s−b+s−c
= 2 s · (s − a) √
(s − a)(s − b)(s − c)
ñ ô
√ 2s − (b + c)
= 2 s · (s − a) √
(s − a)(s − b)(s − c)

s(s − a) A
= 2a = 2a cot . ■
(s − b)(s − c) 2
§ Problem 12.2.12. a2 + b2 + c2 = 2 (bc cos A + ca cos B + ab cos C). ♢
§§ Solution. By Art. 164, we have
b2 + c2 − a2 = 2bc cos A
c2 + a2 − b2 = 2ca cos B, and
a2 + b2 − c2 = 2ab cos C.
Hence, by addition,
a2 + b2 + c2 = 2 (bc cos A + ca cos B + ab cos C) . ■

( ) ( )
§ Problem 12.2.13. a2 − b2 + c 2 tan B = a2 + b2 − c 2 tan C. ♢
§§ Solution.
tan B sin B cos C sin B cos C
= =
tan C cos B sin C sin C cos B
b cos C a2 + b2 − c2 c2 + a2 − b2
= = ÷ .
( 2 c2cos B2 ) 2a( )2a
∴ a − b + c tan B = a + b − c tan C.
2 2 2 ■

C C
§ Problem 12.2.14. c2 = (a − b)2 cos2 + (a + b)2 sin2 . ♢
2 2
12.2. Sides And Angles of A Triangle 184

§§ Solution.
c2 = a2 + b2 − 2ab cos C
( 2 2) ( 2 C C
) (
C C
)
a +b cos + sin2 − 2ab cos2 − sin2
2 2 2 2
( 2 ) 2C ( 2 ) 2C
= a − 2ab + b cos
2 2
+ a + 2ab + b sin
2 2
C 2 C
= (a − b) cos
2 2 2
+ (a + b) sin . ■
2 2
§ Problem 12.2.15. a sin(B − C) + b sin(C − A) + c sin(A − B) = 0. ♢
a b c
§§ Solution. Since = = = k (say), we have
sin A sin B sin C
a sin(B − C) + b sin(C − A) + c sin(A − B)
= k sin A sin(B − C) + k sin B sin(C − A) + k sin C sin(A − B)
= k [sin(B + C) sin(B − C) + sin(C + A) sin(C − A)
+ sin(A + B) sin(A − B)]
[ ]
= k sin2 B − sin2 C + sin2 C − sin2 A + sin2 A − sin2 B
[by Ex. 2, Art. 93]
= 0.
Otherwise thus :
a sin(B − C) + b sin(C − A) + c sin(A − B)
= a (sin B cos C − cos B sin C) + b (sin C cos A − cos C sin A)
+ c (sin A cos B − cos A sin B)
= cos C (a sin B − b sin A) + cos B (c sin A − a sin C)
+ cos A (b sin C − c sin B)
= 0,
∵ a sin B = b sin A, c sin A = a sin C and b sin C = c sin B. ■
a sin(B − C) b sin(C − A) c sin(A − B)
§ Problem 12.2.16. = = . ♢
b2 − c2 c2 − a2 a2 − b2
a b c
§§ Solution. Since = = = k (say), we have
sin A sin B sin C
a sin(B − C) k sin A sin(B − C)
=
b2 − c2 k2 sin2 B − k2 sin2 C
1 sin(B + C) sin(B − C)
= ·
k sin2 B − sin2 C
1 sin2 B − sin2 C
= · , by Ex. 2, Art. 93
k sin2 B − sin2 C
1 b sin(C − A) c sin(A − B)
= = = , similarly. ■
k c2 − a2 a2 − b2
§ Problem 12.2.17.
A B−C B C−A
a sin sin + b sin sin
2 2 2 2
C A−B
+ c sin sin = 0. ♢
2 2
§§ Solution.
A B−C B C−A C A−B
a sin sin + b sin sin + c sin sin
2 2 2 2 2 2
B+C B−C C+A C−A A+B A−B
= a cos sin + b cos sin + c cos sin
2 2 2 2 2 2
12.2. Sides And Angles of A Triangle 185

a b c
= (sin B − sin C) + (sin C − sin A) + (sin A − sin B)
2 2 2
= 0, ∵ a sin B = b sin A, c sin A = a sin C and b sin C = c sin B. ■

(
§ Problem 12.2.18. ) ( )
a2 cos2 B − cos2 C + b2 cos2 C − cos2 A
( )
+ c2 cos2 A − cos2 B = 0. ♢
(
§§ Solution. ) ( ) ( )
a2 cos2 B − cos2 C + b2 cos2 C − cos2 A + c2 cos2 A − cos2 B
[ ( ) ( )
= k2 sin2 A cos2 B − cos2 C + sin2 B cos2 C − cos2 A
( )]
+ sin2 C cos2 A − cos2 B
[ ( ) ( )
= k2 sin2 A sin2 C − sin2 B + sin2 B sin2 A − sin2 C
( 2 )]
+ sin C sin B − sin A
2 2

= k2 [0] = 0. ■

§ Problem 12.2.19.
b2 − c2 c2 − a2
2
sin 2A + sin 2B
a b2
a −b
2 2
+ sin 2C = 0. ♢
c2
§§ Solution.
b 2 − c2 k2 sin2 B − k2 sin2 C
sin 2A = · 2 sin A cos A
a2 k2 sin2 A
2 cos A sin(B + C) sin(B − C)
= , by Ex. 2, Art. 93
sin A
= 2 cos A sin(B − C), ∵ sin(B + C) = sin A
= −2 cos(B + C) sin(B − C) = sin 2C − sin 2B.
c2 − a2
Similarly, sin 2B = sin 2A − sin 2C
b2
a2 − b2
and sin 2C = sin 2B − sin 2A.
c2
Hence the given expression
= sin 2C − sin 2B + sin 2A − sin 2C + sin 2B − sin 2A = 0. ■

A B C
(a + b + c)2 cot + cot + cot
§ Problem 12.2.20. 2 = 2 2 2 . ♢
a + b2 + c2 cot A + cot B + cot C
§§ Solution.
(a + b + c)2 (sin A + sin B + sin C)2
=
a2 + b2 + c2 sin2 A + sin2 B + sin2 C
A B C
16 cos2 cos2 cos2
= 2 2 2 , by §P roblem 9.2.4
sin2 A + sin2 B + sin2 C
A B C sin A sin B sin C
= 2 cot cot cot ×
2 2 2 sin2 A + sin2 B + sin2 C
A B C sin2 A + sin2 B + sin2 C
= 2 cot cot cot ÷
2 2 2 [ sin A sin B sin C ]
A B C sin A sin B sin C
= 2 cot cot cot ÷ + +
2 2 2 sin B sin C sin A sin C sin A sin B
12.2. Sides And Angles of A Triangle 186

[ ]
A B C sin(B + C) sin(A + C) sin(A + B)
= 2 cot cot cot ÷ + +
2 2 2 sin B sin C sin A sin C sin A sin B
A B C
= 2 cot cot cot ÷ [cot C + cot B + cot C + cot A + cot B + cot A]
2 2 2
A B C
cot cot cot
= 2 2 2
cot A + cot B + cot C
A B C
cot + cot + cot
= 2 2 2 , by §P roblem 9.2.13.
cot A + cot B + cot C
Otherwise thus :
(a + b + c)2 (sin A + sin B + sin C)2
=
2 2
a +b +c 2 sin2 A + sin2 B + sin2 C
A B C
16 cos2 cos2 cos2
= 2 2 2 , by §P roblem 9.2.4 and Ex. 3 of Art. 127
2 (1 + cos A cos B cos C)
[ ]
A B C cos A cos B cos C − cos(A + B + C)
= cot cot cot ÷ ,
2 2 2 sin A sin B sin C
by dividing numerator and denominator by 2 sin A sin B sin C,
and putting cos(A + B + C) = −1.

Now
cos A cos B cos C − cos(A + B + C)
sin A sin B sin C
cos(A + B + C)
= cot A cot B cot C −
sin A sin B sin C
= cot A cot B cot C − (cot A cot B cot C − cot A − cot B − cot C) ,
by expanding cos(A + B + C), as in Art. 124;

A B C A B C
also cot cot cot = cot + cot + cot
2 2 2 2 2 2
A B C
(a + b + c)2 cot + cot + cot
∴ 2 = 2 2 2 .

a + b 2 + c2 cot A + cot B + cot C
§ Problem 12.2.21.
a3 cos(B − C) + b3 cos(C − A)
+ c3 cos(A − B) = 3abc. ♢
§§ Solution.
a3 cos(B − C) + b3 cos(C − A) + c3 cos(A − B)
[ ]
= k3 sin3 A cos(B − C) + sin3 B cos(C − A) + sin3 C cos(A − B)
[
=k 3
sin2 A sin(B + C) cos(B − C) + sin2 B sin(C + A) cos(C − A)
]
+ sin2 C sin(A + B) cos(A − B)
k3 [ 2
= sin A (sin 2B + sin 2C) + sin2 B (sin 2C + sin 2A)
2 ]
+ sin2 C (sin 2A + sin 2B)
(
= k3 sin2 A sin B cos B + sin2 A sin C cos C
+ sin2 B sin C cos C + sin2 B sin A cos A
12.2. Sides And Angles of A Triangle 187

)
+ sin2 C sin A cos A + sin2 C sin B cos B
= k3 [sin A sin B (sin A cos B + cos A sin B)
+ sin B sin C (sin B cos C + cos B sin C)
+ sin C sin A (sin A cos C + cos A sin C)]
= k3 [sin A sin B sin(A + B) + sin B sin C sin(B + C)
+ sin C sin A sin(A + C)]
= k3 (sin A sin B sin C + sin B sin C sin A + sin C sin A sin B)
= 3k3 sin A sin B sin C = 3abc. ■


§ Problem 12.2.22. In a triangle whose sides are 3, 4 and 38 feet
respectively, prove that the largest angle is greater than 120◦ . ♢

§§ Solution. If a = 3, b = 4 and c = 38, then C is the largest angle.
We have
a2 + b2 − c2 9 + 16 − 38 13
cos C = = =− .
2ab 2×3×4 24
1 12
Now cos 120◦ = − = − .
2 24
∵ cos C is less than cos 120◦ ,
∴ ∠C > 120◦ . ■

§ Problem 12.2.23. The sides of a right-angled triangle are 21 and


28 feet; find the length of the perpendicular drawn to the hypotenuse
from the right angle. ♢
§§ Solution. Let ABC be the right-angled triangle and CD be the
perpendicular drawn to the hypotenuse from the right angle ∠C.
Let BC = 21 f eet and AC = 28 f eet.
In the triangle ABC, we have
21 3 3
tan A = = , so that sin A = .
28 4 5
In the triangle ACD, we have
3 84 4
CD = AC sin A = 28 × = = 16 f eet. ■
5 5 5
§ Problem 12.2.24. If in any triangle the angles be to one √ another
as 1 : 2 : 3, prove that the corresponding sides are as 1 : 3 : 2. ♢
§§ Solution. Let A, 2A and 3A be the angles, so that A + 2A + 3A =
6A = 180◦ and A = 30◦ ; i.e. the angles are 30◦ , 60◦ and 90◦ . If a, b
and c denote the sides opposite these angles respectively, we have,
by Art. 163,
a b c
= =
sin 30◦ sin 60◦ ◦
sin 90√
1 3 √
∴ a : b : c = sin 30◦ : sin 60◦ : sin 90◦ = : : 1 = 1 : 3 : 2. ■
2 2
§ Problem 12.2.25. In any triangle, if
A 5 B 20
tan = and tan = ,
2 6 2 37
C
find tan and prove that in this triangle a + c = 2b. ♢
2
12.2. Sides And Angles of A Triangle 188

§§ Solution. We have ( )
C A+B A+B
tan = tan 90◦ − = cot
2 2 2
A B
1 1 − tan tan
= = 2 2
A+B A B
tan tan + tan
2 2 2
5 20
1− ·
= 6 37 = 222 − 100 = 122 = 2 × 61 = 2 .
5 20 185 + 120 305 5 × 61 5
+
6 37 … …
A C (s − b)(s − c) (s − a)(s − b)
∵ tan tan = ·
2 2 s(s − a) s(s − c)
5 2 s−b 1 a+c−b
∴ · = ; ∴ =
6 5 s 3 a+c+b
3−1 b
∴ = ; ∴ a + c = 2b. ■
3+1 a+c
§ Problem 12.2.26. In an isosceles right-angled triangle, a straight
line is drawn from the middle point of one of the equal sides to the op-
posite angle. Show that it divides the angle into parts whose cotan-
gents are 2 and 3. ♢
§§ Solution. Let ACB be the triangle, right-angled at C and D be
the middle point of AC. Join DB.
Let AC = a = CB. We then have
BC a
cot ∠DBC = = Ä a ä = 2.
CD
2
Also,
cot ∠ABD = cot (∠ABC − ∠DBC)
cot ∠ABC cot ∠DBC + 1
=
cot ∠DBC − cot ∠ABC
2+1
= = 3,
2−1
BC
∵ cot ∠ABC = = 1. ■
AC
§ Problem 12.2.27. The perpendicular AD to the base of a triangle
ABC divides it into segments such that BD, CD and AD are in the
ratio of 2, 3 and 6; prove that the vertical angle of the triangle is
45◦ . ♢
§§ Solution. Denote the ∠BAD by α and the ∠DAC by β. In the
triangle ABD,
2 1
we have tan α = = .
6 3
3 1
In the ∆ACD, we have tan β = = .
6 2
In the ∆ABC, we have
tan α + tan β
tan A = tan(α + β) =
1 − tan α tan β
12.2. Sides And Angles of A Triangle 189

1 1
+
= 3 2 = 5 = 1 = tan 45◦ .
1 1 5
1− ·
3 2
The angle A is therefore 45◦ . ■

§ Problem 12.2.28. A ring, ten inches in diameter, is suspended


from a point one foot above its center by 6 equal strings attached
to its circumference at equal intervals. Find the cosine of the angle
between consecutive strings. ♢
§§ Solution. Let O be the center of the ring and P be the point of
suspension, so that OP = 12′′ .
Let A, B, C, D, E and F be the points of attachment of the strings
to the circumference. Then A, B, C, D, E and F are the angular
points of a regular hexagon inscribed in the ring.
Consider the points B and C. Join BC.
We have, BC = radius of ring = 5′′ . Also
√ √ √
PB = PC = OP 2 + OC 2 = (12)2 + 52 = 144 + 25 = 13′′ .
(13)2 + (13)2 − 52
∴ cos ∠BP C =
2 × 13 × 13
313
= = the cosine of the angle required. ■
338
§ Problem 12.2.29. If a2 , b2 and c2 be in A. P., prove that cot A,
cot B and cot C are in A. P. also. ♢
§§ Solution. Given a2 − b2 = b2 − c2 , i.e. sin2 A − sin2 B = sin2 B −
sin2 C;
∴ sin(A + B) sin(A − B) = sin(B + C) sin(B − C).
Dividing both sides of this last equation by sin A sin B sin C, we have
sin(A − B) sin(B − C)
=
sin A sin B sin B sin C
sin A cos B − cos A sin B sin B cos C − cos B sin C
∴ =
sin A sin B sin B sin C
∴ cot B − cot A = cot C − cot B
∴ cot A − cot B = cot B − cot C
∴ cot A, cot B and cot C are in A. P.
Otherwise thus :
We have 2b2 = a2 + c2
b2 a + c − b2
2 2 sin2 B
∴ = ; ∴ = 2 cos B
ac ac sin A sin C
sin B 2 cos B sin(A + C) 2 cos B
∴ = ; ∴ =
sin A sin C sin B sin A sin C sin B
cos A cos C 2 cos B
∴ + = ; ∴ cot A + cot C = 2 cot B
sin A sin C sin B
∴ cot A, cot B and cot C are in A. P. ■

A
§ Problem 12.2.30. If a, b and c be in A. P., prove that cos A cot ,
2
B C
cos B cot and cos C cot are in A. P . ♢
2 2
12.2. Sides And Angles of A Triangle 190

§§ Solution.
Given a − b = b − c, ∴ sin A − sin B = sin B − sin C
A+B A−B B+C B−C
∴ 2 cos sin = 2 cos sin .
2 2 2 2
A B C
Dividing both sides of this last equation by 2 sin sin sin , we
2 2 2
have
A−B B−C
sin sin
2 = 2
A B B C
sin sin sin sin
2 2 2 2
A B A B B C B C
sin cos − cos sin sin cos − cos sin
∴ 2 2 2 2 = 2 2 2 2
A B B C
sin sin sin sin
2 2 2 2
B A C B
∴ cot − cot = cot − cot
2 2 2 2
A B B C
∴ cot − cot = cot − cot
2 2 2 2
A B C
∴ cot , cot and cot are in A. P.
2 2 2
Also sin A, sin B and sin C are in A. P.:
A B C
∴ cot − sin A, cot − sin B and cot − sin C are in A. P.
2 2 2
A
A cos
Now cot − sin A = 2 − 2 sin A cos A
2 A 2 2
sin
2
A
cos ( )
= 2 1 − 2 sin2 A = cot A cos A.
A 2 2
sin
2
Similarly,
B B
cot − sin B = cot cot B
2 2
C C
and cot − sin C = cot cot C.
2 2
A B C
Hence cos A cot , cos B cot and cos C cot are in A. P . ■
2 2 2

A
§ Problem 12.2.31. If a, b and c are in H. P., prove that sin2 ,
2
B C
sin2 and sin2 are also in H. P . ♢
2 2
§§ Solution.
Given a:c=a−b:b−c
∴ sin A (sin B − sin C) = sin C (sin A − sin B)
A A B+C B−C
∴ 2 sin cos · 2 cos sin
2 2 2 2
C C A+B A−B
= 2 sin cos · 2 cos sin
2 2 2 2
12.2. Sides And Angles of A Triangle 191

A B+C B−C C A+B A−B


∴ sin2 sin sin = sin2 sin sin
2 ( 2 2 ) 2 ( 2 2)
A B C C A B
∴ sin2 sin2 − sin2 = sin2 sin2 − sin2
2 2 2 2 2 2
A B C
∴ sin2 , sin2 and sin2 are in H. P. ■
2 2 2
§ Problem 12.2.32. The sides of a triangle are in A. P. and the
greatest and least angles are θ and ϕ; prove that
4 (1 − cos θ) (1 − cos θ) = cos θ + cos ϕ. ♢
§§ Solution. The third angle of the triangle is π − (θ + ϕ). We are
given
sin θ + sin ϕ = 2 sin [π − (θ + ϕ)] = 2 sin (θ + ϕ)
θ+ϕ θ−ϕ θ+ϕ θ+ϕ
∴ 2 sin cos = 4 sin cos
2 2 2 2
θ−ϕ θ+ϕ
∴ cos = 2 cos
2 2
θ ϕ θ ϕ θ ϕ θ ϕ
∴ cos cos + sin sin = 2 cos cos − 2 sin sin
2 2 2 2 2 2 2 2
θ ϕ θ ϕ
∴ 3 sin sin = cos cos (12.1)
2 2 2 2
θ ϕ
by adding sin sin to each side of Eq. (12.1):
2 2
θ ϕ θ ϕ θ ϕ θ−ϕ
4 sin sin = cos cos + sin sin = cos (12.2)
2 2 2 2 2 2 2
θ ϕ
by subtracting sin sin from each side of Eq. (12.1):
2 2
θ ϕ θ ϕ θ ϕ θ+ϕ
2 sin sin = cos cos − sin sin = cos (12.3)
2 2 2 2 2 2 2
Hence, from Eq. (12.2) and Eq. (12.3), by multiplication, we have
θ ϕ θ+ϕ θ−ϕ
16 sin2 sin2 = 2 cos cos
2 2 2 2
∴ 4 (1 − cos θ) (1 − cos ϕ) = cos θ + cos ϕ.
Otherwise thus :
1
Let a be the least side. Since b = (a + c), we have
2
a2 + b2 − c2 a2 − c2 b2 a−c a+c 5a − 3c
cos θ = = + = + = , and
2ab 2ab (
2ab a) 4a 4a
b2 + c2 − a2 (c + a)2 + 4 c2 − a2 c + a + 4(c − a) 5c − 3a
cos ϕ = = = = .
2bc 4c(c + a) 4c 4c
(3c − a)(3a − c) 10ac − 3a2 − 3c2
∴ 4 (1 − cos θ) (1 − cos ϕ) = =
4ac 4ac
(5a − 3c)c + (5c − 3a)a 10ac − 3a2 − 3c2
∴ cos θ + cos ϕ = =
4ac 4ac
∴ 4 (1 − cos θ) (1 − cos ϕ) = cos θ + cos ϕ. ■

§ Problem 12.2.33. The sides of a triangle are in A. P. and the


greatest angle exceeds ◦
√ √the least
√ by 90 ; prove that the sides are
proportional to 7 + 1, 7 and 7 − 1. ♢
12.2. Sides And Angles of A Triangle 192

§§ Solution. Let A be the greatest and C be the least angle of the


triangle ABC.
Given, a + c = 2b (12.4)
and A − C = 90◦ (12.5)
From Eq. (12.4), we have
sin A + sin C = 2 sin B = 2 sin(A + C)
A+C A−C A+C A+C
∴ 2 sin cos = 4 sin cos
2 2 2 2
A−C A+C B
∴ cos = 2 cos = 2 sin
2 2 2
1 B
∴ cos 45◦ = √ = 2 sin
2 … 2

B 1 B B 7
∴ sin = √ and cos = 1 − sin2 = √
2 2 2 2 2 2 2

B B 7
∴ sin A + sin C = 2 sin B = 4 sin cos = .
2 2 2
Also
A+C A−C B 1
sin A − sin C = 2 cos sin = 2 sin sin 45◦ = .
2 2 2 2
Hence, by addition, we have
√ √
7+1 7+1
2 sin A = , ∴ sin A =
2 4
and, by subtraction, we have
√ √
7−1 7−1
2 sin C = , ∴ sin C = .
2 4
∴ a : b : c = sin A : sin B : sin C
√ √ √
= 7 + 1 : 7 : 7 − 1. ■

§ Problem 12.2.34. If C = 60◦ , then prove that


1 1 3
+ = .
a+c b+c a+b+c

§§ Solution. We have
1 1 3
+ =
a+c b+c a+b+c
if (2c + a + b)(c + a + b) = 3(c + a)(c + b)
i.e. if a2 − ab + b2 = c2
i.e. if a − ab + b2 = a2 − 2ab cos C + b2
2

i.e. if 2 cos C = 1
i.e. if C = 60◦ . ■

§ Problem 12.2.35. In any triangle ABC, if D be any point of the


base BC, such that BD : DC :: m : n and if ∠BAD = α, ∠DAC = β,
∠CDA = θ and AD = x, prove that
(m + n) cot θ = m cot α − n cot β
= n cot B − m cot C
( )
and (m + n)2 · x2 = (m + n) mb2 + nc2 − mna2 . ♢
§§ Solution. Take the figure of Art. 218, with m and n for x and y
respectively.
12.2. Sides And Angles of A Triangle 193

We have
m BD BD AD sin ∠BAD sin ∠ACD
= = · = ·
n DC AD DC sin ∠ABD sin ∠DAC
sin (θ − B) sin C
= ·
sin B sin (θ + C)
sin θ cot B − cos θ cot B − cot θ
= =
sin θ cot C + cos θ cot C + cot θ
∴ (m + n) cot θ = n cot B − m cot C.
BC
Again, we have BD = BD ×
BC
BC ma
and DC = DC × ; i. e. BD =
BC m+n
na
and DC = .
m+n
Hence in the triangle ADC, we have
( )2
na na
AD2 + − 2AD · · cos θ = b2 (12.6)
m+n m+n
Hence in the triangle ADB, we have
( )2
ma ma
AD2 + + 2AD · · cos θ = c2 (12.7)
m+n m+n
Multiplying Eq. (12.6) by m and Eq. (12.7) by n, by addition,
mna2
(m + n)AD2 + = mb2 + nc2
m+( n )
∴ (m + n)2 AD2 = (m + n) mb2 + nc2 − mna2 . ■

§ Problem 12.2.36. If in a triangle, the bisector of the side c be


perpendicular to the side b, prove that
2 tan A + tan C = 0. ♢
§§ Solution. Let D be the middle point of c. Then the ∠ACD = 90◦ .
Through B, draw BE parallel to DC, meeting AC produced in E.
Then,
∵ AD = DB, ∴ AC = CE.
DC
We have tan A =
AC
BE 2DC
and tan C = − =− = −2 tan A.
CE AC
∴ 2 tan A + tan C = 0. ■

§ Problem 12.2.37. In any triangle, prove that, if θ be any angle,


then
b cos θ = c cos (A − θ) + a cos (C + θ) . ♢
§§ Solution. By Art. 170 and ∵ c sin A = a sin C; we have
b cos θ = (c cos A + a cos C) cos θ + (c sin A − a sin C) sin θ
∴ b cos θ = c (cos A cos θ + sin A sin θ) + a (cos C cos θ − sin C sin θ)
= c cos (A − θ) + a cos (C + θ) . ■

§ Problem 12.2.38. If p and q be the perpendiculars from the an-


gular points A and B on any line passing through the vertex C of the
triangle ABC, then prove that
a2 p2 + b2 q 2 − 2abpq cos C = a2 b2 sin2 C. ♢
12.2. Sides And Angles of A Triangle 194

§§ Solution. Let p and q meet the line through C in M and N re-


spectively. We then have
p q
sin ∠ACM = , and sin ∠BCN = .
b a
Also, ∠ACM + ∠BCN = 180◦ − C
∴ cos (∠ACM + ∠BCN ) = cos (180◦ − C)
∴ cos ∠ACM cos ∠BCN − sin ∠ACM sin ∠BCN = − cos C
… …
p2 q2 p q
∴ 1− 2 1 − 2 − · = − cos C
Å bãÅ a ã bÄ a ä2
p2 q2 pq
∴ 1− 2 1− 2 = − cos C
b a ab
p2 q2 p2 q 2 p2 q 2 pq
∴1− − 2 + 2 2 = 2 2 −2 cos C + cos2 C
b2 a a b a b ( ab )
∴ a2 p2 + b2 q 2 − 2abpq cos C = a2 b2 1 − cos2 C = a2 b2 sin2 C. ■

§ Problem 12.2.39. In the triangle ABC, lines OA, OB and OC are


drawn so that the angles ∠OAB, ∠OBC and ∠OCA are each equal to
ω; prove that
cot ω = cot A + cot B + cot C
and cosec 2 ω = cosec 2 A + cosec 2 B + cosec 2 C. ♢
§§ Solution. Since the ∠OBC = ∠OCA
∴ ∠BOC = 180◦ − ∠OCB − ∠OCA = 180◦ − C.
Similarly, ∠COA = 180◦ − A, and ∠AOB = 180◦ − B.
In the triangle OAB, we have
OA sin (B − ω)
= .
c sin B
In the triangle OAC, we have
b sin A
= .
OA sin ω
Hence, by multiplication, we have
b sin (B − ω) sin A
= ·
c sin ω sin B
sin B · sin B sin (B − ω) sin B cos ω − cos B sin ω
∴ = =
sin C · sin A sin ω sin ω
sin2 B
∴ = sin B cot ω − cos B
sin C sin A
sin B sin(A + C)
∴ cot ω = cot B + = cot B +
sin C sin A sin C sin A
∴ cot ω = cot A + cot B + cot C.
Again,
cosec 2 ω = 1 + cot2 ω
= 1 + cot2 A + cot2 B + cot2 C
+ 2 (cot A cot B + cot B cot C + cot C cot A)
= 3 + cot2 A + cot2 B + cot2 C, by §P roblem 9.2.14
= 1 + cot2 A + 1 + cot2 B + 1 + cot2 C
= cosec 2 A + cosec 2 B + cosec 2 C. ■
Chapter 13
Solution of Triangles

13.1 Solution of Right-Angled Triangles


§ Problem 13.1.1. In a right-angled triangle ABC, where C is the
right angle, if a = 50 and[B = 75◦ , find the ]sides.

tan 75◦ = 2 + 3. ♢
b ◦

§§ Solution. By Art. 177, we have = tan 75 = 2 + 3
( √ ) 50
∴ b = 50 2 + 3 = 50 × 3.73205 = 186.60 . . .
a 50
Also, c= = .
cos B √cos 75◦
3−1
Now by Art. 93, cos 75◦ = √
2 2
√ √ (√ )
100 2 100 2 3+1 (√ √ )
∴c= √ = = 50 6+ 2
3−1 2
= 50 [2.4494 + 1.4142] = 193.18. ■

§ Problem 13.1.2. Solve the triangle of which two-sides are equal


to 10 and 20 feet and of which the included angle is 90◦ ; given that
log 20 = 1.30103, and
L tan 26◦ 33′ = 9.6986847, dif f. f or 1′ = 3160. ♢
§§ Solution. In Art. 176, putting a = 20 and b = 10, we have
10 1
tan B = =
20 2
∴ L tan B = 10 − log 2 = 10 − 1.30103 = 9.69897.
( )
Let B = 26◦ 33′ + x′′ , ∴ L tan 26◦ 33′ + x′′ = 9.69897.
13.1. Solution of Right-Angled Triangles 196

The diff. for x′′ = 9.69897 − 9.6986847 = .0002853.


2853
Hence x = 60′′ × ≈ 54′′
3160
∴ B = 26◦ 33′ 54′′ ; ∴ A = 90◦ − B = 63◦ 26′ 6′′ .
√ √
Also, c = 202 + 102 = 10 5 f eet. ■

§ Problem 13.1.3. The length of the perpendicular from one angle


of a triangle upon the base is 3 inches and the lengths of the sides
containing this angle are 4 and 5 inches. Find the angles, having
given
log 2 = .30103, log 3 = .4771213
L sin 36◦ 52′ = 9.7781186, dif f. f or 1′ = 1684
and L sin 48◦ 35′′ = 9.8750142, dif f. f or 1′ = 1115. ♢
§§ Solution. Let AD(= 3 inches) be the perpendicular from A upon
BC the base of the ∆ABC and let AB = 4 inches and AC = 5 inches.
3 3×2 3 3
We then have sin C = = and sin B = = 2 .
5 10 4 2
∴ L sin C = 10 + log 3 + log 2 − log 10
= 10 + .4771213 + .30103 − 1 = 9.7781513.
( )
Let C = 36◦ 52′ + x′′ , ∴ L sin 36◦ 52′ + x′′ = 9.7781513.
The diff. for x′′ = 9.7781513 − 9.7781186 = .0000327.
327
∴ x = 60′′ × ≈ 12′′ ; ∴ C = 36◦ 52′ 12′′ .
1684
Again, L sin B = 10 + log 3 − 2 log 2
= 10 + .4771213 − .60206 = 9.8750613.
Let B = 48◦ 35′ + x′′
( )
∴ L sin 48◦ 35′ + x′′ = 9.8750613.
The diff. for x′′ = 9.8750613 − 9.8750142 = .0000471.
471
∴ x = 60′′ × ≈ 25′′
1115
∴ B = 48◦ 35′ 25′′ .
Also, A = 180◦ − (B + C) = 94◦ 32′ 23′′ . ■

§ Problem 13.1.4. Find the acute angles of a right-angled triangle


whose hypotenuse is four times as long as the perpendicular drawn
to it from the opposite angle. ♢
§§ Solution. Let ABC be the triangle, C the right angle and CD be
the perpendicular from C upon AB. We are given AB = 4CD.
∴ AC sec A = 4AC sin A
∴ 4 sin A cos A = 1
∴ 2 sin 2A = 1
1
∴ sin 2A = = sin 30◦ or sin 150◦
2
∴ 2A = 30◦ or 150◦
∴ A = 15◦ or 75◦ .
If A = 15◦ , then B = 90◦ − 15◦ = 75◦ .
If A = 75◦ , then B = 90◦ − 75◦ = 15◦ . ■
13.2. Solution of Non-Right-Angled Triangles 197

13.2 Solution of Non-Right-Angled Triangles


§ Problem 13.2.1. If the sides of a triangle be 56, 65, and 33 feet,
find the greatest angle. ♢
§§ Solution. The greatest angle is opposite the greatest side, so
that if a = 56 feet, b = 65 feet and c = 33 feet,then B is the required
angle. We have
1
s = (56 + 65 + 33) = 77, s − a = 21, s − b = 12, and s − c = 44.
2 …
B 44 × 21
∴ tan = = 1 = tan 45◦
2 77 × 12
B
∴ = 45◦ , and ∴ B = 90◦ .
2
Alternative Sol. Since 652 = 562 + 332 , the angle opposite the side
65 is a right angle. ■

√ √
§ Problem 13.2.2. The sides of a triangle are 7, 4 3, and 13 yards
respectively. Find the number of degrees in its smallest angle. ♢
√ √
§§ Solution. If a = 7 yards, b = 4 3 yards, and c = 13 yards, then
C is the required angle.
( √ ) 2 (√ ) 2
72 + 4 3 − 13 49 + 48 − 13
We have cos C = √ = √
2 × 7 × 4 3√ 56 3
84 3 3
= √ = √ = = cos 30◦ .
56 3 2 3 2
∴ C = 30◦ . ■

§ Problem 13.2.3. The sides of a triangle are x2 + x + 1, 2x + 1 and


x2 − 1 ; prove that the greatest angle is 120◦ . ♢
§§ Solution. If θ be the required angle, we have
( )2 ( )2
x2 − 1 + (2x + 1)2 − x2 + x + 1
cos θ =
2 (x2 − 1) (2x + 1)
( )
x4 − 2x2 + 1 + 4x2 + 4x + 1 − x4 + 2x3 + 4x2 + 2x + 1
=
2 (x2 − 1) (2x + 1)
( )
− 2x3 + x2 − 2x − 1 1
= = − = cos 120◦ .
2 (2x3 + x2 − 2x − 1) 2
∴ θ = 120◦ . ■


§ Problem 13.2.4. The sides of a triangle are a, b and a2 + ab + b2
feet; find the greatest angle. ♢
§§ Solution. If θ be the( required angle,
) we have
a2 + b2 − a2 + ab + b2 −ab 1
cos θ = = = − = cos 120◦ .
2ab 2ab 2
∴ θ = 120◦ . ■

√ √
§ Problem 13.2.5. If a = 2, b = 6 and c = 3−1, solve the triangle.

13.2. Solution of Non-Right-Angled Triangles 198

§§ Solution. We have
(√ )2 ( √ )2 √
6 + 3 − 1 − 22 6+4−2 3−4
cos A = √ (√ ) = √ (√ )
2 6 3−1 2 6 3−1
√ √ √ ( )
3− 3 3 3−1 1
= √ (√ ) = √ (√ ) = √ = cos 45◦ .
6 3−1 6 3−1 2
∴ A = 45◦ .
(√ )2 ( √ )2 √
3 − 1 + 22 − 6 4−2 3+4−6
cos B = (√ ) = (√ )
4 3−1 4 3−1

1− 3 1
= (√ ) = − = cos 120◦ .
2 3−1 2
∴ B = 120◦ .
∴ C = 180 − (A + B) = 180◦ − 165◦ = 15◦ .

√ √
§ Problem 13.2.6. If a = 2, b = 6 and c = 3+1, solve the triangle.

§§ Solution. We have
( √ )2 (√ )2 √
6 + 3 + 1 − 22 6+4+2 3−4
cos A = √ (√ ) = √ (√ )
2 6 3+1 2 6 3+1
√ (√ )
3 3+1 1
= √ (√ ) = √ = cos 45◦ .
6 3+1 2
∴ A = 45◦ .
(√ )2 ( √ )2 √
3 + 1 + 22 − 6 4+2 3+4−6
cos B = (√ ) = (√ )
4 3+1 4 3+1

3+1 1
= (√ ) = = cos 60◦ .
2 3+1 2
∴ B = 60◦ .
∴ C = 180 − (A + B) = 180◦ − 105◦ = 75◦ .

§ Problem 13.2.7. If a = 9, b = 10 and c = 11, find B, given


log 2 = .30103, L tan 29◦ 29′ = 9.7523472
and L tan 29◦ 30′ = 9.7526420.

§§ Solution. We have
1
s = (9 + 10 + 11) = 15, s − a = 6, s − b = 5 and s − c = 4
2
… … …
B 4×6 8 25
∴ tan = = =
2 15 × 5 25 100
B 1
∴ L tan = 10 + (5 log 2 − 2) = 9.7525750.
2 2
B
Let = 29◦ 29′ + x′′ , ∴ L tan (29◦ 29′ + x′′ ) = 9.7525750.
2
The diff. for x′′ = 9.7525750 − 9.7523472 = .0002278.
The diff. for 60′′ = 9.7526420 − 9.7523472 = .0002948.
13.2. Solution of Non-Right-Angled Triangles 199

2278
∴ x = 60′′ × ≈ 46.3′′ .
2948
B
∴ = 29◦ 29′ 46.3′′ and B = 58◦ 59′ 33′′ . ■
2
§ Problem 13.2.8. The sides of a triangle are 130, 123 and 77 feet.
Find the greatest angle,having given
log 2 = .30103, L tan 38◦ 39′ = 9.9029376
and L tan 38◦ 40′ = 9.9031966. ♢
§§ Solution. If a = 130 f eet, b = 123 f eet and c = 77 f eet, then A is
the required angle. We have
1
s = (130 + 123 + 77) = 165, s − a = 35, s − b = 42 and s − c = 88
2 …
A 42 × 88 4 23
∴ tan = = =
2 165 × 35 5 10
A
∴ L tan = 10 + 3 log 2 − log 10 = 9.90309.
2
A
Let = 38◦ 39′ + x′′ , so that L tan (38◦ 39′ + x′′ ) = 9.90309.
2
The diff. for x′′ = 9.90309 − 9.9029376 = .0001524.
The diff. for 60′′ = 9.9031966 − 9.9029376 = .0002590.
1524
∴ x = 60′′ × ≈ 35.3′′ .
2590
A
∴ = 38◦ 39′ 35.3′′ and A = 77◦ 19′ 11′′ . ■
2
§ Problem 13.2.9. Find the greatest angle of a triangle whose sides
are 212, 188 and 270 feet, having given
log 2 = .30103, log 3 = .4771213, log 7 = .8450980
L tan 38◦ 20′ = 9.8980104 and L tan 38◦ 19′ = 9.8977507. ♢
§§ Solution. If a = 242 f eet, b = 188 f eet and c = 270 f eet, then C is
the required angle.
We have
1
s = (242 + 188 + 270) = 350, s − a = 108
2
s − b = 162 and s − c = 80.
… …
C 108 × 162 2 × 37
∴ tan = =
2 350 × 80 7 × 103
C 1
∴ L tan = 10 + (7 log 3 + log 2 − log 7 − 3 log 10)
2 2
1
= 10 + (3.3398491 + .3010300 − .8450980 − 3) = 9.8978905.
2
C
Let = 38◦ 19′ + x′′ , so that L tan (38◦ 19′ + x′′ ) = 9.8978905.
2
The diff. for x′′ = 9.8978905 − 9.8977507 = .0001398.
The diff. for 60′′ = 9.8980104 − 9.8977507 = .0002597.
1398
Hence x = 60′′ × ≈ 32.3′′
2597
C
∴ = 38◦ 19′ 32.3′′ and C = 76◦ 39′ 5′′ . ■
2
13.2. Solution of Non-Right-Angled Triangles 200

§ Problem 13.2.10. The sides of a triangle are 2, 3 and 4; find the


greatest angle, having given
log 2 = .30103, log 3 = .4771213
L tan 52◦ 14′ = 10.1108395
and L tan 52◦ 15′ = 10.1111004. ♢
§§ Solution. If a = 2, b = 3 and c = 4, then C is the required angle.
We have
1 9 5 3 1
s = (2 + 3 + 4) = , s − a = , s − b = and s − c =
2 2Õ 2 2 2
5
×
3 … …
C 2 2 = 5 10
∴ tan = = .
2 9 1 3 3×2
×
2 2
C 1
∴ L tan = 10 + (log 10 − log 2 − log 3) = 10.1109243.
2 2
C
Let = 52◦ 14′ + x′′ , so that L tan (52◦ 14′ + x′′ ) = 10.1109243.
2
The diff. for x′′ = 10.1109243 − 10.1108395 = .0000848.
The diff. for 60′′ = 10.1111004 − 10.1108395 = .0002609.
848
∴ x = 60′′ × ≈ 19.5′′
2609
C
∴ = 52◦ 14′ 19.5′′ and C = 104◦ 28′ 39′′ . ■
2

Making use of the tables, find all the angles when


§ Problem 13.2.11. a = 25, b = 26 and c = 27. ♢
§§ Solution. We have
1
s = (25 + 26 + 27) = 39, s − a = 14, s − b = 13 and s − c = 12
2 … …
A 13 × 12 2
∴ tan = =
2 39 × 14 7
A 1
∴ L tan = 10 + (log 2 − log 7) = 9.7279660.
2 2
Now L tan 28◦ 7′ = 9.7278048, diff. for 1′ = 3039.
A
Let = 28◦ 7′ + x′′ , so that L tan (28◦ 7′ + x′′ ) = 9.7279660.
2
The diff. for x′′ = 9.7279660 − 9.7278048 = .0001612.
1612
∴ x = 60′′ × ≈ 32′′
3039
A
∴ = 28◦ 7′ 32′′ and A = 56◦ 15′ 4′′ .
2 … √
B 12 × 14 2 14
Again, tan = =
2 39 × 13 13
B 1
∴ L tan = 10 + log 2 + log 14 − log 13 = 9.7601506.
2 2
Now L tan 29◦ 55′ = 9.7599794, diff. for 1′ = 2922.
B
Let = 29◦ 55′ + x′′ , so that L tan (29◦ 55′ + x′′ ) = 9.7601506.
2
The diff. for x′′ = 9.7601506 − 9.7599794 = .0001712.
13.2. Solution of Non-Right-Angled Triangles 201

1712
Hence x = 60′′ × ≈ 35′′
2922
B
∴ = 29◦ 55′ 35′′ and B = 59◦ 51′ 10′′ .
2
Also, C = 180 − (A + B) = 180◦ − 116◦ 6′ 14′′ = 63◦ 53′ 46′′ .

§ Problem 13.2.12. a = 17, b = 20 and c = 27. ♢


§§ Solution. We have
1
s = (17 + 20 + 27) = 32, s − a = 15, s − b = 12 and s − c = 5
2 … …
A 12 × 5 1
∴ tan = =
2 32 × 15 23
A 1
∴ L tan = 10 + (log 1 − 3 log 2) = 9.5484550.
2 2
Now L tan 19◦ 28′ = 9.5483452, diff. for 1′ = 4019.
A
Let = 19◦ 28′ + x′′ , so that L tan (19◦ 28′ + x′′ ) = 9.5484550.
2
The diff. for x′′ = 9.5484550 − 9.5483452 = .0001098.
1098
∴ x = 60′′ × ≈ 16.4′′
4019
A
∴ = 19◦ 28′ 16.4′′ and A = 38◦ 56′ 33′′ .
2 …
B 5 × 15 5 10
Again, tan = = √ = √
2 32 × 12 27 29
B 1
∴ L tan = 10 + log 10 − (9 log 2) = 9.6453650.
2 2
Now L tan 23 50 = 9.6451743, diff. for 1′ = 3417.
◦ ′

B
Let = 23◦ 50′ + x′′ , so that L tan (23◦ 50′ + x′′ ) = 9.6453650.
2
The diff. for x′′ = 9.6453650 − 9.6451743 = .0001907.
1907
Hence x = 60′′ × ≈ 33.5′′
3417
B
∴ = 23◦ 50′ 33.5′′ and B = 47◦ 41′ 7′′ .
2
Also, C = 180◦ − (A + B) = 180◦ − 86◦ 37′ 40′′ = 93◦ 22′ 20′′ . ■

§ Problem 13.2.13. a = 2000, b = 1050 and c = 1150. ♢


§§ Solution. We have
1
s = (2000 + 1050 + 1150) = 2100, s − a = 100, s − b = 1050
2
and s − c = 950.
… √
A 1050 × 950 19
∴ tan = =
2 2100 × 100 2
A 1
∴ L tan = 10 + log 19 − log 2 = 10.3383468.
2 2
Now L tan 65◦ 21′ = 10.3382897, diff. for 1′ = 3334.
A
Let = 65◦ 21′ + x′′ , so that L tan (65◦ 21′ + x′′ ) = 10.3383468.
2
The diff. for x′′ = 10.3383468 − 10.3382897 = .0000571.
571
∴ x = 60′′ × ≈ 10.276′′
3334
13.3. Two Sides and Included Angle 202

A
∴ = 65◦ 21′ 10.276′′ and A = 130◦ 42′ 20.5′′ .
2 … √
B 950 × 100 19
Again, tan = =
2 2100 × 1050 21
B 1
∴ L tan = 10 + log 19 − log 21 = 9.3171575.
2 2
Now L tan 11◦ 43′ = 9.3167950, diff. for 1′ = 6349.
B
Let = 11◦ 43′ + x′′ , so that L tan (11◦ 43′ + x′′ ) = 9.3171575.
2
The diff. for x′′ = 9.3171575 − 9.3167950 = .0003625.
3625
Hence x = 60′′ × ≈ 34.25′′
6349
B
∴ = 11◦ 43′ 34.25′′ and B = 23◦ 27′ 8.5′′ .
2
Also, C = 180◦ − (A + B) = 180◦ − 154◦ 9′ 29′′ = 25◦ 50′ 31′′ . ■

13.3 Two Sides and Included Angle


§ Problem 13.3.1. If b = 90, c = 70 and A = 72◦ 48′ 30′′ , find B and C,
given
log 2 = .30103, L cot 36◦ 24′ 15′′ = 10.1323111
L tan 9◦ 37′ = 9.2290071
and L tan 9◦ 38′ = 9.2297735. ♢
§§ Solution. We have
B−C b−c A 1 A 1
tan = cot = cot = 3 cot 36◦ 24′ 15′′
2 b+c 2 8 2 2
B−C
∴ L tan = log 1 − 3 log 2 + L cot 36◦ 24′ 15′′
2
= 0 − .90309 + 10.1323111 = 9.2292211.
B−C
Let = 9 37 + x , so that L tan (9◦ 37′ + x′′ ) = 9.2292211.
◦ ′ ′′
2
The diff. for x′′ = 9.2292211 − 9.2290071 = .0002140.
The diff. for 60′′ = 9.2297735 − 9.2290071 = .0007664.
2140
∴ x = 60′′ × ≈ 17′′
7664
B−C
∴ = 9◦ 37′ 17′′ (13.1)
2
B+C A
But = 90◦ − = 90◦ − 36◦ 24′ 15′′ = 53◦ 35′ 45′′ (13.2)
2 2
By adding (13.1) and (13.2), we have B = 63◦ 13′ 2′′ .
By subtracting (13.1) from (13.2), we have C = 43◦ 58′ 28′′ . ■

§ Problem 13.3.2. If a = 21, b = 11 and C = 34◦ 42′ 30′′ , find A and


B, given
log 2 = .30103
and L tan 72◦ 38′ 45′′ = 10.50515. ♢
§§ Solution. We have
A−b a−b C 10
tan = cot = cot 17◦ 21′ 15′′
2 a+b 2 32
13.3. Two Sides and Included Angle 203

A−B
∴ L tan = log 10 − 5 log 2 + L tan 72◦ 38′ 45′′
2
= 1 − 1.50515 + 10.50515 = 10.
A−B
∴ 10 + log tan = 10
2
A−B A−B
∴ log tan = 0, ∴ tan = 1 = tan 45◦
2 2
A−B
∴ = 45◦ (13.3)
2
A+B C
but = 90◦ − = 90◦ − 17◦ 21′ 15′′ = 72◦ 38′ 45′′ (13.4)
2 2
By adding (13.3) and (13.4), we have A = 117◦ 38′ 45′′ .
By subtracting (13.3) from (13.4), we have B = 27◦ 38′ 45′′ . ■

§ Problem 13.3.3. If the angles of a triangle be in A. P. and the


lengths of the greatest and least sides be 24 and 16 feet respectively,
find the length of the third side and the angles, given
log 2 = .30103, log 3 = .4771213
and L tan 19◦ 6′ = 9.5394287, diff. for 1′ = 4084. ♢
§§ Solution. Let A, B and C be the angles. Since they are in A. P.,
we have
2B = A + C
∴ 3B = A + B + C = 180◦ ; ∴ B = 60◦ and A + C = 120◦ .
Let a = 24 f eet and c = 16 f eet. We then have

A−C a−c A+C 2 2 3
tan = tan = tan 60◦ =
2 a+c 2 10 10
A−C 1
∴ L tan = 10 + log 2 + log 3 − log 10
2 2
= 10 + .30103 + .2385607 − 1 = 9.5395907.
A−C A−C
Let = 19◦ 6′ + x′′ , so that L tan = 9.5395907.
2 2
′′
The diff. for x = 9.5395907 − 9.5394287 = .0001620.
The diff. for 60′′ = .0004084.
1620
∴ x = 60′′ × ≈ 24′′
4084
A−C
∴ = 19◦ 6′ 24′′ (13.5)
2
A+C
But = 60◦ (13.6)
2
By adding (13.5) and (13.6), we have A = 79 6 24 . ◦ ′ ′′

By subtracting (13.5) from (13.6), we have C = 40◦ 53′ 36′′ .


Also, b2 = a2 + c2 − 2ac cos B = 576 + 256 − 384 = 448 = 64 × 7

∴ b = 8 7 f eet. ■

§ Problem 13.3.4. If a = 13, b = 7 and C = 60◦ , find A and B, given


that
log 3 = .4771213
and L tan 27◦ 27′ = 9.7155508, diff. for 1′ = 3087. ♢
§§ Solution. We have
√ 3
A−B a−b C 3 3 3 32
tan = cot = cot 30◦ = =
2 a+b 2 10 10 10
13.3. Two Sides and Included Angle 204

A−B 3
∴ L tan = 10 + log 3 − log 10 = 10 + .7156819 − 1 = 9.7156819.
2 2
A−B
Let = 27◦ 27′ + x′′ , so that L tan (27◦ 27′ + x′′ ) = 9.7156819.
2
The diff. for x′′ = 9.7156819 − 9.7155508 = .0001311.
The diff. for 60′′ = .0003087.
1311
∴ x = 60′′ × ≈ 25.5′′
3087
A−B
∴ = 27◦ 27′ 25.5′′ (13.7)
2
A+B ◦ C ◦ ◦ ◦
But = 90 − = 90 − 30 = 60 (13.8)
2 2

By adding (13.7) and (13.8), we have A = 87 27 25.5 . ′ ′′

By subtracting (13.7) from (13.8), we have B = 32◦ 32′ 34.5′′ . ■

§ Problem 13.3.5. If a = 2b and C = 120◦ , find the values of A, B


and the ratio of c to a, given that
log 3 = .4771213
and L tan 10◦ 53′ = 9.2839070, diff. for 1′ = 6808. ♢
§§ Solution. We have
A−B a−b C 2b − b 1 1 1
tan = cot = cot 60◦ = · √ = 3
2 a+b 2 2b + b 3 3 32
A−B 3
∴ L tan = 10 − log 3 = 10.7156819 = 9.2843181.
2 2
A−B
Let = 10◦ 53′ + x′′ , so that L tan (10◦ 53′ + x′′ ) = 9.2843181.
2
The diff. for x′′ = 9.2843181 − 9.283907 = .0004111.
The diff. for 60′′ = .0006808.
4111
∴ x = 60′′ × ≈ 36′′
6808
A−B
∴ = 10◦ 53′ 36′′ (13.9)
2
A+B C
But = 90◦ − = 90◦ − 60◦ = 30◦ (13.10)
2 2
By adding (13.9) and (13.10), we have A = 40◦ 53′ 36′′ .
By subtracting (13.9) from (13.10), we have B = 19◦ 6′ 24′′ .
( )
a2 a 1
Also, c2 = a2 + b2 − 2ab cos C = a2 + − 2a × −
4 2 2
a2 a2 7a2
= a2 + + =
4 2 4 √
c2 7 c 7
∴ 2 = , ∴ =
a 4 √a 2
∴ c : a = 7 : 2. ■

§ Problem 13.3.6. If b = 14, c = 11 and A = 60◦ , find B and C, given


that
log 2 = .30103, log 3 = .4771213
L tan 11◦ 44′ = 9.3174299
and L tan 11◦ 45′ = 9.3180640. ♢
13.3. Two Sides and Included Angle 205

§§ Solution. We have
√ 3
B−C b−c A 3 3 3 3 2 × 22
tan = cot = cot 30◦ = =
2 b+c 2 25 25 100
B−C 3
∴ L tan = 10 + log 3 + 2 log 2 − log 100
2 2
= 10 + .7156819 + .60206 − 2 = 9.3177419.
B−C
Let = 11◦ 44′ + x′′ , so that L tan (11◦ 44′ + x′′ ) = 9.3177419.
2
The diff. for x′′ = 9.3177419 − 9.3174299 = .0003120.
The diff. for 60′′ = 9.3180640 − 9.3174299 = .0006341.
3120
∴ x = 60′′ × ≈ 30′′
6341
B−C
∴ = 11◦ 44′ 30′′ (13.11)
2
B+C A
But = 90◦ − = 90◦ − 30◦ = 60◦ (13.12)
2 2
By adding (13.11) and (13.12), we have B = 71 44 30 . ◦ ′ ′′

By subtracting (13.11) from (13.12), we have C = 48◦ 15′ 30′′ . ■

§ Problem 13.3.7. The two sides of a triangle are 540 and 420 yards
long respectively and include an angle of 52◦ 6′ . Find the remaining
angles, given that
log 2 = .30103, L tan 26◦ 3′ = 9.6891430
L tan 14 20 = 9.4074189 and L tan 14◦ 21′ = 9.4079453.
◦ ′

§§ Solution. If b = 540 yards, c = 420 yards, and A = 52◦ 6′ , we have,
since
B−C A b−c B−C 540 − 420 1 1
tan tan = , tan tan 26◦ 6′ = = = 3
2 2 b+c 2 540 + 420 8 2
B−C
∴ L tan + L tan 26◦ 3′ = 20 + log 1 − 3 log 2
2
B−C
∴ L tan = 20 + 0 − .90309 − 9.6891430 = 9.4077670.
2
B−C
Let = 14◦ 20′ + x′′ , so that L tan (14◦ 20′ + x′′ ) = 9.4077670.
2
The diff. for x′′ = 9.4077670 − 9.4074189 = .0003481.
The diff. for 60′′ = 9.4079453 − 9.4074189 = .0005264.
3481
∴ x = 60′′ × ≈ 40′′
5264
B−C
∴ = 14◦ 20′ 40′′ (13.13)
2
B+C A
But = 90◦ − = 90◦ − 26◦ 3′ = 63◦ 57′ (13.14)
2 2
By adding (13.13) and (13.14), we have B = 78 17 40 . ◦ ′ ′′

By subtracting (13.13) from (13.14), we have C = 49◦ 36′ 20′′ . ■

1
§ Problem 13.3.8. If b = 2 f t., c = 2 f t. and A = 22◦ 20′ , find the
2
other angles and show that the third side is nearly one foot, given
log 2 = .30103, log 3 = .47712
L cot 11◦ 10′ = 10.70465, L sin 22◦ 20′ = 9.57977
L tan 29◦ 22′ 20′′ = 9.75038, L tan 29◦ 22′ 30′′ = 9.75013
13.3. Two Sides and Included Angle 206

and L sin 49◦ 27′ 34′′ = 9.88079. ♢


§§ Solution. We have ( )
1
B−C b−c A 2 1
tan = cot = cot 11◦ 10′ = 2 cot 11◦ 10′
2 b+c 2 4 12 3
B−C
∴ L tan = log 1 − 2 log 3 + L cot 11◦ 10′
2
= 0 − .95424 + 10.70465 = 9.75041.
B−C
Let = 29◦ 22′ 20′′ + x′′ , so that L tan (29◦ 22′ 20′′ + x′′ ) = 9.75041.
2
The diff. for x′′ = 9.75041 − 9.75038 = .00003.
The diff. for 10′′ = 9.75043 − 9.75038 = .00005.
3
∴ x = 10′′ × = 6′′
5
B−C
∴ = 29◦ 22′ 26′′ (13.15)
2
B+C A
But = 90◦ − = 90◦ − 11◦ 10′ = 78◦ 50′ (13.16)
2 2
By adding (13.15) and (13.16), we have B = 108 12 26 . ◦ ′ ′′

By subtracting (13.15) from (13.16), we have C = 49◦ 27′ 34′′ .

c sin A
To find a, we have a=
sin C
∴ log a = log c + L sin A − L sin C = .30103 + 9.57977 − 9.88079
= .00001 ≈ 0
∴ a ≈ 1 f oot. ■


§ Problem 13.3.9. If a = 2, b = 1 + 3 and C = 60◦ , solve the
triangle. ♢
§§ Solution. We have
√ ( √ )1
c2 = a2 + b2 − 2ab cos C = 4 + 4 + 2 3 − 4 1 + 3 =6
√ 2
∴ c = 6.

a 2 2 3 1
Also, sin A = sin C = √ sin 60◦ = √ · = √
c 6 6 2 2
∴ A = 45◦
∴ B = 180◦ − (A + C) = 180◦ − 105◦ = 75◦ . ■

√ √
§ Problem 13.3.10. Two sides of a triangle are 3 + 1 and 3 − 1

and the included angle is 60 ; find the other side and angles. ♢
√ √
§§ Solution. If b = 3 + 1, c = 3 − 1 and A = 60◦ , we have
√ √ 1
a2 = b2 + c2 − 2bc cos A = 4 + 2 3 + 4 − 2 3 − 2 × 2 × = 6
√ 2
∴ a = 6.
√ √ √
c 3−1 3 3−1
Also, sin C = sin A = √ · = √
a 6 2 2 2

∴ C = 15
∴ B = 180◦ − (A + C) = 180◦ − 75◦ = 105◦ . ■
13.3. Two Sides and Included Angle 207


§ Problem 13.3.11. If b = 1, c = 3 − 1 and A = 60◦ , find the length
of the side a. ♢
§§ Solution.
a2 = b2 + c2 − 2bc cos A
√ (√ )1 √
=1+4−2 3−2 3−1 =6−3 3
2
= 6 − 3 (1.73205) = .80385

∴ a = .80385 = .8965. ■

A 17
§ Problem 13.3.12. If b = 91, c = 125 and tan = , prove that
2 6
a = 204. ♢
§§ Solution. We have
A 289
1 − tan2 1−
cos A = 2 = 36 = − 253
A 289 325
1 + tan2 1+
2 36
∴ a = b + c − 2bc cos A
2 2 2

253
= 8281 + 15625 + 2 × 91 × 125 ×
325
= 8281 + 15625 + 17710 = 41616
∴ a = 204. ■

31
§ Problem 13.3.13. If a = 5, b = 4 and cos(A − B) = , prove that
32
the third side c will be 6. ♢
§§ Solution. We have Õ
… 31
1−

A−B 1 − cos(A − B) 32 = 1 1
tan = = = √ .
2 1 + cos(A − B) 31 63 3 7
1+
32
A+B a+b A−B 9 1 3 C
Also, tan = tan = · √ = √ = cot
2 a−b 2 1 3 7 7 2
C 9
cot2 −1 −1 2 1
∴ cos C = 2 = 7 = =
C 9 16 8
cot2 +1 +1
2 7
1
∴ c2 = a2 + b2 − 2ab cos C = 25 + 16 − 2 × 5 × 4 × = 36
8
∴ c = 6. ■

§ Problem 13.3.14. One angle of a triangle


√ is 30◦ and the lengths
of the sides adjacent to it are 40 and 40 3 yards. Find the length of
the third side and the number of degrees
√ in the other angles. ♢
§§ Solution. If b = 40 yards, c = 40 3 yards and A = 30◦ , we have
a2 = b2 + c2 − 2bc cos A

√ 3
= 1600 + 4800 − 2 × 40 × 40 3 × = 1600
2
∴ a = 40 yards = b.
∴ B = A = 30◦ .
∴ C = 180 − (A + B) = 180◦ − 60◦ = 120◦ .


13.3. Two Sides and Included Angle 208

§ Problem 13.3.15. The sides of a triangle are 9 and 3 and the dif-
ference of the angles opposite to them is 90. Find the base and the
angles, having given
log 2 = .30103, log 3 = .4771213
log 75894 = 4.8802074, log 75895 = 4.8802132
L tan 26◦ 33′ = 9.6986847
and L tan 26◦ 34′ = 9.6990006. ♢
§§ Solution. If b = 9, c = 3 and B − C = 90◦ , we have, since
B−C b−c A
tan = cot
2 b+c 2
◦ 1 A
tan 45 = cot
2 2
1 A A 1
∴ 1 = cot , ∴ tan =
2 2 2 2
A
∴ L tan = log 1 − log 2 + 10 = 0 − .30103 + 10 = 9.69897.
2
A ( )
Let = 26◦ 33′ + x′′ , ∴ L tan 26◦ 33′ + x′′ = 9.69897.
2
The diff. for x′′ = 9.69897 − 9.6986847 = .0002853.
The diff. for 60′′ = 9.6990006 − 9.6986847 = .0003159.
2853
∴x= × 60′′ = 54.2′′
3159
A
∴ = 26◦ 33′ 54.2′′ , ∴ A = 53◦ 7′ 48′′
2
B+C A
∴ = 90◦ − = 90◦ − 26◦ 33′ 54′′ = 63◦ 26′ 6′′ (13.17)
2 2
B−C
also, = 45◦ (13.18)
2
By adding (13.17) and (13.18), we have B = 108◦ 26′ 6′′ .
By subtracting (13.18) from (13.17), we have C = 18◦ 26′ 6′′ .
A 1
1 − tan2 1−
Again, cos A = 2 = 4 = 3
A 1 5
1 + tan2 1+
2 4
3
∴ a2 = b2 + c2 − 2bc cos A = 81 + 9 − 2 × 9 × 3 ×
5
162 (24)2 32 × 26
= 90 − = 90 − 32.4 = 57.6 = =
5 10 10
∴ 2 log a = 2 log 3 + 6 log 2 − log 10
1
∴ log a = log 3 + 3 log 2 − log 10
2
= .4771213 + .90309 − .5 = .8802113.
We have log 7.5894 = .8802074 (13.19)
and log 7.5895 = .8802132 (13.20)
Let log (7.5894 + x) = .8802113 (13.21)
From (13.19) and (13.20), the diff. for .0001 = .0000058.
From (13.19)and (13.21), the diff. for x = .0000039.
39 .0039
∴x= × .0001 = ≈ .000067.
58 58
∴ a = 7.589467. ■
13.3. Two Sides and Included Angle 209

§ Problem 13.3.16.
a−b C
If tan ϕ = cot
a+b 2
C
sin
prove that c = (a + b) 2 .
cos ϕ
If a = 3, b = 1 and C = 53◦ 7◦ 48′′ , find c without getting A and B, given
log 2 = .30103, log 25298 = 4.4030862
log 25299 = 4.4031034, L cos 26◦ 33′ 54′′ = 9.9515452
and L tan 26◦ 33′ 54′′ = 9.6989700. ♢
§§ Solution. We have
c2 = a2 + b2 − 2ab cos C
( )( 2 C C
) (
C C
)
= a2 + b2 cos + sin2 − 2ab cos2 − sin2
2 2 2 2
2 C 2 C
= (a − b) cos
2 2
+ (a + b) sin
2ï 2 ò
( )
2 2 C a−b 2 2 C
= (a + b) sin cot +1
2 a+b 2
2 2 C
( 2 )
= (a + b) sin tan ϕ + 1
2
C
= (a + b)2 sin2 sec2 ϕ
2
C
sin
∴ c = (a + b) 2 .
cos ϕ
If a = 3, b = 1 and C = 53◦ 7′ 48′′ , we have
1 C 1
tan ϕ = cot = .
2 2 C
2 tan
2
C
∴ L tan ϕ − 10 = log 1 − log 2 − L tan + 10
2
∴ L tan ϕ = 20 − .30103 − 9.69897 = 10
∴ 10 + log tan ϕ = 10, ∴ log tan ϕ = 0, ∴ tan ϕ = 1
∴ ϕ = 45◦ .
C C C
Also, L sin = L tan + L cos − 10
2 2 2
= 9.69897 + 9.9515452 − 10 = 9.6505152.
C C √
Now c = (a + b) sin sec 45◦ = 4 sin × 2
2 2
C 1
∴ log c = 2 log 2 + L sin − 10 + log 2
2 2
5
= (.30103) + 9.6505152 − 10 = .4030902.
2
Now log 2.5298 = .4030862 (13.22)
and log 2.5299 = .4031034 (13.23)
Let log (2.5298 + x) = .4030902 (13.24)
From (13.22)and (13.23), the diff. for .0001 = .0000172.
From (13.23) and (13.24), the diff. for x = .0000040.
40 .004 .001
∴x= × .0001 = = ≈ .000023.
172 172 43
13.3. Two Sides and Included Angle 210

∴ c = 2.529823. ■

§ Problem 13.3.17. Two sides of a triangle are 237 and 158 feet and
the contained angle is 66◦ 40′ ; find the base and the other angles,
having given
log 2 = .30103, log 79 = 1.89763
log 22687 = 4.35578, L cot 33◦ 20′ = 10.18197
L sin 33◦ 20′ = 9.73998, L tan 16◦ 54′ = 9.48262
L tan 16◦ 55′ = 9.48308, L sec 16◦ 54′ = 10.01917
and L sec 16◦ 55′ = 10.01921.
B−C b+c A
Note : Use either the formula cos = sin or the formula
2 a 2
of the preceding problem. ♢
§§ Solution. If b = 237, c = 158 and A = 66◦ 40′ , we have
B−C b−c A 79 A 1 A 2
tan = cot = cot = cot = cot 33◦ 20′
2 b+c 2 395 2 5 2 10
B−C
∴ L tan = log 2 − log 10 + L cot 33◦ 20′
2
= .30103 − 1 + 10.18197 = 9.48300.
B−C
Let = 16 54 + x , so that L tan (16◦ 54′ + x′′ ) = 9.48300.
◦ ′ ′′
2
The diff. for x′′ = 9.48300 − 9.48262 = .00038.
The diff. for 60′′ = 9.48308 − 9.48262 = .00046.
38
∴ x = 60′′ × ≈ 50′′ .
46
B−C
∴ = 16◦ 54′ 50′′ (13.25)
2
B+C A
But = 90◦ − = 90◦ − 33◦ 20′ = 56◦ 40′ (13.26)
2 2
By adding (13.25) and (13.26), we have B = 73◦ 34′ 50′′ .
subtracting (13.25) from (13.26), we have C = 39◦ 45′ 10′′ .
A B−C
Again, we have a = (b + c) sin sec
2 2
∴ log a = log 395 + L sin 33◦ 20′ + L sec 16◦ 54′ 50′′ − 20
= log 790 − log 2 + L sin 33◦ 20′ + L sec 16◦ 54′ 50′′ − 20.
Now
50
L sec 16◦ 54′ 50′′ = 10.01917 + (10.01921 − 10.01917)
60
5
= 10.01917 + (.00004) = 10.01917 + .00003 = 10.01920
6
∴ log a = 2.89763 − .30103 + 9.73998 + 10.01920 − 20 = 2.35578
∴ a = 226.87 f eet. ■

In the following four examples, the required logarithms must


be taken from the tables :
§ Problem 13.3.18. If a = 242.5, b = 164.3 and C = 54◦ 36′ , solve the
triangle. ♢
§§ Solution. We have
A−B a−b C 39.1
tan = cot = cot 27◦ 18′
2 a+b 2 203.4
13.3. Two Sides and Included Angle 211

A−B
∴ L tan = log 39.1 − log 203.4 + L cot 27◦ 18′
2
= 1.5921768 − 2.3083509 + 10.2872338 = 9.5710597.
Now L tan 20◦ 25′ = 9.5708088, diff. for 1′ = 3863.
A−B ( )
Let = 20◦ 25′ + x′′ , ∴ L tan 20◦ 25′ = 9.5710597.
2
The diff. for x′′ = 9.5710597 − 9.5708088 = .0002509.
2509
∴ x = 60′′ × ≈ 39′′
3863
A−B
∴ = 20◦ 25′ 39′′ (13.27)
2
A+B ◦ C ◦ ◦ ′ ◦ ′
But = 90 − = 90 − 27 18 = 62 42 (13.28)
2 2
By adding (13.27) and (13.28), we have A = 83 7 39 . ◦ ′ ′′

By subtracting (13.27) from (13.28), we have B = 42◦ 16′ 21′′ .


a sin C
To find c, we have c=
sin A
∴ log c = log a + L sin C − L sin A
= log 242.5 + L sin 54◦ 36′ − L sin 83◦ 7′ 39′′ .
39
Now L sin 83◦ 7′ 39′′ = 9.9968584 + × .0000152
60
= 9.9968584 + .0000099 = 9.9968683.
∴ log c = 2.3847117 + 9.9112257 − 9.9968683 = 2.2990691.
Now log 199.09 = 2.2980494, diff. for .01 = .0000219.
Let log (199.09 + x) = 2.2990691.
The diff. for x = 2.2990691 − 2.2980494 = .0000197.
197 1.97
∴x= × .01 = ≈ .009
219 219
∴ c = 199.099. ■

§ Problem 13.3.19. If b = 130, c = 63 and A = 42◦ 15′ 30′′ , solve the


triangle. ♢
§§ Solution. We have
B−C b−c A 67
tan = cot = cot 21◦ 7′ 45′′ .
2 b+c 2 193
B−C
∴ L tan = log 67 − log 193 + L cot 21◦ 7′ 45′′ .
2
45
Now L cot 21◦ 7′ 45′′ = 10.4131853 − × .0003757
60
= 10.4131853 − .0002818 = 10.4129035.
B−C
∴ L tan = 1.8260748 − 2.2855573 + 10.4129035
2
= 9.9534210 = L tan 41◦ 56′ .
B−C
∴ = 41◦ 56′ (13.29)
2
B+C A
But = 90◦ − = 90◦ − 21◦ 7′ 45′′ = 68◦ 52′ 15′′ (13.30)
2 2
By adding (13.29) and (13.30), we have B = 110◦ 48′ 15′′ .
By subtracting (13.29) from (13.30), we have C = 26◦ 56′ 15′′ .
c sin A
To find a, we have a=
sin C
13.3. Two Sides and Included Angle 212

∴ log a = log c + L sin A − L sin C


= log 63 + L sin 42◦ 15′ 30′′ − L sin 26◦ 56′ 15′′ .
30
Now L sin 42◦ 15′ 30′′ = 9.8276063 + × .0001390
60
1
= 9.8276063 + × .0001390
2
= 9.8276758
15
and L sin 26◦ 56′ 15′′ = 9.6560536 + × .0002485
60
1
= 9.6560536 + × .0002485
4
= 9.6561157
∴ log a = 1.7993405 + 9.8276758 − 9.6561157 = 1.9709006.
Now log 93.519 = 1.9708999, diff. for .001 = .0000046.
Let log (93.519 + x) = 1.9709006.
The diff. for x = 1.9709006 − 1.9708999 = .0000007.
7 .007
∴x= × .001 = ≈ .0002, ∴ a = 93.5192. ■
46 46
§ Problem 13.3.20. Two sides of a triangle being 2265.4 and 1779
feet and the included angle 58◦ 17′ , find the remaining angles. ♢
§§ Solution. If b = 2265.4 f eet, c = 1779 f eet and A = 58◦ 17′ , we
have
B−C b−c A 486.4
tan = cot = cot 29◦ 8′ 30′′
2 b+c 2 4044.4
B−C
∴ L tan = log 486.4 − log 4044.4 + L cot 29◦ 8′ 30′′
2
30
Now L cot 29◦ 8′ 30′′ = 10.2538680 − × .0002970
60
1
= 10.2538680 − × .0002970 = 10.2537195
2
B−C
∴ L tan = 2.6869936 − 3.6068541 + 10.2537195 = 9.3338590.
2
Now L tan 12◦ 10′ = 9.3336463, diff. for 1′ = 6128.
B−C ( )
Let = 12◦ 10′ + x′′ , ∴ L tan 12◦ 10′ + x′′ = 9.3338590.
2
The diff. for x′′ = 9.3338590 − 9.3336463 = .0002127.
2127
∴ x = 60′′ × ≈ 21′′
6128
B−C
∴ = 12◦ 10′ 21′′ (13.31)
2
B+C ◦ A ◦ ◦ ′ ′′ ◦ ′ ′′
But = 90 − = 90 − 29 8 30 = 60 51 30 (13.32)
2 2
By adding (13.31) and (13.32), we have B = 73 1 51 . ◦ ′ ′′

By subtracting (13.31) from (13.31), we have C = 48◦ 41′ 9′′ . ■

§ Problem 13.3.21. Two sides of a triangle being 237.09 and 130.96


feet and the included angle 57◦ 59′ , find the remaining angles. ♢
§§ Solution. If b = 237.09,c = 130.96 and A = 57◦ 59′ , we have
B−C b−c A 106.13
tan = cot = cot 28◦ 59′ 30′′
2 b+c 2 368.05
13.4. Ambiguous Case 213

B−C
∴ L tan = log 106.13 − log 368.05 + L cot 28◦ 59′ 30′′ .
2
30
Now L cot 28◦ 59′ 30′′ = 10.2565460 − × .0002980
60
1
= 10.2565460 − × .0002980 = 10.2563970.
2
B−C
∴ tan = 2.0258382 − 2.5659068 + 10.2563970 = 9.7163284.
2
Now L tan 27◦ 29′ = 9.7161682, diff. for 1′ = 3085.
B−C ( )
Let = 27◦ 29′ + x′′ , ∴ L tan 27◦ 29′ + x′′ = 9.7163284.
2
The diff. for x′′ = 9.7163284 − 9.7161682 = .0001602.
1602
∴ x = 60′′ × ≈ 31′′
3085
B−C
∴ = 27◦ 29′ 31′′ (13.33)
2
B+C A
But = 90◦ − = 90◦ − 28◦ 59′ 30′′ = 61◦ 0′ 30′′ (13.34)
2 2
By adding (13.33) and (13.34), we have B = 88◦ 30′ 1′′ .
By subtracting (13.33) from (13.34), we have C = 33◦ 30′ 59′′ . ■

13.4 Ambiguous Case


3
§ Problem 13.4.1. If a = 5, b = 7 and sin A = , is there any ambi-
4
guity ? ♢
§§ Solution. We have
3 21 1
b sin A = 7 × = =5
4 4 4
∴ a < b sin A.
Hence there is no triangle. ■


§ Problem 13.4.2. If a = 2, c = 3 + 1 and A = 45◦ , solve the
triangle. ♢
§§ Solution. √
3+1 1 (√ √ )
∵ c sin A = √ = 6+ 2
2 2
1
= (2.4495 + 1.4142) = 1.9318,
2
we have a > c sin A; also a < c and A is acute.
Hence there are two triangles.
We have √ √
c 3+1 1 3+1
sin C = sin A = ·√ = √
a 2 2 2 2
◦ ◦
∴ C1 = 75 and C2 = 105
∴ B1 = 60◦ and B2 = 30◦
Å √ ã
a sin B1 3 1 √
∴ b1 = = 2× ÷√ = 6
sin A 2 2
( ) √
a sin B2 1 1
and b2 = = 2× ÷ √ = 2. ■
sin A 2 2
13.4. Ambiguous Case 214


§ Problem 13.4.3. If a = 100, c = 100 3 and A = 30◦ , solve the
triangle. ♢
§§ Solution.
√ 1 √
∵ c sin A = 100 3 × = 50 3
2
we have a > c sin A; also a < c and A is acute.
Hence there are two triangles.
We have √ √
c 100 3 1 3
sin C = sin A = · =
a 100 2 2
∴ C1 = 60◦ and C2 = 120◦
∴ B1 = 90◦ and B2 = 30◦
a sin B1 1
∴ b1 = = 100 ÷ = 200
sin A ( 2 )
a sin B2 1 1
and b2 = = 100 × ÷ = 100. ■
sin A 2 2
3
§ Problem 13.4.4. If 2b = 3a and tan2 A = , prove that there are
5
two values to the third side, one of which is double the other. ♢
√ √
3 3
§§ Solution. Since tan A = , ∴ sin A = √ (the positive sign
5 2 2 √
◦ 5
being taken, as A must be < 180 in a triangle) and cos A = √ .
√ … 2 2
3a 3 27
∴ b sin A = · √ = ·a
2 2 2 32
( )
2
∴ a > b sin A and < b ∵ a = b .
3
Also A is acute; for since b > a, ∴ B > A, and if A were obtuse, there
would be two obtuse angles in the triangle, which is impossible. The
two values of c are given by
c2 = a2 − b2 + 2bc cos A
( )
∴ c2 − 2bc cos A + b2 − a2 = 0.

5 9a2
∴ c2 − 3ac · √ + − a2 = 0
2√ 2 4
∴ 4c2 − 3a 10 · c + 5a2 = 0
√ √
3 10 ± 90 − 80 3±1 √
∴c= ·a= · a 10
8 √ √ 8
a 10 a 10
∴c= and .
2 4
Otherwise thus :
Take the figure 3 of Art. 186, putting C for A, A for B and B1 and
B2 for C1 and C2 respectively. We then have
√ √
3a 5 3 5
AD = b cos A = · √ = √ ·a
2 2 2 4√ 2

3a 3 3 3
and CD = b sin A = · √ = √ · a.
2 2 2 4 2
13.4. Ambiguous Case 215

…… √
√ 27 2 5a2 a 5
∴ B1 D = B1 − C2= −
CD 2 a = a2 = √
32 32 4 2
Å √ √ ã √
3 5 5 a 10
∴ AB1 = AD + DB1 = √ + √ a=
4 2 4 2 2
Å √ √ ã √
3 5 5 a 10
and AB2 = AD − DB1 = √ − √ a= .
4 2 4 2 4 ■

§ Problem 13.4.5. If A = 30◦ , b = 8 and a = 6, find c. ♢


1
§§ Solution. Since b sin A = 8 × = 4, we have a > b sin A; also a < b
2
and A is acute; hence there are two triangles. The two values of c
are given by
c2 − 2bc cos A + b2 − a2 = 0

3
∴ c2 − 2 · 8 · c · + 64 − 36 = 0
√ 2
∴ c − 8 3 · c + 28 = 0
2
√ √
8 3 ± 64 × 3 − 4 × 28
∴c=
√ √ 2 √ √ √ √
= 4 3 ± 48 − 28 = 4 3 ± 20 = 4 3 ± 2 5. ■


§ Problem 13.4.6. Given B = 30◦ , c = 150 and b = 50 3, prove that
of the two triangles which satisfy the data, one will be isosceles and
the other right-angled. Find the greater value of the third side.
Would the solution have been ambiguous had
B = 30◦ , c = 150 and b = 75? ♢
§§ Solution. We have
1 √
c 150 × 3
sin C = sin B = √ 2 =
b 50 3 2
∴ C1 = 60◦ and C2 = 120◦
∴ A1 = 90◦ and A2 = 30◦ .
Hence one triangle is isosceles and the other is right-angled. The
greater value of a is when A = 90◦ and we have
√ √ √ √
a= b2 + c2 = (50)2 (3 + 9) = 50 12 = 100 3.
[ √ ]
2
Or thus : a = c sec B = 150 × √ = 100 3.
3
1
If B = 30◦ , c = 150 and b = 75, we have c sin B = 150 × = 75 = b.
2
Hence there is one triangle right-angled. ■

§ Problem 13.4.7. In the ambiguous case given√a, b and A, prove


that the difference between the two values of c is 2 a2 − b2 sin2 A. ♢
§§ Solution. The two values of c (c1 and c2 ) are given by
c2 − 2bc cos A + b2 − a2 = 0
and we have c1 + c2 = 2b cos A and c1 c2 = b2 − a2 .
( )
∴ (c1 ∼ c2 ) = (c1 + c2 )2 − 4c1 c2 = 4b2 cos2 A − 4 b2 − a2
2
( ) ( ) ( 2 )
= 4b2
1 − sin2 A − 4 b2 − a 2
= 4 a − b2 sin2 A
13.4. Ambiguous Case 216


c1 ∼ c2 = 2 a2 − b2 sin2 A.

Otherwise thus : Take f ig. 3 of Art. 186, with C for A, A for B and
B2 and B1 for C2 and C1 respectively. We then have
√ √
c1 ∼ c2 = B1 B2 = 2B2 D = 2 BC 2 − CD 2 = 2 a2 − (b sin A)2 . ■

§ Problem 13.4.8. If a = 5, b = 4 and A = 45◦ , find the other angles,


having given
log 2 = .30103, L sin 33◦ 29′ = 9.7520507
and L sin 33◦ 30′ = 9.7530993. ♢
§§ Solution. Since a > b, there is only one triangle.
We have
√ 5
b 4 1 4 2 22
sin B = sin A = × √ = =
a 5 2 10 10
5
∴ L sin B = 10 + log 2 − log 10 = 10 + .7525750 − 1 = 9.7525750.
2 ( )
Let B = 33◦ 29′ + x′′ , ∴ L sin 33◦ 29′ + x′′ = 9.7525750.
The diff. for 60′′ = 9.7530993 − 9.7520507 = .0010486.
The diff. for x′′ = 9.7525750 − 9.7520507 = .0005243.
5243 1
∴ x = 60′′ × = 60′′ × = 30′′
10486 2
∴ B = 33◦ 29′ 30′′
and C = 180◦ − (A + B) = 180◦ − 78◦ 29′ 30′′ = 101◦ 30′ 30′′ . ■

§ Problem 13.4.9. If a = 9, b = 12 and A = 30◦ , find c, having given


log 2 = .30103, log 3 = .47712
log 171 = 2.23301, log 368 = 2.56635
L sin 11◦ 48′ 39′′ = 9.31108, L sin 41◦ 48′ 39′′ = 9.82391
and L sin 108◦ 11′ 21′′ = 9.97774. ♢
1
§§ Solution. Since b sin A = 12 × = 6, we have a > b sin A; also
2
a < b and A is acute. Hence there are two triangles.
We have
b 6 2
sin B = sin A = =
a 9 3
∴ L sin B = 10 + log 2 − log 3 = 9.82391
∴ B1 = 41◦ 48′ 39′′
and B2 = 180 − 41◦ 48′ 39′′ = 138◦ 11′ 21′′

∴ C1 = 180◦ − (A + B1 ) = 180◦ − 71◦ 48′ 39′′ = 108◦ 11′ 21′′


and C2 = 180◦ − (A + B2 ) = 180◦ − 168◦ 11′ 21′′ = 11◦ 48′ 39′′ .
a sin C1 9 sin C1
∴ c1 = = = 2 × 32 × sin 108◦ 11′ 21′′
sin A 1
2
∴ log c1 = log 2 + 2 log 3 + L sin 108◦ 11′ 21′′ − 10
= .30103 + .95424 + 9.7774 − 10 = 1.23301
∴ c1 = 17.1.
a sin C2 9 sin C2
Again, c2 = = ( ) = 2 × 32 × sin 11◦ 48′ 39′′
sin A 1
2
13.4. Ambiguous Case 217

∴ log c2 = log 2 + 2 log 3 + L sin 11◦ 48′ 39′′ − 10


= .30103 + .95424 + 9.31108 − 10 = .56635
∴ c2 = 3.68. ■

§ Problem 13.4.10. Point out whether or no the solutions of the


following triangles are ambiguous.
Find the smaller value of the third side in the ambiguous case and
the other angles in both cases :
(1) A = 30◦ , c = 250 f eet and a = 125 f eet
(2) A = 30◦ , c = 250 f eet and a = 200 f eet.

Given log 2 = .30103, log 6.03893 = .7809601


L sin 38◦ 41′ = 9.7958800
and L sin 8◦ 41′ = 9.1789001. ♢
c 250 1
§§ Solution. (1) We have sin C = sin A = · = 1; ∴ C = 90◦ .
a 125 2
Hence the triangle is not ambiguous.
c 250 1 5 10 10
(2) We have sin C = sin A = · = = = 4.
a 200 2 8 16 2
Since a > c sin A, a < c, and A is acute, the triangle is ambigu-
ous.
We have L sin C = 10 + log 10 − 4 log 2
= 10 + 1 − 1.2041200 = 9.7958800
∴ C1 = 38◦ 41′
and C2 = 180◦ − 38◦ 41′ = 141◦ 19′
∴ B1 = 180◦ − (A + C1 ) = 180◦ − 68◦ 41′ = 111◦ 19′
and B2 = 180◦ − (A + C2 ) = 180◦ − 171◦ 19′ = 8◦ 41′ .
For the smaller value of b, we have
a sin 8◦ 41′ 200 sin 8◦ 41′
b= = ( ) = 22 × 100 × sin 8◦ 41′
sin A 1
2
∴ log b = 2 log 2 + log 100 + L sin 8◦ 41′ − 10
= .60206 + 2 + 9.1789001 − 10 = 1.7809601
∴ b = 60.3893. ■

§ Problem 13.4.11. Given a = 250, b = 240 and A = 72◦ 4′ 48′′ , find


the angles B and C and state whether they can have more than one
value, given
log 2.5 = .3979400, log 2.4 = .3802112
L sin 72◦ 4′ = 9.9783702, L sin 72◦ 5′ = 9.9784111
and L sin 65◦ 59′ = 9.9606739. ♢
§§ Solution. Since a > b, there is only one triangle.
b 24
We have sin B = sin A = sin 72◦ 4′ 48′′
a 25
∴ L sin B = log 24 + L sin 72◦ 4′ 48′′ − log 25.
To find L sin 72◦ 4′ 48′′ , we have
diff. for 60′′ = 9.9784111 − 9.9783702 = .0000409
48 8
∴ diff. for 48′ = × .0000409 = × .0000409 = .0000327
60 10
13.4. Ambiguous Case 218

∴ L sin 72◦ 4′ 48′′ = 9.9783702 + .0000327 = 9.9784029


∴ L sin B = 1.3802112 + 9.9784029 − 1.3979400 = 9.9606741
∴ B ≈ 65◦ 59′
∴ C = 180◦ − (A + B) = 180◦ − 138◦ 3′ 48′′ = 41◦ 56′ 12′′ . ■

§ Problem 13.4.12. Two straight roads intersect at an angle of 30◦ ;


from the point of junction two pedestrians A and B start at the same
time, A walking along one road at the rate of 5 miles per hour and B
walking uniformly along the other road. At the end of 3 hours, they
are 9 miles apart. Show that there are two rates at which B may
walk to fulfill this condition and find them. ♢
§§ Solution. In the triangle ABC, let B represent the point of in-
tersection of the roads. At the end of 3 hours, let the pedestrians A
and B be at A and C respectively. We then have the ∠ABC = 30◦ ,
BA(c) = 15 miles and AC(b) = 9 miles. The distance walked by B in
3 hours will be given by the value of a.
1
Since c sin B = 15 sin 30◦ = 7 , we have b > c sin B; also b < c and
2
the ∠B is acute. Hence there are two values of a, i.e. two rates at
which B may walk. These values of a are given by
a2 − 2ca cos B + c2 − b2 = 0.

3
We have a2 − 2 × 15 × a × + 225 − 81 = 0
√ 2
∴ a − 15 3 · a + 144 = 0
2
√ √ √ √
15 3 ± 675 − 576 15 3 ± 99
∴a= =
√ 2
√ 2
15 3 ± 3 11
= .
2
Hence the require rates
√ √
a 5 3 ± 11 5 × 1.73205 ± 3.31661
= = =
3 2 2
8.66025 ± 3.31662
= = 5.9884 . . . or 2.6718 . . . miles per hour
2
In f ig. 3 of Art. 186, we see that when the pedestrian A is at A, the
pedestrian B may be either at C2 or C1 to be 9 miles (b) apart. ■

For the following three examples, a book of tables will be re-


quired.
§ Problem 13.4.13. Two sides of a triangle are 1015 feet and 732 feet
and the angle opposite the latter side is 40◦ ; find the angle opposite
the former and prove that more than one value is admissible. ♢
§§ Solution. Let b = 732 f eet, c = 1015 f eet and B = 40◦ . Since, by
the table of natural sines,
c sin B = 1015 × .6427876 ≈ 625,
we have b > c sin B; also b < c and B is acute.
Hence there are two triangles.
c 1015
We have sin C = sin B = sin 40◦ .
b 732
∴ L sin C = log 1015 + L sin 40◦ − log 732
= 3.0064660 + 9.8080675 − 2.8645111 = 9.9500224.
13.4. Ambiguous Case 219

Now L sin 63◦ 2′ = 9.9500095, diff. for 1′ = 643.


( )
Let C = 63◦ 2′ + x′′ , ∴ L sin 63◦ 2′ + x′′ = 9.9500224.
The diff. for x′′ = 9.9500224 − 9.9500095 = .0000129.
129
∴ x = 60′′ × ≈ 12′′ .
643
∴ L sin C = L sin 63◦ 2′ 12′′ .
∴ C = 63 2 12′′ or 180◦ − 63◦ 2′ 12′′ = 116◦ 57′ 48′′ .
◦ ′

§ Problem 13.4.14. Two sides of a triangle being 5374.5 and 1586.6


feet and the angle opposite the latter being 15◦ 11′ , calculate the
other angles of the triangle or triangles. ♢
§§ Solution. Let b = 1586.6, c = 5374.5 and B = 15◦ 11′ . Since, by
the table of natural sines,
c sin B = 5374.5 × .2619085 ≈ 1408,
we have b > c sin B; also b < c and B is acute.
Hence there are two triangles.
c 5374.5
We have sin C = sin B = sin 15◦ 11′ .
b 1586.6
∴ L sin C = log 5374.5 + L sin 15◦ 11′ − log 1586.6
= 3.7303381 + 9.4181495 − 3.2004674 = 9.9480202.
Now L sin 62◦ 31′ = 9.9479947, diff. for 1′ = 657.
( )
Let C = 62◦ 31′ + x′′ , ∴ L sin 62◦ 31′ + x′′ = 9.9480202.
The diff. for x′′ = 9.9480202 − 9.9479947 = .0000255.
255
Hence x = 60′′ × ≈ 23.
657
∴ L sin C = L sin 62◦ 31′ 23′′ .
∴ C = 62◦ 31′ 23′′ or 180◦ − 62◦ 31′ 23′′ = 117◦ 28′ 37′′
∴ C1 = 62◦ 31′ 23′′ and C2 = 117◦ 28′ 37′′ .
( )
∴ A1 = 180◦ − 15◦ 11′ + 62◦ 31′ 23′′ = 102◦ 17′ 37′′
( )
and A2 = 180◦ − 15◦ 11′ + 117◦ 28′ 37 ′′
= 47◦ 20′ 23′′ . ■

§ Problem 13.4.15. Given A = 10◦ , a = 2308.7 and b = 7903.2, find


the smaller value of c. ♢
b 7903.2 ◦
§§ Solution. We have sin B = sin A = sin 10 .
a 2308.7
∴ L sin B = log 7903.2 + L sin 10◦ − log 2308.7
= 3.8978030 + 9.2396702 − 3.3633675 = 9.7741057.
Now L sin 36◦ 28′′ = 9.7740459, diff. for 1′ = 1709.
( )
Let B = 36◦ 28′ + x′′ , ∴ L sin 36◦ 28′ + x′′ = 9.7741057.
The difference for
x′′ = 9.7741057 − 9.7740459 = .0000598.
598
∴ x = 60′′ × ≈ 21′′ .
1709
∴ L sin B = L sin 36◦ 28′ 21′′ .
∴ B = 36◦ 28′ 21′′ or 180◦ − 36◦ 28′ 21′′ = 143◦ 31′ 39′′ ,
and, by the question, we must take B = 143◦ 31′ 39′′ .
( )
∴ C = 180◦ − 10◦ + 143◦ 31′ 39′′ = 26◦ 28′ 21′′ .
13.5. One Side Two Angles vs. Three Angles 220

To find c, we have
a sin C 2308.7 sin 26◦ 28′ 21′′
c= = .
sin A sin 10◦
∴ log c = log 2308.7 + L sin 26 28 21 − L sin 10◦ .
◦ ′ ′′

Now L sin 26◦ 28′ = 9.6490203, diff. for 1′ = 2537.


21
∴ dif f. f or 21′′ = × 2537 ≈ 888
60
◦ ′ ′′
∴ L sin 26 28 21 = 9.6490203 + .0000888 = 9.6491091
∴ log c = 3.3633675 + 9.6491091 − 9.2396702 = 3.7728064.
Now log 5926.6 = 3.7728056, diff. for .1 = 73.
Let log (5926.6 + x) = 3.7728064.
The diff. for x = 3.7728064 − 3.7728056 = .0000008.
8 .8
∴x= × .1 = ≈ .011
73 73
∴ log c = log 5926.611.
Hence the smaller value of c = 5926.611. ■

13.5 One Side Two Angles vs. Three Angles


17 1
§ Problem 13.5.1. If cos A = and cos C = , find the ratio of
22 14
a : b : c. ♢
§§ Solution. … ( )2 √
17 195
We have sin A = 1− =
22 22
… ( ) √
1 2 195
and sin C = 1− =
14 14
∴ sin B = sin (A + C) = sin A cos C + cos A sin C
√ √
195 1 17 195
= · + ·
22
√ 14 22 14
9 195
= .
11 × 14
1 9 1
∴ the required ratio = : : = 7 : 9 : 11. ■
22 11 × 14 14

§ Problem 13.5.2. The angles of a triangle are as √1 : 2 : 7;√prove that


the ratio of the greatest side to the least side is 5 + 1 : 5 − 1. ♢
§§ Solution. If x◦ , 2x◦ and 7x◦ be the angles, we have
x◦ + 2x◦ + 7x◦ = 10x◦ = 180◦
∴ x◦ = 18◦ and 7x◦ = 126◦ .
Hence the greatest side : the least side
= sin 126◦ : sin 18◦
= sin 54◦ : sin 18◦
√ √
5+1 5−1
= :
√ 4 √ 4
= 5 + 1 : 5 − 1. ■
13.5. One Side Two Angles vs. Three Angles 221

§ Problem
√ 13.5.3. If A = 45◦ , B = 75◦ and C = 60◦ , prove that
a + c 2 = 2b. ♢
§§ Solution. We have
a b

=
Å√
sin 45 sinã75◦
b 3+1 2b
∴a= √ ÷ √ = √ .
2 2 2 3+1
c b
Also, ◦
=
√ sin 60Å√ sin 75◦
ã √
b 3 3+1 b 6
∴c= ÷ √ = √ .
2 2 2 3+1
√ ( √ )
√ 2b + b 12 2b 1 + 3
∴a+c 2= √ = √ = 2b. ■
3+1 1+ 3
§ Problem 13.5.4. Two angles of a triangle are 41◦ 13′ 22′′ and 71◦ 19′ 5′′
and the side opposite the first angle is 55; find the side opposite the
latter angle, given
log 55 = 1.7403627, log 79063 = 4.8979775
L sin 41◦ 13′ 22′′ = 9.8188779
and L sin 71◦ 19′ 5′′ = 9.9764927. ♢
§§ Solution. Let A = 41◦ 13′ 22′′ , B = 71◦ 19′ 5′′ and a = 55.
a sin B
We have b=
sin A
∴ log b = log a + L sin B − L sin A
= 1.7403627 + 9.9764927 − 9.8188779
= 1.8979775 = log 79.063.
∴ b = 79.063. ■

§ Problem 13.5.5. From each of two ships, one mile apart, the an-
gle is observed which is subtended by the other ship and a beacon
on shore; these angles are found to be 52◦ 25′ 15′′ and 75◦ 9′ 30′′ respec-
tively. Given
L sin 75◦ 9′ 30′′ = 9.9852635
L sin 52 25 15′′ = 9.8990055, log 1.2197 = .0862530
◦ ′

and log 1.2198 = .0862886,


find the distance of the beacon from each of the ships. ♢
§§ Solution. The third angle of the triangle formed by the ships and
the beacon ( )
= 180◦ − 52◦ 25′ 15′′ + 75◦ 9′ 30′′ = 52◦ 25′ 15′′ .
Thus the triangle is isosceles, so that if a and b be the required dis-
tances respectively, we have b = 1 mile.
1 × sin 75◦ 9′ 30′′
Also, a= .
sin 52◦ 25′ 15′′
∴ log a = log 1 + L sin 75 9 30 − L sin 52◦ 25′ 15′′
◦ ′ ′′

= 0 + 9.9852635 − 9.8990055 = .0862580.


We have log 1.2197 = .0862530 (13.35)
and log 1.2198 = .0862886 (13.36)
Let log (1.2197 + x) = .0862580 (13.37)
From (13.35) and (13.36), the diff. for .0001 = .0000356.
13.5. One Side Two Angles vs. Three Angles 222

From (13.35) and (13.37), the diff. for x = .0000050.


50 .005
∴x= × .0001 = ≈ .000014.
356 356
Hence the required distance = 1.219714 mile. ■

1◦ 1◦
§ Problem 13.5.6. The base angles of a triangle are 22 and 112 ;
2 2
prove that the base is equal to twice the height. ♢
1◦ 1◦
§§ Solution. In the triangle ABC, if A = 22 and B = 112 , then
2 2
C = 45◦ .
Let h denote the height of the triangle. We then have
1◦ 1◦
c sin B sin A c sin 112 sin 22
h = b sin A = = 2 2
sin C sin 45◦
1 ◦ 1◦ 1◦ 1◦
c sin 67 sin 22 c cos 22 sin 22
= 2 2 = 2 2 = c.
sin 45◦ 1◦ 1◦ 2
2 cos 22 sin 22 ■
2 2

For the following five problems, a book of tables is required.


§ Problem 13.5.7. The base of a triangle being seven feet and the
base angles 129◦ 23′ and 38◦ 36′ , find the length of its shorter side. ♢
§§ Solution. The third angle
( of the triangle
)
= 180◦ − 129◦ 33′ + 38◦ 36′ = 12◦ 1′ .
To find the side opposite the angle 38◦ 36′ (a f eet, say), we have
7 sin 38◦ 36′
a=
sin 12◦ 1′
∴ log a = log 7 + L sin 38◦ 36′ − L sin 12◦ 1′
= .8450980 + 9.7951008 − 9.3184728 = 1.3217260.
Now log 20.976 = 1.3217227, diff. for .001 = .0000207.
Let log (20.976 + x) = 1.3217260.
The diff. for x = 1.3217260 − 1.3217227 = .0000033.
33 .033 .011
∴x= × .001 = = ≈ .00016
207 207 69
∴ a = 20.97616 f eet. ■

§ Problem 13.5.8. If the angles of triangle be as 5 : 10 : 21 and the


side opposite the smaller angle be 3 feet, find the other sides. ♢
§§ Solution. If 5x◦ , 10x◦ and 21x◦ be the angles, we have
5x◦ + 10x◦ + 21x◦ = 36x◦ = 180◦ ,
so that x = 5 and the angles are 25◦ , 50◦ and 105◦ .
◦ ◦

sin 25◦ sin 50◦ sin 105◦


We have = = ,
3 b c
where b feet and c feet are the sides opposite the angles 50◦ and 105◦
respectively.
3 sin 50◦ 6 sin 25◦ cos 25◦
∴b= ◦
= = 6 cos 25◦
sin 25 sin 25◦
= 6 × .9063078 = 5.4378468 f eet,
13.5. One Side Two Angles vs. Three Angles 223

by the table of natural cosines.


3 sin 105◦ 3 sin 75◦
Also, c= =
sin 25◦ sin 25◦
∴ log c = log 3 + L sin 75◦ − L sin 25◦
= .4771213 + 9.9849438 − 9.6259483 = .8361168.
Now log 6.8567 = .8361151, diff. for .0001 = .0000064.
Let log (6.8567 + x) = .8361168.
The diff. for x = .8361168 − .8361151 = .0000017.
17 .0017
∴x= × .0001 = ≈ .00003
64 64
∴ c = 6.85673 f eet. ■

§ Problem 13.5.9. The angles of a triangle being 150◦ ,18◦ 20′ and
11◦ 40′ and the longest side being 1000 feet, find the length of the
shortest side. ♢
§§ Solution. Let A = 150◦ , B = 18◦ 20′ , C = 11◦ 40′ and a = 1000 f eet.
To find c, we have
a sin C 1000 sin 11◦ 40′ 1000 sin 11◦ 40′
c= = =
sin A sin 150◦ sin 30◦
1
= [1000 × .2022176] ÷ = 404.4352 f eet. ■
2
§ Problem 13.5.10. To get the distance of a point A from a point
B, a line BC and the angles ∠ABC and ∠BCA are measured and are
found to be 287 yards and 55◦ 32′ 10′′ and 51◦ 8′ 20′′ respectively. Find
the distance AB. ♢
§§ Solution. We have
A = 180◦ − (B + C) = 180◦ − 106◦ 40′ 30′′ = 73◦ 19′ 30′′ .
BC sin C
Also, AB = .
sin A
∴ log AB = log BC + L sin C − L sin A
= log 287 + L sin 51◦ 8′ 20′′ − L sin 73◦ 19′ 30′′ .
20
Now L sin 51◦ 8′ 20′′ = 9.8913191 + × .0001017 = 9.8913530
60
30
and L sin 73◦ 19′ 30′′ = 9.9813229 + × .0000379 = 9.9813419
60
∴ log AB = 2.4578819 + 9.8913530 − 9.9813419 = 2.3678930.
Now log 233.28 = 2.3678775, diff. for .01 = .0000186.
Let log (233.28 + x) = 2.3678930.
The diff. for x = 2.3678930 − 2.3678775 = .0000155.
155 1.55
∴x= × .01 = ≈ .0083
186 186
∴ AB = 233.2883 yards. ■

§ Problem 13.5.11. To find the distance from A to P a distance, AB


of 1000 yards is measured in a convenient direction. At A the angle
∠P AB is found to be 41◦ 18′ and at B the angle ∠P BA is found to be
114◦ 38′ . What is the required distance to the nearest yard ? ♢
13.5. One Side Two Angles vs. Three Angles 224

§§ Solution. The angle ∠AP B = 180◦ − (41◦ 18′ + 114◦ 38′ ) = 24◦ 4′ .
AB sin 114◦ 38′ 1000 sin 65◦ 22′
We have AP = ◦ ′
=
sin 24 4 sin 24◦ 4′
∴ log AP = log 1000 + L sin 65◦ 22′ − L sin 24◦ 4′
= 3 + 9.9585609 − 9.6104465 = 3.3481144.
Hence AP = 2229 yards, to the nearest yard. ■
Chapter 14
Heights and Distances

14.1 Inaccessible Object and Distant Points


§ Problem 14.1.1. A flagstaff stands on the middle of a square
tower. A man on the ground, opposite the middle of one face and
distant from it 100 feet, just sees the flag ; on his receding another
100 feet, the tangents of elevation of the top of the tower and the top
1 5
of the flagstaff are found to be and . Find the dimensions of the
2 9
tower and the height of the flagstaff, the ground being horizontal. ♢
§§ Solution. Let CHKL represent the vertical section of the tower
through its center, HK being the base. Let A and B be the two
positions of the man respectively, opposite H. Let DF (k feet, say)
be the flagstaff, F being the middle point of CL.
Produce DF to meet HK in E. Let h feet be the height, and b
feet be the breadth of the tower, so that CH = F E = LK = h, and
b
HE = EK = .
2
1
We are given BA = AH = 100 feet, tan ∠CBE = , and tan ∠DBE =
2
5
. Also ACD is a straight line.
9
In the ∆CBH, we have CH = BH tan ∠CBE,
1
i.e. h = 200 × = 100′ ;
2
∴ ∠CAH = 45◦ = ∠DCF ;
b
∴k= .
2
In the ∆DBE, we have
DE = BE tan ∠DBE,
14.1. Inaccessible Object and Distant Points 226

( )
b 5
i.e. h+k = 200 + ;
2 9
∴ 9(100 + k) = 5(200 + k);
∴ 4k = 100, ∴ k = 25′ ;
b
and = k = 25, ∴ b = 50′ . ■
2
§ Problem 14.1.2. A man, walking on a level plane towards a tower,
observes that at a certain point the angular height of the tower is 10◦ ,
and, after going 50 yards nearer the tower, the elevation is found to
be 15◦ . Having given
L sin 15◦ = 9.4129962, L cos 5◦ = 9.9983442,
log 25.783 = 1.4113334, and log 25.784 = 1.4113503,
find, to 4 places of decimals, the height of the tower in yards. ♢
§§ Solution. Take the figure of Art. 192. Let P Q (x yards, say) rep-
resent the tower and A and B be the points at which the angles of
elevation are taken.
We are given AB = 50 yards, ∠P AQ = 10◦ and ∠P BQ = 15◦ , so
that ∠AP B = 15◦ − 10◦ = 5◦ .
We then have
x BP sin 10◦ 2 sin 5◦ cos 5◦
= sin 15◦ , and = = = 2 cos 5◦ .
BP 50 sin 5◦ sin 5◦
Hence by multiplication, we have
x
= 2 sin 15◦ cos 5◦
50
∴ x = 100 sin 15◦ cos 5◦
∴ log x = log 100 + L sin 15◦ − 10 + L cos 5◦ − 10
= 2 + 9.4129962 + 9.9983442 − 20 = 1.4113404.

We have log 25.783 = 1.4113334 (14.1)


log 25.784 = 1.4113503 (14.2)
Let log(25.783 + y) = 1.4113404 (14.3)
From (14.1) and (14.2), we have the difference for .001 = .0000169.
From (14.1) and (14.3), we have the difference for y = .0000070.

70 .07
∴y= × .001 = ≈ .0004
169 169
∴ x = 25.7834 yards. ■

§ Problem 14.1.3. DE is a tower standing on a horizontal plane


and ABCD is a straight line in the plane. The height of the tower
subtends an angle θ at A, 2θ at B, and 3θ at C. If AB and BC be
respectively 50 and 20 feet, find the height of the tower and the dis-
tance CD. ♢
§§ Solution. Let the height of the tower be h feet and the distance
CD be x feet.
70 + x 20 + x x
We then have cot θ = , cot 2θ = and cot 3θ = .
h h h
cot2 θ − 1
Now cot 2θ = , so that we have
ï( 2 cot θ )2 ò [ ]
20 + x 70 + x 2(70 + x) (70 + x)2 − h2
= −1 ÷ =
h h h 2h(70 + x)
14.1. Inaccessible Object and Distant Points 227

∴ 2(20 + x)(70 + x) = (70 + x)2 − h2 (14.4)


cot θ cot 2θ − 1
Again, cot 3θ = cot(2θ + θ) = .
[ ] cot[θ + cot 2θ ]
x (70 + x)(20 + x) 70 + x 20 + x
∴ = 2
−1 ÷ +
h h h h
(70 + x)(20 + x) − h2
=
h(90 + 2x)
∴ x(90 + 2x) = (70 + x)(20 + x) − h2 (14.5)
Subtracting (14.5) from (14.4), we have
3(20 + x)(70 + x) − x(90 + 2x) = (70 + x)2
70 1
∴ 40x = 700, and x = = 17 f eet.
4 2
Also, from (14.4), we have
h2 = (70 + x) [(70 + x) − (40 + 2x)] = (70 + x)(30 − x)
( )( )
70 70 5 50 7
= 70 + 30 − = 70 × × = 25 × 25 × .
4 4 4 4 4

25 7
∴h= = 33.07189 f eet. ■
2
§ Problem 14.1.4. A tower, 50 feet high, stands on the top of a
mound ; from a point on the ground the angles of elevation of the
top and bottom of the lower are found to be 75◦ and 45◦ respectively
; find the height of tho mound. ♢
§§ Solution. If h feet be the height of the mound, and x feet be the
distance of the point from the base of the mound, we have
h
= tan 45◦ = 1, ∴ h = x.
x
h + 50 √
Also, = tan 75◦ = 2 + 3
x √
∴ h + 50 = h(2 + 3)

50 50( 3 − 1) 100
∴h= √ = = × .732 . . .
3+1 2 4
∴ h = 18.3 f eet . ■

§ Problem 14.1.5. A vertical pole (more than 100 feet high) consists
1
of two parts, the lower being rd of the whole. From a point in a
3
horizontal plane through the foot of the pole and 40 feet from it, the
1
upper part subtends an angle whose tangent is . Find the height
2
of the pole. ♢
§§ Solution. Let the required height be 3h feet, so that the lower
part of the pole is h feet and the upper part is 2h feet. Then, if α and
β be the angles subtended at the point in the horizontal plane by the
pole and the lower part of it respectively, we have
3h h
tan α = and tan β = .
40 40
tan α − tan β 1
Also, tan(α − β) = = .
1 + tan α tan β 2
14.1. Inaccessible Object and Distant Points 228

( ) ï ò
3h − h 3h2 1
∴ ÷ 1+ =
40 (40)2 2
∴ 3h2 − 160h + 1600 = 0
∴ (3h − 40)(h − 40) = 0.
Since the pole is more than 100 feet high, we must take the solution
h − 40 = 0, ∴ h = 40.
Hence the required height = 3h = 120 f eet. ■

§ Problem 14.1.6. A tower subtends an angle α at a point on the


same level as the foot of the tower, and at a second point, h feet
above the first, the depression of the foot of the tower is β. Find the
height of the tower. ♢
§§ Solution. If x be the required height and y be the distance of the
point from the foot of the tower, we have
y = x cot α and y = h cot β.
Hence x cot α = h cot β
∴ x = h tan α cot β. ■

§ Problem 14.1.7. A person in a balloon, which has ascended verti-


cally from flat land at the sea level, observes the angle of depression
of a ship at anchor to be 30◦ ; after descending vertically for 600
feet, he finds the angle of depression to be 15◦ ; find the horizontal
distance of the ship from the point of ascent. ♢
§§ Solution. Let B and C be the positions of the balloon respec-
tively when the observations are taken and S be the position of the
ship whose horizontal distance SA (x feet, say) is required, A being
the point of ascent.
Draw BN and CM parallel to AS, so that BN and CM are the
horizontal lines passing through B and C respectively.
We are given
∠N BS = 30◦ = ∠BSA, and
∠M CS = 15◦ = ∠CSA.
x
∴ = sin ∠ACS = cos 15◦ , and
CS
CS sin ∠CBS sin 60◦
= = .
600 sin ∠CSB sin 15◦
Hence, by multiplication, we have

x cos 15◦ sin 60◦ ◦ ◦ 3 √
= ◦
= sin 60 cot 15 = (2 + 3).
600 sin 15 √ 2
∴ x = 300(2 3 + 3) = 300 × 6.464 . . .
∴ x = 1939.2 . . . f eet.
Otherwise thus : If CA = h feet, we have
h + 600 1 h √
= tan 30◦ = √ , and = tan 15◦ = 2 − 3,
x 3 x

600 1 √ 4−2 3
∴ = √ −2+ 3= √
x 3 3

∴ x = 300(2 3 + 3) = 1939.2 f eet. ■
14.1. Inaccessible Object and Distant Points 229

§ Problem 14.1.8. P Q is a tower standing on a horizontal plane, Q


being its foot ; A and B are two points on the plane such that the
∠QAB is 90◦ , and AB is 40 feet. It is found that
3 1
cot P AQ = and cot P BQ = .
10 2
Find the height of the tower. ♢
§§ Solution. If h feet be the height of the tower, we have
3 h
AQ = P Q cot ∠P AQ = h, and BQ = P Q cot ∠P BQ = .
10 2
Also, BQ2 = BA2 + AQ2 .
h2 9h2
∴ = (40)2 +
4 100
∴ 16h2 = (400)2 , ∴ h = 100 f eet . ■

§ Problem 14.1.9. A column is E.S.E. of an observer, and at noon


the end of the shadow is North-East of him. The shadow is 80 feet
long and the elevation of the column at the observer’s station is 45◦ .
Find the height of the column. ♢
§§ Solution. Let CB, perpendicular to the plane of the paper, be the
column and O be the position of the observer, so that the ∠COB =
1◦
45◦ , and the ∠EOB = 22 (the column being E.S.E of O) where
2
E is the extremity of the horizontal line due east from O. Let BD
represent the shadow, which lies due N , since at noon the sun is
due S. D, the extremity of the shadow, is N.E, of O, so that the
1◦
∠DOE = 45◦ . We have OD = 80 feet, and the ∠DOB = 67 .
2
◦ ◦ 1◦ 1◦
Also ∠DBO = 180 − 45 − 67 = 67 = ∠DOB.
2 2

∴ OB = BC cot COB = BC cot 45 = BC;
1◦
∴ OB = 2OD cos DOB = 2 × 80 sin 22
√ √ √2
= 80 2 − 2 cos 45◦ = 80 2 − 2

= 80 .5857864 = 80 × .7653 = 61.224 f eet.
= BC, the required height. ■

§ Problem 14.1.10. A tower is observed from two stations A and


B, It is found to be due north of A and north-west of B. B is due east
of A and distant from it 100 feet. The elevation of the tower as seen
from A is the complement of the elevation as seen from B. Find the
height of the tower. ♢
§§ Solution. Let T and R be the top and foot of the tower respec-
tively, and let h feet be its height. Let α and β be the angles of
elevation at A and B respectively.
We have
α + β = 90◦ ,
RA = h cot α, and
RB = h cot β = h tan α.
Also,
∵ ∠ABR = 45◦ , ∴ RA = AB = 100 f eet
14.1. Inaccessible Object and Distant Points 230


∴ RB = 100 2 f eet.

∴ h cot α = 100, and h tan α = 100 2.
Hence, by multiplication, we have

h2 = (100)2 × 2

4
∴ h = 100 2 f eet. ■

§ Problem 14.1.11. The elevation of a steeple at a place due south


of it is 45◦ and at another place due west of the former place the
elevation is 15◦ . If the distance between the two places be a, prove
that the height of the steeple is (√ )
a 3−1
√ .
243 ♢
§§ Solution. Take the figure of Art. 193, with P Q(= x) representing
the steeple,
∠P BQ = 45◦ , ∠P AQ = 15◦ , ∠QBA = 90◦ , and AB = a.
We then have
BQ = x cot 45◦ = x, and

3+1
AQ = x cot 15◦ = x √ .
3−1
Also, AQ2 = BQ2 + AB 2
Å√ ã2
3+1
∴ √ x2 = x2 + a2
3−1
ñÅ √ ã2 ô
3+1
∴ √ − 1 x2 = a2
3−1
ï √ ò
4 3
∴ √ x2 = a2
( 3 − 1)2
ï √ ò
a2 ( 3 − 1)2
∴ x2 = √
4 3
ï√ ò
a 3−1
∴x= √
4
.
2 3 ■

§ Problem 14.1.12. A person stands in the diagonal produced of the


square base of a church tower, at a distance 2a from it, and observes
the angles of elevation of each of the two outer corners of the top of
the tower to be 30◦ , whilst that of the nearest corner is 45◦ . Prove
that the breadth of the tower(√is √ )
a 10 − 2 . ♢
§§ Solution. Let A and C be the two upper corners and B and D
the two lower corners, B being the corner nearest to the observer
at E.
Let x be the required breadth.
We have
BE = 2a, ∠AEB = 45◦ and ∠CED = 30◦ .
Then AB = EB tan 45◦ = 2a = CD

and ED = CD cot 30◦ = 2a 3
14.1. Inaccessible Object and Distant Points 231

also, ∠EBD = 135◦ .


∴ ED2 = EB 2 + BD2 − 2EB.BD cos 135◦
1
12a2 = 4a2 + x2 + 2 × 2a × x × √
√ 2
∴ x2 + 2a 2x − 8a2 = 0
√ √
−2a 2 ± 8a2 + 32a2
∴x=
√ √2 (√ √ )
= −a 2 + 10a2 = a 10 − 2 . ■

§ Problem 14.1.13. A person standing at a point A due south of a


tower built on a horizontal plane observes the altitude of the tower
to be 60◦ . He then walks to B due west of A and observes the altitude
to be 45◦ , and again at C in AB produced he observes it to be 30◦ .
Prove that B is midway between A and C. ♢
§§ Solution. If T and R be the top and foot of the tower respectively
and h be its height.
We have
h
AR = h cot 60◦ = √
3
BR = h cot 45◦ = h, and

CR = h cot 30◦ = h 3.
h2 2h2
∴ AB 2 = BR2 − AR2 = h2 − = , and
3 3
h 2 8h 2
AC 2 = CR2 − AR2 = 3h2 − = .
3 3
∴ AC = 4AB
2 2

∴ AC = 2AB. ■

§ Problem 14.1.14. At each end of a horizontal base of length 2a it


is found that the angular height of a certain peak is θ and that at the
middle point it is ϕ. Prove that the vertical height of the peak is
a sin θ sin ϕ
√ .
sin(ϕ + θ) sin(ϕ − θ) ♢
§§ Solution. Let AB be the base of length 2a, C be its middle point
and P and Q be the top and foot of the peak respectively.
Let the required height, P Q, be h.
We the have
CQ = h cot ϕ, and
AQ = BQ = h cot θ.
∴ ∆ABQ is isosceles
∴ ∠ACQ = 90◦
∴ AQ2 = AC 2 + CQ2
∴ h2 cot2 θ = a2 + h2 cot2 ϕ
( )
∴ h2 cot2 θ − cot2 ϕ = a2
Å ã
cos2 θ cos2 ϕ
∴ h2 − = a2
sin2 θ sin2 ϕ
14.1. Inaccessible Object and Distant Points 232

a2 sin2 θ sin2 ϕ
∴ h2 =
sin2 ϕ cos2 θ − sin2 θ cos2 ϕ
a2 sin2 θ sin2 ϕ
=
(sin ϕ cos θ + cos ϕ sin θ) (sin ϕ cos θ − cos ϕ sin θ)
a2 sin2 θ sin2 ϕ
=
sin(ϕ + θ) sin(ϕ − θ)
a sin θ sin ϕ
∴h= √ .
sin(ϕ + θ) sin(ϕ − θ) ■

§ Problem 14.1.15. A and B are two stations 1000 feet apart ; P and
Q are two stations in the same plane as AB and on the same side of
it ; the angles ∠P AB, ∠P BA, ∠QAB and ∠QBA are respectively 75◦ ,
30◦ , 45◦ , and 90◦ ; find how far P is from Q and how far each is from
A and B. ♢
§§ Solution. Draw a figure as in Art. 194, with the angles and AB
as given.
Since the ∠QAB = 45◦ , we have √
BQ = AB = 1000 f eet; AQ = 1000 2 f eet.
Also, ∠AP B = 180◦ − (75◦ + 30◦ ) = 75◦
∴ P B = AB = 1000 f eet.
∵ ∠P BQ = 60◦
∴ ∆P BQ is equilateral.
∴ P Q = 1000 f eet.
Also, in the isosceles ∆AP B, we have

3−1 (√ √ )
AP = 2 × 1000 cos 75◦ = 1000 × √ = 500 6 − 2 f eet. ■
2
For the following seven examples, a book of tables will be re-
quired.
§ Problem 14.1.16. At a point on a horizontal plane the elevation
of the summit of a mountain is found to be 22◦ 15′ , and at another
point on the plane, a mile further away in a direct line, its elevation
is 10◦ 12′ ; find the height of the mountain. ♢
§§ Solution. Take the figure of Art. 192. Let P Q (x miles, say) be
the mountain and B and A be the two points at which the angles of
elevation are taken.
We are given
BA = 1 mile, ∠P BQ = 22◦ 15′ and ∠P AQ = 10◦ 12′ .
Also, ∠AP B = 22◦ 15′ − 10◦ 12′ = 12◦ 3′ .
From the ∆P BQ, we have
x
= sin 22◦ 15′ (14.6)
BP
From the ∆P AB, we have
PB sin 10◦ 12′
= (14.7)
1 mile sin 12◦ 3′
From (14.6) and (14.7), by multiplication, we have
x sin 10◦ 12′ sin 22◦ 15′
= = sin 10◦ 12′ sin 22◦ 15′ cosec 12◦ 3′
1 sin 12◦ 3′
log x = L sin 10◦ 12′ + L sin 22◦ 15′ + L cosec 12◦ 3′ − 30
= 9.2481811 + 9.5782364 + 10.6809341 − 30 = 1̄.5067594.
14.1. Inaccessible Object and Distant Points 233

Now
log .32118 = 1̄.5067485, and
1̄.5067594 − 1̄.5067485 = .0000109,
for which difference the proportional part is 8.
∴ x = .321188 ≈ .32119 mile. ■

§ Problem 14.1.17. From the top of a hill the angles of depression


of two successive milestones, on level ground and in the same verti-
cal plane with the observer, are found to be 5◦ and 10◦ respectively.
Find the height of the hill and the horizontal distance to the nearest
milestone. ♢
§§ Solution. Take the figure of Art. 192. Draw P N parallel to QA,
so that P N is the horizontal line passing through P . Let P be the
position of the observer on the hill P Q and A and B be the positions
of the milestones.
We are given
AB = 1 mile,
∠P AQ = ∠N P A = 5◦ and
∠P BQ = ∠N P B = 10◦ .
Also,
∠AP B = 10◦ − 5◦ = 5◦ = ∠P AB,
∴ AP = AB = 1 mile.
∴ P Q = AP sin 10◦ = sin 10◦ = .1736482 mile, and
BQ = AP cos 10◦ = cos 10◦ = .9848078 mile,
by the table of natural sines and cosines. ■

§ Problem 14.1.18. A castle and a monument stand on the same


horizontal plane. The height of the castle is 140 feet, and the angles
of depression of the top and bottom of the monument as seen from
the top of the castle are 40◦ and 80◦ respectively. Find the height of
the monument. ♢
§§ Solution. Take the figure of Ex. 3, Art. 45. Let AB be the castle
and CD be the monument.
We are given
AB = 140 f eet, ∠EAC = 40◦ , and
∠EAD = 80◦ = ∠ADB.
We then have
BD = 140 cot ∠ADB = 140 cot 80◦ = AE, and
CE = AE tan 40◦ = 140 cot 80◦ tan 40◦
∴ log CE = log 140 + L cot 80◦ + L tan 40◦ − 20
= 2.1461280 + 9.2463188 + 9.9238135 − 20 = 1.3162603.
Now log 20.713 = 1.3162430, diff. for .001 = .0000210.
Let log(20.713 + x) = 1.3162603.
The diff. for x = 1.3162603 − 1.3162430 = .0000173.
173 .173
∴x= × .001 = ≈ .0008238
210 210
∴ CE = 20.7138238 f eet, and
DE = 140 − CE ≈ 119.2862 f eet. ■
14.1. Inaccessible Object and Distant Points 234

§ Problem 14.1.19. A flagstaff P N stands on level ground. A base


AB is measured at right angles to AN , the points A,B and N being
in the same horizontal plane, and the angles ∠P AN and ∠P BN are
found to be α and β respectively.
Prove that the height of the flagstaff is
sin α sin β
AB √ .
sin(α − β) sin(α + β)
If AB = 100 f eet, α = 70◦ and β = 50◦ , calculate the height. ♢
§§ Solution. Take the figure of Art. 193, with N for Q, A for B and
B for A.
We then have
AN = x cot α, and BN = x cot β.
Also, since the ∠BAN is a right angle,
∴ BN 2 = AN 2 + AB 2
∴ x2 cot2 β = x2 cot2 α + AB 2
( )
∴ x2 cot2 β − cot2 α = AB 2
sin α sin β
∴ x = AB √
sin(α − β) sin(α + β)
as in (14.1.14).
If AB = 100 f eet, α = 70◦ and β = 50◦ , we have
100 sin 70◦ sin 50◦
x= √
sin 20◦ sin 120◦
∴ log x = log 100 + L sin 70◦ − 10 + L sin 50◦ − 10
1
− [L sin 20◦ − 10 + L sin 60◦ − 10]
2
= 2 + 9.9729858 + 9.8842540 − 20
1
− [9.5340517 + 9.9375306 − 20]
2
1
= 1.8572398 − [−.5284177] = 2.1214487.
2
Now log 132.26 = 2.1214285, diff. for .01 = .0000329.
Let log(132.26 + y) = 2.1214487.
The diff. for y = 2.1214487 − 2.1214285 = .0000202.
202 2.02
Hence we have y = × .01 = ≈ .006.
329 329
∴ log x = log(132.26 + .006) = log 132.266
∴ x = 132.266 f eet. ■

§ Problem 14.1.20. A man, standing due south of a tower on a hor-


izontal plane through its foot, finds the elevation of the top of the
tower to be 54◦ 16′ ; he goes east 100 yards and finds the elevation to
be then 50◦ 8′ . Find the height of the tower. ♢
§§ Solution. As in the last example, if h yards be the required height,
we have

100 sin 54◦ 16′ sin 50◦ 8′


h= √
sin(54◦ 16′ + 50◦ 8′ ) sin(54◦ 16′ − 50◦ 8′ )
14.1. Inaccessible Object and Distant Points 235

100 sin 54◦ 16′ sin 50◦ 8′


= √
sin 104◦ 24′ sin 4◦ 8′
∴ log h = log 100 + L sin 54◦ 16′ − 10 + L sin 50◦ 8′ − 10
1[ ]
− L sin 75◦ 36′ − 10 + L sin 4◦ 8′ − 10
2
= 2 + 9.9094190 + 9.8851000 − 20
1
− [9.9861369 + 8.8578010 − 20] = 2.3725500.
2
Now log 235.80 = 2.3725438, diff. for .01 = .0000184.
Let log(235.80 + x) = 2.3725500.
The diff. for x = 2.3725500 − 2.3725438 = .0000062.
62 .62
∴x= × .01 = ≈ .00337
184 184
∴ h ≈ 235.8034 yards. ■

§ Problem 14.1.21. A man in a balloon observes that the angle of


depression of an object on the ground bearing due north is 33◦ ; the
balloon drifts 3 miles due west and the angle of depression is now
found to be 21◦ . Find the height of the balloon. ♢
§§ Solution. Let C be the object, A be the first position of the bal-
loon, AD (h miles) its height and B be the second position of the
balloon, BE (h miles) its height.
We then have
CD = h cot 33◦ and CE = h cot 21◦
also, CE 2 = CD 2 + DE 2
∴ h cot2 21◦ = h2 cot2 33◦ + 32 .
2

Hence, as in § Problem 14.1.14, we have


3 sin 33◦ sin 21◦
h= √
sin 54◦ sin 12◦
∴ log h = log 3 + L sin 33 − 10 + L sin 21◦ − 10

1
− [L sin 54◦ − 10 + L sin 12◦ − 10]
2
= .4771213 + 9.7361088 + 9.5543292 − 20
1
− [9.9079576 + 9.3178789 − 20] = .1546410.
2
Now log 1.4277 = .1546370, diff. for .0001 = .0000304.
Let log(1.4277 + x) = .1546410.
The diff. for x = .1546410 − .1546370 = .0000040.
40 .001
∴x= × .0001 = ≈ .000013
304 76
∴ h = 1.427713 mile. ■

§ Problem 14.1.22. From the extremities of a horizontal base-line


AB, whose length is 1000 feet, the bearings of the foot C of a tower
are observed and it is found that ∠CAB = 56◦ 23′ , ∠CBA = 47◦ 15′ ,
and that the elevation of the tower from A is 9◦ 25′ ; find the height
of the tower. ♢
§§ Solution. Let D be the top of the tower.
We have ( )
∠DAC = 9◦ 25′ , and ∠ACB = 180◦ − 56◦ 23′ + 47◦ 15′ = 76◦ 22′ ,
14.2. Angle Subtended at Two Points 236

AC sin ∠ABC
and =
AB sin ∠ACB
CD
also, = tan ∠DAC.
AC
Hence, by multiplication, we have
CD
= tan ∠DAC sin ∠ABC cosec ∠ACB
AB
∴ CD = 1000 tan 9◦ 25′ sin 47◦ 15′ cosec 76◦ 22′
∴ log CD = log 1000 + L tan 9◦ 25′ + L sin 47◦ 15′ + L cosec 76◦ 22′ − 30
= 3 + 9.2197097 + 9.8658868 + 10.0124124 − 30 = 2.0980089.
Now log 125.31 = 2.0979857, diff. for .01 = .0000346.
Let log(125.31 + x) = 2.0980089.
The diff. for x = 2.0980089 − 2.0979857 = .0000232.
232 2.32
∴x= × .01 = ≈ .0067
346 346
∴ CD = 125.3167 f eet. ■

14.2 Angle Subtended at Two Points


§ Problem 14.2.1. A bridge has 5 equal spans, each of 100 feet mea-
sured from the center of the piers, and a boat is moored in a line with
one of the middle piers. The whole length of the bridge subtends a
right angle as seen from
√ the boat. Prove that the distance of the boat
from the bridge is 100 6 f eet. ♢
§§ Solution. Let AB be the bridge, C be the pier and D be the po-
sition of the boat.
We are given
AC = 200 f t., CB = 300f t., and ∠ADB = 90◦ .
Let the required distance CD be x ft. and ∠ADC = α.
We then have
x = AC cot α = 200 cot α
also x = BC cot(90◦ − α) = 300 tan α.
Hence, by multiplication, we have
x2 = 60000

∴ x = 100 6 f eet. ■

§ Problem 14.2.2. A ladder placed at an angle of 75◦ with the ground


just reaches the sill of a window at a height of 27 f eet above the
ground on one side of a street. On turning the ladder over with-
out moving its foot, it is found that when it rests against a wall on
the other side of the street it is at an angle of 15◦ with the ground.
Prove that the breadth of the street and the length of the ladder are
respectively ( √ ) (√ √ )
27 3 − 3 and 27 6 − 2 f eet. ♢
§§ Solution. Let AC and CE be the two positions of the ladder
(length l f eet) respectively and DB be the breadth of the street, B
being vertically below A and D below E.
We are given
AB = 27 f t., ∠ACB = 75◦ and ∠ECD = 15◦ .
14.2. Angle Subtended at Two Points 237

The triangles ABC and CDE are identically equal.


∴ CD = AB = 27 f t.
( √ )
Also, CB = 27 cot 75◦ = 27 2 − 3 f t.
∴ the breadth of the street ( √ )
= DC + CB = 27 3 − 3 f t.

2 2 (√ √ )
Again, l = 27 cosec 75◦ = 27 × √ = 27 6 − 2 f t.
3+1
Otherwise thus :
The breadth of the street = l cos 75◦ + l cos 15◦
= 2l cos 45◦ cos 30◦

(√ √ ) 1 3
= 54 6− 2 √ ×
2
√ √ 2 ( √ )
= 27( 3 − 1) 3 = 27 3 − 3 f t. ■

§ Problem 14.2.3. From a house on one side of a street observa-


tions are made of the angle subtended by the height of the opposite
house; from the level of the street the angle subtended is the angle
whose tangent is 3 ; from two windows one above the other the angle
subtended is found to be the angle whose tangent is −3 ; the height
of the opposite house being 60 f eet, find the height above the street
of each of the two windows. ♢
§§ Solution. Let A be the top and B be the bottom of the house of
height 60 feet and C be the bottom of the other house, i.e. C is at
the level of the street.
We then have
1 1
CB = AB cot ∠ACB = 60 × = 60 × = 20 f eet.
tan ∠ACB 3
Let W ′ and W be the upper and lower windows respectively and let
x f eet be the height of either window above the street.
Draw W ′ D and W E perpendicular to AB, so that W ′ D = W E =
CB = 20 f eet.
We have
∠AW E + ∠BW E = ∠AW B.
tan ∠AW E + tan ∠BW E
∴ = tan ∠AW B
1 − tan ∠AW E. tan ∠BW E
x 60 − x
+
∴ 20 20 = −3
x 60 − x
1− .
20 20
∴ x2 − 60x + 800 = 0
∴ (x − 20)(x − 40) = 0
∴ x = 20 or 40.

Thus the windows are 20 f t. and 40 f t. above the street.

§ Problem 14.2.4. A rod of given length can turn in a vertical plane


passing through the sun, one end being fixed on the ground ; find
the longest shadow it can cast on the ground.
Calculate the altitude of the sun when the longest shadow it can
1
cast is 3 times the length of the rod. ♢
2
14.2. Angle Subtended at Two Points 238

§§ Solution. Let BA be the rod of length l, AC be its shadow, γ be


the sun’s altitude and θ be the inclination of the rod to the ground.
We then have
l sin(θ + γ)
AC = ,
sin γ
and this is greatest when θ + γ = 90 , i.e. when θ = 90◦ − γ, i.e. when

ABC is a right angle.


∴ AC = l cosec γ.
1
Again, if AC = 3 × l, then
2
7 2
cosec γ = , ∴ sin γ = . ■
2 7
§ Problem 14.2.5. A person on a ship A observes another ship B
leaving a harbour, whose bearing is then N.W. After 10 minutes A,
having sailed one mile N.E., sees B due west and the harbour then
bears 60◦ West of North. After another 10 minutes B is observed to
bear S.W. Find the distances between A and B at the first observation
and also the direction and rate of B. ♢
§§ Solution. Let A, A1 , A2 , B, B1 and B2 be the positions of A and
B respectively as given in the question.
A2 A1 B2 is a straight line since both A1 and B2 are in a direction
S.E. of A2 .
We have
∠BAA1 = 90◦ and ∠BA1 A = 75◦

∴ AB = AA1 tan 75◦ = 1 × (2 + 3) = 3.732 . . . miles.
Also, BB1 = B1 B2
A1 B 1 A1 B1
∴ =
BB1 B1 B2
sin(θ − 30◦ ) sin(θ + 45◦ )
∴ ◦
= where θ ≡ ∠A1 B1 B2
sin 30 sin 45◦

∴ sin θ cot 30 − cos θ = sin θ + cos θ
∴ sin θ(cot 30◦ − 1) = 2 cos θ
2 √
∴ tan θ = √ = 3 + 1.
3−1 √
∴ B ′ s course is at an angle whose tangent √ is 3 + 1, S. of◦ E.
Again, BB2 = AB cosec ∠BB2 A = (2 + 3) cosec (θ + 45 )
√ √ √
2+ 3 2(2 + 3)
= =
sin(θ + 45◦ ) sin θ + cos θ

√ √ 3+1+1 √ √ √
= 2(2 + 3) ÷ √ √ = 2 5+2 3
5+2 3
√ √ √
= 10 + 4 3 = 16.9282 = 4.114 . . . miles.

Hence in 1 hour the distance = 4.114 . . . × 3 = 12.342 . . . miles.

§ Problem 14.2.6. A person on a ship sailing north sees two light-


houses, which are 6 miles apart, in a line due west ; after an hour’s
sailing one of them bears S.W. and the other S.S.W. Find the ship’s
rate. ♢
14.2. Angle Subtended at Two Points 239

§§ Solution. Let S and N be the two positions of the ship respec-


tively, A be the nearer lighthouse and B be the further lighthouse.
We are given
1◦
∠BN S = 45◦ and ∠AN S = 22
2
1◦
∴ ∠N AS = 67 .
2
We then have ( )
1◦
sin 180◦ − 67
BN sin ∠BAN 2
= = ( )
BA sin ∠BN A 1◦
sin 45◦ − 22
2
1◦ 1◦
sin 67 cos 22
2 = cot 22 1 = √2 + 1
BN ◦
∴ = 2 =
6 1◦ 1◦ 2
sin 22 sin 22
2 (√2 )
∴ BN = 6 2+1 .
(√ )
6 2+1 √ (√ )
Also, SN = BN sin 45◦ = √ =3 2 2+1
( √ ) 2
= 3 2 + 2 = 3 × 3.4142 = 10.2426 miles

i.e. the required rate is 10.2426 miles per hour.

§ Problem 14.2.7. A person on a ship sees a lighthouse N.W. of


himself. After sailing for 12 miles in a direction 15◦ south of W. the
lighthouse is seen due N. Find the distance of the lighthouse from
the ship in each position. ♢
§§ Solution. Let S and H be the two positions of the ship and L be
the light house.
We then have
SL sin ∠SHL
=
SH √ ∠SLH
sin
12 sin 75◦ 3+1
∴ SL = ◦
= 12 ×
(sin
√ 45 ) 2
=6 3 + 1 = 6 × 2.73205 = 16.3923 . . . miles
HL sin ∠HSL
Also, =
SH sin ∠SLH
12 sin 60◦ √ √ √
∴ HL = =6 3 2=6 6
sin 45◦
= 6 × 2.4494897 ≈ 14.697 miles ■

§ Problem 14.2.8. A man, traveling west along a straight road, ob-


serves that when he is due south of a certain windmill the straight
line drawn to a distant tower makes an angle of 30◦ with the road.
A mile further on the bearings of the windmill and tower are re-
spectively N.E. and N.W. Find the distances of the tower from the
windmill and from the nearest point of the road. ♢
§§ Solution. Let W be the windmill, T be the tower, M and A be
the two points at which the directions are taken and let T N be per-
pendicular to the road.
14.2. Angle Subtended at Two Points 240

We are given
M A = 1 mile, ∠T M A = 30◦ and ∠W AM = 45◦ = ∠T AN.
We then have
AT sin ∠AM T sin 30◦
= = = 2 cos 15◦
AM sin ∠AT M √sin 15◦
3+1
∴ AT = √ .
2 √

Also, AW = AM sec 45 = 2.
∵ ∠T AW = 90◦ , we have
√ √
T W 2 = AT 2 + AW 2 = 2 + 3 + 2 = 4 + 3.
√ √ √
∴ TW = 4 + 3 = 5.73205 = 2.39 miles.
√ √
3+1 1 3+1
Also, T N = AT sin 45◦ = √ .√ =
2 2 2
2.73205
= = 1.366 miles. ■
2
§ Problem 14.2.9. An observer on a headland sees a ship due north
of him ; after a quarter of an hour he sees it due east and after an-
other half-hour he sees it due south-east; find the direction that the
ship’s course makes with the meridian and the time after the ship
is first seen until it is nearest the observer, supposing that it sails
uniformly in a straight line. ♢
§§ Solution. Let O be the position of the observer and S, H and I
be the three positions of the ship respectively.
Draw OP perpendicular to the straight line SHI ; join OH ; and
draw IA perpendicular to SO produced.
Then P is the nearest position of the ship to the observer after it
is first seen.
1
Also, ∠HOI = 45◦ . We have SH = the distance the ship sails in
4
1
hour and HI = the distance it sails in hour.
2
∴ HI = 2SH
∴ OA = 2OS = AI
AI 2
∴ tan ∠ASI = = ,
AS 3
so that the ship’s course makes with the meridian an angle whose
2
tangent is .
3
Also,
9
SP = OS cos ∠OSP = SH cos2 ∠OSP = SH cos2 ∠ASI = SH × .
( ) 13
9 1 9
Hence the required time = × hour = hour. ■
13 4 52
§ Problem 14.2.10. A man walking along a straight road, which
runs in a direction 30◦ east of north, notes when he is due south of a
certain house ; when he has walked a mile further, he observes that
the house lies due west and that a windmill on the opposite side of
the road is N.E. of him ; three miles further on he finds that he is due
14.2. Angle Subtended at Two Points 241

north of the windmill; prove that the line joining the house and the
windmill makes with the road the angle whose tangent is

48 − 25 3
. ♢
11
§§ Solution. Let A, and B and C be the three positions of the man,
H be the house and W be the windmill.
Let HW meet AC in K and CW meet HB produced in D.
We then have
1
HB = AB sin 30◦ = mile, and
2
3
BD = BC sin 30◦ = mile = DW, ∵ ∠W BD = 45◦
2
∴ HD = 2 miles.
Let ϕ be the required angle, so that
ϕ = ∠CKW = ∠CBD − ∠W HD = 60◦ − ∠W HD.
WD 3 3
We have tan ∠W HD = = ÷2=
HD 2 4
3
∴ tan(60◦ − ϕ) =
4
tan 60◦ − tan ϕ 3
∴ ◦ tan ϕ
=
(√ 1 + tan)60 ( √4 )
∴4 3 − tan ϕ = 3 1 + 3 tan ϕ
√ ( √ )( √ ) √
4 3−3 4 3−3 3 3−4 48 − 25 3
∴ tan ϕ = √ = = . ■
3 3+4 27 − 16 11
§ Problem 14.2.11. A, B, and C are three consecutive milestones
on a straight road from each of which a distant spire is visible. The
spire is observed to bear north-east at A, east at B, and 60◦ east of
south at C. Prove that the shortest distance of the spire from the
road is √
7+5 3
miles. ♢
13
§§ Solution. Let S be the spire and SD (x miles, say) be the shortest
distance of the spire from the road.
Let ∠BSD = θ and tan θ = t.
We have
1 = BC = CD − BD = x tan(30◦ + θ) − x tan θ
Å √ ã Å ã
1+t 3 1 + t2
=x √ −t =x √ .
3−t 3−t
Also,
1 = AB = BD + AD = x tan θ + x tan(45◦ − θ)
( ) Å ã
1−t 1 + t2
=x t+ =x .
1+t 1+t
Hence, equating these Å
values, weã have
Å ã
1 + t2 1 + t2
x √ =x
3−t 1+t

√ √ 3−1
∴ 3 − t = 1 + t, i.e. 2t = 3 − 1, i.e. t = .
2
14.2. Angle Subtended at Two Points 242


3−1
1+t 1+
∴x= = 2√
1 + t2 2− 3
1+
√ √2 √ √
1+ 3 (1 + 3)(4 + 3) 7+5 3
= √ = = . ■
4− 3 16 − 3 13
§ Problem 14.2.12. Two stations due south of a tower, which leans
towards the north, are at distances a and b from its foot; if α and β
be the elevations of the top of the tower from these stations, prove
(
that its inclination to the horizontal is )
b cot α − a cot β
cot−1 .
b−a ♢
§§ Solution. Let T be the top and R be the foot of the tower and let
A and B be the two stations respectively.
Then, if θ be the required inclination, we have
b BR BR T R sin(θ − β) sin α
= = . = .
a AR T R AR sin β sin(θ − α)
sin θ cot β − cos θ cot β − cot θ
= =
sin θ cot α − cos θ cot α − cot θ
∴ b cot α − a cot β = (b − a) cot θ
( )
b cot α − a cot β
∴ θ = cot−1 .
b−a
Otherwise thus :
From T draw T N perpendicular to the ground. Let T N = h and
RN = x. Then
In the ∆T BN , we have b + x = h cot β
In the ∆T AN , we have a + x = h cot α, and
In the ∆T RN , we have x = h cot θ.

∴ b = h(cot β − cot θ) and a = h(cot α − cot θ)


b cot β − cot θ
∴ =
a cot α − cot θ
∴ b cot α − a cot β = (b − a) cot θ
( )
b cot α − a cot β
∴ θ = cot−1 . ■
b−a
§ Problem 14.2.13. From a point A on a level plane the angle of
elevation of a balloon is α, the balloon being south of A ; from a point
B, which is at a distance c south of A, the balloon is seen northwards
at an elevation of β; find the distance of the balloon from A and its
height above the ground. ♢
§§ Solution. If C be the position of the balloon and CD be its height
above the ground,we have
AC sin β
=
AB sin(α + β)
∴ AC = c sin β cosec (α + β).
Also, CD = AC sin α = c sin α sin β cosec (α + β). ■
14.2. Angle Subtended at Two Points 243

§ Problem 14.2.14. A statue on the top of a pillar subtends the same


1
angle α at distances of 9 and 11 yards from the pillar ; if tan α = ,
10
find the height of the pillar and of the statue. ♢
§§ Solution. Take the figure of Art. 196.
Let RP (x yards, say) be the statue and P Q (y yards, say) be the
pillar and let B and A be the points of observation, so that BQ =
9 yards and AQ = 11 yards.
Since the ∠P AR = ∠P BR = α, a circle will go through the four
points A, B, P and R.

We have
y
= tan β (14.8)
11
x+y
= tan(α + β), and (14.9)
11
9
= tan β (14.10)
x+y
From (14.9) and (14.10), by multiplication, we have
9 tan α + tan β
= tan β tan(α + β) = tan β
11 1 − tan α tan β
1
+ tan β tan β + 10 tan2 β
= tan β 10 =
1 10 − tan β
1− tan β
10
∴ 11 tan2 β + 2 tan β − 9 = 0
∴ (11 tan β − 9)(tan β + 1) = 0
∴ 11 tan β − 9 = 0
9
∴ tan β = .
11
Hence, from (14.8), we have y = 9 yards, and, from (14.10), x+y = 11.
∴ x = 11 − y = 2 yards.
Otherwise thus :
Let O be the center of the circle passing though the four points
A, B, P and R and from O draw OM and ON perpendicular to RP
and AB, bisecting them in M and N respectively.
Join OR and OP .
1
∠ROM = ∠ROP = ∠RAP by Euclid. III. 20, and we have
2
1 RM
tan ∠RAP = = tan ∠ROM =
( 10 ) ( OM )
1 1 1 1 1
∴ RM = MO = BQ + AB = × 9 + × 2 = 1 yard
10 10 2 10 2
∴ RP = 2 yards.
Also, by Euclid. III. 36, Cor., we have
QB.QA = QR.QP
∴ 9 × 11 = QP (QP + 2)
∴ QP 2 + 2QP − 99 = 0

−2 ± 4 + 396
∴ QP = = −1 ± 10 = 9 yards. ■
2
14.2. Angle Subtended at Two Points 244

§ Problem 14.2.15. A flagstaff on the top of a tower is observed to


subtend the same angle α at two points on a horizontal plane, which
he on a line passing through the center of the base of the tower and
whose distance from one another is 2a, and an angle β at a point
halfway between them. Prove that the height of the flagstaff is

2 sin β
a sin α .
cos α sin(β − α) ♢
§§ Solution. Take the figure of Art. 196, with N the middle point of
AB. Let P and Q be the top and foot of the tower respectively, and
let P R be the flagstaff.
Let A, B and N be the points of observation, so that
∠RBP = α = ∠RAP, ∠RN P = β and BN = a = N A.
∵ ∠RBP = ∠RAP,
∴ a circle will go through the four points A, B, P and R.
Let O be the center of this circle and from O draw OM and ON
perpendicular to RP and AB, bisecting them in M and N respec-
tively.
Join OR, OP .
Let ∠P N Q = ϕ. We then have the ∠ROP = 2α, so that
∠ROM = α = ∠M OP
x
∴ OM = cot α = QN, where RP = x, and
2
x
OR = cosec α.
2
Also,

√ √ x2
ON = OB 2 − BN 2 = OR2 − BN 2 = cosec 2 α − a2 = M Q
4

x2 x
cosec 2 α − a2 +
4 2
∴ tan(β + ϕ) = x , and
cot α
… 2
x2 x
cosec 2 α − a2 −
4 2
tan ϕ = x .
cot α
2
x 2
∴ tan(β + ϕ) − tan ϕ = x = , and
cot α cot α
2
x2 x2 x2
cosec 2 α − a2 − cot2 α − a2
1 + tan(β + ϕ) tan ϕ = 1 + 4 4 = 2
x2 x2
cot2 α cot2 α
4 4
x 2

tan(β + ϕ) − tan ϕ cot2 α


∴ tan β = tan(β + ϕ − ϕ) = = 2 2
1 + tan(β + ϕ) tan ϕ x
cot2 α − a2
2
2a2
∴ cot β = cot α − 2
x cot α
2a2 1
∴ 2 . = cot α − cot β.
x cot α
14.2. Angle Subtended at Two Points 245

2a2 2a2 sin2 α sin β


∴ x2 = =
cot α(cot α − cot β) cos α sin(β − α)

2 sin β
∴ x = a sin α . ■
cos α sin(β − α)

§ Problem 14.2.16. An observer in the first place stations himself


at a distance a f eet from a column standing upon a mound. He finds
1
that the column subtends an angle, whose tangent is , at his eye
2
which may be supposed to be on the horizontal plane through the
2
base of the mound. On moving a f eet nearer the column, he finds
3
that the angle subtended is unchanged. Find the height of the mound
and of the column. ♢
§§ Solution. Take the figure of Art. 196.
Let P and Q be the top and foot of the mound respectively and let
P R be the column.
Let A and B be the points of observation, so that AQ = a f t. and
2 a
AB = a f t., and therefore BQ = f t.
3 3
∵ ∠P AR = ∠P BR = α, a circle will go through the four points A,
B, P and R.
Let y f t. and x f t. be the heights of the mound and column re-
spectively and β be as marked in the figure.
We then have
1
tan α = .
2
Also, x + y = a tan(α + β) and y = a tan β.
Also, in the ∆RBQ,
a
x + y = cot β.
3
1
1 + tan β 1 + 2 tan β
∴ cot β = tan(α + β) = 2 =
3 1 2 − tan β
1 − tan β
2
1 1 + 2 tan β
∴ =
3 tan β 2 − tan β
∴ 3 tan2 β + 2 tan β − 1 = 0
∴ (3 tan β − 1)(tan β + 1) = 0
1
∴ tan β = .
3
a
∴ y = a tan β = f t., and
3
a a 2a
x = cot β − y = a − = f t.
3 3 3
Otherwise thus:
Let O be the center of the circle passing through the points A, B,
P and R.
From O draw OM and ON perpendicular to RP and AB, bisecting
them in M and N respectively.
Join OR, OP . The ∠ROP = 2α, and ∠ROM = α = ∠M OP .
14.2. Angle Subtended at Two Points 246

[ ]
x x 1
∴ OM = cot α = × 2 ∵ tan α =
2 2 2
2a
= x = NQ = .
3
Also, ON = OM , since the chords RP and AB are equal ;
2a
∴ ON = = MQ
3
a
∴ PQ = . ■
3
§ Problem 14.2.17. A church tower stands on the bank of a river,
which is 150 f eet wide, and on the top of the tower is a spire 30 f eet
high. To an observer on the opposite bank of the river, the spire
subtends the same angle that a pole six feet high subtends when
placed upright on the ground at the foot of the tower. Prove that the
height of the tower is nearly 285 f eet. ♢
§§ Solution. Let AB be the spire, BD be the tower, CD be the pole
and O be the position of the observer, so that
AB = 30 f eet, CD = 6 f eet and OD = 150 f eet.
Let ∠AOB = α = ∠COD, ∠BOC = β and x feet be the required
height.
We then have
6 x
= tan α, = tan(α + β) and
150 150
x 6
x + 30 tan(α + β) + tan α +
= tan(α + β + α) = = 150 150
150 1 − tan(α + β) tan α 6x
1−
(150)2
∴ x2 + 30x − 90000 = 0
∴ x ≈ 285 f eet. ■

§ Problem 14.2.18. A person, wishing to ascertain the height of a


tower, stations himself on a horizontal plane through its foot at a
point at which the elevation of the top is 30◦ . On walking a distance
a in a certain direction he finds that the elevation of the top is the
5
same as before, and on then walking a distance a at right angles to
3
his former direction he finds the elevation ◦
√ of the √top to be 60 . Prove
5 85
that the height of the tower is either a or a. ♢
6 48
§§ Solution. Let T and R be the top and the foot of the tower, whose
height h is required and let A, B and C be the points at which the
angles of elevation are taken respectively, so that
5
∠T AR = 30◦ , ∠T BR = 30◦ , ∠T CR = 60◦ , AB = a and BC = a.
3
We then have
√ h
AR = h cot 30◦ = h 3 = BR and CR = h cot 60◦ = √ .
3
Also, ∠CBR = 90◦ − ∠ABR
∴ cos ∠CBR = sin ∠ABR.
a a
Now cos ∠ABR = ÷ BR = √
2 2h 3
14.2. Angle Subtended at Two Points 247


√ a2
∴ cos ∠CBR = 1− cos2 ∠ABR =
. 1−
12h2
Again, RC = BC + BR − 2BC.BR cos ∠CBR
2 2 2

h2 25a2 5a √ a2
∴ = + 3h2 − 2. .h 3 1 −
3 9 3 12h2
5a √ 8 25a 2
∴ 12h2 − a2 = h2 +
3 √ 3 9
∴ 15a 12h2 − a2 = 24h2 + 25a2
( )
∴ 225a2 12h2 − a2 = 576h4 + 1200a2 h2 + 625a4
∴ 288h4 − 750a2 h2 + 425a4 = 0

750 ± 72900 750 ± 270
∴ h2 = a2 . = a2 .
576 576
1020 480
= a2 . or a2 .
576 576
85 5
= a2 . or a2 .
48 … 6 …
85 5
∴h=a or a . ■
48 6
§ Problem 14.2.19. The angles of elevation of the top of a tower,
standing on horizontal plane, from two points distant a and b from
the base and in the same straight line √ with it are complementary.
Prove that the height of the tower is ab f eet, and, if θ be the angle
subtended at the top of the tower by the line joining the two points,
then
a∼b
sin θ = .
a+b ♢
§§ Solution. Let h denote the height of the tower and α the greater
angle of elevation.
We then have
a = h cot α and b = h cot(90◦ − α) = h tan α, where a < b.
Hence, by multiplication,

h2 = ab, ∴ h = ab.
Again, θ = α − (90 − α) = 2α − 90◦ .

sin2 α − cos2 α tan α − cot α


∴ sin θ = − cos 2α = 2 α + cos2 α
=
( ) (sin ) tan α + cot α
b a b a b−a
= − ÷ + = .
h h h h b+a
If a > b, we have
b = h cot α and a = h tan α
( ) ( )
a b a b a−b
∴ sin θ = − ÷ + =
h h h h a+b
a∼b
∴ sin θ = . ■
a+b
§ Problem 14.2.20. A tower 150 feet high stands on the top of a
cliff 80 feet high. At what point on the plane passing through the
foot of the cliff must an observer place himself so that the tower and
14.2. Angle Subtended at Two Points 248

the cliff may subtend equal angles, the height of his eye being 5 feet
? ♢
§§ Solution. Let A be the top of the tower, B and C be the top and
the foot of the cliff respectively, O be the position of the observer’s
eye and x feet be the required distance.
From O, draw OD perpendicular to BC, so that OD = x f eet.
Then, since OB bisects the ∠AOC, we have, by Euclid V I. 3,
OA AB
=
√ OC BC
OD 2 + AD2 150 15
∴ √ = =
x2 + 5 2 80 8
x2 + (150 + 80 − 5)2 225
∴ =
x 2 + 52 64
2
x + (225) 2 225
∴ =
x 2 + 52 64
∴ 161x2 = 225 × 25(64 × 9 − 1) = 225 × 25 × 575
∴ (7 × 23)x2 = 225 × (25)2 × 23
15 × 25 375
∴x= √ = √ f eet.
7 7
Otherwise thus :
Let E be the point on the ground vertically below O.
Let the ∠AOB = α = ∠BOC and the ∠ECO = θ = ∠COD.

We then have
5 75 225
= tan θ, = tan(α − θ) and = tan(2α − θ).
x x x
Now 2α − θ − θ = 2(α − θ)
∴ tan(2α − θ − θ) = tan 2(α − θ)
tan(2α − θ) − tan θ 2 tan(α − θ)
∴ =
1 + tan(2α − θ) tan θ 1 − tan2 (α − θ)
225 5 150

x x

225 5
= (x )2
75
1+ . 1−
x x x
220 150
∴ 2 = 2
x + 1125 x − 5625
375
∴ x = √ , as before. ■
7
§ Problem 14.2.21. A statue on the top of a pillar, standing on level
ground, is found to subtend the greatest angle α at the eye of an
observer when his distance from the pillar is c f eet ; prove that the
height of the statue is 2c tan α feet, and find the height of the pillar. ♢
§§ Solution. Let P and Q be the top and the foot of the pillar re-
spectively and let P R be the statue.
If D be the point at which the greatest angle is subtended by P R,
then D must be the point where a circle drawn through P and R
touches the ground.
Also, O the center of this circle (to which QD is a tangent) is in
the vertical line through D.
14.2. Angle Subtended at Two Points 249

Draw OM perpendicular to RP , bisecting it in M . Join OR and


OP .
We have OM = DQ = c; also the ∠ROP = 2 × ∠RDP = 2α, so that
the ∠ROM = α = ∠M OP .
∴ P R = P M + M R = 2.OM tan α = 2c tan α.
Also, ∵ OD = OR = c sec α and P M = c tan α
( )
1 sin α
∴ P Q = c(sec α − tan α) = c − = c(1 − sin α) sec α.
cos α cos α
Otherwise thus :
We may obtain this last result thus :
By Euclid III. 36,
QP.QR = QD2
∴ QP (QP + P R) = QD2
∴ QP 2 + QP.2c tan α = c2
( )
∴ QP 2 + QP.2c tan α + c2 tan2 α = c2 1 + tan2 α = c2 sec2 α
∴ QP = c(sec α − tan α) = c(1 − sin α) sec α. ■

§ Problem 14.2.22. A tower stood at the foot of an inclined plane


whose inclination to the horizon was 9◦ . A line 100 feet in length was
measured straight up the incline from the foot of the tower, and at
the end of this line the tower subtended an angle of 54◦ . Find the
height of the tower, having given
log 2 = .30103, log 114.4123 = 2.0584726,
and L sin 54◦ = 9.9079576. ♢
§§ Solution. Let AB (h feet, say) be the tower at the foot of the
inclined plane AC.
∠BAC = 90◦ − 9◦ = 81◦ .
Hence, if AC be the line 100 feet in length, so that
∠BCA = 54◦
∴ ∠ABC = 180◦ − (∠BAC + ∠BCA) = 180◦ − (81◦ + 54◦ ) = 45◦ .
We then have
h AC
=
sin 54◦ √sin 45◦
∴ h = 100 2 sin 54◦
1
∴ log h = log 100 + log 2 + L sin 54◦ − 10
2
= 2 + .150515 + 9.9079576 − 10
= 2.0584726 = log 114.4123.
Hence the required height = 114.4123 f eet. ■

§ Problem 14.2.23. A vertical tower stands on a declivity which is


inclined at 15◦ to the horizon. From the foot of the tower a man as-
cends the declivity for 80 feet, and then finds that the tower subtends
an angle of 30◦ . Prove that the
√ height
√ of the tower is
40( 6 − 2) f eet. ♢
§§ Solution. Let AB (h feet, say) be the tower and AC be the dis-
tance 80 feet.
We have ∠BAC = 90◦ − 15◦ = 75◦ and ∠BCA = 30◦
Also, ∠ABC = 180◦ − (75◦ + 30◦ ) = 75◦ .
14.2. Angle Subtended at Two Points 250

h 80
We then have =
sin 30◦√ sin 75◦ √
3+1 160 2
∴ 2h = 80 ÷ √ = √
√ √ 2 2 3+1
80 2( 3 − 1) √ √
∴h= = 40( 6 − 2) f eet. ■
3−1
§ Problem 14.2.24. The altitude of a certain rock is 47◦ , and after
walking towards it 1000 feet up a slope inclined at 30◦ to the horizon
an observer finds its altitude to be 77◦ . Find the vertical height of
the rook above the first point of observation, given that
sin 47◦ = .73135. ♢
§§ Solution. Let B and C be the top and the foot of the rock respec-
tively and A be the first point of observation, so that ∠BAC = 47◦ .
Let D be the second point of observation, so that
AD = 1000 f eet, ∠DAC = 30◦ and ∠BDE = 77◦ ,
where DE is parallel to AC, meeting BC in E.
The ∠BAD = 47◦ − 30◦ = 17◦ .
In the ∆BAC, C is a right angle and ∠BAC = 47◦
∴ ∠ABC = 90◦ − 47◦ = 43◦ .
In the ∆BDE, E is a right angle and ∠BDE = 77◦
∴ ∠DBE = 90◦ − 77◦ = 13◦ .
∴ ∠ABD = 43◦ − 13◦ = 30◦ , and
∠BDA = 180◦ − (17◦ + 30◦ ) = (180◦ − 47◦ )
From the ∆ABD, we have
AB AD
=
sin ∠BDA sin ∠ABD
AD sin 47◦
∴ AB = = 2000 sin 47◦ .
sin 30◦
From the ∆ABC, we have
BC = AB sin 47◦ = 2000 sin2 47◦ = 2000 × (.73135)2 = 1069.745645 f t.■

§ Problem 14.2.25. A man observes that when he has walked c feet


up an inclined plane the angular depression of an object in a hori-
zontal plane through the foot of the slope is α, and that, when he
has walked a further distance of c feet, the depression is β. Prove
that the inclination of the slope to the horizon is the angle whose
cotangent is
(2 cot β − cot α) . ♢
§§ Solution. Let O be the object, M be the foot of the inclined plane
and A and N be the two points of observation.
From A and N draw AP and N Q perpendicular to the horizontal
plane which contains O and M .
Draw AR and N S parallel to M O, so that AR and N S are the hor-
izontal lines passing through A and N .
We are given
∠RAO = α = ∠AOP, ∠SN O = β = ∠N OQ, and
M A = c f eet = AN.
Let the distance OM be x feet and the required ∠N M Q be θ(= ∠AM P ).
We then have
M P = P Q = c cos θ, AP = c sin θ and N Q = 2c sin θ.
14.2. Angle Subtended at Two Points 251

In the ∆OAP , we have OP = AP cot α,


∴ x + c cos θ = c sin θ cot α (14.11)
In the ∆ON Q, we have OQ = N Q cot β,
∴ x + 2c cos θ = c sin θ cot β (14.12)
(14.13)
From (14.11) and (14.12), by subtraction, we have
c cos θ = c sin θ (2 cot β − cot α) .
∴ cot θ = 2 cot β − cot α
∴ θ = cot−1 (2 cot β − cot α) . ■

§ Problem 14.2.26. A regular pyramid on a square base has an 150


feet long, and the length of the side of its base is 200 feet. Find the
inclination of its face to the base. ♢
§§ Solution. Let V be the vertex and F be the center of the base
ABCD of the pyramid.
Let E be the middle point of AB. We then have
200 √
AV = 150 f t., AE = 100 f t., AF = √ = 100 2 f t.,
√ √
2
and VF = V A2 − F A2 = 50 9 − 8 = 50 f t.
VF 50 1
∴ tan ∠V EF = = = ,
EF 100 2
1
i.e. the required inclination is the angle whose tangent is . ■
2

§ Problem 14.2.27. A pyramid has for base a square of side a ;


its vertex lies on a line through the middle point of the base and
perpendicular to it, and at a distance h from it ; prove that the angle
α between the two lateral faces is given by the equation

2h 2a2 + 4h2
sin α = .
a2 + 4h2 ♢
§§ Solution. Let V be the vertex and F be the center of the base
ABCD of the pyramid.
Draw BK and DK perpendicular to V A, so that α = ∠BKD ; and
let the ∠V AF = ϕ. ( )
a

α BF 2 1 VA
tan = = = =
2 FK AF sin ϕ sin ϕ VF

a2
√ + h2 √
F A2
+V F2 2 a2 + 2h2
= = = √
VF h h 2
α
2 tan
∴ sin α = 2
α
1 + tan2
√ 2 Å ã
2a2 + 4h2 a2 + 2h2
= ÷ 1+
h 2h2

2h 2a2 + 4h2
= . ■
a2 + 4h2
14.2. Angle Subtended at Two Points 252

§ Problem 14.2.28. A flagstaff, 100 feet high, stands in the center


of an equilateral triangle which is horizontal. From the top of the
flagstaff each side subtends an√angle of 60◦ ; prove that the length
of the side of the triangle is 50 6 f eet. ♢
§§ Solution. Let ABC be the triangle, D be its center and ED be
the flagstaff.
We have EA = EB = EC and the angles at E each 60◦ .
Therefore the triangles EBC, ECA and EAB are equilateral.
Let x f eet = BC = CA = AB = EA = EB = EC. We then have

2 2 x 3 x
AD = .x sin 60◦ = . = √ f eet.
3 3 2 3
Also, AD2 + DE 2 = AE 2
x2
∴ + (100)2 = x2
3
∴ 2x2 = 3 × (100)2

3 √
∴ x = 100 = 50 6 f eet.
2 ■

§ Problem 14.2.29. The extremity of the shadow of a flagstaff, which


is 6 feet high and stands on the top of a pyramid on a square base,
just reaches the side of the base and is distant 56 and 8 feet respec-
tively from the extremities of that side. Find the sun’s altitude if the
height of the pyramid be 34 feet. ♢
§§ Solution. Let ABCD be the square base and O be its center.
Let E be the top of the pyramid and EF be the flag staff and let α
be the sun’s altitude, so that
FO 6 + 34 40
tan α = = = ,
GO GO GO
where G is on AB, such that GA = 8 f eet and GB = 56 f eet.
To find GO, we have AOB a right-angled isosceles triangle,
64 √
AB = (56 + 8) f eet = 64 f eet and AO = OB = √ = 32 2 f eet.
2
Also, ∠GAO = 45◦ .
∴ GO2 = OA2 + AG2 − 2OA.AG cos 45◦
( √ )2 √ 1
= 32 2 + 82 − 2 × 32 2 × 8 × √
2
= 2048 + 64 − 512 = 1600
∴ GO = 40 f eet.
40
Hence tan α = = 1, so that the sun’s altitude = 45◦ . ■
40
§ Problem 14.2.30. The extremity of the shadow of a flagstaff, which
is 6 feet high and stands on the top of a pyramid on a square base,
just reaches the side of the base and is distant x feet and y feet re-
spectively from the ends of that side; prove that the height of the
pyramid is

x2 + y 2
tan α − 6,
2
where α is the elevation of the sun. ♢
14.2. Angle Subtended at Two Points 253

§§ Solution. Let ABCD be the square base and O be its center.


Let E be the top of the pyramid, EF be flagstaff and h be the
required height EO.
FO
We have tan α = , where G is on AB, such that AG = x feet
GO
and GB = y feet.
Thus GO = F O cot α = (6 + h) cot α.
Also, ∠GAO = 45◦ , and
x+y
AO = (x + y) cos 45◦ = √ .
2
∵ GO2 = OA2 + AG2 − 2OA.AG cos 45◦

(x + y)2 2(x + y)x 1
∴ (6 + h) cot α = + x2 − √ .√
2 2 2

x2 + 2xy + y 2
= + x2 − x2 − xy
2

x2 + y 2
=
2

x2 + y 2
∴h= tan α − 6. ■
2

§ Problem 14.2.31. The angle of elevation of a cloud from a point


h feet above a lake is α, and the angle of depression of its reflexion
in the lake is β ; prove that its height is
sin(β + α)
h .
sin(β − α) ♢
§§ Solution. Let C be the cloud and CD the perpendicular upon the
surface of the water.
Produce CD to L so that CD and DL are equal.
Then, by the laws of Optics, L is the image of C in the water.
Let O be the observer and draw OP perpendicular to CD.
Then ∠P OC = α and ∠P OL = β.
If x be the height of the cloud, we have CD = DL = x.
x−h CP OP tan α tan α
∴ = = =
x+h PD OP tan β tan β
x tan α + tan β sin(α + β)
∴ = = .
h tan β − tan α sin(β − α) ■

§ Problem 14.2.32. The shadow of a tower is observed to be half


the known height of the tower and sometime afterwards it is equal
to the known height ; how much will the sun have gone down in the
interval, given
log 2 = .30103, L tan 63◦ 26′ = 10.3009994,
and dif f. f or 1′ = 3159? ♢
§§ Solution. At the first observation, the elevation of the sun =
tan−1 2 = α, say, so that tan α = 2.
∴ L tan α = 10 + log 2 = 10.3010300.
( )
Let α = 63◦ 26′ + x′′ , so that L tan 63◦ 26′ + x′′ = 10.3010300.
The diff. for x′′ = 10.3010300 − 10.3009994 = .0000306.
14.2. Angle Subtended at Two Points 254

306
∴ x = 60′′ × ≈ 6′′
3159
◦ ′ ′′
∴ α = 63 26 6 .
At the second observation, the elevation of the sun = 45◦ .
Hence the required difference = 63◦ 26′ 6′′ − 45◦ = 18◦ 26′ 6′′ . ■

§ Problem 14.2.33. An isosceles triangle of wood is placed in a


vertical plane, vertex upwards, and faces the sun. If 2a be the base
of the triangle, h its height, and 30◦ the altitude of the sun, prove
that the tangent of the angle at the apex of the shadow is

2ah 3
.
3h2 − a2 ♢
§§ Solution. Let C be the vertex of the triangle and D be the middle
point of its base, so that AD = DB = a and CD = h.
Let E be the apex of the shadow, so that √
DE = h cot 30◦ = h 3.
We then have, if 2θ be the ∠BEA,
BD a
tan ∠BED = tan θ = = √
DE h 3
2 tan θ
∴ tan ∠BEA = tan 2θ =
Å1 − tan2 ã

2a a
= √ ÷ 1−
h 3 3h2

2ah 3
= . ■
3h2 − a2

§ Problem 14.2.34. A rectangular target faces due south, being


vertical and standing on a horizontal plane. Compare the area of
the target with that of its shadow on the ground when the sun is β ◦
from the south at an altitude of α◦ . ♢
§§ Solution. Let ACBD be the target and ACB ′ D′ be its shadow.
∠DD′ A = the sun’s altitude = α and ∠D′ AC = 90◦ − β.
We then have
area of target AC.AD AC.AD
= =
area of shadow AC.AD ′ sin(90◦ − β) AC.AD ′ cos β
AC.AD 1
= = = tan α sec β
AC.AD cot α cos β cot α cos β
∴ area of target : area of shadow = tan α sec β : 1. ■

§ Problem 14.2.35. A spherical ball, of diameter δ, subtends an


angle α at a man’s eye when the elevation of its center is β ; prove
that the height of the center of the ball is
1 α
δ sin β cosec . ♢
2 2
§§ Solution. Let O be the man’s eye and A be the center of the ball.
δ α
The angle subtended by at O = , and the angle subtended by
2 2
h, the height of the center of the ball, at A = β.
δ α
∴ h = OA sin β = cosec sin β. ■
2 2
14.2. Angle Subtended at Two Points 255

§ Problem 14.2.36. A man standing on a plane observes a row of


equal and equidistant pillars, the 10th and 17th of which subtend the
same angle that they would do if they were in the position of the first
1 1
and were respectively and of their height. Prove that, neglecting
2 3
the height of the man’s eye, the line of pillars is inclined to the line
drawn from his eye to the first at an angle whose secant is nearly
2.6. ♢
§§ Solution. Let A, B and C be the positions of the feet of the first,
the tenth and the seventeenth pillars respectively, and let d be the
distance between any two consecutive pillars, so that
AB = 9d and AC = 16d.
Let O be the position of the observer, h be the height of a pillar and α
and β be the angles subtended by the pillars at B and C respectively.
We then have
h h
OB = h cot α, OA = cot α, OC = h cot β and OA = cot β
2 3
∴ OB = 2.OA = 2a, say, and OC = 3.OA = 3a, say.
Let θ be the angle OA makes with CBA produced. We have
in the ∆OAB, (2a)2 = a2 + (9d)2 + 2a.9d cos θ,
in the ∆OAC, (3a)2 = a2 + (16d)2 + 2a.16d cos θ
∴ a2 = 27d2 + 6ad cos θ (14.14)
and a2 = 32d2 + 4ad cos θ (14.15)
From (14.14) and (14.15), by subtraction, we have
2a cos θ
0 = −5d2 + 2ad cos θ, so that d = .
5
Substituting this value of d in (14.15), we have
128a2 cos2 θ 8a2 cos2 θ
a2 = +
25 5
25
∴ cos2 θ =
168
168
∴ sec2 θ =
√ 25
2 42 4 × 6.4809
∴ sec θ = = ≈ 2.6. ■
5 10
For the following four examples a book of tables will be re-
quired.
§ Problem 14.2.37. A and B are two points, which are on the banks
of a river and opposite to one another, and between them is the mast,
P N , of a ship ; the breadth of the river is 1000 feet, and the angular
elevation of P at A is 14◦ 20′ and at B it is 8◦ 10′ . What is the height
of P above AB ? ♢
§§ Solution. Let h be the required height of P N .
AN
Then = cot 14◦ 20′ , ∴ AN = h cot 14◦ 20′ ,
h
BN
and = cot 8◦ 10′ , ∴ BN = h cot 8◦ 10′ .
h
Hence, by addition, we have
( )
AN + BN = AB = 1000 = h cot 14◦ 20′ + cot 8◦ 10′
14.2. Angle Subtended at Two Points 256

1000 1000
∴h= =
cot 14◦ 20′ + cot 8◦ 10′ 3.9136420 + 6.9682335
1000
= ≈ 91.896 f eet
10.8818755
Otherwise thus :
We have ( )
∠AP B = 180◦ − 14◦ 20′ + 8◦ 10′ = 157◦ 30′ .
PB sin ∠P AB PN
Now = and = sin ∠P BN.
AB sin ∠AP B PB
Hence, by multiplication, we have
PN sin ∠P AB sin ∠P BN
=
AB sin ∠AP B
∴ h = 1000 sin 14◦ 20′ sin 8◦ 10′ cosec 22◦ 30′
∴ log h = log 1000 + L sin 14◦ 20′ + L sin 8◦ 10′ + L cosec 22◦ 30′ − 30
= 3 + 9.3936852 + 9.1524507 + 10.4171603 − 30 = 1.9632962.
∴ h ≈ 91.896 f eet. ■

§ Problem 14.2.38. AB is a line 1000 yards long; B is due north of


A and from B a distant point P bears 70◦ east of north ; at A it bears
41◦ 22′ east of north ; find the distance from A to P . ♢
§§ Solution. We have
AP sin ∠ABP sin (180◦ − 70◦ ) sin 70◦
= = ◦ ◦ ′
=
AB sin ∠AP B sin (70 − 41 22 ) sin 28◦ 38′
1000 sin 70◦
∴ AP =
sin 28◦ 38′
∴ log AP = log 1000 + L sin 70◦ − L sin 28◦ 38′
= 3 + 9.9729858 − 9.6805191 = 3.2924667.
Now log 1960.9 = 3.2924554, dif f. f or .1 = .0000222.
Let log(1960.9 + x) = 3.2924667.
The diff. for x = 3.2924667 − 3.2924554 = .0000113.
113 11.3
∴x= × .1 = ≈ .05
222 222
∴ log AP = log(1960.9 + .05) = log 1960.95
∴ AP = 1960.95 yards. ■

§ Problem 14.2.39. A is a station exactly 10 miles west of B, The


bearing of a particular rock from A is 74◦ 19′ east of north, and its
bearing from B is 20◦ 51′ west of north. How far is it north of the line
AB ? ♢
§§ Solution. Let R denote the rock and let RQ be drawn perpen-
dicular to AB.
We have
∠RAQ = 90◦ − 74◦ 19′ = 15◦ 41′ , ∠RBA = 90◦ − 26◦ 51′ = 63◦ 9′
( )
∠ARB = 180◦ − 15◦ 41′ + 63◦ 9′ = 180◦ − 78◦ 50′ = 101◦ 10′ .
RQ
Now = sin ∠RAQ = sin 15◦ 41′
AR
AR sin ∠ABR sin 63◦ 9′ sin 63◦ 9′
and = = = .
AB sin ∠ARB sin 101◦ 10′ sin 78◦ 50′
14.2. Angle Subtended at Two Points 257

Hence by multiplication, we have


RQ sin 15◦ 41′ sin 63◦ 9′
=
AB sin 78◦ 50′
10 sin 15◦ 41′ sin 63◦ 9′
∴ RQ =
sin 78◦ 50′
log RQ = log 10 + L sin 15◦ 41′ + L sin 63◦ 9′ − 10 − L sin 78◦ 50′
= 1 + 9.4318788 + 9.9504583 − 10 − 9.9916991 = .3906380.
Now log 2.4583 = .3906349, dif f. f or .0001 = .0000176.
Let log(2.4583 + x) = .3906380.
The diff. for x = .3906380 − .3906349 = .0000031.
31 .0031
∴x= × .0001 = ≈ .00002,
176 176
Hence, the required distance = 2.45832 miles.
Otherwise thus :
We have
AQ = RQ cot ∠RAQ = RQ cot 15◦ 41′ and
BQ = RQ cot ∠RBQ = RQ cot 63◦ 9′ .
Hence, by addition,
( )
AQ + BQ = RQ cot 15◦ 41′ + cot 63◦ 9′
and, by the table of natural cotangents, we have
10 = RQ(3.5615900 + .5062322) = RQ(4.0678222)
10
∴ RQ = = 2.45832 miles.
4.0678222
Notes :

cos A cos B
∵ cot A + cot B = +
sin A sin B
sin B cos A + cos B sin A sin(B + A)
= = ,
sin A sin B sin A sin B
◦ ′
10 sin 15 41 sin 63 9◦ ′
We have RQ = , as in the first solution. ■
sin 78◦ 50′
§ Problem 14.2.40. The summit of a spire is vertically over the mid-
dle point of a horizontal square enclosure whose side is of length a
feet ; the height of the spire is h feet above the level of the square.
If the shadow of the spire just reach a corner of the square when the
sun has an altitude θ, prove that√
h 2 = a tan θ.
Calculate h, having given a = 1000 f eet and θ = 25◦ 15′ . ♢
§§ Solution. We have
h
= tan θ
a cos 45◦
a tan θ
∴ h = a tan θ cos 45◦ = √
√ 2
∴ h 2 = a tan θ.
If a = 1000 f eet, and θ = 25◦ 15′ , we have
1
log h + log 2 = log a + L tan θ − 10.
2
1
∴ log h = log 1000 + L tan 25◦ 15′ − 10 − log 2
2
14.2. Angle Subtended at Two Points 258

= 3 + 9.6736020 − 10 − .1505150 = 2.5230870.


Now log 333.49 = 2.5230828, dif f. f or .01 = .0000130.
Let log(333.49 + x) = 2.5230870.
The diff. for x = 2.5230870 − 2.5230828 = .0000042.
42 .42
∴x= × .01 = ≈ .0032
130 130
∴ h = 333.4932 f eet. ■
Chapter 15
Properties of A Triangle

15.1 Area of a Given Triangle


Find the area of the triangle ABC when
§ Problem 15.1.1. a = 13, b = 14 and c = 15. ♢
13 + 14 + 15
§§ Solution. s = = 21, s − a = 8, s − b = 7 and s − c = 6
√ 2 √
∴ ∆ = 21 × 8 × 7 × 6 = 42 × 4 × 42 = 42 × 2 = 84. ■

§ Problem 15.1.2. a = 18, b = 24 and c = 30. ♢


18 + 24 + 30
§§ Solution. s = = 36, s−a = 18, s−b = 12 and s−c = 6
√ 2 √
∴ ∆ = 36 × 18 × 12 × 6 = 36 × 36 × 36 = 6 × 6 × 6 = 216. ■

§ Problem 15.1.3. a = 25, b = 52 and c = 63. ♢


25 + 52 + 63
§§ Solution. s = = 70, s−a = 45, s−b = 18 and s−c = 7
√ 2 √
∴ ∆ = 70 × 45 × 18 × 7 = 7 × 2 × 5 × 45 × 18 × 7

= 49 × 36 × 225 = 7 × 6 × 15 = 630. ■

§ Problem 15.1.4. a = 125, b = 123 and c = 62. ♢


125 + 123 + 62
§§ Solution. s = = 155, s − a = 30, s − b = 32 and s −
2
c = 93 √ √
∴ ∆ = 155 × 30 × 32 × 93 = 31 × 5 × 15 × 2 × 32 × 31 × 3

= 31 × 31 × 15 × 15 × 64 = 31 × 15 × 8 = 3720. ■

§ Problem 15.1.5. a = 15, b = 36 and c = 39. ♢


15.1. Area of a Given Triangle 260

15 + 36 + 39
§§ Solution. s = = 45, s − a = 30, s − b = 9 and s − c = 6
√ 2 √
∴ ∆ = 45 × 30 × 9 × 6 = 270 × 270 = 270. ■

§ Problem 15.1.6. a = 287, b = 816 and c = 865. ♢


287 + 816 + 865
§§ Solution. s = = 984, s−a = 697, s−b = 168 and s−
2
c = 119 √
∴ ∆ = 984 × 697 × 168 × 119

= 24 × 41 × 17 × 41 × 24 × 7 × 17 × 7
= 24 × 41 × 17 × 7 = 117096. ■

§ Problem 15.1.7. a = 35, b = 84 and c = 91. ♢


35 + 84 + 91
§§ Solution. s = = 105, s−a = 70, s−b = 21 and s−c =
2
14 √
∴ ∆ = 105 × 70 × 21 × 14

= 15 × 7 × 7 × 10 × 7 × 3 × 7 × 2

= 7 × 7 × 7 × 7 × 900 = 7 × 7 × 30 = 1470. ■

√ √
√ √ 6+ 2
§ Problem 15.1.8. a = 3, b = 2 and c = . ♢
√ √ √2
2 3+3 2+ 6
§§ Solution. ∴ 2s = a + b + c =
√ √ √ 2
2 3+3 2+ 6
∴s=
√ √4 √
3 2+ 6−2 3
s−a=
√ √4 √
2 3− 2+ 6
s−b= and
√ 4
√ √
2 3+ 2− 6
s−c=
1
»[(4 √ √ ) √ ] [( √ √ ) √ ]
∴∆= 3 2+ 6 +2 3 3 2+ 6 −2 3 ×
»[16√ (√ √ )] [ √ (√ √ )]
2 3+ 6− 2 2 3− 6− 2
1
»( √ )( √ )
= 24 + 12 3 − 12 12 − 8 + 4 3
16 √
1 √ √ 1√ √ √
= 4(3 + 3 3).4(1 + 3) = (1 + 33.(1 + 3)
16 4
1 √ √ 1 √ 1
= (1 + 3) 3 = (3 + 3) = × 4.732 . . . = 1.183 . . . . ■
4 4 4

§ Problem 15.1.9. If B = 45◦ , C = 60◦ and √ a = 2( 3 + 1) inches,
prove that the area of the triangle is 6 + 2 3 sq. inches. ♢

§§ Solution. B = 45◦ , C = 60◦ , a = 2( 3 + 1) ins., A = 180◦ − (B +
C) = 75◦
1 1 a sin B
∴ ∆ = ab sin C = a. . sin C
2 2 sin A
a2 sin B sin C
= .
2 sin A
15.1. Area of a Given Triangle 261


1 3
√ .
√ 2 2
= 2(4 + 2 3) Å √ ã
3+1

2 2
√ √ √
= 2( 3 + 1) 3 = (6 + 2 3) sq. ins. ■

§ Problem 15.1.10. The sides of a triangle are 119, 111, and 92


yards; prove that its area is 10 sq. yards less than an acre. ♢
119 + 111 + 92
§§ Solution. s = = 161 yards, s − a = 42 yards,
2
s − b = 50 yards, and
√ s − c = 69 yards
∴ ∆ = 161 × 42 × 50 × 69 sq. yds.

= 23 × 7 × 14 × 3 × 25 × 2 × 23 × 3 sq. yds.

= 23 × 23 × 14 × 14 × 3 × 3 × 25 sq. yds.
= (23 × 14 × 3 × 5) sq. yds. = 4830 sq. yds.
= 10 sq. yds. less than an acre. ■

§ Problem 15.1.11. The sides of a triangular field are 242, 1212 and
1450 yards; prove that the area of the field is 6 acres. ♢
242 + 1212 + 1450
§§ Solution. s = = 1452 yards
2
s − a = 1210 yards, s − b = 240 yards and s − c = 2 yards

∴∆= 1452 × 1210 × 240 × 2 sq. yds.

= 11 × 11 × 12 × 11 × 11 × 10 × 8 × 3 × 10 × 2 sq. yds.

= 11 × 11 × 11 × 11 × 36 × 10 × 10 × 16 sq. yds.
= (11 × 11 × 6 × 10 × 4) sq. yds.
= (4840 × 6) sq. yds.
= 6 acres. ■

§ Problem 15.1.12. A workman is told to make a triangular enclo-


sure of sides 50, 41 and 21 yards respectively ; having made the first
side one yard too long, what length must he make the other two sides
in order to enclose the prescribed area with the prescribed length
of fencing ? ♢
50 + 41 + 21
§§ Solution. We have s = = 56 yards.
2
Also, 51 + b + c = 112
∴ b + c = 61 yards (15.1)
√ √
s(s − 51)(s − b)(s − c) = s × 6 × 15 × 35
[ ]
∴ 5 s2 − (b + c)s + bc = 6 × 15 × 35
∴ s2 − (b + c)s + bc = 6 × 3 × 35
∴ s(s − 61) + bc = 6 × 3 × 35
∴ 56(−5) + bc = 630
∴ bc = 630 + 280 = 910
∴ (b − c)2 = (b + c)2 − 4bc = 3721 − 3640 = 81.
∴ b − c = 9 yards (15.2)
From (15.1) and (15.2), we have b = 35 yds. and c = 26 yds. ■
15.1. Area of a Given Triangle 262

§ Problem 15.1.13. Find, correct to .0001 of an inch, the length of


one of the equal sides of an isosceles triangle on a base of 14 inches
having the same area as a triangle whose sides are 13.6, 15 and 15.4
inches. ♢
§§ Solution. If a be the length of one of the equal sides of the isosce-
les triangle, we have
1
s = (13.6 + 15 + 15.4) = 22 ins., and
2
1
s′ = (a + a + 14) = a + 7.
2
Equating areas, we have
√ √
(a + 7) × 7 × 7 × (a − 7) = 22 × 8.4 × 7 × 6.6
∴ a2 − 7 = 22 × 1.2 × 6.6
∴ a2 = 49 + 174.24 = 223.24
∴ a = 14.941 . . . ins. ■

§ Problem 15.1.14. Prove that the area of a triangle is


1 2 sin B sin C
a .
2 sin A √
If one angle of a triangle be 60◦ , the area 10 3 square feet, and the
perimeter 20 feet, find the lengths of the sides. ♢
§§ Solution.
1
∆ = ab sin C
2
1 a sin B
= a. . sin C
2 sin A
2
a sin B sin C
= . .
√ 2 sin A
Given A = 60◦ , ∆ = 10 3 sq. f t. and a + b + c = 20 f t., we have
1 √
bc sin A = 10 3
2 √
1 3 √
∴ bc. = 10 3
2 2
∴ bc = 40 (15.3)
Also, a2 = b2 + c2 − 2bc cos 60◦
= b2 + c2 − bc = (b + c)2 − 3bc
= (20 − a)2 − 3 × 40 = 280 − 40a + a2
∴ 40a = 280, ∴ a = 7 f eet.
∴ b + c = 20 − 7 = 13 f eet (15.4)
From (15.3) and (15.4), we have b = 8 f eet and c = 5 f eet. ■

§ Problem 15.1.15. The sides of a triangle are in A. P. and its area


3
is th of an equilateral triangle of the same perimeter ; prove that
5
its sides are in the ratio 3 : 5 : 7, and find the greatest angle of the
triangle. ♢
§§ Solution. Let x − y, x and x + y denote the sides of the triangle.
3x
Then 2s = x − y + x + x + y = 3x, ∴ s = ,
2
15.1. Area of a Given Triangle 263


3x Ä x ä Äx ä x√
and ∆= +y x −y = 3 (x2 − 4y 2 ).
2 2 2 4
Also, the area of an equilateral triangle of the same perimeter, i.e.
with side x is √ 2
1 3.x
= x2 sin 60◦ = .
2 4√
x √ 3 3.x2
∴ 3 (x2 − 4y 2 ) = .
4 ( ) 5 4
∴ 25 x2 − 4y 2 = 9x2
∴ 16x2 = 100y 2
5y
∴x= .
2
Thus the sides are in the ratio
3y 5y 7y
, , , i.e. 3 : 5 : 7.
2 2 2
Also, if θ be the greatest angle, we have
32 + 52 − 72 9 + 25 − 49 15 1
cos θ = = =− =−
2.3.5 30 30 2
∴ θ = 120◦ . ■

§ Problem 15.1.16. In a triangle the least angle is 45◦ and the tan-
gents of the angles are in A. P . If its
√ area√ be 3 square yards, prove
that the lengths of the sides are 3 5, 6 2 and 9 feet, and that the
tangents of the other angles are respectively 2 and 3. ♢
§§ Solution. In the triangle ABC, let the ∠ = 45◦ and let
tan A = 1, tan B = 1 + d, tan C = 1 + 2d.
We have
B + C = 180◦ − A = 135◦ , ∴ C = 135◦ − B
−1 − tan B
∴ tan C =
1 − tan B
−1 − 1 − d 2+d
∴ 1 + 2d = =
1−1−d d
∴ d + 2d2 = 2 + d, ∴ 2d2 = 2, ∴ d = 1
∴ tan B = 2 and tan C = 3
1 2 3
∴ sin A = √ , sin B = √ and sin C = √ .
2 5 10
1 sin B sin C
∵ ∆ = a2
2 sin A
1
2×3×9× √
2 √
∴a =
2
= 9 × 5; ∴ a = 3 5 f eet.
2 3
√ ×√
5 10
a sin B √ 2 √ √
Also, b= = 3 5 × √ × 2 = 6 2 f eet
sin A 5
a sin C √ 3 √
and c= = 3 5 × √ × 2 = 9 f eet.
sin A ■
10

§ Problem
√ 15.1.17. The lengths of two sides of a triangle are one
foot and 2 feet respectively, and the angle opposite side the shorter
15.2. The Circles Connected With A Triangle 264

side is 30◦ ; prove that there are two triangles satisfying these con-
ditions, find their angles, and√ show√that their areas are in the ratio
3 + 1 : 3 − 1. ♢

§§ Solution. Given a = 1 f t., b = 2 f t., A = 30◦ , we have
√ 1
b sin A 2×
sin B = = 2 = √1
a 1 2
∴ B1 = 45◦ , C1 = 105◦ ; B2 = 135◦ , C2 = 15◦ .
1 √
∆1 ab sin C1 sin C1 sin 105◦ 3+1
∴ = 2 = = = √ .
∆2 1 sin C2 sin 15◦ 3−1
ab sin C2 ■
2
§ Problem 15.1.18. Find by the aid of the tables the area of the
larger of the two triangles given by the data
A = 31◦ 15′ , a = 5 ins. and b = 7 ins. ♢
§§ Solution. We have
b 7 14
sin B = sin A = sin A = sin 13◦ 15′
a 5 10
∴ L sin B = log 14 + L sin 31◦ 15′ − log 10
= 1.1461280 + 9.7149776 − 1 = 9.8611056.
Now L sin 46◦ 34′ = 9.8610412, dif f. f or 1′ = 1196.
Let B = 46◦ 34′ + x′′
( )
∴ L sin 46◦ 34′ + x′′ = 9.8611056.
The diff.for x′′ = 9.8611056 − 9.8610412 = .0000644.
644
∴ x = 60′′ × ≈ 32′′ , ∴ B = 46◦ 34′ 32′′
1196
and C = 180◦ − (A + B) = 180◦ − 77◦ 49′ 32′′ = 102◦ 10′ 28′′ .
1
∴ the required area = ab sin C
2
1 35
= × 5 × 7 × sin 77◦ 49′ 32′′ = sin 77◦ 49′ 32′′
2 2
∴ log(area) = log 35 + L sin 77◦ 49′ 32′′ − 10 − log 2.
32
Now L sin 77◦ 49′ 32′′ = 9.9901067 + × .0000272 = 9.9901212
60
∴ log(area) = 1.5440680 + 9.9901212 − 10 − .3010300 = 1.2331592.
Now log 17.106 = 1.2331485, dif f. f or .001 = .0000254.
Let log(17.106 + x) = 1.2331592.
The diff. for x = 1.2331592 − 1.2331485 = .0000107.
107 .107
∴x= × .001 = ≈ .0004.
254 254
∴ log(area) = log(17.106 + .0004)
∴ the required area = 17.1064 sq. ins. ■

15.2 The Circles Connected With A Triangle


§ Problem 15.2.1. In a triangle whose sides are 18, 24, and 30 inches
respectively, prove that the circumradius, the inradius and the radii
15.2. The Circles Connected With A Triangle 265

of the three escribed circles are respectively 15, 6, 12, 18 and 36


inches. ♢
§§ Solution.
18 + 24 + 30
s= = 36 ins., s − a = 18 ins.,
2
s − b = 12 ins., and s − c = 6 ins.
√ √
S = 36 × 18 × 12 × 6 = 36 × 36 × 36 = 6 × 6 × 6 = 216 sq. ins.
18 × 24 × 30
∴R= ins = 15 ins.
4×6×6×6
6×6×6
r= ins. = 6 ins.
36
6×6×6
r1 = ins. = 12 ins.
18
6×6×6
r2 = ins. = 18 ins.
12
6×6×6
r3 = ins. = 36 ins. ■
6
§ Problem 15.2.2. The sides of a triangle are 13, 14 and 15 feet ;
prove that
1
(1) R = 8 f t.
8
(2) r = 4 f t.
1
(3) r1 = 10 f t.
2
(4) r2 = 12 f t. and

(5) r3 = 14 f t.

§§ Solution.
13 + 14 + 15
s= = 21 ins., s − a = 8 ins.,
2
s − b = 7 ins., and s − c = 6 ins.
√ √
S = 21 × 8 × 7 × 6 = 42 × 4 × 42 = 42 × 2 = 84 sq. f t.
13 × 14 × 15 65 1
(1) R = f t. = f t. = 8 f t.
4 × 42 × 2 8 8
42 × 2
(2) r = f t. = 4 f t.
21
42 × 2 21 1
(3) r1 = f t. = = 10 f t.
8 2 2
42 × 2
(4) r2 = f t. = 12 f t.
7
42 × 2
(5) r3 = f t. = 14 f t.
6 ■

5
§ Problem 15.2.3. In a ∆ABC if a = 13, b = 4 and cos C = − , find
13
R, r, r1 , r2 and r3 . ♢
15.2. The Circles Connected With A Triangle 266

5
§§ Solution. a = 13, b = 4 and cos C = −
13
∴ c2 = a2 + b2 − 2ab cos C
5
= 169 + 16 + 2 × 13 × 4 × = 185 + 40 = 225
13
∴ c = 15
13 + 4 + 15
∴s= = 16, s − a = 3, s − b = 12 and s − c = 1
2√ √
∴ S = 16 × 3 × 12 × 1 = 16 × 36 = 4 × 6 = 24
13 × 4 × 15 65 1
∴R= = =8
4×4×6 8 8
24 3 1
r= = =1
16 2 2
24
r1 = =8
3
24
r2 = = 2 and
12
24
r3 = = 24. ■
1
§ Problem 15.2.4. In the ambiguous case of the solution of triangles
prove that the circumcircles of the two triangles are equal. ♢
a
§§ Solution. Given a, b and A, we have R = , which is the
2 sin A
same for both triangles.
For the two triangles have the same value for a and the same
value of A. ■

Prove that
§ Problem 15.2.5. r1 (s − a) = r2 (s − b) = r3 (s − c) = rs = S. ♢
§§ Solution. By Arts. 202 and 205, we have
S S S S
r1 = , r2 = , r3 = and r =
s−a s−b s−c s
∴ r1 (s − a) = S = r2 (s − b) = r3 (s − c) = rs. ■

rr1 A
§ Problem 15.2.6. = tan2 . ♢
r2 r3 2
§§ Solution.[( ) ( )]
rr1 S S S S
= . ÷ .
r2 r3 s s−a s−b s−c
S2 (s − b)(s − c) (s − b)(s − c) A
= × = = tan2 . ■
s(s − a) S2 s(s − a) 2
§ Problem 15.2.7. rr1 r2 r3 = S 2 . ♢
§§ Solution.
S4
rr1 r2 r3 =
s(s − a)(s − b)(s − c)
S4
= 2 = S2. ■
S
A B C
§ Problem 15.2.8. r1 r2 r3 = r3 cot2 cot2 cot2 . ♢
2 2 2
15.2. The Circles Connected With A Triangle 267

r1 s r2 s r3 s
§§ Solution. We have = , = and = .
r s−a r s−b r s−c
r1 r2 r3 s3
∴ =
r3 (s − a)(s − b)(s − c)
s(s − a) s(s − b) s(s − c)
= · ·
(s − b)(s − c) (s − c)(s − a) (s − a)(s − b)
A B C
= cot2 cot2 cot2 .
2 2 2
A B C
∴ r1 r2 r3 = r3 cot2 cot2 cot2 . ■
2 2 2
A
§ Problem 15.2.9. rr1 cot = S. ♢
2
§§ Solution. …
A S S s(s − a)
∴ rr1 cot = ·
2 s s−a (s − b)(s − c)

s(s − a)(s − b)(s − c) s(s − a)
=
s(s − a) (s − b)(s − c)

= s(s − a)(s − b)(s − c) = S. ■

§ Problem 15.2.10. r2 r3 + r3 r1 + r1 r2 = s2 . ♢
§§ Solution. ( )
1 1 1 1 1 1
r1 r2 + r2 r3 + r3 r1 = S 2 · + · + ·
[ s − a s − b s − b] s − c s − c s − a
s−c+s−a+s−b
= S2
[ (s − a)(s − b)(s − c) ]
2 3s − (a + b + c)
=S
(s − a)(s − b)(s − c)
= s(3s − 2s) = s2 .
Otherwise thus :
A B C
r1 = s tan , r2 = s tan and r3 = s tan .
2 2 2
By § Problem 9.2.12, we have
A B B C C A
tan tan + tan tan + tan tan = 1.
2 2 2 2 2 2
r1 r2 r2 r3 r3 r1
∴ · + · + · =1
s s s s s s
∴ r1 r2 + r2 r3 + r3 r1 = s . 2 ■

1 1 1 1
§ Problem 15.2.11. + + − = 0. ♢
r1 r2 r3 r
§§ Solution.
1 1 1 1 s−a s−b s−c s
+ + − = + + −
r1 r2 r3 r S S S S
1 1
= [3s − (a + b + c) − s] = (3s − 2s − s) = 0. ■
S S
§ Problem 15.2.12. a (rr1 + r2 r3 ) = b (rr2 + r3 r1 ) = c (rr3 + r1 r2 ).

15.2. The Circles Connected With A Triangle 268

§§ Solution. ï ò
S2 S2
a (rr1 + r2 r3 ) = a +
s(s − a) (s − b)(s − c)
= a[(s − b)(s − c) + s(s − a)]
[ ]
= a 2s2 − (a + b + c)s + bc
( )
= a 2s2 − 2s2 + bc = abc. ■

C C
§ Problem 15.2.13. (r1 + r2 ) tan = (r3 − r) cot = c. ♢
2 2
§§ Solution. ( )
C A B C
(r1 + r2 ) tan = s tan
+ s tan tan
2 Ñ A 2
B
2 é 2C
sin sin sin
=s 2 + 2 2
A B C
cos cos cos
Ñ 2
A+B
é2
C
2
sin sin
=s 2 2
A B C
cos cos cos
2 2 2
C C
s sin (a + b + c) sin
= 2 = 2
A B A B
cos cos 2 cos cos
2 2 2 2
C
sin A + sin B + sin C sin
=c· · 2
sin C A B
2 cos
cos
2 2
A B C C
4 cos cos cos sin
=c· 2 2 2 · 2 = c.
C C A B
2 sin cos 2 cos cos
2 2 2 2
C
Similarly, (r3 − r) cot = c.
2
Otherwise thus :

B C C A
a coscos b cos cos
r1 + r2 = 2 2 + 2 2
A B
cos cos
2 2
A B C B C A
= 4R sin cos cos + 4R sin cos cos (Art. 207, Cor.)
2 2 2 2 2 2
C A+B C
= 4R cos · sin = 4R cos2
2 2 2
C C C
∴ (r1 + r2 ) tan = 4R sin cos = 2R sin C = c
2 2 2
A B A B
c cos cos c sin sin
Again, r3 − r = 2 2 − 2 2
C C
cos cos
2 2
15.2. The Circles Connected With A Triangle 269

C
c A+B c sin
= · cos = 2 = c tan C
C 2 C 2
cos cos
2 2
C C C
∴ (r3 − r) cot = c tan · cot = c.
2 2 2
]…
Otherwise thus :
[
C S S (s − a)(s − b)
(r1 + r2 ) tan = +
2 s−a s−b s(s − c)
[ ]…
2s − (a + b) (s − a)(s − b)
=S
(s − a)(s − b) s(s − c)
Sc
= √ = c.
s(s − a)(s − b)(s − c)
[ ]…
C S S s(s − c)
Similarly, (r3 − r) cot = −
2 s−c s (s − a)(s − b)

Sc s(s − c)
= = c.
s(s − c) (s − a)(s − b) ■

§ Problem 15.2.14. S = 2R2 sin A sin B sin C. ♢


§§ Solution.
1 1
S = a · b sin C = (2R sin A · 2R sin B · sin C)
2 2
= 2R2 sin A sin B sin C. ■

§ Problem 15.2.15. 4R sin A sin B sin C = a cos A + b cos B + c cos C. ♢


§§ Solution.
4R sin A sin B sin C = R (sin 2A + sin 2B + sin 2C) , by Art. 127, Ex. 1
= 2R sin A cos A + 2R sin B cos B + 2R sin C cos C
= a cos A + b cos B + c cos C,
∵ a = 2R sin A, b = 2R sin B, c = 2R sin C, by Art. 200. ■

A B C
§ Problem 15.2.16. S = 4Rr cos cos cos . ♢
2 2 2
§§ Solution.
1 1
S = rs = r · (a + b + c) = r · (2R sin A + 2R sin B + 2R sin C)
2 2
= Rr (sin A + sin B + sin C)
A B C
= 4Rr cos cos cos , by §P roblem 9.2.4.
2 2 2
Otherwise thus : … … …
A B C abc S s(s − a) s(s − b) s(s − c)
4Rr cos cos cos =4· ·
2 2 2 4S s bc ca ab

= s(s − a)(s − b)(s − c) = S. ■

1 1 1 1 a2 + b2 + c2
§ Problem 15.2.17. 2
+ 2 + 2 + 2 = . ♢
r r1 r2 r3 S2
15.2. The Circles Connected With A Triangle 270

§§ Solution.
1 1 1 1 s2 (s − a)2 (s − b)2 (s − c)2
+ 2 + 2 + 2 = 2 + + +
r2 r1 r2 r3 S S2 S2 S2
4s2 − 2(a + b + c)s + a2 + b2 + c2
=
S2
4s2 − 4s2 + a2 + b2 + c2 a2 + b2 + c2
= = . ■
S2 S2
§ Problem 15.2.18. r1 + r2 + r3 − r = 4R. ♢
§§ Solution. [ ]
1 1 1 1
r1 + r2 + r3 − r = S + + −
[ s − a s − b s − c s]
s−b+s−a s−s+c
=S +
(s − a)(s − b) s(s − c)
[ ]
2s − (a + b) c
=S +
[(s − a)(s − b) s(s − c) ]
1 1
= cS +
(s − a)(s − b) s(s − c)
cS
= 2 [s(s − c) + (s − a)(s − b)]
S[ ]
c
= 2s2 − (a + b + c)s + ab
S
c [ 2 ] abc
= 2s − 2s2 + ab = = 4R.
S S
Otherwise thus :
[
A B C B C A
r1 + r2 + r3 − r = 4R sin cos cos + sin cos cos
2 2 2 2 2 2 ]
C A B A B C
+ sin cos cos − sin sin sin
2 2 2 2 2 2
1
= 4R sin (A + B + C), by Art. 124
2
= 4R sin 90◦ = 4R. ■

§ Problem 15.2.19. (r1 − r) (r2 − r) (r3 − r) = 4Rr2 . ♢


§§ Solution.
(r1 − r) (r2 − r) (r3 − r)
[ ][ ]
A A B B
= s tan − (s − a) tan s tan − (s − b) tan ×
[2 2 ] 2 2
C C
s tan − (s − c) tan
2 2
A B C
= a tan · b tan · c tan
…2 2 … 2 …
(s − b)(s − c) (s − c)(s − a) (s − a)(s − b)
= abc
s(s − a) s(s − b) s(s − c)
abc √ abc abc S 2
= 2 s(s − a)(s − b)(s − c) = 2 · S = 4 · · = 4Rr2 .
s s 4S s2
Otherwise thus :
( )( )( )
S S S S S S
(r1 − r) (r2 − r) (r3 − r) = − − −
s−a s s−b s s−c s
15.2. The Circles Connected With A Triangle 271

aS bS cS
= · ·
s(s − a) s(s − b) s(s − c)
abcS abcr2
= 2
= = 4Rr2 . ■
s S
1 1 1 1
§ Problem 15.2.20. + + = . ♢
bc ca ab 2Rr
§§ Solution.
1 1 1 c+a+b
+ + =
ab bc ca abc
2s
=
abc
2S
= ÷ 4RS
r
1
= . ■
2Rr
r1 r2 r3 1 1
§ Problem 15.2.21. + + = − . ♢
bc ca ab r 2R
§§ Solution.
r1 r2 r3 ar1 + br2 + cr3
+ + =
bc ca[ ab abc ]
1 aS bS cS
= + +
abc [ s − a s−b s−c ]
S a(s − b)(s − c) + b(s − a)(s − c) + c(s − a)(s − b)
=
abc (s − a)(s − b)(s − c)
s [ ]
= (a + b + c)s2 − {a(b + c) + b(a + c) + c(a + b)} s + 3abc
abcS [ ]
s
= 2s3 − 2s(bc + ca + ab) + 3abc
abcS [ ]
s 2s −s3 + s(bc + ca + ab) − abc
= −
S [ abcS ]
1 2s −s3 + (s − a)(s − b)(s − c) − s3 + (a + b + c)s2
= − ,
{ r abcS }
∵ (s − a)(s − b)(s − c) = s − (a + b + c)s + (ab + bc + ca)s − abc
3 2

1 2s(s − a)(s − b)(s − c)


= −
r abcS
1 2S
= −
r abc
1 1
= − .
r 2R
Otherwise thus :
B C
r1 a cos cos
= 2 2
bc A
bc cos
2
B C
2R sin A cos cos
= 2 2
A
2R sin B · 2R sin C cos
2
15.3. Orthocenter and Pedal Triangle 272

A
1 sin
= · 2 .
4R B C
sin sin
 A
2 2
B C

sin sin sin

r1
+
r2
+
r3
=
1  2
+ + 2 2 
bc ca ab 4R B C C A A B
sin sin sin sin sin sin
 2 2 2 2  2 2
sin 2 A + sin2 B + sin2 C
1  2 2 2 
=
4R A B C
sin sin sin
2 2 2
A B C
1 − 2 sin sin sin
= 2 2 2 , by §P roblem 9.2.10
A B C
4R sin sin sin
2 2 2
1 1
= −
A B C 2R
4R sin sin sin
2 2 2
1 1
= − , by Art. 204, Cor. ■
r 2R
§ Problem 15.2.22. r2 + r12 + r22 + r32 = 16R2 − a2 − b2 − c2 . ♢
§§ Solution.
r2 + r12 + r22 + r32
= (r1 + r2 + r3 − r)2 + 2r (r1 + r2 + r3 ) − 2 (r2 r3 + r3 r1 + r1 r2 ) .
Now r1 + r2 + r3 − r = 4R, by §P roblem 15.2.18,
and r2 r3 + r3 r1 + r1 r2 = s2 , by §P roblem 15.2.10.
Also,
ï ò
S2 S2 S2
2r (r1 + r2 + r3 ) = 2 + +
s(s − a) s(s − b) s(s − c)
= 2[(s − b)(s − c) + (s − c)(s − a) + (s − a)(s − b)]
[ ]
= 2 3s2 − 2(a + b + c)s + bc + ca + ab
= −2s2 + 2(bc + ca + ab).
∴ r2 + r12 + r22 + r32 = 16R2 − 2s2 + 2(bc + ca + ab) − 2s2
[ ]
= 16R2 − (a + b + c)2 − 2(bc + ca + ab)
= 16R2 − a2 − b2 − c2 . ■

15.3 Orthocenter and Pedal Triangle


If I, I1 , I2 , and I3 be respectively the centers of the incircle
and the three escribed circles of a triangle ABC, prove that
A
§ Problem 15.3.1. AI = r cosec . ♢
2
§§ Solution. Taking the figure of Art. 202, we have
IE A
= sin
AI 2
A
∴ AI = r cosec . ■
2
15.3. Orthocenter and Pedal Triangle 273

A B C
§ Problem 15.3.2. IA · IB · IC = abc tan tan tan . ♢
2 2 2
§§ Solution. With the same figure, we have
A B C
IA · IB · IC = r cosec · r cosec · r cosec
2 2 2
B C C A A B
a sin sin b sin sin c sin sin
= 2 2 · 2 2 · 2 2
A A B B C C
cos sin cos sin cos sin
2 2 2 2 2 2
A B C
= abc tan tan tan . ■
2 2 2
A
§ Problem 15.3.3. AI1 = r1 cosec . ♢
2
§§ Solution. Taking the figure of Art. 205, we have
I1 E1 A
= sin
AI1 2
A
∴ AI1 = r1 cosec . ■
2
A
§ Problem 15.3.4. II1 = a sec . ♢
2
§§ Solution.
A A
II1 = AI1 − AI = r1 cosec − r cosec
[ 2 ] 2
A A A
= s tan − (s − a) tan cosec
2 2 2
A
= a sec . ■
2
A
§ Problem 15.3.5. I2 I3 = a cosec . ♢
2
§§ Solution. If E2 be the point of contact of the circle whose center
is I2 with the side AC of the triangle ABC, we ( have )
A
AI2 = AE2 sec I2 AE2 = AE2 sec 90◦ −
2
A
= (s − c) cosec .
2
A
Similarly, AI3 = (s − b) cosec
2
∴ I2 I3 = AI2 + AI3
A
= (2s − b − c) cosec
2
A
= a cosec .
2
Otherwise thus :
We have ( )
r2 A A r3
= sin 90◦ − = cos =
AI2 2 2 AI3
A
∴ I2 I3 = AI2 + AI3 = (r2 + r3 ) sec
)…
2
(
1 1 bc
=S +
s−b s−c s(s − a)
15.3. Orthocenter and Pedal Triangle 274


aS bc
= √
(s − b)(s − c) s(s − a)

a bc A
= √ = a cosec .
(s − b)(s − c) 2 ■

§ Problem 15.3.6. II1 · II2 · II3 = 16R2 r. ♢


§§ Solution.
A B C
∴ II1 · II2 · II3 = a sec · b sec · c sec
2 2 2
A B C
= abc sec sec sec
2 2 2
A B C
= 8R3 sin A sin B sin C sec sec sec
2 2 2
A A B B C C
8R · 8 sin cos sin
3 cos sin cos
= 2 2 2 2 2 2
A B C
cos cos cos
2 2 2
3 A B C
= 64R sin sin sin
2 2 2
A B C
= 16R2 × 4R sin sin sin
2 2 2
= 16R2 r, by Art. 204, Cor. ■

§ Problem 15.3.7. (I2 I3 )2 = 4R (r2 + r3 ). ♢


§§ Solution.
( )
A 2
(I2 I3 )2 = a cosec , by §P roblem 15.3.5
2
[ ]
a · bc
2 s−b+s−c
= = abc
(s − b)(s − c)
[ ] (s − b)(s
[ − c) ]
1 1 abc S S
= abc + = +
s−b s−c S s−b s−c
= 4R (r2 + r3 ) . ■

B+C
§ Problem 15.3.8. ∠I3 I1 I2 = . ♢
2
§§ Solution.
∠I3 I1 I2 = ∠BI1 C = 180◦ − (I1 BC + I1 CB)
( )
B C B+C
= 180◦ − 90◦ − + 90◦ − = . ■
2 2 2
§ Problem 15.3.9. II12 + I2 I32 = II22 + I3 I12 = II32 + I1 I22 . ♢
§§ Solution. By § Problem
( 15.3.4 and §
) Problem 15.3.5, we have
A A
II12 + I2 I32 = a2 sec2 + cosec 2
Ñ 2A A
2
é Ñ é2
sin 2 + cos 2
2 2 a
= a2 =
2 A 2
A A A
sin cos sin cos
( ) 2 2 2 2
2a 2
=
sin A
= (4R)2 = II22 + I3 I12 = II32 + I1 I22 , similarly. ■
15.3. Orthocenter and Pedal Triangle 275

A B C abc
§ Problem 15.3.10. Area of ∆I1 I2 I3 = 8R2 cos cos cos = .
2 2 2 2r

§§ Solution.
1
∆I1 I2 I3 = I2 I3 × AI1
2
1 A A
= · a cosec · r1 cosec , by §P roblem 15.3.5 and §P roblem 15.3.3
2 2 2
A B C
1 2R sin A 4R sin cos cos
= · · 2 2 2
2 A A
sin sin
2 2
A B C
= 8R2 cos cos cos
2 2 2
= 2R2 (sin A + sin B + sin C) , by §P roblem 9.2.4
abc
= R(a + b + c) = R · 2s = · 2s
4S
2S abc
= abc ÷ = .
s 2r
Otherwise thus : … … …
A B C s(s − a) s(s − b) s(s − c)
8R2 cos cos cos = 8R2
2 2 2 bc ac ab
sS
= 8R × 2
abc
abcs abc
= = . ■
2S 2r
II1 · I2 I3 II2 · I3 I1 II3 · I1 I2
§ Problem 15.3.11. = = . ♢
sin A sin B sin C
§§ Solution.
A A
II1 · I2 I3 a sec · a cosec
= 2 2
sin A sin A
2a2
=
A A
2 cos sin sin A
2 2
2a2
=
sin2 A
= 2 (2R)2 = 8R2
II2 · I3 I1 II3 · I1 I2
= = , similarly. ■
sin B sin C
If I, O, and P be respectively the incenter, circumcenter and
orthocenter, and G the centroid of the triangle ABC, prove
that
§ Problem 15.3.12. IO2 = R2 (3 − 2 cos A − 2 cos B − 2 cos C). ♢
§§ Solution. By Art. ( 217, we have )
A B C
IO2 = R2 1 − 8 sin sin sin
[ ( 2 2 2 ) ]
A−B A+B C
= R2 1 − 4 cos − cos sin
( 2 2 2)
A−B A+B C
= R2 1 − 4 cos cos + 4 sin2
2 2 2
15.3. Orthocenter and Pedal Triangle 276

= R2 [1 − 2(cos A + cos B) + 2(1 − cos C)]


= R2 (3 − 2 cos A − 2 cos B − 2 cos C) . ■

§ Problem 15.3.13. IP 2 = 2r2 − 4R2 cos A cos B cos C. ♢


§§ Solution. We have
IP 2 = AP 2 + AI 2 − 2AP · AI · cos ∠IAP
B B C
= 4R2 cos2 A + 16R2 sin2 sin2 sin2
2 2 2
B C C−B
− 16R2 cos A sin sin cos
( 2 2 2
2 B 2 C B C C B
2 2
= 4R cos A + 4 sin sin − 4 cos A sin sin cos cos
2 2 ) 2 2 2 2
2 B 2 C
−4 cos A sin sin
[ 2 2 ]
B 2 C
2 2
= 4R cos A + 4 sin 2
sin (1 − cos A) + cos A sin B sin C
( 2 2 )
2 A 2 B 2 C
2
= 4R 8 sin sin sin + cos2 A − cos A sin B sin C
2 2 2
= 2r2 + 4R2 cos A (cos A − sin B sin C)
= 2r2 − 4R2 cos A [cos(B + C) + sin B sin C]
= 2r2 − 4R2 cos A cos B cos C.
Notes : In the above,
A A 1
∠IAP = − ∠P AC = − (90◦ − C) = (C − B). ■
2 2 2
(
1 2 )
§ Problem 15.3.14. OG2 = R2 − a + b 2 + c2 . ♢
9
1
§§ Solution. By the figure of Art. 215, we have OG = OP
3
1 1( 2 )
∴ OG2 = · OP 2 = R − 8R2 cos A cos B cos C
9 9
1
= R2 [1 − 4 {cos(A + B) + cos(A − B)} cos C]
9
1 [ ]
= R2 1 + 4 cos2 C + 4 cos(A − B) cos(A + B)
9
1
= R2 [1 + 2(1 + cos 2C) + 2(cos 2A + cos 2B)]
9
1
= R2 (3 + 2 cos 2A + 2 cos 2B + 2 cos 2C)
9
1
= R2 [9 − 2(1 − cos 2A) − 2(1 − cos 2B) − 2(1 − cos 2C)]
9
4R2 ( 2 )
= R2 − sin A + sin2 B + sin2 C
9
1( 2 )
= R2 − a + b2 + c2 . ■
9
B−C C−A A−B
§ Problem 15.3.15. Area of ∆IOP = 2R2 sin sin sin .
2 2 2

§§ Solution.
∆IOP = ∆AP I + ∆AIO − ∆AP O
1 1
= AP · AI · sin ∠AP I + AI · AO · sin ∠AIO
2 2
15.3. Orthocenter and Pedal Triangle 277

1
− AP · AO sin ∠AP O
2
1 B C B−C
= · 8R2 cos A sin sin sin
2 2 2 2
1 B C B−C 1
+ · 4R2 sin sin sin − · 2R2 cos A sin(B − C)
[ 2 2 2 2 2 ]
B C
∵ AP = 2R cos A, AI = 4R sin sin and AO = R
2 2
2 B C B−C
= 2R sin sin sin (2 cos A + 1)
2 2 2
B − C B−C
− 2R2 cos A sin cos
[ 2 2
B−C B C
= 2R2 sin (2 cos A + 1) sin sin
2 ( 2 2 )]
B C B C
− cos A cos cos + sin sin
[ ( 2 2 2 2)
B−C B C B C
= 2R2 sin cos A sin sin − cos cos
2 2 ]2 2 2
B C
+ sin sin
( 2 2 )
B−C B+C B C
= 2R2 sin − cos A cos + sin sin
2 [ ( 2 ) 2( 2 )
B−C B+C B+C
2
= R sin − cos A + − cos A − +
2 2 ] 2
B−C B+C
cos − cos
[ 2( 2)
B − C A B+C
2
= R sin sin − cos A −
2 2 2 ]
B−C A
+ cos − sin
2 2
2 B−C A−C B−A
= 2R sin sin sin
2 2 2
B − C C − A A−B
= 2R2 sin sin sin . ■
2 2 2
4 B−C C−A A−B
§ Problem 15.3.16. Area of ∆IP G = R2 sin sin sin .
3 2 2 2

2
§§ Solution. Cf. figure of Art. 215. We have P G = OP .
3
2 2
∴ ∆IP G = ∆IOP = result of §P roblem 15.3.15
3 3
4 B−C C−A A−B
= R2 sin sin sin .
3 2 2 2
2 2 B−C C−A A−B
Also, ∆IOG = R sin sin sin . ■
3 2 2 2
§ Problem 15.3.17. Prove that the distance of the center of the
R√
nine-point circle from the angle A is 1 + 8 cos A sin B sin C. ♢
2
§§ Solution. Let N be the middle point of OP , where O and P are
respectively the circumcenter and orthocenter of the ∆ABC.
The distance AN is required.
15.3. Orthocenter and Pedal Triangle 278

We have ( )
2 AN 2 + P N 2 = AP 2 + AO2
OP 2
∴ 2AN 2 + = AP 2 + AO2 .
2
R2 1( 2 )
∴ AN 2 = 2R2 cos2 A + − R − 8R2 cos A cos B cos C
2 4
R2 ( )
= 8 cos2 A + 2 − 1 + 8 cos A cos B cos C
4
R2
= [1 + 8 cos A (cos A + cos B cos C)]
4
R2
= [1 + 8 cos A {cos B cos C − cos(B + C)}]
4
R2
= [1 + 8 cos A(sin B sin C)]
4
R√
∴ AN = 1 + 8 cos A sin B sin C. ■
2
§ Problem 15.3.18. DEF is the pedal triangle of ABC ; prove that
(1) its area is 2S cos A cos B cos C
R
(2) the radius of its circumcircle is and
2
(3) the radius of its incircle is 2R cos A cos B cos C. ♢
§§ Solution. For the pedal triangle, if a′ , b′ , c′ , A′ , B ′ and C ′ de-
note the sides and angles respectively, we have, by Art. 210:
a′ = a cos A, A′ = 180◦ − 2A,
b′ = b cos B, B ′ = 180◦ − 2B, and
c′ = c cos C, and C ′ = 180◦ − 2C.
so that, if ∆′ , R′ and r′ denote its area, radius of its circumcircle and
radius of its incircle respectively, we have
(1)
1 ′ ′ 1
∆= a b sin C ′ = ab cos A cos B sin (180◦ − 2C)
2 2
1
= ab cos A cos B sin 2C = ab cos A cos B sin C cos C
2
1
= 2S cos A cos B cos C, ∵ S = ab sin C
2
a′ a cos A a R
(2) R′ = = = = .
2 sin A′ 2 sin 2A 4 sin A 2
(3)
A′ B′ C′
r′ = 4R′ sin sin sin
2 2 2
R
=4· · cos A cos B cos C = 2R cos A cos B cos C.
2 ■

§ Problem 15.3.19. O1 O2 O3 is the triangle formed by the centers


of the escribed circles of the triangle ABC; prove that
A B C
(1) its sides are 4R cos , 4R cos and 4R cos ,
2 2 2
15.3. Orthocenter and Pedal Triangle 279

π A π B π C
(2) its angles are − , − and − , and
2 2 2 2 2 2
(3) its area is 2Rs. ♢
§§ Solution. ABC is the pedal triangle of the triangle O1 O2 O3
[Art. 211], so that, if a′ , b′ , c′ , A′ , B ′ and C ′ , denote the sides and
angles of the ∆O1 O2 O3 , we have
a = a′ cos A′ , A = 180◦ − 2A′ ,
b = b′ cos B ′ , B = 180◦ − 2B ′ , and
c = c′ cos C ′ , and C = 180◦ − 2C ′ .
Hence
A B C
(1) A′ = 90◦ − , B ′ = 90◦ − and C ′ = 90◦ −
2 2 2
(2)
A
a′ = a sec A′ = a cosec
2
A A
A 2R · 2 sin cos
= 2R sin A cosec = 2 2 = 4R cos A .
2 A 2
sin
2
B C
Similarly, b′ = 4R cos and c′ = 4R cos .
2 2
1
(3) ∆O1 O2 O3 = O2 O3 · AO1 .
2
A
Now O2 O3 = a cosec
2
A A A A
and AO1 = r1 cosec = s tan cosec = s sec .
2 2 2 2
1 as as
∴ ∆O1 O2 O3 = · = = 2Rs.
2 A A sin A
sin cos
2 2 ■

§ Problem 15.3.20. DEF is the triangle formed by joining the points


of contact of the incircle with the sides of the triangle ABC, prove
that
A B C
(1) its sides are 2r cos , 2r cos and 2r cos ,
2 2 2
π A π B π C
(2) its angles are − , − and − , and
2 2 2 2 2 2
2S 3 1 r
(3) its area is , i.e. S.
abcs 2R ♢
§§ Solution. If I be the center of the incircle and r its radius, we
have
∠EIF 180◦ − A A
(1) EF = 2r sin = 2r sin = 2r cos .
2 2 2
B C
So F D = 2r cos , and DE = 2r cos .
2 2
15.3. Orthocenter and Pedal Triangle 280

1 A
(2) The ∠F DE = ∠F IE = 90◦ − , as in (1).
2 2
B C
So ∠DEF = 90◦ − , and the ∠DF E = 90◦ − .
2 2
(3)
Area = ∆EIF + ∆F ID + ∆DIE
1 1 1
= r2 sin ∠EIF + r2 sin ∠F ID + r2 sin ∠DIE
2 2 2
1
= r2 (sin A + sin B + sin C)
2 ( )
1 a+b+c r2 s rS
= r2 = =
2 2R 2R 2R
rS 2rS 2 2S 3
= = = .
2abc abc abcs
4S ■

§ Problem 15.3.21. D, E and F are the middle points of the sides


of the triangle ABC ; prove that the centroid of the triangle DEF is
the same as that of ABC and that its orthocenter is the circumcenter
of ABC. ♢
1
§§ Solution. AD bisects EF , at K, say, and DK = AD.
2
2 1
If G be the centroid of the ∆DEF , then DG = DK = AD;
3 3
∴ G is the centroid of the ∆ABC.
Again, draw DL perpendicular to EF and therefore perpendicular to
BC ; also draw EM perpendicular to F D, and therefore perpendic-
ular to AC.
The orhtocenter of the triangle DEF lies in DL, also in EM ; but
DL and EM meet at the circumcenter of the ∆ABC ; hence the or-
thocenter of the ∆DEF is the circumcenter of the ∆ABC. ■

In any ∆ABC prove that


§ Problem 15.3.22. The perpendicular from A divides BC into por-
tions which are proportional to the cotangents of the adjacent an-
gles, and that it divides the angle A into portions whose cosines are
inversely proportional to the adjacent sides. ♢
§§ Solution. Let the perpendicular from A meet BC in D and let
the ∠BAD and ∠DAC be denoted by β and γ respectively. We have
BD = DA cot B and CD = DA cot C
∴ BD : DC = cot B : cot C.
AD
Also, cos β =
AB
∴ AB cos β = AD = AC cos γ
1 1
∴ cos β : cos γ = AC : AB = : . ■
AB AC
§ Problem 15.3.23. The median through A divides it into angles
whose cotangents are 2 cot A + cot C and 2 cot A + cot B, and makes
1
with the base an angle whose cotangent is (cot C ∼ cot B). ♢
2
15.3. Orthocenter and Pedal Triangle 281

§§ Solution. Let β and γ be the angles which the median AD makes


with CA and AB respectively.
BD CD
We have =
DA DA
sin γ sin β
∴ =
sin B sin C
sin(A − β) sin B sin(C + A)
∴ = =
sin β sin C sin C
[∵ γ = A − β and B = 180◦ − (C + A)]
∴ sin A cot β − cos A = cos A + cot C sin A
∴ cot β − cot A = cot A + cot C
∴ cot β = 2 cot A + cot C.
Similarly, cot γ = 2 cot A + cot B.
Otherwise thus :
Draw BH and DG perpendicular to AC, so that CG = GH and
BH = 2DG.
AG 2AG 2AH + HC
cot β = = =
DG BH BH
2AH HC
= + = 2 cot A + cot C.
BH BH
Similarly,
cot γ = 2 cot A + cot B.
Again, if AM be the perpendicular from A on BC, we have
DM 1 CM ∼ BM 1
cot ∠ADB = = · = (cot C ∼ cot B) . ■
AM 2 AM 2
§ Problem 15.3.24. The distance between the middle point of BC
and the foot of the perpendicular from A is
b2 ∼ c2
. ♢
2a
§§ Solution. With the figure of § Problem 15.3.23, we have
1 1 (CM ∼ BM ) (CM + BM )
DM = (CM ∼ BM ) = ·
2 2 ( CM ) + (BM )
1 CM 2 ∼ BM 2 b2 − AM 2 ∼ c2 − AM 2
= · =
2 BC 2a
b ∼c
2 2
= .
2a
Otherwise thus :

a a c2 + a2 − b2 b 2 ∼ c2
DM = BM ∼ BD = ∼ c cos B = ∼ = . ■
2 2 2a 2a
§ Problem 15.3.25. O is the orthocenter of a triangle ABC ; prove
that the radii of the circles circumscribing the triangles BOC, COA,
AOB, and ABC are all equal. ♢
§§ Solution. The angles at O are known.
The radius of the circle circumscribing the ∆BOC
BC a a
= = = = R.
2 sin ∠BOC 2 sin(B + C) 2 sin A
Similarly, the radii of the circles circumscribing the triangles COA
and AOB each = R. ■
15.3. Orthocenter and Pedal Triangle 282

§ Problem 15.3.26. AD, BE, and CF are the perpendiculars from


the angular points of a triangle ABC upon the opposite sides; prove
that the diameters of the circumcircles of the triangles AEF , BDF
and CDE are respectively a cot A, b cot B and c cot C and that the
perimeters of the ∆DEF and ∆ABC are in the ratio r : R. ♢
§§ Solution. We have AE = c cos A, AF = b cos A and ∠F AE = A.
Thus the ∆AEF is similar to the ∆ABC and has its sides equal to
a cos A, b cos A and c cos A.
Hence the radius of the circumcircle of the ∆AEF
a a
= R cos A = · cos A = cot A,
2 sin A 2
and therefore the diameter = a cot A.
Similarly, for the diameters of the other two triangles.
Again, the perimeter of the ∆DEF
= a cos A + b cos B + c cos C
= 2R sin A cos A + 2R sin B cos B + 2R sin C cos C
= R (sin 2A + sin 2B + sin 2C) = 4R sin A sin B sin C
a b c abc 4S 2S
= 4R · · · = = = .
2R 2R 2R 2R2 2R R
Hence the perimeter of the ∆DEF : the perimeter of the ∆ABC
2S S
= : 2s = : R = r : R. ■
R s
§ Problem 15.3.27. Prove that the product of the distances of the
incenter from the angular points of a triangle is 4Rr2 . ♢
§§ Solution. We have
A B C
AI · BI · CI = r cosec · r cosec · cosec
2 2 2
A B C A B C
= r2 · 4R sin sin sin · cosec cosec cosec
[ 2 2 2 ] 2 2 2
A B C
∵ r = 4R sin sin sin
2 2 2
= 4Rr2 . ■

§ Problem 15.3.28. The triangle DEF circumscribes the three es-


cribed circles of the ∆ABC ; prove that
EF FD DE
= = . ♢
a cos A b cos B c cos C
§§ Solution. Let P , Q and R be the centers of the three escribed
circles which are respectively opposite to A, B and C, and to D, E
and F .
From P , draw P L, P K, P H, P N and P M respectively, perpendic-
ular to DF , AB, BC, AC and ED.
Since RP , AC and F D meet, if produced, at a center of similitude
of circles centers R and P , we have
∠LP B = ∠BP N ; but ∠KP B = ∠HP B
∴ the remainder , ∠LP K = the remainder, ∠N P H = C.
Similarly, ∠KP C = ∠M P C; but ∠HP C = ∠N P C
∴ the remainder , ∠KP H(= B) = the remainder, ∠M P N
∴ ∠LP M = 360◦ − 2B − 2C = 2A
∴ D = 180◦ − 2A; so E = 180◦ − 2B, and F = 180◦ − 2C.
15.3. Orthocenter and Pedal Triangle 283

Hence the sides are as


sin (180◦ − 2A) : sin (180◦ − 2B) : sin (180◦ − 2C)
≡ sin 2A : sin 2B : sin 2C
≡ 2 sin A cos A : 2 sin B cos B : 2 sin C cos C
≡ a cos A : b cos B : c cos C. ■

§ Problem 15.3.29. If a circle be drawn touching the inscribed and


circumscribed circles of a triangle and the side BC externally, prove
that its radius is
∆ A
tan2 . ♢
a 2
§§ Solution. Let K be the center and x be the radius of the circle,
and I and O be the centers of the incircle and circumcircle respec-
tively.
Let D be the point of contact of the circle, center K, the inscribed
circle and the side BC.
Then IDK is perpendicular to BC.
Draw OA′ L parallel to IDK, meeting BC in A′ and draw KL per-
pendicular to IK, meeting OA′ L in L.

We then have
OK = R − x, OL = OA′ + x = R cos A + x, and
a a − 2s + 2b b−c
KL = A′ D = BA′ − BD = − (s − b) = = .
2 2 2
2 2 2
Now OL + LK = OK
( )
b−c 2
∴ (R cos A + x)2 + = (R − x)2
2
( )
b−c 2
∴ R2 cos2 A + 2Rx cos A + x2 + = R2 − 2Rx + x2
2
( )
b−c 2
∴ 2Rx(1 + cos A) = R2 sin2 A −
2
Ä a ä2 ( )
A b−c 2 a2 − b2 − c2 + 2bc
∴ 4Rx cos2 = − =
2 2 2 4
2bc − 2bc cos A 2 A
= = bc sin
4 2
bc A ∆ 2 A
∴x= tan2
= tan .
4R 2 a 2
Otherwise thus :

We have
OK 2 = IK 2 + OI 2 − 2IK · OI cos ∠OIK
( )
∴ (R − x)2 = (r + x)2 + R2 − 2Rr − 2(r + x) (r − R cos A)
For, draw OQ perpendicular to IK and we have
OQ
cos ∠OIK = and OQ = IK − QK
OI
= (r + x) − OL = (r + x) − (R cos A + x)
∴ R2 − 2Rx + x2 = r2 + 2rx + x2 + R2 − 2Rr − 2r2 − 2rx
+ 2Rr cos A + 2Rx cos A
15.3. Orthocenter and Pedal Triangle 284

∴ 2Rx(1 + cos A) = r2 + 2Rr(1 − cos A)


[ ]
ax(1 + cos A) a(1 − cos A)
∴ =r r+
sin A
 B C A
 sin A
a sin sin a sin
= r 2 
A 2 2 +
∴ ax cot
2 A A
cos cos
( 2 2 )
ar B C B+C ar B C
= sin sin + cos = · cos cos
A 2 2 2 A 2 2
cos cos
2 2
ar
= (sin A + sin B + sin C)
A
4 cos2
2
A
ar sin ( )
= 2 · sin A + sin B + sin C
A sin A
2 cos
2 ( )
ar A a+b+c A A
= tan = rs tan = ∆ tan .
2 2 a 2 2
∆ A
∴x= tan2 . ■
a 2
§ Problem 15.3.30. If a, b and c be the radii of three circles which
touch one another externally, and r1 and r2 be the radii of the two
circles that can be drawn to touch these three, prove that
1 1 2 2 2
− = + + .
r1 r2 a b c ♢
§§ Solution. Let A, B and C be the centers of the three circles re-
spectively and O be the center of a circle touching them.
Join AB, BC and CA.
Let ∠BOC = α, ∠AOC = β and ∠AOB = γ.
Now if O be in the plane of the ∆ABC, whether within or without
∆ABC, we have
β + γ = 180◦ − α or β + γ = α.
Hence, in all cases,
cos(β + γ) = cos α
∴ cos β cos γ − cos α = sin β sin γ.
Squaring, we have
cos2 β cos2 γ − 2 cos α cos β cos γ + cos2 α
( )( )
= 1 − cos2 β 1 − cos2 γ
= 1 − cos2 β − cos2 γ + cos2 β cos2 γ
∴ cos α + cos β + cos2 γ − 2 cos α cos β cos γ = 1
2 2
(15.5)
The symmetry of this expression shows that it is true for all positions
of O.
α
Now cos α = 1 − 2 sin2 = 1 − x, say
2
β
cos β = 1 − 2 sin2 = 1 − y, say
2
γ
cos γ = 1 − 2 sin2 = 1 − z, say
2
15.3. Orthocenter and Pedal Triangle 285

∴ from (15.5), we have


(1 − x)2 + (1 − y)2 + (1 − z)2 − 2(1 − x)(1 − y)(1 − z) = 1
∴ x2 + y 2 + z 2 − 2(yz + zx + xy) + 2xyz = 0 (15.6)
Now AB = a + b, BC = b + c and CA = c + a
For the smaller circle (O within ∆ABC), in the ∆OBC the sides are
b + c, c + r1 , r1 + b
∴ s = r1 + b + c

α bc
∴ sin
= (15.7)
2 (r1 + b)(r1 + c)
For the larger circle (O not necessarily within ∆ABC, in the ∆OBC
the side are
b + c, r2 − c, r2 − b
∴ s = r2

α bc
∴ sin = (15.8)
2 (r2 − b)(r2 − c)
From (15.7) and (15.8) :
2 2
α r12 r22
∴ 2 sin2 = ( )( ) or ( )( ) = x.
2 1 1 1 1 1 1 1 1
+ + − −
b r1 c r1 b r2 c r2
Hence, if r denote r1 or −r2 , from (15.6), we have
 2
 2

( )(
r 2
) + ... + ...
1 1 1 1
+ +

r b r c 
4
 4 
− 2( ) ( r )( ) + . . . + . . .
1 1 2 1 1 1 1
+ + +
r a r b r c
8
6
+2· ( ) ( r )2 ( ) =0
1 1 2 1 1 1 1 2
+ + +
ï( r a)2 r b ò r c
4 1 1
∴ 4 + + ... + ...
r r a
[( )( ) ]
8 1 1 1 1 16
− 4 + + + · · · + . . . + 6 = 0.
r ( r b r ) ( c )r
3 2 1 1 1 1 1 1
∴ 2 + + + + 2
+ +
r [ r a (b c )a b2 c2 ]
3 2 1 1 1 1 1 1 4
−2 2 + + + + + + + 2 = 0,
r r a b c bc ca ab r
1 1
so that and − are roots of the equation
r1 ( r2 ) ( ) ( )
1 2 1 1 1 1 1 1 1 1 1
− + + + + + − 2 + + = 0.
r2 r a b c a2 b2 c2 bc ca ab
15.3. Orthocenter and Pedal Triangle 286

Hence, by the theory of quadratic


( equations,
) we have
1 1 1 1 1
− =2 + + . ■
r1 r2 a b c
§ Problem 15.3.31. If ∆0 be the area of the triangle formed by join-
ing the points of contact of the inscribed circle with the sides of the
given triangle, whose area is ∆ and ∆1 , ∆2 and ∆3 the corresponding
areas for the escribed circles, prove that
∆1 + ∆2 + ∆3 − ∆0 = 2∆. ♢
§§ Solution. Take the figure of Art. 205. Join F1 D1 , D1 E1 and F1 E1 .
We then have
∆1 = ∆F1 D1 I1 + ∆D1 I1 E1 − ∆F1 I1 E1
1
= r12 (sin B + sin C − sin A)
2 ( )
1 b+c−a
= r12
2 2R
1 ∆ 2(s − a)
= r1 · ·
2 s−a 2R
r1 ∆
= .
2R
Similarly, for ∆2 and ∆3 .
r∆
Also, ∆0 = , by § Problem 15.3.20.
2R

∴ ∆1 + ∆ 2 + ∆ 3 − ∆0 = (r1 + r2 + r3 − r)
2R

= · 4R, by §P roblem 15.2.18
2R
= 2∆. ■

§ Problem 15.3.32. If the bisectors of the angles of a ∆ABC meet


the opposite sides in A′ , B ′ and C ′ , prove that the ratio of the areas
of the triangles A′ B ′ C ′ and ABC is
A B C A−B B−C C−A
2 sin sin sin : cos cos cos . ♢
2 2 2 2 2 2
§§ Solution. By Euclid V I. 3, [cf. Art. 218], we have
bc
AB ′ = ,
c+a
bc
AC ′ = ,
a+b
ca
BC ′ = ,
a+b
′ ca
BA =
b+c
ab
CB ′ =
c+a
ab
and CA′ = ;
b+c
1 bc bc
∴ ∆AB ′ C ′ = · · sin A
2 c+a a+b
bc
= · ∆ABC,
(c + a)(a + b)
15.3. Orthocenter and Pedal Triangle 287

ca
∴ ∆A′ BC ′ = · ∆ABC, and
(a + b)(b + c)
ab
∆A′ CB ′ = · ∆ABC;
(b + c)(c + a)
∴∆A′ B ′ C ′
[ ]
bc ca ab
= ∆ABC 1 − − −
(c + a)(a + b) (a + b)(b + c) (b + c)(c + a)
[ ]
bc(b + c) + ca(c + a) + ab(a + b)
= ∆ABC 1 −
(b + c)(c + a)(a + b)
[ ]
2abc
= ∆ABC
(b + c)(c + a)(a + b)
∆A′ B ′ C ′ 2 sin A sin B sin C
∴ =
∆ABC (sin B + sin C)(sin C + sin A)(sin A + sin B)
A A B B C C
16 sin cos sin cos sin cos
= 2 2 2 2 2 2
B+C B−C C+A C−A A+B A−B
8 sin cos sin cos sin cos
2 2 2 2 2 2
A B C
2 sin sin sin
= 2 2 2
B−C C−A A−B
cos cos cos
2 2 2
∴∆A′ B ′ C ′ : ∆ABC
A B C A−B B−C C−A
= 2 sin sin sin : cos cos cos . ■
2 2 2 2 2 2
§ Problem 15.3.33. Through the angular points of a triangle are
drawn straight lines which make the same angle α with the opposite
sides of the triangle; prove that the area of the triangle formed by
them is to the area of the original triangle as
4 cos2 α : 1. ♢
§§ Solution. Let ABC be the original triangle and A′ B ′ C ′ be the
new triangle whose sides are a′ , b′ , c′ .
∵ ∠A′ B ′ C ′ = ∠B ′ AC + ∠B ′ CA
= α − C + π − A + α = π − A − C = B.
∴ ∆A′ B ′ C ′ is similar to the ∆ABC.
We then have
AB ′ sin(α + A) AC ′ sin(α − A)
= and = .
AC sin B AB sin C
b
∵ AB ′ = · sin(α + A) = 2R sin(α + A), and
sin B
′ c
AC = · sin(α − A) = 2R sin(α − A).
sin C
∴ a = AB − AC ′ = 2R · 2 cos α sin A = 2a cos α.
′ ′

Also, since the triangles are similar, we have, by Euclid V I. 19,


∆′ a′2 4a2 cos2 α
= 2 = = 4 cos2 α
∆ a a2
∴ ∆′ : ∆ = 4 cos2 α : 1. ■
15.3. Orthocenter and Pedal Triangle 288

§ Problem 15.3.34. Two circles, of radii a and b, cut each other at


an angle θ. Prove that the length of the common chord is
2ab sin θ
√ .
a2 + b2 + 2ab cos θ ♢
§§ Solution. Let A and B be the centers of the two circles respec-
tively and C be a point of intersection.
The ∠θ is the angle between the tangents at C.
π π
Also, the ∠ACB = + − θ = π − θ.
2 2
If the length of the common chord be d, we then have
1 d 1
· AB · = ∆ACB = ab sin ∠ACB.
2 2 2
2ab sin θ
∴d=
AB
2ab sin θ
= √
a2 + b2 − 2ab cos ∠ACB
2ab sin θ
= √ . ■
a + b2 + 2ab cos θ
2

§ Problem 15.3.35. Three equal circles touch one another; find the
radius of the circle which touches all three. ♢
§§ Solution. Let A, B and C be the centers of the three equal circles
of radius r, D be the point of contact of circles whose centers are B
and C, and O be the center of the required circles.
Join AB, BC and CA.
ABC is an equilateral ∆.
√ 2 √
Also, AD = r tan 60◦ = r 3 and AO = r 3.
3
∴ the required radii
2 √
= r 3±r
3
= r(1.1547 ± 1)
= 2.1547r, or .1547r. ■

§ Problem 15.3.36. Three circles, whose radii are a, b and c, touch


one another externally and the tangents at their points of contact
meet in a point ; prove that the distance of this point from either of
their points of contact is
( )1
abc 2
.
a+b+c ♢
§§ Solution. Let A, B and C be the centers of three circles respec-
tively, and ID, IE and IF be the three common tangents.
Join AB, BC and CA.
Since ID, IE and IF are perpendicular to BC, CA and AB respec-
tively, and are equal to one another.
∴ I is the incenter of the ∆ABC.
Now the sides of the ∆ABC are
b + c(= a′ , say), c + a(= b′ ) and a + b(= c′ )
∴ 2s = 2(a + b + c) and s = a + b + c.
Hence the required distance
∆ABC
= IE = ID = IF =
s
15.3. Orthocenter and Pedal Triangle 289


(s − a′ )(s − b′ )(s − c′ )
=
s

abc
= . ■
a+b+c
§ Problem 15.3.37. On the sides BC, CA, AB are taken three points
A′ , B ′ , C ′ such that
BA′ : A′ C = CB ′ : B ′ A = AC ′ : C ′ B = m : n;
prove that if AA′ , BB ′ and CC ′ be joined they will form by their
intersections a triangle whose area is to that of the triangle ABC as
(m − n)2 : m2 + mn + n2 . ♢
§§ Solution. Let BB and CC ′ , CC ′ and AA′ , and AA′ and BB ′ , meet

respectively in the points D, E and F .


Let the lengths of the perpendiculars from F upon the sides of the
triangle be called α, β and γ, and let the area of the ∆ABC be ∆.

β AF sin ∠A′ AC
∴ =
γ AF sin ∠A′ AB
AD sin ∠A′ AC
=
AD sin A′ AB
= ratio of perpendiculars from A′ on the sides
n sin C nc
= = .
m sin B mb
γ na
∴ =
α mc
cγ aα bβ aα + bβ + cγ
∴ = m = n = m n
1 + +1
n m n m
twice sum of the areas of ∆′ s AF B, BDC, CEA
= m n
+ +1
n m
mn
= 2∆ × 2
m + mn + n2
1 mn
∴ area ∆AF B = cγ = ∆ × 2
2 m + mn + n2
So each of the areas BDC and CEA is equal to the same quantity;
∴ area ∆DEF = ∆ − sum of the areas of ∆AF B, BDC and CEA
[ ]
3mn
=∆ 1− 2 2
m + mn + n
(m − n)2
=∆× 2 . ■
m + mn + n2
§ Problem 15.3.38. The circle inscribed in the triangle ABC touches
the sides BC, CA and AB in the points A1 , B1 and C1 respectively;
similarly the circle inscribed in the triangle A1 B1 C1 touches the sides
in A2 , B2 , C2 respectively, and so on ; if An Bn Cn be the nth triangle
so formed, prove
Ä that its angles are
π −n πä π Ä πä π Ä πä
+ (−2) A− , + (−2)−n B − and + (−2)−n C − .
3 3 3 3 3 3
Hence prove that the triangle so formed is ultimately equilateral. ♢
15.3. Orthocenter and Pedal Triangle 290

§§ Solution. If I be the center of the incircle of the ∆ABC, we have


1 1 π A π 1Ä πä
A1 = B1 IC1 = (π − A) = − = − A− .

2 ä2 3 Ä2
πä
2 2 3
π A1 π π π 1
∴ A2 = − = − A1 − = + A− .
2 2 3 2 3Ä 3 ä 4 3
π π
If An = + (−2)−n A −
3 3
π An
then An+1 = −
1 îπ Ä π äó
2 2
π
= − + (−2)−n A −
2
π
2 3 Ä πä
3
= + (−2)−n−1 A − .
3 3
Hence the expression is true for n + 1, if true for n.
But it is true when n = 1 and 2 ; ∴ if n = 3, 4 . . . .
Similarly for the other angles.
π π π
If n be very large, 2−n = 0, and the angles are , , , i.e. ulti-
3 3 3
mately the triangle becomes equilateral. ■

§ Problem 15.3.39. A1 B1 C1 is the triangle formed by joining the


feet of the perpendiculars drawn from ABC upon the opposite sides
; in like manner A2 B2 C2 is the triangle obtained by joining the feet
of the perpendiculars from A1 , B1 and C1 on the opposite sides, and
so on. Find the values of the angles An , Bn and Cn in the nth of these
triangles. ♢
§§ Solution. Here we have Ä
π πä
A1 = π − 2A = − 2 A − , and

πä Ä πä
3
2π π
A2 = π − 2A1 = π − +4 A− = +4 A− .
π Ä 3π ä 3 3 3
If An = + (−2) A −n
,
3 3
2π Ä πä
then An+1 = π − 2An = π − + (−2)n+1 A −
π Ä 3 πä 3
= + (−2)n+1 A − .
3 3
Hence the expression is true for n + 1, if true for n.
But it is true when n = 1 and 2 ; ∴ if n = 3, 4 . . . .
Similarly for the other angles. ■
Chapter 16
On Quadrilaterals And Regular
Polygons

16.1 Area of a Quadrilateral


§ Problem 16.1.1. Find the area of a quadrilateral, which can be
inscribed in a circle, whose sides are

(1) 3, 5, 7, and 9 feet ; and

(2) 7, 10, 5, and 2 feet. ♢


§§ Solution. (1) If a = 3 f t., b = 5 f t., c = 7 f t., and d = 9 f t.,
then
s = 12 f t., s − a = 9 f t., s − b = 7 f t.,
s − c = 5 f t., and s − d = 3 f t.
√ √
∴ the area = 9 × 7 × 5 × 3 = 3 105 sq. f t.

(2) If a = 7 f t., b = 10 f t., c = 5 f t., and d = 2 f t.,


then
s = 12 f t., s − a = 5 f t., s − b = 2 f t.,
s − c = 7 f t., and s − d = 10 f t.
√ √
∴ the area = 5 × 2 × 7 × 10 = 10 7 sq. f t. ■

§ Problem 16.1.2. The sides of a quadrilateral are respectively 3, 4,


opposite angles is 120◦ ; prove
5, and 6 f eet, and the sum of a pair of√
that the area of the quadrilateral is 3 30 square feet. ♢
16.1. Area of a Quadrilateral 292

§§ Solution. If a = 3 f t., b = 4 f t., c = 5 f t., and d = 6 f t.,


then
s = 9 f t., s − a = 6 f t., s − b = 5 f t.,
s − c = 4 f t., and s − d = 3 f t.

∴ the area = 6 × 5 × 4 × 3 − 3 × 4 × 5 × 6 × cos2 60◦

= 6 × 5 × 4 × 3 (1 − cos2 60◦ )

= 6 × 5 × 4 × 3 × sin 60◦

√ 3
= 6 10 ×
√ 2
= 3 30 sq. f t. ■

§ Problem 16.1.3. Tho sides of a quadrilateral which can he in-


scribed in a circle are 3, 3, 4 and 4 feet ; find the radii of the incircle
and circumcircle. ♢
§§ Solution. Let ABCD be the quadrilateral. Take
BA = 3 f t., DA = 4 f t., BC = 3 f t. and DC = 4 f t.

By symmetry, BD is diameter and is equal to 42 + 32 = 5 f t.
5 1
Hence the radius of the circumcircle = f t. = 2 f t.
2 2
Also, since AB + CD = BC + DA, a circle can be inscribed in
the quadrilateral and if r be the radius of this circle, we have (as in
Art. 202),
the area of the quadrilateral
r
= (3 + 4 + 4 + 3).
√2
∴ 7r = 3 × 4 × 4 × 3 = 3 × 4
12 5
∴r= = 1 f t. ■
7 7
§ Problem 16.1.4. Prove that the area of any quadrilateral is one-
half the product of the two diagonals and the sine of the angle be-
tween them. ♢
§§ Solution. Take the figure of Art. 221. Let the diagonals AC and
BD intersect in the point O and let the ∠AOD = α = ∠BOC.
We then have
∆ADC = ∆AOD + ∆COD
1 1
= DO · AO sin α + DO · CO sin(π − α)
2 2
1
= DO(AO + CO) sin α
2
1
= DO · AC sin α.
2
1
Similarly, ∆ABC = BO · AC sin α.
2
Hence the area of the quadrilateral
1 1
= (DO + BO)AC sin α = BD · AC sin α. ■
2 2
16.1. Area of a Quadrilateral 293

§ Problem 16.1.5. If a quadrilateral can be inscribed in one √circle


and circumscribed about another circle, prove that its area is abcd
and that the radius of the latter circle is

2 abcd
. ♢
a+b+c+d
§§ Solution. See Art. 222.
If r be the radius of the incircle, we have
1 1 1 1 √
ra + rb + rc + rd = abcd
2 2 2 √ 2
2 abcd
∴r= . ■
a+b+c+d

§ Problem 16.1.6. A quadrilateral ABCD is described about a cir-


cle; prove that
A B C D
AB sin sin = CD sin sin . ♢
2 2 2 2
§§ Solution. If r be the radius of the circle, we have
A+B
A B sin
AB = r cot + r cot =r 2
2 2 A B
sin sin
2 2
A B A+B
∴ AB sin sin = r sin .
2 2 2
C D C+D
Similarly, CD sin sin = r sin .
2 2 2
∵ A + B = 360◦ − (C + D)
A+B C+D
∴ = 180◦ −
2 2
A+B C+D
∴ sin = sin .
2 2
A B C D
∴ AB sin sin = CD sin sin . ■
2 2 2 2
§ Problem 16.1.7. a, b, c and d are the sides of a quadrilateral
taken in order, and α is the angle between the diagonals opposite to
b or d; prove that the area of the quadrilateral is
1( 2 )
a − b2 + c2 − d2 tan α. ♢
4
§§ Solution. Take the figure of Art. 221. Let the diagonals AC and
BD intersect in the point O and let the ∠AOD = α = ∠BOC.
We then have
a2 = AO2 + BO2 + 2AO · BO cos α
b2 = BO2 + CO 2 − 2BO · CO cos α
c2 = CO 2 + DO2 + 2CO · DO cos α, and
d2 = DO2 + AO2 − 2DO · AO cos α
∴ a − b + c − d2
2 2 2
( ) ( )
= a2 + c2 − b2 + d2
= 2 cos α [(AO · BO + CO · DO) + (BO · CO + DO · AO)]
= 2 cos α [AO(BO + DO) + CO(DO + BO)]
= 2AC · BD cos α.
16.1. Area of a Quadrilateral 294

Also, by § Problem 16.1.4,


the area of the quadrilateral
1
= AC · BD sin α.
2
Hence, by substitution, we have
1( 2 )
area = a − b2 + c2 − d2 tan α. ■
4
§ Problem 16.1.8. If a, b, c and d be the sides and x and y the
diagonals of a quadrilateral, prove that its area is
1î 2 2 ( 2 )2 ó 12
4x y − b + d2 − a2 − c2 . ♢
4
§§ Solution. By § Problem 16.1.4,

1 1 2 2
area = xy sin α = x y sin2 α
2 4

1 2 2 1
= x y − x2 y 2 cos2 α
4 4
1√ 2 2
= 4x y − 4x2 y 2 cos2 α
4√
1
= 4x2 y 2 − (a2 − b2 + c2 − d2 )2 , by §P roblem 16.1.7. ■
4
§ Problem 16.1.9. If a quadrilateral can be inscribed in a circle,
prove that the angle
î between its diagonals is ó

sin−1 2 (s − a)(s − b)(s − c)(s − d) ÷ (ac + bd) .
If the same quadrilateral can also be circumscribed about a circle,
prove that this angle is then
ac − bd
cos−1 .
ac + bd

§§ Solution. If x and y be the diagonals and α be the angle between
them, we have √
the area = (s − a)(s − b)(s − c)(s − d), [Art. 219] ,
1
and also = xy sin α. [ §P roblem 16.1.4.]
2
But xy = ac + bd [Euclid. V I. D.]

2 area 2 (s − a)(s − b)(s − c)(s − d)
∴ sin α = =
î √xy ac + bd ó
∴ α = sin−1 2 (s − a)(s − b)(s − c)(s − d) ÷ (ac + bd) .
In the second case, we have the area

= abcd

2 abcd
∴ sin α =
ac + bd
( )
4abcd ac − bd 2
∴ cos α = 1 −
2
2
=
(ac + bd) ac + bd
ac − bd
∴ cos α =
ac + bd
16.1. Area of a Quadrilateral 295

( )
ac − bd
∴ α = cos−1 . ■
ac + bd

§ Problem 16.1.10. The sides of a quadrilateral are divided in order


in tho ratio m : n, and a new quadrilateral is formed by joining the
points of division ; prove that its area is to the area of the original
figure as
m2 + n2 to (m + n)2 . ♢
§§ Solution. Let the sides AB, BC, CD and DA of the quadrilateral
ABCD be divided at the points A′ , B ′ , C ′ and D′ respectively, so that
AA′ BB ′ CC ′ DD′ m

= ′ = ′ = ′ = .
AB BC C D DA n
We then have
A′ B n BB ′ m
= , and =
AB m+n BC m+n
1
∴ ∆A′ BB ′ = A′ B · BB ′ sin B
2
1 mn
= · · AB · BC sin B
2 (m + n)2
mn
= · ∆ABC.
(m + n)2
mn
Similarly, ∆C ′ DD′ = · ∆ADC
(m + n)2
mn
∴ ∆A′ BB ′ + ∆C ′ DD′ = · area of ABCD.
(m + n)2
mn
∴ ∆A′ AD′ + ∆C ′ CB ′ = · area of ABCD.
(m + n)2
Thus the area of the four triangles at the corners
2mn
= · area of ABCD.
(m + n)2
′ ′ ′ ′
Hence the area of A B C D [ ]
2mn
= area of ABCD 1 −
(m + n)2
m2 + n2
= area of ABCD ×
(m + n)2
′ ′ ′ ′
∴ area of A B C D : area of ABCD = m2 + n2 : (m + n)2 . ■

§ Problem 16.1.11. If ABCD be a quadrilateral inscribed in a cir-


cle, prove that

B (s − a)(s − b)
tan =
2 (s − c)(s − d)
and that the product of the segments into which one diagonal is di-
vided by the other diagonal is
abcd(ac + bd)
.
(ab + cd)(ad + bc) ♢
§§ Solution. By Art. 219, we have
a2 + b2 − c2 − d2
cos B =
2(ab + cd)
B 1 − cos B (c + d)2 − (a − b)2
∴ tan2 = =
2 1 + cos B (a + b)2 − (c − d)2
16.2. Regular Polygons 296

(c + d + a − b)(c + d − a + b)
=
(a + b + c − d)(a + b − c + d)
2(s − b)2(s − a)
=
2(s − d)2(s − c)

B (s − a)(s − b)
∴ tan = .
2 (s − c)(s − d)
Again, take the figure of Art. 221 and let O be the intersection of the
diagonals. We then have
AO ∆AOB ∆AOD ∆BAD ad
= = = =
OC ∆COB ∆COD ∆BCD bc
ad bc
∴ AO = · AC and OC = · AC
ad + bc ad + bc
abcd
∴ AO · OC = · AC 2
(ad + bc)2
abcd (ac + bd)(ad + bc)
= ·
(ad + bc)2 (ab + cd)
abcd(ac + bd)
= .
(ab + cd)(ad + bc) ■

§ Problem 16.1.12. If a, b, c and d be the sides of a quadrilateral


taken in order, prove that
d2 = a2 + b2 + c2 − 2ab cos α − 2bc cos β − 2ca cos γ
where α, β and γ denote the angles between the sides a and b, b and
c, and c and a respectively. ♢
§§ Solution. Take the figure of Art. 221. Produce BA and CD to
meet in E, so that the ∠AED = γ.
Let AE = m and DE = n. We then have
d2 = m2 + n2 − 2mn cos γ (16.1)
b2 + c2 − 2bc cos β = BD2 = (a + m)2 + n2 − 2(a + m)n cos γ, and
(16.2)
a2 + b2 − 2ab cos α = AC 2 = (c + n)2 + m2 − 2(c + n)m cos γ. (16.3)
From (16.2) and (16.3), by addition, we have
a2 + 2b2 + c2 − 2bc cos β − 2ab cos α
= (a + m)2 + (c + n)2 + m2 + n2 − 2(an + cm + 2mn) cos γ (16.4)
Also, b2 = (a + m)2 + (c + n)2 − 2(a + m)(c + n) cos γ (16.5)
From (16.4) and (16.5), by subtraction, we have
a2 + b2 + c2 − 2bc cos β − 2ab cos α
= m2 + n2 + 2ac cos γ − 2mn cos γ = d2 + 2ac cos γ
∴ d2 = a2 + b2 + c2 − 2ab cos α − 2bc cos β − 2ca cos γ. ■

16.2 Regular Polygons


§ Problem 16.2.1. Find, correct to .01 of an inch, the length of the
perimeter of a regular decagon which surrounds a circle of radius
one foot. [tan 18◦ = ·32492.] ♢
§§ Solution. If a be a side of the decagon, we have
π
a = 2r tan = (2 × 12 tan 18◦ ) inches
n
16.2. Regular Polygons 297

∴ the length of the perimeter = (240 tan 18◦ ) inches.


Now √
5−1
tan 18◦ =√ √ [Art. 120]
10 + 2 5
  √ … √
6−2 5 3− 5
= √ √ = √
10 + 2 5 5+ 5
√( √ )( √ )
5− 5
3− 5
= ( )( √ )√
5 5− 5
5+
… √ … √
20 − 8 5 5−2 5
= =
25 − 5 5
… …
5 − 2 × 2.2360680 ·5278640 √
= = = ·1055728
5 5
= ·3249 . . .
[this may be obtained at once from the table of
natural tangents] .
Hence the length of the perimeter
= (240 × ·3249) inches ≈ 77.98 inches. ■

§ Problem 16.2.2. Find to 3 places of decimals the length of the side


of a regular polygon of 12 sides which is circumscribed to a circle of
unit radius. ♢
§§ Solution.
Here
π π
a = 2r tan = 2 tan = 2 tan 15◦
( √ )
n 12
= 2 2 − 3 = 2(2 − 1.73205) = 2(.26795) = .5359. ■

§ Problem 16.2.3. Find the area of

(1) a pentagon,

(2) a hexagon,

(3) an octagon,

(4) a decagon and

(5) a dodecagon,

each being a regular figure of side 1 foot.


[cot 18◦ = 3.07768 . . . ; cot 36◦ = 1.37638 . . .] ♢
§§ Solution. (1)
a2 π 1 π 5
Area = n. cot = 5 × × cot = cot 36◦
4 n 4 5 4
5
= × 1.3763819 = 1.720 . . . sq.f t.
4
1 π 3 3
(2) Area = 6× ×cot = cot 30◦ = ×1.7320508 = 2.598 . . . sq.f t.
4 6 2 2
16.2. Regular Polygons 298

1 π 1◦
(3) Area = 8× ×cot = 2 cot 22 = 2×2.4142136 = 4.8284 . . . sq.f t.
4 8 2
1 π 5 5
(4) Area = 10× ×cot = ×cot 18◦ = ×3.0776835 = 7.694 . . . sq.f t.
4 10 2 2
1 π
(5) Area = 12× ×cot = 3 cot 15◦ = 3×3.7320508 = 11.196 . . . sq.f t.
4 12

§ Problem 16.2.4. Find the difference between the areas of a reg-


ular octagon and a regular hexagon if the perimeter of each be 24
feet. ♢
24
§§ Solution. For the octagon, the length of a side (a) = f eet =
8
3 f eet
9 π 1◦
∴ the area = 8 × × cot = 18 cot 22 = 18 × 2.4142136 =
4 8 2
43.4558448 sq.f t.
24
For the hexagon, the length of a side (a) = f eet = 4 f eet
6
16 π
∴ the area = 6 × × cot = 24 cot 30◦ = 24 × 1.7320508 =
4 6
41.5692192 sq.f t.
Hence the required difference
= (43.4558448 − 41.5692192) sq.f t. = 1.8866 . . . sq.f t.
Otherwise thus :
Area of a polygon
na2 π (na)2 π p2 π
= cot = cot = cot , where p is the perimeter.
4 n 4n n 4n n
Hence the required difference
(24)2 π (24)2 π
= cot − cot
4×8 [ 8 4×6 6]
1 π 1 π
= 24 × 6 cot − cot
ï 8√ 8 6√ ò 6
2+1 3
= 24 × 6 −
8 6
[ (√ ) √ ]
=6 3 2 + 1 − 4 3 = 6 [7.2426408 − 6.9282032]
= 6 × .3144376 = 1.8866 . . . sq.f t. ■

§ Problem 16.2.5. A square, whose side is 2 feet, has its corners


cut away so as to form a regular octagon ; find its area. ♢
§§ Solution. If a be a side of the octagon, we have
a + 2a cos 45◦ = 2
1
∴ a + 2a × √ = 2
2
2 (√ )
∴a= √ =2 2−1 .
2+1
Hence the required area
a2 π (√ )2 (√ )
=8× × cot = 8 × 2−1 × 2+1
4 8
16.2. Regular Polygons 299

(√ )
=8 2 − 1 = 8 × .4142136 = 3.3137 . . . sq.f t. ■

§ Problem 16.2.6. Compare the areas and perimeters of octagons


which are respectively inscribed in and circumscribed to a given cir-
cle, and
√ show√that the areas of the inscribed hexagon and octagon
are as 27 to 32. ♢
§§ Solution. We have, if ρ be the radius of the circle,
8 2 2π
area oct. insc. ρ sin
= 2 8
area oct. circ. π
8ρ2 tan
8
π π
2 sin cos
= 8 8
π
sin
2 8
π
cos
8
π π √
2 cos2 1 + cos
= 8 = 4 = 2 + 2.
2 2 4
Again,
π
side oct. insc. 2ρ sin
= 8
side oct. circ. π
2ρ tan
8 √
π π
2 cos 2 + 2 cos
8 4
= =
√2 √ 2
2+ 2
= .
2
Also,
6 2 2π π
area hex. insc. R sin 3 sin
= 2 6 =
π
3
area oct. insc. 8 2 2π 4 sin
R sin
4
Å
2 √ ã8
3 3
√ … …
2 3 6 54 27
= ( ) = = = .
4 8 64 32
√ ■
2

§ Problem 16.2.7. Prove that the radius of the circle described


17
about a regular pentagon is nearly ths of the side of the pen-
20
tagon. ♢
a π
§§ Solution. We have R = cosec .
2 5 √

R 1 2 10 − 2 5
∴ = π = 1 ÷
a 2 sin 4
5 √ √
2 2 10 + 2 5
= √ √ =
10 − 2 5 100 − 20
16.2. Regular Polygons 300

√ √ √ √
10 + 2 5 50 + 10 5
= √ =
20 10

72.36 8.5
= ≈ .
10 10
17
∴R=a× . ■
20
§ Problem 16.2.8. If an equilateral triangle and a regular hexagon
have the same perimeter, prove that their areas are as 2 : 3. ♢
§§ Solution. If 6a = the perimeter of the triangle and of the hexagon,
then a side of the triangle = 2a, and a side of the hexagon = a.
na2 π (na)2 π
Area = cot = cot , where na = the perimeter.
4 n 4n n
Hence we have ( )
1 π π 2
cot 2 cot √
area of triangle 12 3 3 3 2
area of hexagon
=
1 π
= π = √3 = 3 .
cot cot
6 ■
24 6
§ Problem 16.2.9. If a regular pentagon and a regular√decagon
have the same perimeter, prove that their areas are as 2 : 5. ♢
§§ Solution. If 10a = the perimeter of the pentagon and of the
decagon, then a side of the pentagon = 2a, and a side of the decagon
= a.
Hence we have
1 π π
area of pentagon cot 2 cot
= 20 5 =
π
5
area of decagon 1 π cot
cot
40 10 10
π π
cos sin
5 10
=2 π · π
sin cos
5 10
π
2 cos
= 5
π
2 cos2
10
π (√ ) ï √ ò
2 cos 2 5+1 5+1
= 5 ÷ 1+
π = 4 4
1 + cos
(√ 5 )
2 5+1 2
= √ (√ )=√ .
5 5+1 5 ■

§ Problem 16.2.10. Prove that the sum of the radii of the circles,
which are respectively inscribed in and circumscribed about a reg-
ular polygon of n sides, is
a π
cot ,
2 2n
where a is a side of the polygon. ♢
16.2. Regular Polygons 301

§§ Solution. We have
a π a π
r + R = cot + cosec
2[ n 2 ] n
π
a cos n + 1
= π
2 sin
[ n
2 π
]
a 2 cos
= 2n
2 2 sin π cos π
2n 2n
a π
= cot . ■
2 2n
§ Problem 16.2.11. Of two regular polygons of n sides, one cir-
cumscribes and the other is inscribed in a given circle. Prove that
the perimeters of the circumscribing polygon, the circle, and the in-
scribed polygon are in the ratio
π π π
sec : cosec : 1,
n n n
π
and that the areas of the polygons are in the ratio cos2 : 1. ♢
n
§§ Solution. If ρ be the radius of the circle, the perimeters are as
π π
2nρ tan : 2nρ sin : 2πρ
n n
π π π
∴ tan : sin :
n n n
π π π
∴ sec : 1 : cosec
n n n
or, in descending order of magnitude,
π π π
∴ sec : cosec : 1.
n [n n ]
area pol. insc. n 2 2π î πó
Also, = ρ sin ÷ nρ2 tan
area pol. circ. 2 n n
π π
sin cos
= ( n π) n
sin
n
π
cos
n
π
= cos2 : 1. ■
n
§ Problem 16.2.12. Given that the area of a polygon of n sides cir-
cumscribed about a circle is to the area of the circumscribed polygon
of 2n sides as 3 : 2, find n. ♢
§§ Solution. If ρ be the radius of the circle, we have
π π
nρ2 tan : 2nρ2 tan =3:2
n 2n
π
π π 2 tan
∴ 3 tan = tan = 2n
2n n π
1 − tan2
2n
π π π 1
where tan = 0, or 2 = 3 − 3 tan2 , so that tan = ±√ .
2n 2n 2n 3
16.2. Regular Polygons 302

π 1 π
The only value admissible is tan = √ = tan .
2n 3 6
∴ 2n = 6 and n = 3. ■

§ Problem 16.2.13. Prove that the area of a regular polygon of 2n


sides inscribed in a circle is a mean proportional between the areas
of the regular inscribed and circumscribed polygons of n sides. ♢
§§ Solution. If ρ be the radius of the circle, the areas of the poly-
gons are
π n 2π π
nρ2 sin : ρ2 sin and nρ2 tan
n 2 n n
respectively. Now

π π
nρ2 sin = n2 ρ4 sin2
n √ n
π π π
= nρ2 tan × nρ2 sin cos
… n n n
π n 2π
= nρ2 tan × ρ2 sin . ■
n 2 n
§ Problem 16.2.14. The area of a regular polygon of n sides in-
scribed in a circle is to that of the same number of sides circum-
scribing the same circle as 3 is to 4. Find the value of n. ♢
§§ Solution. If ρ be the radius of the circle, we have
n 2 2π π
ρ sin : nρ2 tan :: 3 : 4
2 n n
2π π π
sin 2 sin cos
∴ n = n n = cos2 π = 3
π π n 4
2 tan 2 sin
n n
π
cos
√n
π 3 π
∴ cos = = cos
n 2 6
∴ n = 6. ■

§ Problem 16.2.15. The interior angles of a polygon are in A. P.


; the least angle is 120◦ and the common difference is 5◦ ; find the
number of sides. ♢
§§ Solution. If n be the number of sides of the polygon, the angles
of the polygon are 120◦ , 125◦ , 130◦ , . . . to n terms.
n
Their sum = [2 × 120◦ + (n − 1)5◦ ] = (2n − 4)90◦
2
[By Euc. I. 32, Cor. 1].
( )5
∴ 120n + n2 − n = 180n − 360
2
∴ n − 25n + 144 = 0
2

∴ n = 9 or 16.
Now the greatest angle of the polygon must be less than 180◦ and
with n = 9, this would be 120◦ + 40◦ , i.e. 160◦ and with n = 16, the
greatest angle would be 120◦ + 75◦ , i.e. 195◦ .
Hence the value 9 of n is applicable only, i.e. the polygon has 9
sides. ■
16.2. Regular Polygons 303

§ Problem 16.2.16. There are two regular polygons the number of


sides in one being double the number in the other, and an angle of
one polygon is to an angle of the other as 9 to 8 ; find the number of
sides of each polygon. ♢
§§ Solution. Let 2n and n be the number of sides in the two poly-
gons respectively. By Art. 223, we then have
(4n − 4)90 (2n − 4)90
: =9:8
2n n
∴ 16n − 16 = 18n − 36
∴ n = 10.
So that the polygons have 20 sides and 10 sides respectively. ■

§ Problem 16.2.17. Show that there are eleven pairs of regular


polygons such that the number of degrees in the angle of one is to
the number in the angle of the other as 10 : 9. Find the number of
sides in each. ♢
§§ Solution. Let m and n be the number of sides in the polygons,
so that
(2m − 4)90 (2n − 4)90
: = 10 : 9
m n
36 40
∴ 18 − = 20 −
m n
∴ mn = 20m − 18n
( )
m + 18 − 18 360
∴ n = 20 = 20 − .
m + 18 m + 18
Now n must be positive; also m and n must be > 2, since there must
be more than 2 sides in a polygon. Hence,
∵ 360 = 360 × 1 = 180 × 2 = 120 × 3 = 90 × 4
= 72 × 5 = 60 × 6 = 45 × 8 = 40 × 9
= 36 × 10 = 30 × 12 = 24 × 15 = 20 × 18
∴ m + 18 = 360, 180, 120, 90, 72, 60, 45, 40, 36, 30, or 24
∴ m = 342, 162, 102, 72, 54, 42, 27, 22, 18, 12, or 6.
Also, the corresponding values of
20m
n= = 19, 18, 17, 16, 15, 14, 12, 11, 10, 8, or 5.
m + 18
Thus there are eleven pairs of polygons. ■

§ Problem 16.2.18. The side of a base of a square pyramid is a feet


and its vertex is at a height of h feet above the center of the base ; if
θ and ϕ be respectively the inclinations of any face to the base, and
of any two faces to one another, prove that

2h ϕ a2
tan θ = and tan = 1+ . ♢
a 2 2h2
§§ Solution. Let V be the vertex and E be the center of the base
ABCD of the pyramid. Let H be the middle point of AD. We then
have
h 2h
tan θ = tan ∠V HE = Ä a ä = .
a
2
16.2. Regular Polygons 304

Again, draw BF and DF perpendicular to V A, so that ϕ = ∠BF D and


let the ∠V AE = α. ( )
a

ϕ BE 2 1 VA
∴ tan = = = =
2 EF AE sin α …sin α VE

√ h2 +
a2 …
2
V E + AE 2 2 a2
= = = 1+ . ■
VE h 2h2
§ Problem 16.2.19. A pyramid stands on a regular hexagon as base.
The perpendicular from the vertex of the pyramid on the base passes
through the center of the hexagon, and its length is equal to that of
a side of the base. Find the tangent of the angle between the base
and any face of the pyramid, and also of half the angle between any
two side faces. ♢
§§ Solution. Let V be the vertex and E be the center of the base of
the pyramid.
Let AB and BC be the two sides of the base and D be the middle
point of AB.
Then, if θ be the angle between the base and any face of the pyra-
mid and a be a side of the base, we have
VE a 2 2√
tan θ = = = √ = 3.
ED a sin 60◦ 3 3
Again, since V E = EB, ∴ ∠V BE = 45◦ .
Draw AH perpendicular to V B and AK perpendicular to EB ; then
HK is perpendicular to V B and to AK and if ϕ be the angle between
any two side faces, we have
ϕ AK a sin 60◦
tan = tan ∠AHK = = ◦
2 HK
√ ( ) √BK sin 45
a 3 a 1
= ÷ ·√ = 6. ■
2 2 2
§ Problem 16.2.20. A. regular pyramid has for its base a polygon of
n sides, each of length a and the length of each slant side is l ; prove
that the cosine of the angle between two adjacent lateral faces is

4l2 cos + a2
n . ♢
4l2 − a2
§§ Solution. Let V be the vertex and E be the center of the base of
the polygon.
Let AB and BC be two sides of the base. ( )
2π 1 2π π π
We have the ∠AEB = and the ∠ABE = π− = − .
n 2 n 2 n
Draw AH perpendicular to V B and AK perpendicular to EB ; then
HK is perpendicular to V B and to AK and if ϕ be the angle between
two adjacent lateral faces, we have Äπ πä π
ϕ AK a sin − a cos
tan = = 2 n = n
2 HK BK sin ∠V BE π VE
a sin ·
n l
16.2. Regular Polygons 305

π π
l cot l cot
= √ n √ Äa
n
π ä2
=
l2 − BE 2
l2 − cosec
2 n
ϕ π π
tan2 l2 cot2 4l2 cos2
∴ 2 = n = n
1 a2 π π
l2 −cosec 2 4l2 sin2 − a2
4 n n
ϕ
1 − tan2
∴ cos ϕ = 2
2
ϕ
1 + tan

π πä
−a2 − 4l2 cos2 − sin2
= Ä π
n
πä
n
2
4l sin 2 + cos 2 − a2
n n

a2 + 4l2 cos
=− n .
4l2 − a2
The cosine of the acute angle between two faces

4l2 cos + a2
= n . ■
4l2 − a2
Chapter 17
Trigonometrical Ratios of Small
Angles, Area of A Circle, DIP of
The Horizon.

17.1 Ratios of Small Angles


[ ]
1
π = 3.14159265; = .31831 . . .
π
Find, to 5 places of decimals, the value of
§ Problem 17.1.1. sin 7′ . ♢
7◦ 7π c
§§ Solution. Since 7′ = = , we have
( )c
60 180 × 60
7π 7π 7 × 3.14159265
sin 7′ = sin = = = .00204. ■
180 × 60 180 × 60 180 × 60
§ Problem 17.1.2. sin 15′ . ♢
1′ 1◦ πc
§§ Solution. Since 15′′
= = = , we have
( 4 4 × )c
60 180 × 4 × 60
π
sin 15′′ = sin
180 × 4 × 60
π 3.14159265
= = = .00007. ■
180 × 4 × 60 180 × 4 × 60
§ Problem 17.1.3. sin 1′ . ♢
1◦ πc
§§ Solution. Since 1′ = = , we have
60 ( × 60 )c
180
π
sin 1′ = sin
180 × 60
17.1. Ratios of Small Angles 308

π
= = .00029. ■
180 × 60
§ Problem 17.1.4. cos 15′ . ♢
15◦ 1◦ πc
§§ Solution. Since 15′ = = = , we have
( )c
60 4 180 ×4
π
sin 15′ = sin
180 × 4
π
= ≈ .0043633.
180 × 4
√ 1
Also, cos 15′ = 1 − sin2 15′ = [1 − .00001904] 2
By the Binomial Theorem,
1
≈ 1 − [.00001904]
2
= 1 − .00000952 = .99999 to 5 places of decimals. ■

§ Problem 17.1.5. cosec 8′′ . ♢


1 1
§§ Solution. cosec θ = ≈ .
sin θ θ
8 π
Here θ= ×
60 × 60 180
1 900 180 900
∴ = × = × 57.2957795 = 25783.10077. ■
θ 2 π 2
§ Problem 17.1.6. sec 5′ . ♢
1◦ πc
§§ Solution. Since 5′ = = .
12 180 × 12
We have ( )c
π π
sin 5′ = sin = ≈ .001454.
180 × 12 180 × 12
1 [ ]− 1
Also, sec 5′ = = 1 − sin2 5′ 2
cos 5′
1
= [1 − .000002114]− 2
By the Binomial Theorem,
1
≈ 1 + [.000002114] = 1 + .000001057 ≈ 1.0000011. ■
2

Solve approximately the equations


§ Problem 17.1.7. sin θ = .01. ♢
§§ Solution. Here θ is very small.
180◦
∴ θ ≈ .01c = .01 ×
π
= .01 × 206265′′ ≈ 2063′′ = 34′ 23′′ . ■

§ Problem 17.1.8. sin θ = .48. ♢


1
§§ Solution. Since sin θ is nearly equal to , θ must be nearly equal
2
π
to .
6
π
Let then θ = − x, where x is very small;
6 √
Äπ ä π π 1 3
∴ .48 = sin − x = sin cos x − cos sin x = cos x − sin x.
6 6 6 2 2
17.1. Ratios of Small Angles 309

Since x is very small, we have



1 3
cos x = 1 and sin x ≈ x; ∴ .48 = − x
Å √ ãc 2 2
2 .04 × 3
∴ x = .02 × √ radians = = .023094c ≈ 1◦ 19′ 23′′ .
3 3
∴ θ = 30◦ − 1◦ 29′ 23′′ = 28◦ 40′ 37′′ . ■

Äπ ä
§ Problem 17.1.9. cos + θ = .49. ♢
Äπ ä3
§§ Solution. cos + θ = .49.
3
1 π
Since .49 is very nearly equal to , which is the value of cos , it
2 3
follows that θ must be very small.
The equation may be written

1 3
cos θ − sin θ = .49.
2 2
Also, since θ is very small, we have
cos θ = 1 and sin θ ≈ θ

1 3
∴ − θ = .49
2 2Å √ ãc
2 .02 × 3
∴ θ = .01 × √ radians = = .011547c
3 3
= (.011547 × 206265)′′ ≈ 2382′′ ≈ 39′ 42′′ . ■

§ Problem 17.1.10. cos θ = .999. ♢


§§ Solution. cos θ = .999. Hence θ is very small.
θ2
We have 1− = 1 − .001;
2
θ 2
∴ = .001; ∴ θ2 = .002
2
∴ θ ≈ .0447213c
= (.0447213 × 57.2957795)◦ = 2◦ 33′ 44′′ . ■

§ Problem 17.1.11. Find approximately the distance at which a


halfpenny, which is an inch in diameter, must be placed so as to
just hide the moon, the angular diameter of the moon, that is the
angle its diameter subtends at the observer’s eye, being taken to be
30′ . ♢
§§ Solution. If x be the required distance in inches, we have
1 30 π
= the number of radians in 30′ = ×
x 60 180
180
∴x=2× = 2 × 57.2957795 = 114.59 . . . inches. ■
π
§ Problem 17.1.12. A person walks in a straight line toward a very
distant object and observes that at three points A, B and C the angles
of elevation of the top of the object are α, 2α and 3α respectively;
prove that
AB ≈ 3BC. ♢
17.1. Ratios of Small Angles 310

§§ Solution. Let E be the top of the object.


In the ∆ABE, we have the ∠AEB = ∠BAE = α, so that BE = AB.
In the ∆BEC, the ∠BEC = α and we have
BE sin 3α
=
BC sin α
AB sin 3α
∴ = = 3 − 4 sin2 α.
BC sin α
Since the object is very distant, α is very small.
AB
∴ ≈ 3, ∴ AB ≈ 3BC. ■
BC
§ Problem 17.1.13. If θ be the number of radians in an angle which
is less than a right angle, prove that
θ2 θ4
cos θ is < 1 − + . ♢
2 16
§§ Solution. We have
θ θ θ θ3
cos θ = 1 − 2 sin2 and sin > −
2
Å ã22 2 32
θ θ3
∴ cos θ < 1 − 2 −
2 32
ï ò
θ2 θ4 θ6
<1−2 − +
4 32 (32)2
θ2 θ4 2θ6
<1− + − .
2 16 (32)2
θ2 θ4
∴ cos θ < 1 − + . ■
2 16
§ Problem 17.1.14. Prove the theorem of Euler, viz. that
θ θ θ
sin θ = θ · cos · cos 2 · cos 3 . . . ad. inf. ♢
2 2 2
§§ Solution. We have
θ θ θ θ θ
sin θ = 2 sin cos = 22 sin 2 cos 2 cos
2 2 2 2 2
θ θ θ θ
= 23 sin 3 cos 3 cos 2 cos = . . . . . .
2 2 2 2
θ θ θ θ θ
= 2n sin n × cos · cos 2 · cos 3 . . . . . . cos n .
2 2 2 2 2
Make n indefinitely great so that, by Art. 228 Cor.,
θ
2n sin n = θ.
2
θ θ θ
∴ sin θ = θ · cos · cos 2 · cos 3 . . . ad. inf. ■
2 2 2

(
§ Problem )(
17.1.15. )(
Prove that )
θ θ θ
1 − tan2 1 − tan2 2 1 − tan2 3 . . . ad. inf. = θ · cot θ. ♢
2 2 2
§§ Solution. We have
θ
2 tan
tan θ = 2
θ
1 − tan 2
2
17.2. Area of A Circle 311

θ
θ 2 tan
tan = 22
2 θ
1 − tan2
22
θ
θ 2 tan
tan 2 = 23
2 θ
1 − tan2
23
... = ...
Hence, by multiplication, we have
θ
2n tan
tan θ = ( )( 2n )
θ θ
1 − tan2 1 − tan2 ...
2 22
θ
θ tan n
Now 2 tan n = θ ×
n 2
2 θ
2n
= θ, when n is indefinitely increased;
θ
∴ tan θ = ( )( )
θ θ
1 − tan 2 1 − tan2 2 . . .
( )( 2 ) 2
θ θ
∴ 1 − tan2 1 − tan2 2 . . . = θ cot θ. ■
2 2

17.2 Area of A Circle


[ ]
1
= .31831 and log π = .49715.
Assume that π = 3.14159 . . . ,
π
§ Problem 17.2.1. Find the area of a circle whose circumference is
74 feet. ♢
37
§§ Solution. We have 2πR = 74 f eet, ∴ R = f eet.
π
(37)2 (37)2
∴ the area = πR2 = π × =
π2 π
= 1369 × .31831 ≈ 435.77 sq. f t. ■

§ Problem 17.2.2. The diameter of a circle is 10 feet; find the area


1◦
of a sector whose arc is 22 . ♢
2
§§ Solution. The required area
1
22
= 2 × area of whole circle
360
1 ( ) 100π
= × π × 52 =
16 64
314.1592
= = 4.9087 sq. f t. ■
64
§ Problem 17.2.3. The area of a certain sector of a circle is 10
square feet; if the radius of the circle be 3 feet, find the angle of
the sector. ♢
17.2. Area of A Circle 312

1
§§ Solution. We have × 32 × α = 10;
2
20 20 180◦ 20
∴α= radians = × = × 57.2957795◦ ≈ 127◦ 19′ 26′′ . ■
9 9 π 9
§ Problem 17.2.4. The perimeter of a certain sector of a circle is 10
feet; if the radius of the circle be 3 feet, find the area of the sector. ♢
§§ Solution. We have
4
2R + Rα = 10 and R = 3 f eet; ∴ 3α = 4, ∴ α = .
3
Hence the required area
1 1 4
= R2 · α = × 9 × = 6 sq. f t. ■
2 2 3
§ Problem 17.2.5. A strip of paper, two miles long and .003 of an
inch thick, is rolled up into a solid cylinder; find approximately the
radius of the circular ends of the cylinder. ♢
§§ Solution. If d be the width of the paper and r be the radius of
the cylinder, we have
πr 2 d = 2 × 1760 × 36 × d × .003
6 × 176 × 36
∴ r2 =
… 100π
6 × 11 √
∴r =6×4× = 6 × 4 × 6 × 11 × .0031831
100π
= 6 × 4 × .45835 = 11.0004 inches. ■

§ Problem 17.2.6. A strip of paper, one mile long, is rolled tightly up


into a solid cylinder, the diameter of whose circular ends is 6 inches;
find the thickness of the paper. ♢
§§ Solution. If x inch be the required thickness, we have
π × 32 = 1760 × 36 × x
π 3.14159265
∴x= = = .00044625 inch. ■
1760 × 4 11 × 80 × 8
§ Problem 17.2.7. Given two concentric circles of radii r and 2r;
two parallel tangents to the inner circle cut off an arc from the outer
circle; find its length. ♢
§§ Solution. Let O be the center of the circles and AB be the arc
cut off and let the tangents through A and B meet the inner circle in
C and D respectively.
r 1
We the have cos ∠AOC = = ,
2r 2
π π
∴ ∠AOC = = ∠BOD; ∴ ∠AOB = .
3 3
π 2
∴ the arc AB = 2r × = πr. ■
3 3
§ Problem 17.2.8. The circumference of a semi-circle is divided
into two arcs such that the chord of one is double that of the other.
Prove that the sum of the areas of the two segments cut off by these
chords is to the area of the semicircle
[ ]as 27 is to 55.
22
π= . ♢
7
17.2. Area of A Circle 313

§§ Solution. Take the figure of Art. 106. Let the circumference


OP Q be divided into the two arcs OP and P Q, such that the chord
OP (2x, say) is double the chord P Q(x). The ∠OP Q is a right angle,
being an angle in a semicircle and the other angles are as marked in
the figure.
x 1 1
We then have tan A = = and ∴ sin A = √
2x 2 5
Also, if r be the radius, then
2r
x = 2r sin A = √ .
5
The sum of the areas of the two segments OP and OQ
= the area of the semicircle − ∆OP Q
πr 2 1 πr 2
= − · 2x · x = − x2
2
2
2
2 ( 2 )
πr 4r π 4
= − = r2 −
2( 5 ) 2 5
11 4 27 2
=r 2
− = r .
7 5 35
Hence the required ratio
27 2 πr 2 27 11
= r : = : = 27 : 55. ■
35 2 35 7
§ Problem 17.2.9. If each of three circles, of radius a, touch the
other two, prove that the area included between them is nearly equal
4 2
to a . ♢
25
§§ Solution. Let A, B and C be the centers of the three equal circles
and D, E and F be the points of contact of circles A and B, B and C
and C and A respectively.
Join AB, BC and CA. ABC is an equilateral triangle of side 2a.
The required area = the area of the triangle ABC - the areas of
the three equal sectors DAF , F CE and DBE Å ã
1 π a2 π
∴ the required area = (2a)2 sin − 3 ×
2 3 2 3
√ Ä√ πä

=a 3−a
2
=a 2
3−
2 2
= a2 (1.732 − 1.5708) ≈ a2 × .16
16a2 4 2
= = a . ■
100 25
§ Problem 17.2.10. Six equal circles, each of radius a, are placed
so that each touches two others, their centers being all on the cir-
cumference of another circle; prove that the area which they enclose
is ( √ )
2a2 3 3 − π . ♢
§§ Solution. Let A, B, C, D, E and F be the centers of the six
equal circles and let G, H, K, L, M and N be the points of contact of
circles A and B, B and C, C and D, D and E, E and F and F and A
respectively.
Join AB, BC, CD, DE, EF and F A.
Then ABCDEF is a regular hexagon of side 2a.
17.3. Dip of The Horizon 314

The required area = the area of the hexagon ABCDEF - the areas
of the six equal sectors GAN , HBG, KCH, LDK, M EL and N F M
4a2 π Ä πä
∴ the required area = 6 × cot − 6 a2 ×
√4 6 ( √3 )
= 6a2 3 − 2a2 π = 2a2 3 3 − π . ■

§ Problem 17.2.11. From the vertex A of a triangle, a straight line


AD is drawn making an angle θ with the base and meeting it at D.
Prove that the area common to the circumscribing circles of the tri-
angles ABD and ACD is
1( 2 )
b γ + c2 β − bc sin A cosec 2 θ,
4
where β and γ are the number of radians in the angles B and C
respectively. ♢
§§ Solution. Let O and O′ be the centers and R and R′ be the radii
of the circles respectively. We then have
AB c b
R= = and R′ = .
2 sin ∠ADB 2 sin θ 2 sin θ

Also, ∠AOD = 2∠ABD = 2β, and ∠AO D = 2∠ACD = 2γ.
The required area
= area of sector AOD − area of triangle AOD
+ area of sector AO′ D − area of triangle AO′ D
1 1
= R2 β − R2 sin 2B + R′2 γ − R′2 sin 2C
2( 2 )
1 1 2 1 2
= c 2
β − c sin 2B + b 2
γ − b sin 2C
4 sin2 θ 2 2
1( 2 )
= b γ + c β − c sin B cos B − b sin C cos C cosec 2 θ
2 2 2
4
1( 2 )
= b γ + c2 β − bc sin C cos B − bc sin B cos C cosec 2 θ
4
1[ 2 ]
= b γ + c2 β − bc sin(B + C) cosec 2 θ
4
1( 2 )
= b γ + c2 β − bc sin A cosec 2 θ. ■
4

17.3 Dip of The Horizon


[Unless otherwise stated, the earth’s radius may be taken to
be 4000 miles.]
§ Problem 17.3.1. Find in degrees, minutes and seconds, the dip
of the horizon from the top of a mountain 4200 feet high, the earth’s
radius being 21 × 106 feet. ♢
§§ Solution. The dip of the horizon
… …
2h 2 × 4400
= radians =
r 21 × 106
Ç … å
2 22 180
= · degrees
102 21 π
2 × 1.02
= × 57.2957795◦ = 1◦ 10′ 8′′ . ■
100
17.3. Dip of The Horizon 315

§ Problem 17.3.2. The lamp of a lighthouse is 196 feet high; how


far off can it be seen ? ♢
§§ Solution. The required distance
√ √ √
= 2hr = 2 × 196 × 21 × 106 f eet = 14000 42 f eet
14000 × 6.4807
= miles ≈ 17.2 miles. ■
1760 × 3
§ Problem 17.3.3. If the radius of the earth be 4000 miles, find the
height of a balloon when the dip is 1◦ .
Find also the dip when the balloon is 2 miles high. ♢
√ √
2h π 2h
§§ Solution. Since the dip = radians, we have = ,
r 180 4000
so that Ä π ä2
h = 2000 miles ≈ .61 mile.
180
Again, the dip
… … …
2h 2×2 1
= radians = =
r 4000 1000
√ Å√ ã◦
10 10
= radian = × 57.2957795 ≈ 1◦ 48′ .
100 100
[ √ ]
N. B. 10 = 3.1622777. ■

§ Problem 17.3.4. From the top of the mast of a ship, which is 66


feet above the sea, the light of a lighthouse which is known to be
132 feet high can just be seen; prove that its distance is 24 miles
nearly. ♢
§§ Solution. If x and y be the distances in miles of the horizon seen
from the top of the mast and the lighthouse respectively, we have

2 × 66 × 4000
x= miles
1760 × 3

2 × 132 × 4000
and y= miles.
1760 × 3
Hence the required distance

2 × 66 × 4000 ( √ )
= (x + y) miles = 1+ 2
1760 × 3
( √ )
= 10 1 + 2 ≈ 24 miles. ■

§ Problem 17.3.5. From the top of the mast, 66 feet above the sea,
the top of the mast of another ship can just be seen at a distance of
20 miles; prove that the heights of the masts are the same. ♢
§§ Solution. If x and y be the distances in miles of the horizon seen
from the tops of the masts respectively, we have

2 × 66 × 4000
x= miles = 10 miles
1760 × 3
and x + y = 20 miles
∴ y = 10 miles.
17.3. Dip of The Horizon 316

Hence the height of the second mast


y2 100 × 1760 × 3
= = = 66 f eet. ■
2r 2 × 4000
§ Problem 17.3.6. From the top of the mast of a ship which is 44
feet above the sea-level, the light of a lighthouse can just be seen;
after sailing for 15 minutes the light can just be seen from the deck
which is 11 feet above the sea-level; prove that the rate of sailing of
the ship is nearly 16.33 miles per hour. ♢
§§ Solution. Let S and S ′ be the positions on the sea of the vessel
at the two instants spoken of.
Let P be the top of the mast at the first instant; if we draw a
tangent P A to the earth, meeting it in A, then P A produced will
pass through the top L of the lighthouse.
At the second instant, let P ′ be the point on the deck at which L
can just be seen and hence P ′ lies on P A between P and A.
Then SP = 44 f eet and S ′ P ′ = 11 f eet.
… …
2 × 44 × 4000 2
∴ PA = miles = 10 miles
1760 × 3 3
… …
2 × 11 × 4000 1
and P ′A = miles = 10 miles
1760 × 3 6
( )
2−1
∴ SS ′ = SA − S ′ A = P A − P ′ A = 10 √ miles
√ 6
10 6 24.494897
= = = 4.082483 miles,
6 6
i.e. the ship sails ≈ 4.0825 miles in 15 minutes or 16.33 miles per
hour. ■

§ Problem 17.3.7. Prove that, if the height of the place √of observa-
3n
tion be n feet, the distance that the observer can see is miles
2
nearly. ♢
§§ Solution.
The required distance
… …
2n × 4000 100n
= miles = miles
1760 × 3 66
… …
99n 3n
≈ miles ≈ miles. ■
66 2
§ Problem 17.3.8. There are 10 million meters in a quadrant of the
earth’s circumference. Find approximately the distance at which the
top of the Eiffel tower should be visible, its height being 300 meters.

§§ Solution. If r be the radius of the earth, we have
1 2 × 107
× 2πr = 107 meters, ∴ r = meters.
4 π
The required distance

√ 2 × 107
= 2hr = 2 × 300 × meters
π
17.3. Dip of The Horizon 317


30 √
= 2 × 10 ×
4
meters = 2 × 104 × 30 × .31831 meters
π
1
≈ 2 × 104 × 3.09 meters ≈ 61800 meters ≈ 38 miles. ■
2
§ Problem 17.3.9. Three vertical posts are placed at intervals of
a mile along a straight canal, each rising to the same height above
the surface of the water. The visual line joining the tops of the two
extreme posts cuts the middle post at a point 8 inches below its top.
Find the radius of the earth to the nearest mile. ♢
§§ Solution. Take the figure of Art. 235, with T , O and T ′ represent-
ing the tops of the posts. Let T T ′ meet CA in N .
We then have, very approximately.

√ 8
1 mile = line AT = 2r · AN = 2× ×r
12 × 3 × 1760
∴ r = (6 × 3 × 220) miles = 3960 miles. ■
Chapter 18
Inverse Circular Functions

18.1 Identities and Equations


Prove that
3 8 77
§ Problem 18.1.1. sin−1 + sin−1 = sin−1 . ♢
5 17 85
(
§§ Solution. )
3 8
sin sin−1 + sin−1
(5 ) 17( ) ( ) ( )
−1 3 8 3 8
= sin sin cos sin−1 + cos sin−1 sin sin−1
… 5 … 17 5 17
3 64 9 8 3 15 4 8 77
= × 1− + 1− × = × + × =
5 289 25 17 5 17 5 17 85
3 8 77
∴ sin−1 + sin−1 = sin−1 . ■
5 17 85
( )
5 7 253
§ Problem 18.1.2. sin−1 + sin−1 = cos−1 . ♢
13 25 325
(
§§ Solution. )
5 7
cos sin−1 + sin−1
(13 ) 25 ( ) ( ) ( )
−1 5 7 5 7
= cos sin cos sin−1 − sin sin−1 sin sin−1
… 13 … 25 13 25
25 49 5 7
= 1− × 1− − ×
169 625 13 25
12 24 5 7 253
= × − × =
13 25 13 25 325
18.1. Identities and Equations 320

( )
5 7 253
∴ sin−1 + sin−1 = cos−1 . ■
13 25 325
4 3 27
§ Problem 18.1.3. cos−1 + tan−1 = tan−1 . ♢
5 5 11
§§ Solution.
4 3 3 3
cos−1 + tan−1 = tan−1 + tan−1
5 5 4 5
3 3
+ 15 + 12 27
= tan−1 4 5 = tan−1 = tan−1 .
3 3 20 − 9 11
1− ×
4 5
4 3
N. B. Such relations as cos−1 = tan−1 can be seen at once by
5 4
drawing a figure as in Art. 240. ■

4 12 33
§ Problem 18.1.4. cos−1 + cos 6−1 = cos−1 . ♢
5 13 65
(
§§ Solution. )
4 12
cos cos−1 + cos 6−1
(5 ) 13( ) ( ) ( )
−1 4 12 4 12
= cos cos cos cos−1 − sin cos−1 sin cos−1
…5 … 13 5 13
4 12 16 144 4 12 3 5 33
= × − 1− × 1− = × − × =
5 13 25 169 5 13 5 13 65
−1 4 12 −1 33
∴ cos + cos 6−1 = cos . ■
5 13 65
√ √
1−x 1+x
§ Problem 18.1.5. cos−1 x = 2 sin−1 = 2 cos−1 . ♢
2 2
§§ Solution. … …
1−x 1−x
Let sin−1 = α, ∴ sin α = .
2
( )2
1−x
Then cos 2α = 1 − 2 sin2 α = 1 − 2 =x
… 2
1−x
∴ 2α = cos−1 x, ∴ 2 sin−1 = cos−1 x.
2
… …
−1 1+x 1+x
Again, let cos = α, ∴ cos α = .
2
( ) 2
1+x
Then cos 2α = 2 cos2 α − 1 = 2 −1=x
… 2
1+x
∴ 2α = cos−1 x, ∴ 2 cos−1 = cos−1 x. ■
2
3 16 1 7
§ Problem 18.1.6. 2 cos−1 √ + cot−1 + cos−1 = π. ♢
13 63 2 25
18.1. Identities and Equations 321

§§ Solution.
3 16 1 7
2 cos−1 √ + cot−1 + cos−1
13 63 2 25

1 1−x
For, as in the previous problem, cos−1 x = tan−1 .
2 1+x
Õ
7
2 63 1−
−1
= 2 tan + tan−1 + tan−1 25
3 16 7
1+
25
4 …
−1 −1 63 18
= tan 3 + tan + tan−1
4 16 32
1−
9
12 3
12 63 3 +
= tan−1 + tan−1 + tan−1 = tan−1 5 4 + tan−1 63
5 16 4 12 3 16
1− ×
( ) 5 4
63 63 63 63
= tan−1 − + tan−1 = π − tan−1 + tan−1 = π. ■
16 16 16 16
1 1 1
§ Problem 18.1.7. tan−1 + tan−1 = sin−1 √ + cot−1 3 = 45◦ . ♢
2 3 5
1 1
1 1 +
§§ Solution. tan−1 + tan−1 = tan−1 2 3 = tan−1 1 = 45◦ .
2 3 1 1
1− ×
2 3
From a figure, we see at once that
1 1 1
tan−1 = sin−1 √ , and cot−1 3 = tan−1 . ■
2 5 3
1 1 2
§ Problem 18.1.8. tan−1 + tan−1 = tan−1 . ♢
7 13 9
§§ Solution.
1 1
tan−1 + tan−1
7 13
1 1
+ 20 2
−1
= tan 7 13 = tan−1 = tan−1 .
1 1 90 9
1− × ■
7 13
2 1 12
§ Problem 18.1.9. tan−1 = tan−1 . ♢
3 2 5
4
2 12
§§ Solution. 2 tan−1 = tan−1 3 = tan−1 .
3 4 5
1−
9
2 1 12
∴ tan−1 = tan−1 . ■
3 2 5
1 2 1 3
§ Problem 18.1.10. tan−1 + tan−1 = cos−1 . ♢
4 9 2 5
18.1. Identities and Equations 322

§§ Solution.
1 2
1 2 +
−1
tan + tan−1 = tan−1 4 9
4 9 1 2
1− ×
4 9
17 1 1
= tan−1 = tan−1 = α (say), ∴ tan α = .
34 2 2
1
1 − tan α 2 1 −
∴ cos 2α = = 4 = 3
1 + tan2 α 1 5
1+
4
3 1 3
∴ 2α = cos−1 , and α = cos−1
5 2 5
1 2 1 3
∴ tan−1 + tan−1 = cos−1 . ■
4 9 2 5
1 1 1 π
§ Problem 18.1.11. 2 tan−1 + tan−1 + 2 tan−1 = . ♢
5 7 8 4
§§ Solution.
1 1 1
2 tan−1 + tan−1 + 2 tan−1
5 ( 7 8 )
1 1 1
= 2 tan−1 + tan−1 + tan−1
5 8 7
1 1
+
= 2 tan−1 5 8 + tan−1 1 = 2 tan−1 1 + tan−1 1
1 1 7 3 7
1− ×
5 8
2
= tan −1 3 + tan−1 1 = tan−1 3 + tan−1 1
1 7 4 7
1−
9
3 1
+ π
= tan−1 4 7 = tan−1 1 = .
3 1 4
1− × ■
4 7
3 3 8 π
§ Problem 18.1.12. tan−1 + tan−1 − tan−1 = . ♢
4 5 19 4
§§ Solution.
3 3 8
tan−1 + tan−1 − tan−1
4 5 19
3 3
+
= tan −1 4 5 − tan−1 8
3 3 19
1− ×
4 5
27 8
= tan−1 − tan−1
11 19
27 8
− π
−1
= tan 11 19 = tan−1 1 = .
27 8 4
1+ × ■
11 19
1 1 1 1 π
§ Problem 18.1.13. tan−1 + tan−1 + tan−1 + tan−1 = . ♢
3 5 7 8 4
18.1. Identities and Equations 323

§§ Solution.
1 1 1 1
tan−1 + tan−1 + tan−1 + tan−1
3 5 7 8
1 1 1 1
+ + 4 3
−1 −1
= tan 3 5 + tan 7 8 = tan−1 + tan−1
1 1 1 1 7 11
1− × 1− ×
3 5 7 8
4 3
+ π
= tan−1 7 11 = tan−1 1 = .
4 3 4
1− × ■
7 11
1 1 π 1
§ Problem 18.1.14. 3 tan−1 + tan−1 = − tan−1 . ♢
4 20 4 1985
1 1 1
§§ Solution. Let tan−1 = α, tan−1 = β, and tan−1 = γ,
4 20 1985
1 1 1
∴ tan α = , tan β = , and tan γ = .
4 20 1985
3 1
3 tan α − tan3 α −
∴ tan 3α = = 4 64 = 47
1 − 3 tan2 α 3 52
1−
16
47 1
tan 3α + tan β +
∴ tan (3α + β) = = 52 20 = 992 .
1 − tan 3α tan β 47 1 993
1− ·
52 20
1
Äπ ä 1 − tan γ 1 − 992
Again, tan −γ = = 1985 = .
4 1 + tan γ 1 993
1+
1985
π
∴ 3α + β = − γ
4
1 1 π 1
∴ 3 tan−1 + tan−1 = − tan−1 . ■
4 20 4 1985
1 1 1 π
§ Problem 18.1.15. 4 tan−1 − tan−1 + tan−1 = . ♢
5 70 99 4
1 120
§§ Solution. If tan−1 = α, then tan 4α = .
5 119
[Ex. 3, Art. 240].
1 1
Let tan−1 = β, and tan−1 =γ
70 99
1 1
∴ tan β = , and tan γ = .
70 99
1 1

Then tan (β − γ) = 70 99 = 29 = 1 .
1 1 6931 239
1+ ·
70 99
tan 4α − tan (β − γ)
Again, tan [4α − (β − γ)] =
1 + tan 4α tan (β − γ)
120 1

= 119 239 = 28561 = 1 = tan π .
120 1 28561 4
1+ ·
119 239
π
∴ 4α − β + γ =
4
18.1. Identities and Equations 324

1 1 1 π
∴ 4 tan−1 − tan−1 + tan−1 = . ■
5 70 99 4
−1 120 −1 5
§ Problem 18.1.16. tan = 2 sin . ♢
119 13
120 120
§§ Solution. Let tan−1 = α, so that tan α = .
119 119
5 5 5
Let sin−1 = β, ∴ sin β = , and ∴ tan β = .
13 ( )
13 12
10
2 tan β 12 120
∴ tan 2β = = = = tan α
1 − tan2 β 25 119
1−
144
120 5
∴ α = 2β, i.e. tan−1 = 2 sin−1 . ■
119 13
m m−n π
§ Problem 18.1.17. tan−1 − tan−1 = . ♢
n m+n 4
m m
§§ Solution. Let tan−1 = α, so that tan α = .
Ä n π ä tan α − 1 m −nn
Then tan α − = =
4 tan α + 1 m+n
π m − n
∴ α − = tan−1
4 m+n
m m−n π
∴ tan−1 − tan−1 = . ■
n m+n 4
§ Problem 18.1.18.
2t 3t − t3 1 √
tan−1 t + tan−1 = tan−1 , t > 0, if t < √ or > 3
1−t 2 1 − 3t 2
3
−1 3t − t
3 1 √
= π + tan , if t > √ and < 3.
1 − 3t2 3

2t
t+
2t 1 − t2 3t − t3
§§ Solution. tan−1 t+tan−1 = tan−1 = tan−1 .
1−t 2 2t2 1 − 3t2
1−
1 − t2
1 3t − t3 3t − t3
If t < √ , then is positive and tan−1 lies between
3 1 − 3t 2 1 − 3t2

0 and 90 .◦
1 3t − t3 3t − t3
If t > √ , then is negative and tan−1 is a negative
3 1 − 3t 2 1 − 3t2
3t − t 3
angle; also π+tan−1 is a positive angle with the same tangent,
1 − 3t2
so that
2t 3t − t3
tan−1 t + tan−1 = π + tan−1 . ■
1−t 2 1 − 3t2


§ Problem 18.1.19. …
a(a + b + c) b(a + b + c)
tan−1 + tan−1
bc ca

−1 c(a + b + c)
+ tan = π. ♢
ab
18.1. Identities and Equations 325


§§ Solution. …
−1 a(a + b + c) b(a + b + c)
tan + tan−1
bc ca
… …
a(a + b + c) b(a + b + c)
+
bc ca
= tan −1

ab(a + b + c)2
1−
abc2
√ √
−1
a c(a + b + c) + b c(a + b + c)
= tan √ √
c ab − ab(a + b + c)
Ç √ å
(a + b) c(a + b + c)
= tan−1 − √
ab(a + b)

c(a + b + c)
= π − tan−1
ab
… …
−1 a(a + b + c) b(a + b + c)
∴ tan + tan−1
bc ca

c(a + b + c)
+ tan−1 = π. ■
ab
ab + 1 bc + 1 ca + 1
§ Problem 18.1.20. cot−1 +cot−1 +cot−1 = 0. ♢
a−b b−c c−a
§§ Solution.
ab + 1 a−b
cot−1 = tan−1 = tan−1 a − tan−1 b
a−b 1 + ab
bc + 1 b−c
cot−1 = tan−1 = tan−1 b − tan−1 c
b−c 1 + bc
ca + 1 c−a
and cot−1 = tan−1 = tan−1 c − tan−1 a.
c−a 1 + ca
Hence, by addition, we obtain the required result. ■

( )
§ Problem 18.1.21. tan−1 n + cot−1 (n + 1) = tan −1 n2 + n + 1 . ♢
§§ Solution.
1
tan−1 n + cot−1 (n + 1) = tan−1 n + tan−1
n+1
1
n+
n + 1
= tan−1 n
1−
n+1
2
−1 n + n + 1
( )
= tan = tan−1 n2 + n + 1 . ■
n+1−n
( ) ( )
1 1
§ Problem 18.1.22. cos 2 tan−1 = sin 4 tan−1 . ♢
7 3
1 1
§§ Solution. Let tan−1 = α, so that tan α = .
7 7
1
1 − tan α
2 1 −
Then cos 2α = = 49 = 48 = 24 .
1 + tan2 α 1 50 25
1+
49
18.1. Identities and Equations 326

1 1
Let tan−1 = β, so that tan β = .
3 3( )
2
3 3
Then tan 2β = =
1 4
1−
9 ( )
3
2 tan 2β 242
and sin 4β = = =
1 + tan2 2β 9 25
1+
( ) 16 ( )
1 1
∴ cos 2α = sin 4β, i.e. cos 2 tan−1 = sin 4 tan−1 . ■
7 3
§ Problem 18.1.23. [ ]
β
2 tan−1 tan (45◦ − α) tan
[ 2 ]
−1 sin 2α + cos β
= cos .
[ 1 + sin 2α]cos β ♢
−1 ◦ β
§§ Solution. Let tan tan (45 − α) tan = θ,
2
β 1 − tan α β
∴ tan θ = tan (45◦ − α) tan = · tan
… 2 1 + tan α 2
cos α − sin α 1 − cos β
= ·
cos α + sin α 1 + cos β
1 − sin 2α 1 − cos β
∴ tan θ =
2
·
1 + sin 2α 1 + cos β
2
tan θ 1 − cos β − sin 2α + sin 2α cos β
∴ =
1 1 + cos β + sin 2α + sin 2α cos β
1 − tan2 θ sin 2α + cos β
∴ cos 2θ = =
1 + tan2 θ 1 + sin 2α cos β
sin 2α + cos β
∴ 2θ = cos−1
1 + sin 2α cos β
[ ] [ ]
β sin 2α + cos β
∴ 2 tan−1 tan (45◦ − α) tan = cos−1 . ■
2 1 + sin 2α cos β
[ ]
§ Problem 18.1.24. tan−1 x = 2 tan−1 cosec tan−1 x − tan cot−1 x .

−1 x = θ, so that tan θ = x.
[
§§ Solution. Let tan ]
∴ 2 tan−1 cosec tan−1 x − tan cot−1 x
î Äπ äó
= 2 tan−1 cosec θ − tan −θ
2 [ ]
1 − cos θ
= 2 tan−1 [cosec θ − cot θ] = 2 tan−1
[ ] [ ] sin θ
−1 θ θ −1
= 2 tan tan =2 = θ = tan x. ■
2 2
§ Problem 18.1.25. [ ( )]
α π β
2 tan−1 tan tan −
2 4 2
sin α cos β
= tan−1 .
sin β + cos α ♢
18.1. Identities and Equations 327

[ ( )]
α π β
§§ Solution. Let tan−1 tan tan − = θ,
( 2 ) 4 2
α π β
∴ tan θ = tan tan −
2 4 2 Ä ä
π
1 − cos α 1 − cos −β 1 − cos α 1 − sin β
= · Äπ 2 ä = · .
sin α sin −β sin α cos β
(2 )( )
1 − cos α 1 − sin β
2
2 tan θ sin α cos β
∴ tan 2θ =
1 − tan2 θ
= ( ) ( )
1 − cos α 2 1 − sin β 2
1−
sin α cos β
2 sin α cos β (1 − cos α) (1 − sin β)
= ( )
(1 − cos2 α) 1 − sin2 β − (1 − cos α)2 (1 − sin β)2
2 sin α cos β
=
(1 + cos α) (1 + sin β) − (1 − cos α) (1 − sin β)
sin α cos β
=
cos α + sin β
sin α cos β
∴ 2θ = tan−1
[ ( cos
)]α + sin β
α π β sin α cos β
∴ 2 tan−1 tan tan − = tan−1 . ■
2 4 2 sin β + cos α
§ Problem… 18.1.26. Show… that …
−1 a−x −1 x−b a−x
cos = sin = cot−1
a−b a−b x−b

1 2 (a − x)(x − b)
= sin−1 .
−b ♢
√ 2 a√
−1 a−x a−x
§§ Solution. Let cos = α, so that cos α =
a−b … … a−b
√ a − x x − b
∴ sin α = 1 − cos2 α = 1− = .
a−b a−b
… … …
−1 x−b a−x x−b
∴ α = sin , i.e cos−1 = sin−1 .
a−b a−b a−b
… … …
cos α a−x x−b a−x
Again, cot α = = ÷ =
sin α a−b a−b x−b

−1 a−x
∴ α = cot ,
x−b
… … …
−1 a−x x−b a−x
i.e. cos = sin−1 = cot−1 .
a−b a−b x−b

(x − b)(a − x)
∴ sin 2α = 2 sin α cos α = 2
√ a−b
2 (x − b)(a − x)
∴ 2α = sin−1
√ a−b
1 2 (x − b)(a − x)
∴ α = sin−1
2 a−b
18.1. Identities and Equations 328

… … …
−1 a−x x−b a−x
∴ cos = sin−1 = cot−1
x−b a−b x−b

1 2 (a − x)(x − b)
sin−1
= . ■
2 a−b
x y
§ Problem 18.1.27. If cos−1 + cos−1 = α, prove that
a b
x2 2xy y2
− cos α + = sin2 α. ♢
a2 ab b2
x y
§§ Solution. Given cos−1 = α − cos−1
a b …
x Ä yä y y2
∴ = cos α − cos−1 = cos α · + sin α 1−
a b b b2

x y y2
∴ − cos α = sin α 1 − 2 .
a b b
Squaring both sides, we have
x2 2xy y2 y2
− cos α + 2 cos2 α = sin2 α − 2 sin2 α.
a2 ab b b
x2 2xy y2
∴ 2 − 2
cos α + 2 = sin α. ■
a ab b

Solve the equations :


√ √
1 + x2 − 1 − x2
§ Problem 18.1.28. tan−1 √ √ = β. ♢
1+ x2 + 1 − x2
§§ Solution. We have
√ √
1 + x2 − 1 − x2
√ √ = tan β
1+ + 1 − x2
x2

1 + x2 1 + tan β cos β + sin β
∴ √ = =
1 − x2 1 − tan β cos β − sin β
( )
1 + x2 cos β + sin β 2 1 + sin 2β
∴ = =
1 − x2 cos β − sin β 1 − sin 2β

∴ x2 = sin 2β, i.e. x = ± sin 2β. ■

π
§ Problem 18.1.29. tan−1 2x + tan−1 3x = . ♢
4
§§ Solution.
π
tan−1 2x + tan−1 3x =
4
2x + 3x
∴ tan−1 = tan−1 1
1 − 2x × 3x
5x
∴ = 1; ∴ 6x2 + 5x − 1 = 0
1 − 6x2
1
∴ (6x − 1) (x + 1) = 0; ∴ x = or − 1.
6
The latter value of x is inadmissible, by Art. 240, Ex. 4, since in this
case we should have ab > 1. ■

x−1 x+1 π
§ Problem 18.1.30. tan−1 + tan−1 = . ♢
x−2 x+2 4
18.1. Identities and Equations 329

§§ Solution. We have
x−1 x+1
+
−1 x−2 x+2
tan = tan−1 1
x−1 x+1
1− ×
x−2 x+2
(x − 1)(x + 2) + (x + 1)(x − 2)
∴ =1
x2 − 4 − (x2 − 1)
2x − 4
2
∴ = 1; ∴ 2x2 = 1
−3
1 1
∴ x2 = , i.e. x = ± √ . ■
2 2
4 3
§ Problem 18.1.31. tan−1 (x+1)+cot−1 (x−1) = sin−1 +cos−1 . ♢
5 5
§§ Solution.
4 3
tan−1 (x + 1) + cot−1 (x − 1) = sin−1 + cos−1
( ) 5 5
1 4 4 4
∴ tan−1 (x + 1) + tan−1 = tan−1 + tan−1 = 2 tan−1
x−1 3 ( ) 3 3
1 8
x+1+
x−1 3
∴ tan−1 = tan−1
x+1 16
1− 1−
x−1 9
x2 − 1 + 1 24
∴ =−
x − 1 − (x + 1) 7

x2 24 48 3
∴ = , i.e. x2 = , i.e. x = ±4 . ■
2 7 7 7
8
§ Problem 18.1.32. tan−1 (x + 1) + tan−1 (x − 1) = tan−1 . ♢
31
§§ Solution.
8
tan−1 (x + 1) + tan−1 (x − 1) = tan−1
31
x+1+x−1 8
∴ tan−1 = tan−1
1 − (x + 1)(x − 1) 31
2x 8
∴ = , i.e. 4x2 + 31x − 8 = 0
2 − x2√ 31
−31 ± 961 + 128 −31 ± 33 1
∴x= = = −8 or . ■
8 8 4
§ Problem 18.1.33. 2 tan−1 (cos x) = tan−1 (2 cosec x). ♢
§§ Solution.
Let tan−1 (cos x) = α and tan−1 (2 cosec x) = β
∴ tan α = cos x and tan β = 2 cosec x.
We are given 2α = β
∴ tan 2α = tan β
2 cos x 2 cos x 1
∴ = ; ∴ =
1 − cos2 x sin x sin2 x sin x
∴ sin x = 0 or cos x = sin x.
If sin x = 0 then x = nπ.
π
If cos x = sin x, then tan x = 1 = tan
4
18.1. Identities and Equations 330

π
∴ x = nπ + . ■
4
2
§ Problem 18.1.34. tan−1 x + 2 cot−1 x = π. ♢
3
§§ Solution.
2
tan−1 x + 2 cot−1 x = π
Ä3 π ä
Let tan−1 x = θ; then cot−1 x = −θ .
2
The equation becomes
2π π
θ + π − 2θ = , i.e. θ =
3 √ 3
π
∴ x = tan θ = tan = 3. ■
3
1
§ Problem 18.1.35. tan cos x = sin cot−1 .
−1 ♢
2
§§ Solution. Let cos−1 x = α, so that
√ cos α = x,
1 − x2
∴ tan α = .
x
1 1
Let cot−1 = β, ∴ cot α =
2 2
2
∴ sin α = √ .
5
The equation becomes

1 − x2 2
= √ .
x 5 √
5
∴ 5 − 5x2 = 4x2 , i.e. 9x2 = 5, i.e. x = .
√ 3
5
The value x = − is inadmissible; for then cos−1 x would be be-
3
π
tween and π and therefore tan cos−1 x would be negative. ■
2

§ Problem 18.1.36. cot−1 x − cot−1 (x + 2) = 15◦ . ♢


§§ Solution.
cot−1 x − cot−1 (x + 2) = 15◦
1 1
∴ tan−1 − tan−1 = 15◦
x x+2
1 1

−1 x x+2
∴ tan = 15◦
1
1+
x(x + 2)
x+2−x √
∴ 2 = tan 15◦ = 2 − 3
x + 2x + 1
2 √
∴ 2
=2− 3
(x + 1)
2 ( √ ) √ (√ )2
∴ (x + 1)2 = √ =2 2+ 3 =4+2 3= 3+1
2− 3 √ (√ )
∴ x = 3 or − 3+2 . ■

x2 − 1 2x 2π
§ Problem 18.1.37. cos−1 + tan−1 2 = . ♢
x2 + 1 x −1 3
18.1. Identities and Equations 331

§§ Solution.
x2 − 1 2x 2π
cos−1 + tan−1 2 = .
x2 + 1 x −1 3
If x = cot θ, we have
x2 − 1 cot2 θ − 1
= = cos 2θ
x2 + 1 cot2 θ + 1
2x 2 cot θ 1
and = = = tan 2θ.
x2 − 1 cot2 θ − 1 cot 2θ
Thus the equation becomes
2π 2π π
2θ + 2θ = ; ∴ 4θ = ; ∴θ=
3 3 √ 6
π
∴ x = cot θ = cot = 3. ■
6
( )
§ Problem 18.1.38. cot−1 x + cot −1 n2 − x + 1 = cot−1 (n − 1). ♢
§§ Solution. ( )
cot−1 x + cot−1 n2 − x + 1 = cot−1 (n − 1)
1 1 1
∴ tan−1 + tan−1 2 = tan−1
x n −x+1 n−1
1 1
+ 2
−1 x n −x+1 1
∴ tan = tan−1
1 1 n−1
1− · 2
x n −x+1
n2 + 1 1
∴ 2 =
n x − x2 + x − 1 n−1
∴ n2 x − x2 + x = n3 − n2 + n
( ) ( )
∴ x2 − n2 + 1 x + n n2 − n + 1 = 0
[ ( )]
∴ (x − n) x − n2 − n + 1 =0
∴ x = n or n2 − n + 1. ■

π
§ Problem 18.1.39. sin−1 x + sin−1 2x = . ♢
3
§§ Solution.
π
sin−1 x + sin−1 2x =
3
( ) 1
∴ cos sin−1 x + sin−1 2x =
√ √ 1
2
∴ 1 − x2 × 1 − 4x2 − x × 2x =
2
( )( ) 1
∴ 1 − x2 1 − 4x2 = 4x4 + 2x2 +
4
1
∴ 1 − 5x2 + 4x4 = 4x4 + 2x2 +
4
3 1 3
∴ 7x2 = ; ∴ x2 = ×
4 … 4 7
1 3
∴x=± .
2 7
The negative value is inadmissible, since x is necessarily positive.
Otherwise thus :
18.1. Identities and Equations 332

π
sin−1 2x =
− sin−1 x
3 √
Äπ ä 3√ 1
∴ 2x = sin − sin−1 x = 1 − x2 − × x
3√ ( ) 2 2
∴ 5x = 3 1 − x2 ; ∴ 25x2 = 3 − 3x2

3 1 3
∴ 28x2 = 3, i.e. x2 = , i.e. x = . ■
28 2 7
5 12 π
§ Problem 18.1.40. sin−1 + sin−1 = . ♢
x x 2
§§ Solution.
5 12 π
sin−1+ sin−1 =
x x 2
5 12
Let sin−1 = α and sin−1 =β
x x
5 12
∴ sin α = and sin β = .
x x
π
Now α + β = ; ∴ sin β = cos α
2
∴ sin2 β = 1 − sin2 α; ∴ sin2 α + sin2 β = 1
25 144
∴ 2 + 2 = 1; ∴ x2 = 169; ∴ x = 13. ■
x x
a b c d π
§ Problem 18.1.41. tan−1 + tan−1 + tan−1 + tan−1 = . ♢
x x x x 2
§§ Solution. We have
a b c d
+ +
tan−1 x x + tan−1 x x = π
ab cd 2
1− 2 1− 2
x x
(a + b)x π (c + d)x
∴ tan−1 2 = − tan−1 2
x − ab 2 x − cd
(a + b)x x2 − cd
∴ 2 =
x − ab
( (c + d)x
)( )
∴ (a + b)(c + d)x2 = x2 − ab x2 − cd
∴ x4 − (ab + ac + ad + bc + bd + cd)x2 + abcd = 0. ■

x x
§ Problem 18.1.42. sec−1 − sec−1 = sec−1 b − sec−1 a. ♢
a b
§§ Solution. The given equation may be written
a 1 b 1
cos−1 + cos−1 = cos−1 + cos−1
( x a) ( x b )
a 1 b 1
∴ cos cos−1 + cos−1 = cos cos−1 + cos−1
… x a
… …x b

a 1 a2 1 b 1 b2 1
∴ × − 1− × 1− = × − 1− × 1−
x a x2 a2 x b x2 b2
( )( ) ( )( )
x2 − a2 a2 − 1 x2 − b2 b2 − 1
∴ =
( a22 x2 2 ) ( b 2 x2 )
∴ a −b x = a2 b2 a2 − b2
2

∴ x2 = a2 b2 , i.e. x = ab. ■
18.1. Identities and Equations 333

§ Problem 18.1.43. cosec −1 x = cosec −1 a + cosec −1 b. ♢


§§ Solution. The given equation may be written
1 1 1
sin−1 = sin−1 + sin−1
( x) ( a b )
1 1 1
∴ sin sin−1 = sin sin−1 + sin−1
… x… a b
1 1 1 1 1 1 Ä√ 2 √ ä
∴ = 1− + 1− = b − 1 + a2 − 1
x a b2 b a 2 ab
ab
∴x= √ √ . ■
b2 − 1 + a2 − 1
1 − a2 1 − b2
§ Problem 18.1.44. 2 tan−1 x = cos−1 − cos−1 . ♢
1 + a2 1 + b2
§§ Solution.
1 − a2 1 − b2
∵ cos−1 2
= 2 tan−1 a and cos−1 = 2 tan−1 b,
1+a 1 + b2
the equation becomes
a−b
tan−1 x = tan−1 a − tan−1 b = tan−1
1 + ab
a−b
∴x= . ■
1 + ab
Chapter 19
On Some Simple Trigonometrical
Series

19.1 Simple Series


Sum the series :
§ Problem 19.1.1. cos θ + cos 3θ + cos 5θ + . . . to n terms. ♢
§§ Solution. By Art. 242, we have
cos θ + cos 3θ + cos 5θ + . . . to n terms
[ ]
n−1 n · 2θ
cos θ + · 2θ sin
2 2
=

sin
2
cos nθ sin nθ 1
= = sin 2nθ cosec θ. ■
sin θ 2
A 7A
§ Problem 19.1.2. cos + cos 2A + cos + . . . to n terms. ♢
2 2
§§ Solution.
A 7A
cos + cos 2A + cos + . . . to n terms
2 [ 2 ]
A n − 1 3A n 3A
cos + · sin ·
2 2 2 2 2
=
1 3A
sin ·
2 2
3n − 1 3n 3
= cos A sin A cosec A. ■
4 4 4
19.1. Simple Series 336

Prove that :
sin α + sin 2α + sin 3α + . . . + sin nα n+1
§ Problem 19.1.3. = tan α.
cos α + cos 2α + . . . + cos nα 2

§§ Solution. The given expression
 [ n−1
]
nα  
[
n−1
]
nα 
 sin α + α sin   cos α + α sin 
2 2 2 2
= α ÷ α
 sin   sin 
[ 2] 2
n−1 n+1
= tan α + α = tan α. ■
2 2

§ Problem 19.1.4.
sin α + sin 3α + sin 5α + . . . + sin(2n − 1)α
cos α + cos 3α + cos 5α + . . . + cos(2n − 1)α
= tan nα. ♢
§§ Solution. The given expression ]
 [ n−1 n · 2α 
 sin α + · 2α sin 
2 2
=
 sin
2α 
 [ ]
n · 2α 
2
n−1
 cos α + · 2α sin 
2 2
÷
 sin
2α 
2
= tan [α + (n − 1)α] = tan nα. ■

§ Problem 19.1.5.
sin α − sin (α + β) + sin (α + 2β) + . . . to n terms
cos α − cos (α + β) + cos (α + 2β) + . . . to n terms
[ ]
n−1
= tan α + (β + π) .
2 ♢
§§ Solution. The given expression
sin α + sin (α + β + π) + sin (α + 2β + 2π) + . . . to n terms
=
cos α + cos (α + β + π) + cos (α + 2β + 2π) + . . . to n terms
[ ]
n−1 n 1
sin α + (β + π) sin (β + π) cosec (β + π)
2 2 2
= [ ]
n−1 n 1
cos α + (β + π) sin (β + π) cosec (β + π)
[ 2 ] 2 2
n−1
= tan α + (β + π) . ■
2

Sum the following series :


π 3π 5π
§ Problem 19.1.6. cos +cos +cos +. . . to n terms.
2n + 1 2n + 1 2n + 1

§§ Solution.
π 3π 5π
cos + cos + cos + . . . to n terms
2n + 1 2n + 1 2n + 1
19.1. Simple Series 337

{ [( ) ( )] }
π n−1
2π n 2π
cos + · sin ·
2n + 1 2
2n + 1 2 2n + 1
= π
sin
2n + 1
nπ nπ π
= cos sin cosec
2n + 1 2n + 1 2n + 1
1 2nπ π
= sin cosec
2 2n + 1 2n + 1
1 π π 1
= sin cosec = . ■
2 2n + 1 2n + 1 2

§ Problem 19.1.7. cos α − cos (α + β) + cos (α + 2β) − . . . to 2n terms.



§§ Solution.
cos α − cos (α + β) + cos (α + 2β) − . . . to 2n terms
= cos α + cos (α + β + π) + cos (α + 2β + π) + . . . to 2n terms
[ ]
2n − 1 2n 1
= cos α + (β + π) sin (β + π) cosec (β + π)
[ ( 2 ) 2] 2 ( )
1 π β π
= cos α + n − β + nπ − sin (nβ + nπ) cosec +
[ ( 2) ] 2 2 2
1 β
= sin α + n − β + nπ sin (nβ + nπ) sec
[ ( 2) ] 2
1 β
= sin α + n − β cos nπ · sin nβ cos nπ sec
[ ( 2 ) ] 2
1 β
= sin α + n − β sin nβ sec , ∵ cos2 nπ = +1. ■
2 2
n−4 n−6
§ Problem 19.1.8. sin θ + sin θ + sin θ + . . . to n terms. ♢
n−2 n−2
§§ Solution.
n−4 n−6
sin θ + sin θ + sin θ + . . . to n terms
[ n − 2 ( −2
n)] ( ) ( )
n−1 −2θ n −2θ −θ
= sin θ + sin cosec
[ 2 ]n − 2 2 n−2 n−2
n−1 nθ θ
= sin θ − θ sin cosec
( n− )2 n−2 n−2
−θ nθ θ nθ
= sin sin cosec = − sin .
n−2 n−2 n−2 n−2 ■

§ Problem 19.1.9. cos x + sin 3x + cos 5x + sin 7x + . . . + sin(4n − 1)x. ♢


§§ Solution.
cos x + sin 3x + cos 5x + sin 7x + . . . + sin(4n − 1)x
= (cos x + cos 5x + . . . to n terms)
+ (sin 3x + sin 7x + . . . to n terms)
[ ] Ä ä
n−1 n 4x
= cos x + · 4x sin · 4x cosec
2[ 2 ]
n−1 Ä n 2ä 4x
+ sin 3x + · 4x sin · 4x cosec
2 2 2
= {cos(2n − 1)x + sin(2n + 1)x} sin 2nx cosec 2x
= (cos 2nx cos x + sin 2nx sin x
+ sin 2nx cos x + cos 2nx sin x) sin 2nx cosec 2x
= (cos 2nx + sin 2nx) (cos x + sin x) sin 2nx cosec 2x. ■
19.1. Simple Series 338

§ Problem 19.1.10.
sin α sin 2α + sin 2α sin 3α
+ sin 3α sin 4α + . . . to n terms. ♢
§§ Solution.
If
S = sin α sin 2α + sin 2α sin 3α + sin 3α sin 4α + . . . to n terms
then
2S = cos α − cos 3α + cos α − cos 5α + cos α − cos 7α + . . . to n terms
= n cos α − (cos 3α + cos 5α + cos 7α + . . . to n terms)
= n cos α − cos [3α + (n − 1)α] sin nα cosec α
n
= sin 2α cosec α − cos(n + 2)α sin nα cosec α
2
1
= [n sin 2α − sin(2n + 2)α + sin 2α] cosec α
2
1
∴ S = [(n + 1) sin 2α − sin(2n + 2)α] cosec α. ■
4
§ Problem 19.1.11.
cos α sin 2α + sin 2α cos 3α + cos 3α sin 4α
+ sin 4α cos 5α + . . . to 2n terms. ♢
§§ Solution.
If
S = cos α sin 2α + sin 2α cos 3α + cos 3α sin 4α
+ sin 4α cos 5α + . . . to 2n terms
then
2S = sin 3α + sin α + sin 5α − sin α + sin 7α + sin α
+ sin 9α − sin α + . . .
= sin 3α + sin 5α + sin 7α + sin 9α + . . . to 2n terms
= sin [3α + (2n − 1)α] sin 2nα cosec α
1
∴ S = sin(2n + 2)α sin 2nα cosec α. ■
2
§ Problem 19.1.12.
sin α sin 3α + sin 2α sin 4α
+ sin 3α sin 5α + . . . to n terms. ♢
§§ Solution.
S = sin α sin 3α + sin 2α sin 4α + sin 3α sin 5α + . . . to n terms
∴ 2S = cos 2α − cos 4α + cos 2α − cos 6α + cos 2α − cos 8α + . . .
= n cos 2α − (cos 4α + cos 6α + cos 8α + . . . to n terms)
= n cos 2α − cos [4α + (n − 1)α] sin nα cosec α.
n 1
∴ S = cos 2α − cos(n + 3)α sin nα cosec α. ■
2 2
§ Problem 19.1.13.
cos α cos β + cos 3α cos 2β
+ cos 5α cos 3β + . . . to n terms. ♢
19.1. Simple Series 339

§§ Solution.
S = cos α cos β + cos 3α cos 2β + cos 5α cos 3β + . . . to n terms
∴ 2S = cos (α − β) + cos (α + β) + cos (3α − 2β) + cos (3α + 2β)
+ cos (5α − 3β) + cos (5α + 3β) + . . .
= {cos (α − β) + cos (3α − 2β) + cos (5α − 3β) + . . . to n terms}
+ {cos (α + β) + cos (3α + 2β) + cos (5α + 3β) + . . . to n terms}
[ ]
n−1 n 1
= cos (α − β) + (2α − β) sin (2α − β) cosec (2α − β)
[ 2 ] 2 2
n−1 n 1
+ cos (α + β) + (2α + β) sin (2α + β) cosec (2α + β)
[ 2] ( )2 ( ) 2
n+1 nβ β
cos nα − β sin nα − sin α +
[ 2 ] ( 2 ) ( 2 )
n+1 nβ β
+ cos nα + β sin nα + sin α −
2 2 2
=
2α − β 2α + β
sin sin
( )2 2
n+1
cos nα − β
[ ( 2 ) ( )]
n+1 n−1
cos nα − α − β − cos nα + α − β
( 2) 2
n+1
+ cos nα + β
[ ( 2 ) ( )]
n+1 n−1
cos nα − α + β − cos nα + α + β
2 2
=
(cos β − cos 2α)
cos [2nα − α − (n + 1)β]
+ cos α − cos (2nα + α − nβ) − cos (α − β)
+ cos [2nα − α + (n + 1)β]
+ cos α − cos (2nα + α + nβ) − cos (α + β)
∴ 4S =
(cos β − cos 2α)
cos (2nα − α) cos(n + 1)β − cos (2nα + α) cos nβ
+ cos α (1 − cos β)
∴S= .
2 (cos β − cos 2α) ■

§ Problem 19.1.14. sin2 α + sin2 2α + sin2 3α + . . . to n terms. ♢


§§ Solution.
S = sin2 α + sin2 2α + sin2 3α + . . . to n terms
∴ 2S = 1 − cos 2α + 1 − cos 4α + 1 − cos 6α + . . .
= n − (cos 2α + cos 4α + cos 6α + . . . to n terms)
= n − cos [2α + (n − 1)α] sin nα cosec α
= {n sin α − cos(n + 1)α sin nα} cosec α
∴ 4S = {2n sin α − sin(2n + 1)α + sin α} cosec α
1
∴ S = {(2n + 1) sin α − sin(2n + 1)α} cosec α. ■
4
§ Problem 19.1.15. sin2 θ+sin2 (θ + α)+sin2 (θ + 2α)+. . . to n terms.

§§ Solution.
S = sin2 θ + sin2 (θ + α) + sin2 (θ + 2α) + . . . to n terms
19.1. Simple Series 340

∴ 2S = 1 − cos 2θ + 1 − cos (2θ + 2α) + 1 − cos (2θ + 4α) + . . .


= n − {cos 2θ + cos (2θ + 2α) + cos (2θ + 4α) + . . . to n terms}
= n − cos [2θ + (n − 1)α] sin nα cosec α
n 1
∴ S = − cos [2θ + (n − 1)α] sin nα cosec α. ■
2 2
§ Problem 19.1.16. sin3 α + sin3 2α + sin3 3α + . . . to n terms. ♢
§§ Solution. ∵ sin 3α = 3 sin α − 4 sin3 α, i.e. 4 sin3 α = 3 sin α − sin 3α,
S = sin3 α + sin3 2α + sin3 3α + . . . to n terms
∴ 4S = 3 (sin α + sin 2α + sin 3α + . . . to n terms)
− (sin 3α + sin 6α + sin 9α + . . . to n terms)
î αó nα α
= 3 sin α + (n − 1) sin cosec
[ 2 ] 2 2
n−1 3nα 3α
− sin 3α + · 3α sin cosec
2 2 2
3 (n + 1)α nα α
∴ S = sin sin cosec
4 2 2 2
1 3(n + 1)α 3nα 3α
− sin sin cosec . ■
4 2 2 2
§ Problem 19.1.17. sin4 α + sin4 2α + sin4 3α + . . . to n terms. ♢
§§ Solution.
S = sin4 α + sin4 2α + sin4 3α + . . . to n terms
( )2
∵ 4 sin4 α = 2 sin2 α = (1 − cos 2α)2
= 1 − 2 cos 2α + cos2 2α
∴ 8 sin4 α = 2 − 4 cos 2α + (1 + cos 4α)
= 3 − 4 cos 2α + cos 4α
∴ 8S = 3n − 4 (cos 2α + cos 4α + cos 6α + . . . to n terms)
+ (cos 4α + cos 8α + cos 12α + . . . to n terms)
[ ]
n−1
= 3n − 4 cos 2α + · 2α sin nα cosec α
[ 2 ]
n−1
+ cos 4α + · 4α sin 2nα cosec 2α
2
1
∴ S = {3n − 4 cos(n + 1)α sin nα cosec α
8
+ cos(2n + 2)α sin 2nα cosec 2α} . ■

§ Problem 19.1.18. cos4 α + cos4 2α + cos4 3α + . . . to n terms. ♢


§§ Solution.
Let S = cos4 α + cos4 2α + cos4 3α + . . . to n terms
( )2
∵ 4 cos4 α = 2 cos2 α = (1 + cos 2α)2
= 1 + 2 cos 2α + cos2 2α
∴ 8 cos α = 2 + 4 cos 2α + (1 + cos 4α)
4

= 3 + 4 cos 2α + cos 4α
∴ 8S = 3n + 4 (cos 2α + cos 4α + . . . to n terms)
+ (cos 4α + cos 8α + . . . to n terms)
= 3n + 4 cos [2α + (n − 1)α] sin nα cosec α
+ cos [4α + (n − 1)2α] sin 2nα cosec 2α
19.1. Simple Series 341

1
∴S= {3n + 4 cos(n + 1)α sin nα cosec α
8
+ cos(2n + 2)α sin 2nα cosec 2α} . ■

§ Problem 19.1.19.
cos θ cos 2θ cos 3θ + cos 2θ cos 3θ cos 4θ
+ . . . to n terms. ♢
§§ Solution.
S = cos θ cos 2θ cos 3θ + cos 2θ cos 3θ cos 4θ + . . . to n terms
∴ 2S = cos 2θ (cos 2θ + cos 4θ) + cos 3θ (cos 2θ + cos 6θ) + . . . to n terms
∴ 4S = 2 cos 2θ (cos 2θ + cos 3θ + . . . to n terms)
+ cos 2θ + cos 6θ + cos 3θ + cos 9θ + . . . to 2n terms
= (2 cos 2θ + 1) (cos 2θ + cos 3θ + . . . to n terms)
+ (cos 6θ + cos 9θ + . . . to n terms)
[ ]
n−1 nθ θ
= (2 cos 2θ + 1) cos 2θ + θ sin cosec
[ ] 2 2 2
n−1 3nθ 3θ
+ cos 6θ + · 3θ sin cosec
( 2 2 )
2
nθ n+3 n+3 θ
= sin 2 cos 2θ cos θ + cos θ cosec
2 2 2 2
3nθ 3n + 9 3θ
+ sin cos θ cosec
2( 2 2 )
1 nθ n−1 n+7 n+3 θ
∴ S = sin cos θ + cos θ + cos θ cosec
4 2 2 2 2 2
1 3nθ 3n + 9 3θ
+ sin cos θ cosec . ■
4 2 2 2
§ Problem 19.1.20.
sin α sin (α + β) − sin (α + β) sin (α + 2β)
+ . . . to 2n terms. ♢
§§ Solution.
S = sin α sin (α + β) − sin (α + β) sin (α + 2β) + . . . to 2n terms
∴ 2S = cos β − cos (2α + β) − cos β + cos (2α + 3β) + . . . to 2n terms
= − {cos (2α + β) − cos (2α + 3β) + . . . to 2n terms}
= − {cos (2α + β) + cos (2α + 3β + π) + . . . to 2n terms}
[ ] Ä
2n − 1 πä
= − cos (2α + β) + (2β + π) sin n (2β + π) cosec β +
Ä 2
πä
2
= − cos 2α + 2nβ + nπ − sin (2nβ + nπ) sec β
2
= − sin (2α + 2nβ + nπ) sin (2nβ + nπ) sec β

= − sin (2α + 2nβ) cos nπ sin 2nβ cos nπ sec β


1
∴ S = − sin (2α + 2nβ) sin 2nβ sec β, ∵ cos2 nπ = +1. ■
2
§ Problem 19.1.21. From the sum of the series
sin α + sin 2α + sin 3α + . . . to n terms,
deduce (by making α very small) the sum of the series
1 + 2 + 3 + . . . + n. ♢
19.1. Simple Series 342

§§ Solution.
sin α + sin 2α + sin 3α + . . . + sin nα
[ ]
n−1 nα n+1 nα
sin α + α sin sin α sin
2 2 2 2
= α = α
sin sin
2 2
Dividing by α, we have
sin α sin 2α sin nα
+2· + ... + n ·
α 2α nα
n+1 nα
n + 1 sin 2 α n sin 2
· · · nα
2 n+1 2
α
2 2
= α
1 sin
· α2
2
2
Hence, making α very small, we have
n+1 n
·
1 + 2 + ... + n = 2 2 = n(n + 1) .
1 2 ■
2
§ Problem 19.1.22. From the result of the example of Art. 241, de-
duce the sum of
1 + 3 + 5 . . . to n terms. ♢
§§ Solution.
sin2 nα
sin α + sin 3α + sin 5α + . . . + sin(2n − 1)α = .
sin α
Dividing by α, we have
( )
sin nα 2
n2
sin α sin 3α sin(2n − 1)α nα
+3· + . . . + (2n − 1) = .
α 3α (2n − 1)α sin α
α
Hence, making α very small, we have
1 + 3 + 5 . . . to n terms = n2 . ■


§ Problem 19.1.23. If α = ,
17
prove that 2 (cos α + cos 2α + cos 4α + cos 8α)
and 2 (cos 3α + cos 5α + cos 6α + cos 7α)
are the roots of the equation
x2 + x − 4 = 0. ♢

§§ Solution. ∵ α = , ∴ 17α = 2π.
17
Let p = 2 (cos α + cos 2α + cos 4α + cos 8α) ,
and q = 2 (cos 3α + cos 5α + cos 6α + cos 7α) .
Then, by the relation 17α = 2π, we have
p = 2 (cos α + cos 15α + cos 13α + cos 9α)
and q = 2 (cos 3α + cos 5α + cos 11α + cos 7α) .
∴ p + q = 2 (cos α + cos 3α + cos 5α + . . . to 8 terms)
19.1. Simple Series 343

2 cos (α + 7α) sin 8α


=
sin α
2 cos 8α sin 8α sin 16α
= = = −1.
sin α sin α
 cos 4α + cos 2α + cos 18α + cos 12α + cos 16α + cos 10α
+ cos 12α + cos 6α + cos 6α + cos 4α + cos 20α + cos 10α
+ cos 18α + cos 8α + cos 14α + cos 4α + cos 12α + cos 10α
∴ pq = 2  
+ cos 26α + cos 4α + cos 24α + cos 2α + cos 20α + cos 2α 
+ cos 8α + cos 6α + cos 22α + cos 8α + cos 20α + cos 6α
+ cos 16α + cos 2α
∴ pq = 8 (cos 2α + cos 4α + cos 6α + . . . to 8 terms) ,
∵ 17α = 2π and 34α = 4π
8 cos (2α + 7α) sin 8α
=
sin α
8 cos 8α sin 8α 4 sin 16α
= = = −4.
sin α sin α

Hence the equation whose roots are p and q is x2 + x − 4 = 0.

§ Problem 19.1.24. ABCD . . . is a regular polygon of n sides which


is inscribed in a circle, whose center is O and whose radius is r and
P is any point on the arc AB such that ∠P OA is θ. Prove that
PA · PB + PA · PC + PA · PD + ... + PB · PC + ...
[ ( ) ]
θ π π
= r2 2 cos2 − cosec 2 −n .
2 2n 2n ♢

§§ Solution. Each of the angles ∠AOB, ∠BOC . . . = = α, say.
n
Hence we have
θ α−θ 2α − θ
P A = 2r sin , P B = 2r sin , P C = 2r sin , ...
2 2 2
∴ PA + PB + PC + ...
[ ( ) ( ) ]
θ π θ 2π θ
= 2r sin + sin − + sin − + . . . to n terms
( 2 ) n 2 n 2
θ θ
∵ sin = sin π − = the last term,
2 [ (2 ) ( ) ]
π θ 2π θ
= 2r sin − + sin − + . . . to n terms
[ (n 2) n ]2
π θ n−1 π n π π
= 2r sin − + · sin · cosec
( n 2 2 ) n 2 n 2n
π θ π π π π
= 2r sin − + − sin cosec
(n 2 ) 2 2n 2 2n
π θ π
= 2r cos − cosec
2n 2 2n
( )
θ π π
∴ (P A + P B + P C + . . .)2 = 4r2 cos2 − cosec 2 (19.1)
2 2n 2n
∴ P B2 + P C 2 + . . .
[ ( ) ( ) ]
π θ 2π θ
= 4r2 sin2 − + sin2 − + . . . to n terms
[ n( 2 ) n (2 ) ]
2π 4π
= 2r2 1 − cos − θ + 1 − cos − θ + ...
[ (n )n ]
2π n − 1 2π π
= 2r2 n − cos −θ+ · sin π cosec
n 2 n n
19.1. Simple Series 344

∴ P B 2 + P C 2 + . . . = 2r2 n, ∵ sin π = 0 (19.2)


Hence, subtracting Eq. (19.2) from Eq. (19.1) and canceling 2 on
each side, we have
PA · PB + PA · PC + PA · PD + ... + PB · PC + ...
[ ( ) ]
θ π π
= r2 2 cos2 − cosec 2 −n . ■
2 2n 2n
§ Problem 19.1.25. Two regular polygons, each of n sides, are cir-
cumscribed to and inscribed in a given circle. If an angular point of
one of them be joined to each of the angular points of the other, then
the sum of the squares of the straight lines so drawn is to the sum
of the areas of the polygons as

2 : sin . ♢
n
§§ Solution. Let O be the center of the circle and let the angular
point A of the in-polygon be joined to each of the angular points
(A′ , B ′ , . . .) of the circum-polygon.
Ä πä
Let OA = r and OA′ = r′ = r sec .
n
Let the ∠AOK = α, where K is the point of contact of the side
A′ B ′ of the circum-polygon with the circle. ÄWe have ä
π
AA′2 = r2 + r′2 − 2rr′ cos −α
(n )

and AB ′2 = r2 + r′2 − 2rr′ cos −α
n
...........................
For the sum of the series of cosines vanishes, since the difference of

the angles = ,
n ( )
∴ AA′2 + AB ′2 + . . . = n r2 + r′2
π
Ä ä cos2 + 1
2 2 π 2 n
= nr 1 + sec = nr π .
n cos2
n
Also, the sum of the areas of the polygons
n 2π π
= r2 sin + nr2 tan
2 ( n n )
π
π π sin
= nr 2
sin cos + n
n n π
cos
n
cos 2 π +1
π n
= nr2 sin · π .
n cos
n
1 2
Hence the required ratio = π π = .
sin cos 2π ■
sin
n n n
§ Problem 19.1.26. A1 , A2 , . . . A2n+1 are the angular points of a
regular polygon inscribed in a circle and O is any point on the cir-
cumference between A1 and A2n+1 ; prove that
OA1 + OA3 + . . . + OA2n+1 = OA2 + OA4 + . . . + OA2n . ♢
19.1. Simple Series 345

§§ Solution. Let C be the center and r be the radius of the circle;



the ∠OCA1 = α, and the ∠A1 CA2 = θ = . We have
2n + 1
OA1 + OA3 + . . . + OA2n+1
α Äα ä
= 2r sin + 2r sin + θ + . . . to (n + 1) terms
(
2 ) 2
α nθ (n + 1)θ θ
= 2r sin + sin cosec
2 2 2 2
OA2 + OA4 + . . . + OA2n
α+θ α + 3θ
= 2r sin + 2r sin + . . . to n terms
[ 2 2]
α+θ (n − 1)θ nθ θ
= 2r sin + sin cosec
( 2 ) 2 2 2
α nθ nθ θ
= 2r sin + sin cosec .
2 2 2 2
(n + 1)θ nθ (2n + 1)θ
Now + = = π.
2 2 2
(n + 1)θ nθ
∴ sin = sin .
2 2 ■
Hence the series are equal.
§ Problem 19.1.27. If perpendiculars be drawn on the sides of a
regular polygon of n sides from any point on the inscribed circle
whose radius is a, prove that
2 Ä p ä2 2 Ä p ä3
Σ = 3, and Σ = 5. ♢
n a n a
§§ Solution. Let KA, KB, KC . . . be the perpendiculars from K,
the center of the circle, on the sides of the polygon.
Let O be the point on the circumference of the circle (take O be-
tween A and B) and OM , ON . . . be the perpendiculars on the sides
(produced) passing through B, C, . . .; also let the ∠AKO = α.

We have the ∠AKB = = ∠BKC = . . .
n [ ( )]

∴ p1 = OM = KB − OK cos ∠OKB = a 1 − cos −α (19.3)
n
[ ( )]

Similarly, p2 = a 1 − cos −α , and so on.
n
From Eq. (19.3): ( ) ( )
Ä p1 ä2 2π 2π
= 1 − 2 cos − α + cos2 −α
Ä pa1 ä2 (n ) ( n )
2π 4π
∴2 = 3 − 4 cos − α + cos − 2α
Ä pa1 ä2 [ n( ) (
n ) ]
2π 4π
∴ 2Σ = 3n − 4 cos − α + cos − α + . . . to n terms
a [ ( n) ( n) ]
4π 8π
+ cos − 2α + cos − 2α + . . . to n terms
n n
= 3n, ∵ each series vanishes (Art. 243) .
2 Ä p ä2
∴ Σ = 3.
n a
Again,
Ä p1 ä3 ( ) ( ) ( )
2π 2π 2π
= 1 − 3 cos − α + 3 cos2 − α − cos3 −α
a n n n
19.1. Simple Series 346

( ) [ ( )]
2π 3 4π
= 1 − 3 cos −α + 1 + cos −α
[n ( 2 ) (n )]
1 2π 2π
− cos 3 − α + 3 cos −α
4 n n
Ä p ä3 3
∴Σ = n + n. ∵ the series vanishes as bef ore,
2 Ä p ä3
a 2
∴ Σ = 5. ■
n a
Chapter 20
Elimination

20.1 Elimination of Unknown Quantity


Eliminate θ from the equations :
§ Problem 20.1.1. a cos θ + b sin θ = c, and b cos θ − a sin θ = d. ♢
§§ Solution. Solving for cos θ and sin θ, we have
cos θ sin θ 1
= = 2
ac + bd bc − ad a + b2
(ac + bd)2 + (bc − ad)2
∴ 1 = cos2 θ + sin2 θ =
(a2 + b2 )2
a c + b d + 2abcd + b2 c2 + a2 d2 − 2abcd
2 2 2 2
=
2 2 2
( 2 2 ) 2 ((a2 + b2 )) 2
a +b c + a +b d c2 + d2
= 2
= 2
(a2 + b2 ) a + b2
∴ a2 + b2 = c2 + d2 .
Otherwise thus :
Squaring both sides of each of the given equations, we have
a2 cos2 θ + 2ab cos θ sin θ + b2 sin2 θ = c2 ,
and b2 cos2 θ − 2ab cos θ sin θ + a2 sin2 θ = d2 .
Hence, by addition, we have
( ) ( )
a2 cos2 θ + sin2 θ + b2 cos2 θ + sin2 θ = c2 + d2
∴ a2 + b2 = c2 + d2 . ■

§ Problem 20.1.2. x = a cos (θ − α) and y = b cos (θ − β). ♢


20.1. Elimination of Unknown Quantity 348

§§ Solution. From the given equations, we have


x
= cos θ cos α + sin θ sin α
a
y
and = cos θ cos β + sin θ sin β.
b
x y
∴ sin β − sin α = cos θ sin (β − α)) (20.1)
a b
x y
∴ cos β − cos α = sin θ sin (α − β)) (20.2)
a b
Squaring and adding Eq. (20.1) and Eq. (20.2), we have
x 2 y 2 2xy
+ 2 − (cos α cos β + sin α sin β) = sin2 (α − β)
a2 b ab
x2 y2 2xy
∴ 2 + 2 − cos (α − β) = sin2 (α − β) . ■
a b ab
§ Problem 20.1.3. a cos 2θ = b sin θ and c sin 2θ = d cos θ. ♢
§§ Solution.
a cos 2θ = b sin θ (20.3)
c sin 2θ = d cos θ (20.4)
From Eq. (20.4), we have
2c sin θ cos θ = d cos θ
d
∴ sin θ = (20.5)
( ) 2c
But Eq. (20.3) is a 1 − 2 sin2 θ = b sin θ (20.6)
Substituting from Eq. (20.5) in Eq. (20.6), we have
Å ã
d2 bd
a 1− =
2c2 2c
( )
∴ a 2c2 − d2 = bdc. ■

§ Problem 20.1.4. a sin α − b cos α = 2b sin θ and a sin 2α − b cos 2θ =


a. ♢
§§ Solution. The second equation may be written
2a sin α cos α − b + 2b sin2 θ = a (20.7)
From the first equation, we have
a sin α − b cos α
sin θ = (20.8)
2b
Hence, by substituting from Eq. (20.8) in Eq. (20.7), we have
(a sin α − b cos α)2
2a sin α cos α − b + =a
2b
∴ 4ab sin α cos α − 2b + a sin α − 2ab sin α cos α + b2 cos2 α = 2ab
2 2 2

∴ (a sin α + b cos α)2 = 2ab + 2b2



∴ a sin α + b cos α = 2b(a + b). ■

√ sin2 θ cos2 θ
§ Problem 20.1.5. x sin θ − y cos θ = x2 + y 2 and + =
a2 b2
1
. ♢
x2 + y 2
20.1. Elimination of Unknown Quantity 349

§§ Solution. Squaring the first equation, we have


x2 sin2 θ − 2xy sin θ cos θ + y 2 cos2 θ = x2 + y 2
( ) ( )
∴ x2 1 − sin2 θ + y 2 1 − cos2 θ + 2xy sin θ cos θ = 0
∴ x2 cos2 θ + y 2 sin2 θ + 2xy sin θ cos θ = 0
∴ (x cos θ + y sin θ)2 = 0
x
∴ x cos θ + y sin θ = 0, i.e. tan θ = − .
y
x2 y2
∴ sin θ = 2
2 2
and cos θ = 2 .
x + y2 x + y2
2 2
Substituting these values of sin θ and cos θ in the second equation,
we have Å 2 ã
1 x y2 1
+ = 2
x2 + y 2 a2 b2 x + y2
x2 y2
∴ 2
+ 2 = 1. ■
a b
§ Problem 20.1.6.
x cos θ y sin θ
+ =1
a √ b
and x sin θ − y cos θ = a2 sin2 θ + b2 cos2 θ. ♢
§§ Solution. Squaring the given equations, we have
x2 2xy y2
2
cos2 θ + sin θ cos θ + 2 sin2 θ = 1 = sin2 θ + cos2 θ (20.9)
a ab b
and x sin θ − 2xy sin θ cos θ + y cos θ = a sin θ + b cos θ (20.10)
2 2 2 2 2 2 2 2

From Eq. (20.9) and Eq. (20.10), we have


x2 − a2 2xy y 2 − b2
2
cos2 θ + sin θ cos θ + sin2 θ = 0 (20.11)
a ab b2
x −a
2 2 2xy y −b
2 2
and sin2 θ − sin θ cos θ + cos2 θ = 0 (20.12)
ab ab ab
From Eq. (20.11)Å and Eq. (20.12), ã by addition,
Å we have ã
x2 − a2 cos2 θ sin2 θ y 2 − b2 sin2 θ cos2 θ
+ + + =0
a a b b b a
x2 y2 x2 y2
∴ −a+ − b = 0; i.e. + = a + b. ■
a b a b
§ Problem 20.1.7. sin θ − cos θ = p and cosec θ − sin θ = q. ♢
§§ Solution.
sin θ − cos θ = p (20.13)
cosec θ − sin θ = q (20.14)
From Eq. (20.14), we have
1 − sin2 θ = q sin θ (20.15)
Adding Eq. (20.13) and Eq. (20.14), we have
cosec θ − cos θ = p + q
∴ 1 − sin θ cos θ = (p + q) sin θ (20.16)
Squaring Eq. (20.13), we have
1 − 2 sin θ cos θ = p2 (20.17)
20.1. Elimination of Unknown Quantity 350

Multiply Eq. (20.16) by 2 and subtract Eq. (20.17) from the result,
we have
1 = 2(p + q) sin θ − p2
p2 + 1
∴ sin θ = (20.18)
2(p + q)
Subtracting this value of sin θ in Eq. (20.15), we have
( )2 ( )
p2 + 1 q p2 + 1
1− =
4(p + q)2 2(p + q)
( )2 ( )
∴ 4(p + q) − p + 12 2
= 2q(p + q) p2 + 1
( )2 ( )
∴ p2 + 1 + 2q p2 + 1 (p + q) = 4(p + q)2 . ■

§ Problem 20.1.8. x = a cos θ + b cos 2θ and y = a sin θ + b sin 2θ. ♢


§§ Solution. We have
x + b = a cos θ + b (1 + cos 2θ)
= a cos θ + 2b cos2 θ
∴ x + b = cos θ (a + 2b cos θ) (20.19)
and y = sin θ (a + 2b cos θ) (20.20)
Squaring and adding Eq. (20.19) and Eq. (20.20), we have
(x + b)2 + y 2 = (a + 2b cos θ)2 (20.21)
Squaring and adding the original equations, we have
x2 + y 2 = a2 + b2 + 2ab (cos θ cos 2θ + sin θ sin 2θ)
= a2 + b2 + 2ab cos θ
∴ x + y − b = a2 + 2ab cos θ
2 2 2

= a (a + 2b cos θ)
( )2
∴ x2 + y 2 − b2 = a2 (a + 2b cos θ)2 (20.22)
Substituting from Eq. (20.22) in Eq. (20.21), we have
[ ] ( )2
a2 (x + b)2 + y 2 = x2 + y 2 − b2 . ■

§ Problem 20.1.9.
If m = cosec θ − sin θ and n = sec θ − cos θ,
prove that
2 2 2
m 3 + n 3 = (mn)− 3 . ♢
§§ Solution. From the first equation, we have
1 1 − sin2 θ cos2 θ
m= − sin θ = = .
sin θ sin θ sin θ
From the second equation, we have
1 1 − cos2 θ sin2 θ
n= − cos θ = =
cos θ cos θ cos θ
∴ mn = cos θ sin θ.
4 4
2 2 cos 3 θ sin 3 θ cos2 θ + sin2 θ 1 2
∴ m3 + n3 = 2
+ 2
= 2
= 2
= (mn)− 3 .

sin θ3 cos θ 3 (sin θ cos θ) 3 (mn) 3

§ Problem 20.1.10. Prove that the result of eliminating θ from the


equations
x cos (θ + α) + y sin (θ + α) = a sin 2θ,
and y cos (θ + α) − x sin (θ + α) = 2a cos 2θ
20.1. Elimination of Unknown Quantity 351

is
2 2 2
(x cos α + y sin α) 3 + (x sin α − y cos α) 3 = (2a) 3 . ♢
§§ Solution. The given equations may be written
(x cos α + y sin α) cos θ − (x sin α − y cos α) sin θ = 2a sin θ cos θ
and (x sin α − y cos α) cos θ + (x cos α + y sin α) sin θ = −2a cos 2θ
i.e. m cos θ − n sin θ = 2a sin θ cos θ (20.23)
and m sin θ + n cos θ = −2a cos 2θ (20.24)
where m = x cos α + y sin α and n = x sin α − y cos α.
Multiplying Eq. (20.23) by cos θ, Eq. (20.24) by sin θ and adding, we
have ( )
m = 2a sin θ cos2 θ − cos 2θ sin θ
( )
= 2a sin θ cos2 θ − 2 cos2 θ + 1 (20.25)
( )
= 2a sin θ 1 − cos2 θ = 2a sin3 θ (20.26)
Again, multiplying Eq. (20.23) by sin θ, Eq. (20.24) by cos θ and sub-
tracting, we have ( )
n = −2a cos 2θ cos θ + sin2 θ cos θ
( )
= −2a cos θ 1 − 2 sin2 θ + sin2 θ
( )
= −2a cos θ 1 − sin2 θ = −2a cos3 θ (20.27)
From Eq. (20.26) and Eq. (20.27), we have
Ä m ä 13
= sin θ (20.28)
2a
Ä n ä 13
= − cos θ (20.29)
2a
Squaring and adding Eq. (20.28) and Eq. (20.29), we have
Ä m ä 23 Ä n ä 23
+ = sin2 θ + cos2 θ = 1
2a 2a
2 2 2
∴ (x cos α + y sin α) 3 + (x cos α − y sin α) 3 = (2a) 3 . ■

Eliminate θ and ϕ from the equations:


§ Problem 20.1.11. sin θ+sin ϕ = a, cos θ+cos ϕ = b and θ−ϕ = α. ♢
§§ Solution. Squaring and adding the first two equations, we have
2 + 2 (sin θ sin ϕ + cos θ cos ϕ) = a2 + b2
∴ a2 + b2 = 2 + 2 cos (θ − ϕ) = 2 + 2 cos α. ■

§ Problem 20.1.12. tan θ + tan ϕ = x, cot θ + cot ϕ = y and θ + ϕ =


α. ♢
§§ Solution. We have
x tan θ + tan ϕ (tan θ + tan ϕ) tan θ tan ϕ
= = = tan θ tan ϕ.
y cot θ + cot ϕ tan θ + tan ϕ
Also,
tan θ + tan ϕ x xy
tan α = tan (θ + ϕ) = = x = y−x
1 − tan θ tan ϕ 1−
y
∴ xy = (y − x) tan α. ■
20.1. Elimination of Unknown Quantity 352

§ Problem 20.1.13.
a cos2 θ + b sin2 θ = c, b cos2 ϕ + a sin2 ϕ = d
and a tan θ = b tan ϕ. ♢
§§ Solution.
a cos2 θ + b sin2 θ = c (20.30)
b cos2 ϕ + a sin2 ϕ = d (20.31)
a tan θ = b tan ϕ (20.32)
From Eq. (20.30), we have ( )
a cos2 θ + b sin2 θ = c cos2 θ + sin2 θ
∴ (a − c) cos2 θ = (c − b) sin2 θ
a−c
∴ tan2 θ = (20.33)
c−b
From Eq. (20.31), we have ( )
b cos2 ϕ + a sin2 ϕ = d cos2 ϕ + sin2 ϕ
∴ (b − d) cos2 ϕ = (d − a) sin2 ϕ
b−d
∴ tan2 ϕ = (20.34)
d−a
From Eq. (20.32), we have
a2 tan2 θ = b2 tan2 ϕ (20.35)
Substituting from Eq. (20.33) and Eq. (20.34) in Eq. (20.35), we have
( ) ( )
a−c b−d
a2 = b2
c−b d−a
∴ a2 (a − c)(a − d) = b2 (b − c)(b − d). ■

§ Problem 20.1.14.
cos θ + cos ϕ = a, cot θ + cot ϕ = b
and cosec θ + cosec ϕ = c. ♢
§§ Solution.
cos θ + cos ϕ = a (20.36)
cot θ + cot ϕ = b (20.37)
cosec θ + cosec ϕ = c (20.38)
From Eq. (20.36), we have
θ+ϕ θ−ϕ
2 cos cos =a (20.39)
2 2
From Eq. (20.37), we have
cos θ sin ϕ + cos ϕ sin θ
=b
sin θ sin ϕ
∴ sin (θ + ϕ) = b sin θ sin ϕ (20.40)
From Eq. (20.38), we have
sin θ + sin ϕ = c sin θ sin ϕ (20.41)
From Eq. (20.40) and Eq. (20.41), by division, we have
sin (θ + ϕ) b
=
sin θ + sin ϕ c
θ+ϕ θ+ϕ
2 sin cos b
∴ 2 2 = (20.42)
θ+ϕ θ−ϕ c
2 sin cos
2 2
20.1. Elimination of Unknown Quantity 353

From Eq. (20.39) and Eq. (20.42), we have


θ+ϕ ab θ−ϕ ac
2 cos2 = and 2 cos2 =
2 c 2 b
ab
∴ 1 + cos (θ + ϕ) = (20.43)
c
ac
and 1 + cos (θ − ϕ) = (20.44)
b
From Eq. (20.43) and Eq. (20.44), by subtraction,Å 2 2 we ã have
( )
c b c −b
2 sin θ sin ϕ = a − =a .
b c bc
Hence from Eq. (20.40), we have
a ( 2 )
sin (θ + ϕ) = c − b2 .
2c
ab
But cos (θ + ϕ) = − 1.
( 2 2 )2 c ( )2
a c −b
2
ab
∴1= + − 1
4c2 c
which reduces to the form î
( )2 ó
8bc = a 4b2 + b2 − c2 . ■

§ Problem 20.1.15.
a sin θ = b sin ϕ, a cos θ + b cos ϕ = c
and x = y tan (θ + ϕ) . ♢
§§ Solution.
a sin θ = b sin ϕ (20.45)
a cos θ + b cos ϕ = c (20.46)
x = y tan (θ + ϕ) (20.47)
From Eq. (20.45), we have
a sin θ − b sin ϕ = 0 (20.48)
Squaring and adding Eq. (20.46) and Eq. (20.48), we have
a2 + 2ab cos (θ + ϕ) + b2 = c2
c2 − a2 − b2
∴ cos (θ + ϕ) = (20.49)
2ab
Now tan2 (θ + ϕ) = sec2 (θ + ϕ) − 1.
Hence, from Eq. (20.47) and Eq. (20.49), we have
( )2 ( )2
x2 2ab (2ab)2 − c2 − a2 − b2
= −1=
y2 c2 − a2 − b2 (c2 − a2 − b2 )2
( )( )
2ab + c2 − a2 − b2 2ab − c2 + a2 + b2
=
(c2 − a2 − b2 )2
[2 ][ ]
c − (a − b)2 (a + b)2 − c2
=
(c2 − a2 − b2 )2
(c + a − b)(c − a + b)(a + b + c)(a + b − c)
=
(c2 − a2 − b2 )2
( ) √
∴ x c2 − a2 − b 2
=y (a + b + c)(a − b + c)(a + b − c)(−a + b + c).
Otherwise thus :
20.1. Elimination of Unknown Quantity 354

Draw a triangle ABC whose sides are a, b and c. Then clearly


θ = ∠B and ϕ = ∠C.
x
∴ = tan (θ + ϕ) = tan(B + C) = − tan A.
y
On substituting the values of Arts. 164 and 169, we have the same
answer as before. ■

§ Problem 20.1.16.
x y x y
cos θ + sin θ = 1, cos ϕ + sin ϕ = 1
a b a b
θ ϕ θ ϕ
and a2 sin sin + b2 cos cos = c2 . ♢
2 2 2 2
§§ Solution.
x y
cos θ + sin θ = 1 (20.50)
a b
x y
cos ϕ + sin ϕ = 1 (20.51)
a b
θ ϕ θ ϕ
a2 sin sin + b2 cos cos = c2 (20.52)
2 2 2 2
From Eq. (20.50) and Eq. (20.51), by addition, we have
x θ+ϕ θ−ϕ y θ+ϕ θ−ϕ
cos cos + sin cos =1
a 2 2 b 2 2
x θ+ϕ y θ+ϕ θ−ϕ
∴ cos + sin = sec (20.53)
a 2 b 2 2
From Eq. (20.50) and Eq. (20.51), by subtraction, we have
x θ+ϕ ϕ−θ y θ+ϕ θ−ϕ
sin sin + cos sin =0
a 2 2 b 2 2
x θ+ϕ y θ+ϕ
∴ sin = cos (20.54)
a 2 b 2
From Eq. (20.53) and Eq. (20.54), we have
Äxä Äyä x θ+ϕ y θ+ϕ θ−ϕ
cos + sin sec
a = b = a 2 b 2 = 2
θ+ϕ θ+ϕ 2
θ+ϕ 2 θ+ϕ 1
cos sin cos + sin
2 2 2 2
(20.55)
Again, from Eq. (20.52), we have
( ) ( )
θ−ϕ θ+ϕ θ−ϕ θ+ϕ
a2 cos − cos + b2 cos + cos = 2c2
2 2 2 2
( ) θ−ϕ ( 2 ) θ+ϕ
∴ a2 + b2 cos + b − a2 cos = 2c2 (20.56)
2 2
Also, each fraction in Eq. (20.55)
Õ
x2 y2 √
+ 2 b2 x2 + a2 y 2
= a2 b =
θ+ϕ θ+ϕ ab
cos 2 + sin2
2 √2
θ−ϕ b2 x2 + a2 y 2
∴ sec =
2 ab
θ−ϕ ab
∴ cos = √
2 b x + a2 y 2
2 2
Äxä
θ+ϕ bx
and cos = a = √ .
2 θ−ϕ b x + a2 y 2
2 2
sec
2
20.1. Elimination of Unknown Quantity 355

θ−ϕ θ+ϕ
Hence, substituting these values of cos and cos in
2 2
Eq. (20.56), we have ( ) ( )
ab a2 + b2 + b2 − a2 bx
√ = 2c2
b2 x2 + a2 y 2
[ ( ) ( )]2 ( )
∴ b2 x b2 − a2 + a a2 + b2 = 4c4 b2 x2 + a2 y 2 . ■

▲▲▲

▼▼▼
T
h
is
pa
ge
is
in
te
n
ti
on
al
ly
le
ft
bl
an
k.
T
h
is
pa
ge
is
in
te
n
ti
on
al
ly
le
ft
bl
an
k.

You might also like